MJDF MCQ Collection

Download as pdf or txt
Download as pdf or txt
You are on page 1of 481

2020

MJDF MCQs , MFDS


DR ABDULRAHMAN ALMUALM

Rev. Dr nagham

MJDF WHATSAPP GROUP \\2020


MJDF MCQS WITH ANSWERS

This is for every enthusiastic Dentist who is working hard to


become better version of him self

To all Doctors in our beautiful Group


Those who gave their efforts
And time

THANK YOU A LOT


I can say that “ The beautiful results are from the Beautiful
Teams “

Special thanks to the doctors who gave their


time ,work,energy ,who were committed to this file who
were correcting and participating in the knowledge spread
for the sake of knowledge
Contributors & reviewers :
1. Dr Nagham katran
1. Dr Mujtaba Ali mohammed Osman
2. Dr Naima Khalid
3. -Dr Mohammed Hamza
4. -DR Rana Ahmed
5. DR Y.Rodwan
6. DR Rana Fayez
7. Dr Uffaq Rao

#Stay safe -Stay home

BY DR.ABDULRAHMAN ALMUALM
MJDF MCQS WITH ANSWERS

We are not perfect people but we always seek to be better


I highly recommend that everyone should verify answers
based on your intuition and knowledge or references
The aim was to collect every question from past papers
Some questions without answers , you can search for
answers or post the question on our what’sapp group
Good luck for every one.and I hope this file can help you
out

Dr.Almualm

1. What is the mode of action of penicillin?

a) It breaks down cell walls


b) It inhibits β-lactamases
c) It inhibits cell wall synthesis
d) It inhibits transcription
e) It inhibits translation
C
2.Which one of these biochemical changes is associated with Paget’s disease?
a) Anaemia
b)Vitamin increase
c) Raised alkaline phosphate
d) Low serum calcium
e) High serum phosphate
C
3)A bone biopsy shows irregular Chinese characters. Which one of the
following is the diagnosis?
a) Paget’s disease
b) Giant cell granuloma
c) Cherubism
d) Fibrous dysplasia e) Osteogenesis imperfect
D
4) Which cranial nerves pass through the cavernous sinus?
a) III, IV and V
b) III, IV, V and VI
c) IV, V and VI
d) IV and VI
e) VII
Ans is B

BY DR.ABDULRAHMAN ALMUALM
MJDF MCQS WITH ANSWERS

5 )Which is the most appropriate image to use when assessing third molars?
a) Bitewing radiograph
b) Long cone CT
c) Long cone periapical radiograph
d) Orthopantomogram (OPG)
e) PA mandible
Ans is D

6) Patient with prosthetic heart valve taking 7.5 mg warfarin.


She has????. Patient needs extraction. What is your
management
A. 3g Amoxicilin, suture after surgical removal
B. 3g Amoxicillin Gentamycin/vancomycin cover, stop warfarin, give heparin and
suture
later
D. Ampicillin cover, stop warfarin, give heparin and suture later
E. Gentamycin/vancomycin cover, stop warfarin and suture later
According to new nice guidelines high risk patient and high risk procedures then you
can give prophylactic antibiotic 3g Amoxicillin or clindamycin in case of allergy 600
mg .see NICE GUIDELINES FOR FURTHER INFOS
Ans is A

7. A Female patient with history of hip fracture and osteoporosis taking oral
bisphosphonates
from about 2 years, no other medications. She want to extract a tooth:
a) Extract with no alteration but advice to keep her mouth clean
b) Stop the drug 3 days before extraction after consultation with GP
c) Consider 3 months drug holiday after consultation with GP
d) Never to extract, RCT and crown amputation
e) Extract under antibiotic cover

BONJ is very rare incidence it results from using BIPHOSPHONATE DRUG


TREATMENT
Ans is A

BY DR.ABDULRAHMAN ALMUALM
MJDF MCQS WITH ANSWERS

Antibiotics has no evidence to prevent BONJ / ALWAYS TREAT PATIENT ANS


MAKE HIM FIT BEFORE BIPHOSPHONATE TREATMENT
After extraction always make it a atruamatic avoid raising flaps ,follow up with
patient after 4 weeks to check healing ,if no healing in 4 -6 weeks refer to
maxillofacial with patient medical and dental history

8) Which type of cells does an abscess contain


A. Mast cells
B. Polymorphonuclear leukocytes
C. Eosinophils
D. Epithelial cells
ANS B

9) . Irrigation solution for RCT when there infection and draining is :


a) Sodium hypochlorite
b) Iodine potassium
c. Naocl& h2o2
d) Normal saline
ANS IS A
10) Ideal INR check if the pt is undergoing surgical removal of impacted 3
rd molar
a) On day of operation
b) Within 24 hours
c) 72 hours
ans is B //ideal is 24 ..limit within 72.

11) Child aged 10 with 1 mm pulp exposure of upper c;entral ,which is the preferred
treatment option
-='
a) Cvek pulpotomy
b) Direct pulp capping
c) Indirect pulp capping
d) Complete pulpotomy
A

12) A 65 year old patient needs extraction of 44; he has taken


insulin in the morning. What preoperative advice you shou
give
A. Take more sugar
B. Maintain normal diet
C. Antibiotic 2 hours before
D. Medication increases preoperatively
B
13 When a RPD is terminally seated, the retentive clasp tips should

BY DR.ABDULRAHMAN ALMUALM
MJDF MCQS WITH ANSWERS

a) Apply retentive force to the body


B) Exert no force
C) Be invisible
D) Resist torque through the long axis of the teeth
Ans is B

14 ) The biting load of a denture base to the gingival tissue compared to teeth are
a) Ten times more
b) Ten times less
c) equal
Ans B

15 If a child’s teeth do not form; this would MOSTLY affects the


growth of:
A. Alveolar bone
B. Whole face
C. Mandible
D. Maxilla
Ans is A

16 Patients comes to you with an ulcer on the lateral border of his tongue. He wears
a
lower full
denture and is complaining that his denture is loose and sore. How would you
manage?
a) Refer the patient as an emergency case
b) Refer the patient to the oncologist
c) Relieve the denture and ask the patient to come in for a review
d) Send the patient to the Oral Maxillofacial surgery department
e) Take biopsy
Ans is C

17 Haemophilia is characterised with


A. Daughters affected from their carriers fathers
B. Presents on “y” chromosome
C. Hemarthrosis is common finding
D. Deficiency of factor VII
E. Neutrophil defect
Ans is C Hemarthrosis, or articular bleeding, means bleeding into the joints. It can
occur after an injury, but is also a complication of a genetic bleeding disorder known
as hemophilia

BY DR.ABDULRAHMAN ALMUALM
MJDF MCQS WITH ANSWERS

18) Patients on Oral Bisphosphonates


a) Must not be treated in general dental practice
b) Will develop osteonecrosis of the jaw after undergoing
tooth

extraction
c) Require antibiotic prophylaxis with3g Amoxicillin provided they are
not allergic to penicillin
d) Most probably have metastatic bone disease or osteoporosis
E) Must have their bisphosphonate medication stopped 24 hours
prior to dental treatment
D is the answer

19) Which of the following has highest five year survival rate for class V restoration?
A. Amalgam
B. Bonded amalgam
C. Composite (flowable)
D. Conventional GIC
E. Resin modified GIC
Ans is E

20 Which of the following nerves is at risk during the extra-oral drainage of a


submandibular abscess?
a) Cervical branch of the facial nerve
b) Lingual nerve
c) Mandibular branch of the trigeminal nerve
d) Marginal mandibular branch of the facial nerve
e) Mental nerve
ans is D

21 Smallest angle for amalgam is


a) 70
b) 90
c) 110
Ans A

22) A pt presents with a history of clicking from their tmj. This click occurs midway
through the

BY DR.ABDULRAHMAN ALMUALM
MJDF MCQS WITH ANSWERS

opening cycle and is consistent. There is some preauricular pain and the lateral
pterygoid muscle on
the affected side is tender to resisted movement test. There is no trismus and the
click is not present
when the patient opens from a incisor edge to edge relationship ,instead of her
normal class-1
occlusion. The patient would like treatment, the most appropriate occlusal splint for
this patient would
be
a) Stabilisation splint
b) localised occlusal interference splint
c) Bite raiser
d) Soft biteGuard
e) Anterior Repositioner Splint
ans is e

23) Blow to mandible causing fracture in molar’s right side


region, you expect a second fracture of
A. Sub condylar of right side
B. Sub-condylar of left side
C. Fracture of symphysis
Ans B

24) Which salivary gland do not secrete mucous saliva


a) Parotid
b) Submandibular
c) Sublingual
d) Minor palatal salivary gland
e) Labial
Ans is A

25 If u suspected a child abuse in your patient,u check for:


a) Bruises in neck
b) Scratches in knee and elbow.
c) Child distress and agitated
ans is A most of Non accidental injuries signs are bruises on neck

26 A study was conducted with the following objective: To determine the individual
and combined

BY DR.ABDULRAHMAN ALMUALM
MJDF MCQS WITH ANSWERS

effects of potential risk factors in relation to the temporomandibular disorder, Pain


Dysfunction
Syndrome (PDS). The first group comprised new referrals between May 1997 and
August 1999 to the
temporomandibular disorder clinic of the University Dental Hospital, diagnosed with
PDS. The second
group consisted of patients without PDS randomly selected from twenty-four dental
practices. Using a
postal questionnaire information was collected on socio-demographic, local
mechanical, psychological
factors, co-morbidities and illness behaviour.What type of study is this?
a) Case series
b) Cross-sectional study
c) Case-control study
d) Case report
e) Randomised clinical trial
Ans c

27 Which antibiotic associated with infectious mononucleosis or glandular fever


a) Penicillin
a) Ampicillin
b) Tetracycline
Ans is ampicillin

The top 3 reported drugs were ampicillin, azithromycin, andamoxicillin. Incidence of


this adverse effect was higher in the 1960s (55.6%, 45%, and 33%) than in 2013
(33% and 15%) ref NCBI.nlm.nih.gov

28What is the usual range of orthodontic force for bodily tooth movements?
• A. 25-50g
• B. 50-75g
C.75-100g
D.100-150g
E.>150g
Ans is D

BY DR.ABDULRAHMAN ALMUALM
MJDF MCQS WITH ANSWERS

29 In bpe if there is no bleeding no calculus or overhang restoration,only pocket


2mm, BPE is
a) 0
b) 1
C) 2
d) 3
ans is A =0

30) Which of the following is regarded by British Dental Association as risk factor for
oral cancer?
A Age
B Candida infection
C Chronic sepsis in mouth
D Excessive exposure to sunlight and radiation
E Immune deficiency disease
Ans is D

31) Attached gingiva is the thinnest in the mouth in the following region of the
mouth?
a) Maxillary anterior buccal region
b) Maxillary molar buccal region
c) Mandibular anterior buccal region
d) mandibular anterior lingual region
E mandibular molar lingual region
Ans is C

32 Question about AED and what you are gonna do aftrer fierst given shock..
• A) give another shock
• B)check ABCDE and start CPR
• C) recovery position
• D) give two breath
Ans is B

33 Patient had his history of epilepsy and presents to your practice with a painful
dental abscess. She is currently on Carbamazepine. Which of the following antibiotic
is contraindicated?
a. Amoxicilin

BY DR.ABDULRAHMAN ALMUALM
MJDF MCQS WITH ANSWERS

b. Erythromycin
c. Clindamycin
d. Metrodinazone
e. Tetracycline
Erythromycin interferes with liver microsomal metabolism of carbamazepine,
causing decreased carbamazepine clearance, an elevated
serum carbamazepine level, and clinical toxicity. Erythromycin-
inducedcarbamazepine toxicity usually manifests as drowsiness, lethargy, ataxia,
vomiting, and nystagmus.

34 After doing pulpectomy, you performed a readymade stainlesteal crown and


space maintainer So
under which band you should treat this child:
a) Band1
b) Band 2
c) Band3
d) Band 4
e) No claim
Ans is A

35 what is the best distance to achieve good apical area preparation


a) 0.5mm
B) 1.0mm
C) 1.5mm
D 2.5 mm
Ans B
36 Best way of giving oral health education
a) Nurse advice
b) Leaflet
c) Diagrams
d) Makesure thepatient understands what causes gum disease and dental caries
Ans is D

37 Which of the following is NOT an ideal requirement for an interocclusal record


a) Does not displace teeth during intercuspation
B) May shrink to about 5% after setting
C) Should reproduce occlusal and incisal surfaces accurately
D) Remain rigid after setting
E) Offers minimal resistance during closure in order to Reduce mandibular flexion or
displacement
Ans is B
38 Which of the following cannot be done by a Level 2 Practitioner?
a. Extraction of an 8 that is distoangular impacted
b. Surgical removal of a cyst
c. surgical treatment of a disorder involving orofacial pain
ans is C

39) 12.After consultation with the patient’s cardiologist regarding the risk of infective
endocarditis, antibiotic prophylaxis was advised, which of the

BY DR.ABDULRAHMAN ALMUALM
MJDF MCQS WITH ANSWERS

following antibiotics would least likely cause an alteration of the patient’s


INR?
a. Metronidazole.
b. Erythromycin.
c. Amoxicillin. (According to BDA SHEETS AMOXICILLIN RAISE INR)
d. Tetracycline.
e. Clarithromycin.
Ans is

40A 55 year old lady has attended requesting extraction of lower right first
molar which has a questionable restorability. Her medical history reveals
that she has diabetes mellitus, post-menopausal osteoporosis, and
rheumatoid arthritis. She has been taking the following medications orally
since 2009; metformin, alendronic acid and methotrexate. What would be
the risk of having MRONJ from the extraction of this tooth?
a. High.
b. Moderate.
c. Low.
d. Negligible.
Ans is A

41.Which of the following medications is most likely to interfere with one or


more of the patient’s medications in question 40?
a. Paracetamol (acetaminophen).
b. Ibuprofen.
c. Codeine.
d. Oxycodone.
e. Meperidine.
Ans is B (methotrexate interact with ibuprofen)

42 Vasoconstrictor in local anesthetics (LA) solution:


a) Reduces toxic effects of LA solution.
b) Decreases depth & duration of LA.
c) Increases bleeding.
d) Increases toxic effects of LA.
e) Has no effect on efficacy of LA solution
Ans is A

43 Which one of the following is an absolute contraindication to the


administration of 2% lignocaine with 1:80 000 adrenaline as local anaesthesia
for an oral surgery procedure?
a) Hypertension
b) Myocardial infarct 3 years ago
c) Previous coronary artery bypass graft (CABG)
d) Previous ‘faint’ following administration
e) Proven allergy to the local anaesthetic
ans is E

44) What is the first-line treatment for suspected anaphylactic shock?


a) Cardiopulmonary resuscitation

BY DR.ABDULRAHMAN ALMUALM
MJDF MCQS WITH ANSWERS

b) Intramuscular administration of adrenaline 1 mg/1 ml 1:1000


c) Intramuscular administration of adrenaline 1 mg/1 ml 1:10 000
d) Intravenous administration of adrenaline 1 mg/1 ml 1:1000
e) Intravenous administration of adrenaline 1 mg/1 ml 1:10 000
Ans is B
45) 1 which test is the most reliable test to indicate the presence of active hepatitis:
a) Hbs(surface)antigen
b) Hbc(core)antigen
c) Hbe antigen
d) Antibody to HBs antigen
Ans is C
46) 7 What is the medical condition of a man who came to your clinic asking you to
remove radioreceptor
from his upper premolar
a) Mania
b) Anexity
c) Depression
d) Schizophrenia
Ans A
47 Canine Miller Cl 2 recession which flap :
A-full thickness with full split
B-full thickness with partial split
C-connective tissue graft with full split
D-connective tissue graft with partial split
Ans is C

48 68) Patient has an over jet of above 9mm, which IOTN band it will be
a) Band 1
b) Band 2
c) Band 3
D)Band 4
e)Band 5
Ans is C band 3 |
IOTN 1 and 2 --> no claim
IOTN 3 with aesthetic component 1-4 --> no claim
IOTN 3 with aesthetic component 5-10 --> band 3
IOTN 4 and 5 --> band 3
Overjet 6-9 is IOTN 4
Both IOTN 4 and 5 are eligible for NHS orthodontic treatment under band 3

49 Regarding drug metabolism, which statement is FALSE?


a) Many are metabolised in the liver
b) The liver may bio-transform inert pro-drugs into more active metabolites
C) The kidney is the main site for drug excretion
d) After absorption of a drug in the GIT, it is distributed in the blood stream bound to
hemoglobin
ans is D
50) A 15 year old child and her mother are demanding veneers to close a gap. You
feel that
orthodontic treatment would be better, how would you best rnanaqe the situation?

BY DR.ABDULRAHMAN ALMUALM
MJDF MCQS WITH ANSWERS

a) Arrange for a second opinion


B) Refuse to treat the patient
C) Treat according to the patient's wishes
D) Refer for an orthodontic opinion
Ans is D

51)Success of post retained crown on a CENTRAL incisor is dependent on


a) Post length greater than crown height
b) At least 5mm apical seal
c) Ferrule effect
d) Tough or serrated post
e) All the above
Ans is E

52 When an individual is exposed to radiation a certain amount of radiation is


needed before clinical signs of damage to somatic cells appear.
For these effects to occur a minimum radiation dose has to be exceeded and this is
known as:
a) Background radiation dose
b) Threshold dose
c) Equivalent dose
d) Absorbed dose
e) Effective
Ans is B
53. 87) Most common complication withorthodontic treatment
a) Pulpitis
b) Root resorption
c) Tmj problem.
Ans is B
54.Cervical line and dark lines on Central incisors, molars and incisal edges, most
possible reason
could be
a) Ameologensis
b) Dentinogenesis
C) Childhood illness
D) Hypoplasia
Ans is C

55 Patient who has WBC count of just over 100000 is most


likely suffering from
A. Leucopoenia
B. Leukaemia
C. Polycythemia
Ans is B

56.diabetic with abscess complain of failure of root canal therapy and during the
examination the filling was leaking? What the reason
. a) Lack of coronal seal
b) Lack of apical seal
c) Medical condition

BY DR.ABDULRAHMAN ALMUALM
MJDF MCQS WITH ANSWERS

Ans is a

57.Which is NOT CORRECT in regards to lingual nerve


A. It is posterior and medial to the inferior alveolar nerve
B. It passes close to the mandibular 3rd molar
C. It may be anaesthetized by the mandibular nerve block
D. It provides supply to the lingual gingiva
Ans is A (its anterior and medial )

58.In regards to Benzodiazepines


A. Increases R.E.M. sleep
B. Has a hangover effects because of active metabolism
C. Includes carbamazepine
D. Can be used safely on children as it achieves reliable effects
Ans is B

59.Patient with haemophilia presents which of the following


findings**
A. Increased prothrombin time
B. Increased bleeding time
C. Increased clotting time
E.increased partial thromboplastin time PTT
Ans is.E hemophilia patient have all.normal but ptt elevated

60. Gagging reflex is caused by


A. Trigeminal nerve
B. Glossopharyngeal
C. Facial nerve
D. Recurrent laryngeal
Ans is b

61 The characteristic feature of basal cell carcinoma is


A. Blood metastasis
B. Does not erode bone
C. Intensive involvement / inveterately characteristic/
D. Radio resistant
Not sure b or c

62.. What is not correct about Long Buccal Nerve**


A. Passes through two heads of pterygoids muscles
B. Supplies mucosa over lower and upper molars
C. Supplies the buccinator muscle
D. Supplies skin over buccinator
Ans is B

63. Antibiotics should be used routinely to prevent infection

BY DR.ABDULRAHMAN ALMUALM
MJDF MCQS WITH ANSWERS

arising from oral surgery in patients suffering from all the


following EXCEPT
A. Agranulocytosis
B. Sever uncontrolled diabetes
C. Aplastic anaemia
D. Mumps
E. Leukaemia
Ans is D
64. The most serious complications which may occur from
abscess of max canine is :
A. Cellulitis
B. Cavernous sinus thrombosis
C. Lacrimal duct stenosis
D. Damage to infra orbital nerves
Ans is B

65.Important histological feature of carcinoma in situ.


a) Intact basement membrane.
b) Absence of metastasis.
c) Absence of connective tissues involvement.
Ans is A

66.The maxillary and mandibular teeth get their blood supply


from**
A. Separate branches of S. Palatina artery
B. Separate branches of maxillary artery
C. Branches of maxillary and mandibular arteries
Ans is B

67. Toxicity as a result of anaesthetic solution can be seen more


when
A. Injection in supine position
B. Injection into vascular area
C. Injection without vasoconstrictors
D. Intravenous injection
Ans is D

68. Elongation of pulp chamber seen in:


a.gemination
b.turner teeth
c.taurodontism
d.dilaceration
ans is C

69. NSAIDs do not interact with


a) Phenytoin
b) Antihypertensive drugs

BY DR.ABDULRAHMAN ALMUALM
MJDF MCQS WITH ANSWERS

c) Methotrexate
d) Systemic Corticosteroids
e) Fluoxetine
Ans is e

70.Which of the following interfere with contraceptive pills:


a) Carbamazbine
b) Alcohol
c) Ampicillin
d) Phenytoin
ans is C

71. In dental epidemiology, The main limitation of DMF index is


a) It does not allow statistical analysis
b) It gives equal weight to decayed,missing,filled teeth
c) It is difficult to calibrate examiners
d) It is reversible
e) There is no gold standard
Ans us B

72. ) You have just completed conventional endodontics on UL7. The tooth had
originally been
restored with an extensive MOD amalgam and is now significantly further weakened.
What is the best
immediate restoration for this tooth?
a) Direct compositewith cusp reduction
b) Direct composite with full cuspal coverage
c) Indirect composite or porcelain onlay
d) MOD amalgam with Nayyar core capping all cusps
e) Porcelain fused to metal crown on amalgam core

Ans D

73) Which is the best cantilever bridge for missing max canine, the abutment on
a) Both premolars
B) Lateral and Central Incisors
C) Lateral incisor
D) First premolar
Ans is A

74 .The purpose of recording protrusive relation is


a) To register the condylar path and to adjust the' inclination of incisal guidance
b) To aid in determining the freeway space and to adjust the inclination of incisal
guidance
C) To register the condylar path and to adjust the condylar guides of the articulator
so that they are equal to the condylar paths of the patient
d) To aid in establishing the occlusal vertical dimension and to adjust the condylar
guides of the articulator so that they are equivalent to the condylar paths of the
patient.

BY DR.ABDULRAHMAN ALMUALM
MJDF MCQS WITH ANSWERS

Ans is C

75. according to the recent radiation protection recommendations, The annual dose
limit for non-classified workers shouldn't exceed:
a) 20 MSV (classified workers)
b) 50 MSV
c) 6 MSV
d) 26 MSV ans is C || general public =1 msv
76. Which odontogenic pathology is associated with Afro-Caribbean men?
a) Ameloblastoma
b) Odontomes
c) Solitary bone cyst
d) Dentigerous cyst
e) Keratocyst
Ans is A

77. What percentage of the population has a missing wisdom tooth? a) 0.5%
b) 2%
c) 5%
d) 10%
e) 20%
Ans e

78. A 78-year-old woman presents for the extraction of her lower left 6. She
is deaf with a ‘cotton wool’ bone appearance on OPG. Which one of the
following is the diagnosis?
a) Paget’s disease
b) Giant cell granuloma
c) Cherubism
d) Fibrous dysplasia
e) Osteogenesis imperfecta
ans is A

79. 20. Which one of the following syndromes is associated with multiple tori?

a) Eagle’s syndrome
b) Frey’s syndrome
c) Gardner’s syndrome
d) Golin–Goltz syndrome
e) Crouzon’s syndrome
Ans is C

80.. Which one of these is not an NHS justification for implant placement?
a) Prosthetics tolerance issues
b) Trauma
c) Hypodontia
d) teaching
e) Oncology

BY DR.ABDULRAHMAN ALMUALM
MJDF MCQS WITH ANSWERS

Ans is D

81. 17. What is Vicryl made from?


a) Polyglycolic acid
b) Polydioxanone
c) Polyglactin
d) Polyglecaprone
e) Polyglyconate
ans is C

82. Treatment for patient of cherubism is:


a) Surgical excision.
b) Enblock dissection.
c) Radiation therapy.
d) Cosmetic surgery.
e) Do not require any treatment.
Ans is E Treatment of cherubism consists of local curettage of the lesions, jaw
contouring, intralesional steroid injections, and systemic calcitonin
administration as well. Calcitonintherapy for central giant cell granuloma of
the jaws is well documented, and favorable results have been achieved.

83. 6. Moved buccally while pushing coronally, but frequently need a varied
rocking movement to extract:
a) Upper 1, 2, 3
b) Upper 4, 5
c) Upper 6, 7
d) Lower 4, 5
e) Lower 6, 7
ans is C

84. Blanching at the site of injection is caused by:


a) Increased tissue tension.
b) Local effects of reducing agent in LA solution.
c) Decreased tissue tension.
d) Infection at the site of injection.
e) Warm LA solution.
Ans is A

85. Most common complication after tooth extraction is:


a) Bleeding.
b) Alveolar osteitis.
c) Condensing osteitis.
d) Infection.
e) Swelling.
Ans is B

86. Most common complication of rheumatoid arthritis involving


TMJ is:

BY DR.ABDULRAHMAN ALMUALM
MJDF MCQS WITH ANSWERS

a) Ankylosis.
b) Subluxation.
c) Osteoma of condyle.
d) Recurrent dislocation.
e) TMJ pain dysfunction syndrome.
Ans is A

87. Local anesthetic solution with highest tissue irritancy is:


a) Procaine.
b) Lignocaine.
c) Bupivacaine.
d) Mepivacaine.
e) Chloroprocaine.
Ans is E

88. Reducing agent in LA solution is:


a) Methyl paraben.
b) Sodium metabisulphite.
c) Thymol.
d) Adrenaline.
e) Ringer lactate.
Ans is B
89. Who is required to be registerd with GDC?
• A) receptionist
• B) techincan
• C) cleaner
• D) manager
Ans is B

90) What is the level of evidence for expert opinion


• A)1
• B)2
C)5
D)4
Ans is C

91. Manifestation of Coelic disease in the mouth:


• A) mucosal tags
• B) pigmentation
• C) attrition
• D )abrasion
• E) hypodontia
Ans is A

BY DR.ABDULRAHMAN ALMUALM
MJDF MCQS WITH ANSWERS

92.Your dental nurse is qualified to take radiograph. Which of the following is


required for her to carry out her task?
• A. Verbal prescription from you
• B. Written prescription from you
• C. Written prescription from lead nurse
• D. Written prescription from practice manager
• E. Written prescription from radiation protection supervisor
Ans is B

93. A 15 year old female patient has a 8mm overjet and bilateral impacted canines
a) The patient would be placed in the Index of Orthodontic Treatment Need DHC
grade 5
b) The patient is at an appropriate age to commence functional appliance therapy
c) The canine position is best assessed by using a lateral cephalometric radiograph
d) The canines are almost certainly buccally placed
e) Bilateral impacted canines are seen more frequently then unilateral impacted
canines
Ans is A

94. During the restoration of a deep carious cavity in UL6, to minimise the risk of
bacteria gaining access to the pulp, the dentist may
a) Carry out direct pulp capping
b) Carry out indirect pulp capping
c) Remove the caries from the floor of the cavity before the cavity wall
d) Give the patient a course of antibiotics for a week
e) Ask the patient to rinse with a fluoride mouthwash one hour prior the cavity
ppreparation
Ans is B

95. A 35 year old female patient requires replacement of 6 upper anterior crowns
with a conformative occlusal scheme. What would be the
most appropriate way to articulate study and working casts?
a) Average value articulator with incisal guidance table.
b) Fully adjustable articulator
c) Plane line articulator.
d) Semi adjustable articulator with facebow transfer and custom incisal guidance
table.
e) Semi adjustable articulator with facebow transfer.
Ans is D

96. Regarding root fractures, which of the following statements are LEAST
appropriate
a) Vertical root fractures occur most commonly in sound teeth as a result of direct
physical trauma
b) They occur more commonly in endodontically treated teeth
c) They could occur due to corrosion of metal posts
d) Wedging effects during post cementation are known to cause vertical root fracture
e) Horizontal root fractures are most commonly the results of direct physical trauma
to the dentition
Ans is A

BY DR.ABDULRAHMAN ALMUALM
MJDF MCQS WITH ANSWERS

97. Regarding inhalation sedation, which of the following statements is MOST


appropriate
a) Nitrous oxide sedation is only useful for children under 12 years of age.
b) Nitrous oxide should not be used on patients with liver and kidney disease.
c) Flumazenil may be used to reverse sedation produced by nitrous oxide.
d) 100% oxygen should be administered to the patient for 3-5 minutes once the
nitrous oxide has been terminated.
e) Nitrous oxide should not be used on patients with cerebral palsy
ans is D?

98. Secondary research papers in which all the primary studies on a particular
subject have been hunted out and critically appraised
according to rigorous criteria are called
a) Randomised controlled trials
b) Case control studies
c) Systemic reviews and meta-analysis
d) Cross sectional surveys
e) Cohort studies
Ans is C

99. Surgical endodontic treatment


a) Is indicated for all failed RCTs
b) Has a success rate of 20%
c) Is indicated if there is a periapical radiolucency seen on the radiograph
d) Is indicated to prevent removal of extensive coronal restorative work
e) Is contraindicated in all multi-rooted teeth
Ans is D

100. When should handpieces be placed and sealed in an autoclave pouch/bag?


a) Whenever they are autoclaved
b) Never
c) Once a day at the end of the session
d) When autoclaved in a vacuum autoclave
e) When autoclaved in a downward displacement type autoclave
Ans is D

#follow up and recall


: high caries risk --> 6 months in pedo and adult
Low caries risk --> 12-18 months in primary dentition, 2 years in permanent dentition
: moderate caries risk --> 12 months for both

101. Which of the following is NOT a potential consequence of submerged


deciduous teeth
a) Over eruption
b) Centre line shift
c) Tipping of adjacent teeth
d) Premature exfoliation
e) Increased difficulty of extraction
ans is D

BY DR.ABDULRAHMAN ALMUALM
MJDF MCQS WITH ANSWERS

102. Worst periodontal prognosis usually with??


A first mandibular molar
B upper second premolar
C upper first molar
D lower second premolar
Ans is c?

103. Which interleukin is most responsible fo neutrophil chemotaxis?


A. IL-1
B. IL-4
C. IL-8
D. IL-15
E. IL-20 ans is c
104. Which complement component is found in high levels in gingival crevicular fluid
and cause chemotaxis and vasodilation?
A. C2
B. C4a
C. C4b
D. C5a
E. C5b
Ans is D

105. What is legal amount (unit)of alcohol intake to men?


Ans was 21 per week for men
14 for women
Recent guidelines are 14 for both men and women

106.which fluoride varnish aggravates effects of ANUG?


Duraphat

107)which antifungal shouldn’t be given with diazepam?


Itraconazole, fluconazole ?
Ans itraconazole ( all azoles are avoided with diazepam but itraconazole seek
alternative) fluconazole = monitor

108. Basal cell carcinoma is characterised by


A. Rapid growth and metastasis
B. Local cutaneous invasion
C. Inability to invade bone
D. Poor prognosis
E. Radiation resistance
F. Can not metastasise to the bone
Ans is B

109. Benign migratory glossitis or Geographic Tongue, manifests


itself in the oral cavity as
A. Irregularly outlined areas of hyperkeratosis of the dorsal surface of
the tongue
B. Furrows outlined the dorsal surface radiating out from a central groove
in the centre of the tongue

BY DR.ABDULRAHMAN ALMUALM
MJDF MCQS WITH ANSWERS

C. Loss (atrophy) of filiform papillae in multiple irregularly outlined areas


D. Irregularly outlined erythematous area of hyper trophic fungiform
E. A fibrinous exudate on the dorsal surface
F. Grooves (fissures) radiating from a central fissure
G. Irregular area in the midline of the tongue
Ans is C

110. In regards to HIV infection, which of the following is the


earliest finding
A. Kaposi sarcoma on the palate
B. Reduced haemoglobin
C. Infection with pneumocystic carinii
D. Reduction in white cells count
E. B cell lymphoma
Ans is D

111. Endogenous morphine like substance which can control pain is


known as
A. Bradykinins
B. Peptides
C. Prostaglandins
D. Serotonins
E. Enkephalins
Ans is E

112. -A patient who recently had a calculus removed from the


kidney presented with radiolucent area in the left maxilla with
clinical evidence of swelling. The disease that you would
immediately suggest is
A. Diabetes
B. Thyrotoxicosis
C. Hyperparathyroidism
D. Osteoporosis
E. Adrenal insufficiency
Ans is C

113. Trichloroacetic acid, a strong acid, has been used by dentists


for chemical cautery of hypertrophic tissue and aphthous
ulcers; its mechanism of action is
A. Thermodynamic action
B. Activation of tissue enzymes
C. Osmotic pressure
D. Protein precipitation PPT
E. Neutralization
Ans is D :ref boucher mcq book

114. X-ray films have an emulsion on one or both side of a support


material. The emulsion contains particles of
A. Silver nitrate crystal

BY DR.ABDULRAHMAN ALMUALM
MJDF MCQS WITH ANSWERS

B. Metallic silver in gelatine


C. Silver bromide in gelatine
D. Silver nitrate in gelatine
E. Potassium bromide in gelatine
Ans is c

115. The inverse Square Law is concerned with intensity of


radiation using type D film of 200mm target to film distance,
the exposure time was 0.25s. What would be the exposure for
the same situation with 400mm target to film distance
A. 0.5s
B. 1.0s
C. 2.0s
D. 0.25s
E. 0.125s
Ans is B

116. Exposure of the patient to ionising radiation when taking a


radiograph is NOT REDUCED by
A. The use of fast film
B. The addition of filtration
C. Collimation of the beam
D. The use of an open and lead lined cone
E. Decreasing the kilovoltage KvP
Ans is E

117. In the inferior alveolar block the needle goes through or close
to which muscles
A. Buccinator and superior constrictor
B. Medial and lateral pterygoid
C. Medial pterygoid and superior instructor
D. Temporal and lateral pterygoid
E. Temporal and medial pterygoid
Ans is A

118. A patient attends with pain of four days duration in a carious upper molar tooth.
The pain is constant and is not relieved by paracetamol. Sleep has been disturbed
by the pain. The tooth is tender to percussion and gives a positive response to Ethyl
Chloride. What is the most likely diagnosis? .,
A-Pericoronitis
B-Apical periodontitis
C-Marginal periodontitis
D-Reversible pulpitis
E-Irreversible pulpitis
Ans is E

119.Lady in 40s got TMJpain for about few months, she got divorced recently what is
the first line treatment

BY DR.ABDULRAHMAN ALMUALM
MJDF MCQS WITH ANSWERS

A.amitrptyline
B.physiotherapy
C.occlusal splint
Ans B ( first line of treatment.)

120.In a class II div 2 malocclusion, which bridge design would be contraindicated


for a missing lateral upper incisor?
A.Cantilever bridge
B.Maryland bridge
Ans is B

121. Q4 A 30 year old man with unknown allergy to latex goes into anaphylactic
shock whilst being treated in the dental surgery.

Which drug and route of administration is of most benefit in this situation?

A. Hydrocortisone - orally
B. Chlorphenamine – intramuscularly
C. Chlorphenamine - orally
D. Epinephrine – intravenously
E. Epinephrine – intramuscularly
Ans is e
122. days after preparation and filling of a shallow class I
amalgam cavity the patient complains of pain on biting. You
would:
A.Perform vitality test.
B.Replace filling.
C.Check for premature contacts.
D.Remove all occlusal contacts from this filling
E.Tell the patient to wait 2-4 weeks, the pain will go away.
Ans is c

123. A diabetic patient complains of pain on a root canal treated


tooth. Root canal is done 2 years ago. On examination the
restoration on the tooth seems leaking. Reasons for pain and
failure?
A) No proper obturation
B) Medical condition
C) Lack of coronal seal
D) Inadequate biomechanical preparation
Ans is C

124.A 90-year old gentleman presented to clinic who is


edentulous and has dentures upper and lower full 15 years old. Denture bit
uncomfortable, tooth structures little bit worn out, freeway space 2-4mm, polished
surfaces satisfactory and occlusal wear is minimal. How would you proceed?
A) Copy dentures
B) Hard reline
C) Soft reline
D) Construction of new dentures

BY DR.ABDULRAHMAN ALMUALM
MJDF MCQS WITH ANSWERS

Ans is A

125 .Secondary impressions for edentulous patient close fit tray


with hyper gap reflex. Which material you use for taking an
impression?
A) Alginate
B) Silicone rubber based material
C) Plaster of Paris
D) Zinc oxide eugenol
Ans is B / some selected D ..due to close fit tray

126.The optimum cavosurface angle for occlusal amalgam surface is


A. 45-60°
B. 70-85°
C. 45-80°
D. 90-100°
E. 130-150°
Ans is 90-100 D

127.For lower premolars, the purpose of inclining the handpiece lingually is to ?

A. Avoid buccal pulp horn


B. Avoid lingual pulp horn
C. Remove unsupported enamel
D. Conserve lingual dentin
Ans is A
128The retention pin in an amalgam restoration should be placed
A. Parallel to the outer wall
B. Parallel to the long axis of tooth
C. Near the marginal ridge
ANs is A ?

129 the pulp horn most likely to be exposed in the preparation of large cavity in
permanent molar tooth is
A. Mesio-lingual in upper first molar
B. Mesio-buccal in upper first molar
C. Disto-buccal in lower first molar
D. Mesio-lingual in lower first molar
E. Mesio-buccal in lower first molar
Ans is B

130

BY DR.ABDULRAHMAN ALMUALM
MJDF MCQS WITH ANSWERS

When preparing class III for composite restoration; which situation acid Etching
should be placed

A. Always should be performed to minimise marginal leakage


B. Should not be performed because it might damage the adjacent tooth
C. When extra retention is required
D. Only in situations where cavity is shallow to avoid pulp irritation
Ans is A

131Which pin system has proven to be the most retentive

A. Self tapping threaded pin


B. Friction peak pin
C. Cemented pin
Ans is A

132when do you finish campsite resin restorations

A. Immediately after curing


B. After 24 hours
C. A week after placement
Ans is A

133 .The technique of placing Gutta-Percha cones against the root canal walls
providing space for additional Gutta Percha is termed

A. Lateral Condensation
B. One major Gutta Percha point
C. Laterally above condensed
Ans is A

134The best way of getting good retention in full veneer crown is by

A. Tapering
B. Long path of insertion
C. Pin retention
Ans is B (long path of insertion means more length of abutment

135
What is the most sensible approach to treatment?A patient reports that his post
crown has fallen out. This crown had been present for many years. You note that
there appears to be a hairline vertical fracture of the root. The tooth is symptomless :
A. Replace the post crown using a resin-reinforced glass ionomer material
B. Replace the post crown using a polycarboxylate cement
C.Replace the post crown using a dentine bonding agent and a resin-reinforced
glass ionomer material
D.Replace the post crown using a resin composite luting agent
E.Arrange to extract the tooth

BY DR.ABDULRAHMAN ALMUALM
MJDF MCQS WITH ANSWERS

Ans is E

136 While attempting to cement a PFM crown, the crown has accidently
slipped into the patient’s throat. The patient has suddenly grasped his
throat with both hands and he is unable to speak. How would you manage
this situation?
a. Use McGill forceps to extract the crown.
b. Encourage the patient to cough.
c. Back slaps only.
d. Abdominal thrusts only.
e. Alternating back slaps with abdominal thrusts.
Ans is E (medical emergency : helmich manuever

137 An apical radiolucency (2mm) is noticed as an incidental radiographic finding


associated with the apex of the mesiobuccal root of the lower right first molar. The
tooth has been root filled but is 2mm short of the radiographic apex. There are no
other clinical or radiographic findings and the patient is fit and well. What is the most
appropriate course of action?
A. Extract the tooth.
B. Redo the root filling
C. Perform periapical surgery.
D. Advise the patient of the situation and monitor clinically and radiographically
E. Prescribe antibiotics then review

Ans is D

138 Conventional radiographs provide a three dimensional image of the pupal


anatomy.

A.False
B.True
Ans is A

139 Which of the following materials has the greatest resistance to wear in the oral
environment?

A. Compomers
B. Conventional glass ionomer cements
C. Resin composites
D. Resin modified glass ionomer cements
E. All have equal water resistance
Ans is C

140 A patient attends your clinic complaining of the appearance of his discolored
root filled upper right lateral incisor.which one of the following options is the most
appropriate?

A. scale and polish


B. all ceramic crown
C. Veneer preparation with dentine

BY DR.ABDULRAHMAN ALMUALM
MJDF MCQS WITH ANSWERS

D. Extraction and conventional bridge


E. Internal and external bleaching
Ans is E

141 Stiffness refers to

a. Resistance to elastic deformation.


b. Degree of elastic deformation.
c. Expandability on heating.
d. Shrinkage on cooling.
e. Expansion on cooling
ans is A

142 The minimal labial tooth reduction for satisfactory aesthetics with porcelain fused
to metal crown is,
A. 1mm
B. The full thickness of enamel
C. 1.5 mm
D. 2.5mm
E. One third of the dentine thickness
Ans is C

143 Which is the minimum distance between the major connector


on a maxillary RPD and the gingival margins?
A) 1mm
B ) 6 mm
C) 12 mm
D) 4 mm
Ans is B

144 Which should be covered by the denture base of a mandibular distal extension
RPD?
A) All undercuts areas and engage them for retention
B) Hammular notch
C) retromolar pad
D) pterygomandibular raphe
Ans is C????

.145 patient presents with a radiolucency in the posterior maxilla region. You suspect
an ameloblastoma. Which of the following feature
might support your suspicion.
a) The patient is 17 years old
b) The radiolucency is multilocular
c) There is evidence of calcification within the radiolucency
d) There is no evidence of tooth resorption
e) There is parasthesia in the region of the distribution of the infraorbital nerve

BY DR.ABDULRAHMAN ALMUALM
MJDF MCQS WITH ANSWERS

ans is B

146) Which of the following does not affect the elasticity of retentive clasp?
a) Length of the arm1
b) The cross section of the shape
C) The material used
D) The undercut area
Ans is D

147A pt on examination was found to have swollen gingiva around a crown that had
been present for
several years.The papillae were particularly enlarged, The most important feature of
the crown
responsible for this is
a) Material of the crown
b) The occlusion
c) Proximal contour
d) labial contour
e) Surface finish
ans is C

148 When do you do not extract impacted 8's


a) Extraction of 8's recommended by orthodontist
B) Extraction of 8's recommended in patient undergoing renal transplant
immunosupression
C) 47 years old with moderate periodontittis
D) Second permanent molar undergoing internal resorption due to third molar
Ans is c

149 Pt got metallic taste after oral meditation, the reason is


a) Oral dysthsia
b) Burning mouth syndrome .
C) Oral dysguesia
ans is c

150 Randomised controlled trials enable us to establish cause and effect.


The main reason for using this study design is to:

A Ensure that making a Type II error is minimised.


B Ensure that test and control groups are similar.
C Ensure that test and control groups are of equal size.
D Help recruitment of patients into the trial.
E Provide sufficient power for the study.
Ans is E

BY DR.ABDULRAHMAN ALMUALM
MJDF MCQS WITH ANSWERS

151) All the following drugs should be present in emergency drug box in dental office
except
a) Adrenalin
b) Midazolam
c) Corticosteroids
d) Salbutamol
e) Aspirin
ans is C

152What is not true for diazepam –


a.patient complain headache ,
B.profound amnesic action no side effects,
c. active metabolites sedation 8hr post op,
d. safe in children
ans is B

153child consumed 5mg/kg fluoride what is the appropriate emergent management :


A.give salty drink
B.give sugar drink
C.give milk
Ans is C / exact management is give patient Ca+

154. Reasonof failure canine eruption without early loss of deciduous teeth?
1. Supernumerary
2. Ectopic
3. Crypt
4. Primary failure
5. supplementary
Ans is 3

155 In advanced periodontitis with marked mobility, teeth may be splinted:


A.To improve comfort for the patient
B.splinting helps in transmitting the force to the adjucent teeth to reduce the load on
the involved teeth
Ans is A (ref oxford also.B is correct )

.156 30 yr-old patient attends complaining of pain from the lower left quadrant.
Clinical
examination reveals a dentition with generally good oral hygiene. There is no
significant periodontal pocketing other:.than an isolated defect in the region of the
furcation of lower left first molar which is non-mobile. The gingival tissue in thiS area
appears erythematous and slightly hyperplastic with a purulent exudate on probing.
From the list below, which is the most appropriate next step?
A-Obtain a radiograph

BY DR.ABDULRAHMAN ALMUALM
MJDF MCQS WITH ANSWERS

B-Biopsy the gingival tissue


C-Remove the restoration
D-Vitality testing
E-Prescribe antibiotics
Ans is D
Vitality testing. We want to reach a diagnosis as to whether the lesion is endodontic
or periodontal. The radiograph may show a radioloucency in the furcal area in both
cases (Primary endo lesions can cause furcal or periapical radiolucency. Furcation
involvement also causes furcal radiolucency) but will still not give us a definite
diagnosis. We initially need to know whether the tooth is vital or not, then carry on
with radiographic investigation.

157 A 58 year old male presents at your surgery complaining of a sharp pain of no
more than 30 minutes duration arising from his upper left molar region. The pain is
brought on by cold stimuli but persists after-the stimulus is removed. It does not
seem to occur spontaneously. He has tried taking paracetemol and this does
temporarily stop the pain from recurring. The upper left 6' reacted to a lower current
on electronic pulp testing than the upper right 6, upper left ~ or the lower left molars.
What is the most likely cause of the patient's pain? -
A-Acute/reversible pulpitis
B-Dentine sensitivity
C-Chronic/irreversible pulpitis
D-Periapical periodontitis
E-Trigeminal neuralgia
Ans is A

158 The normal un stimulated salivary flow rate is


A.0.02 ml/min
B.0.2 ml/min
C. 2 ml/min
Ans is B

159A dental practitioner can be


A-Radiology adviser+physicist+operator
B-Supervisor + adviser+physicist
C-practitioner+refer+operator
Ans is C

160Little tooth preparation required to be


Can be made using the lost wax technique
Angle of tooth preparation should be almost vertical with sharp line angles
Has potentially the strongest properties of all ceramic crowns allow transillumination
which is the one with those characteristics:
A.Zirconion crowns
B.Dentine bonded crowns
C.Leucite reinforced crowns
D.Pressed ceramic crowns inceram
Ans is either D

161Stiffness of material is measured by

BY DR.ABDULRAHMAN ALMUALM
MJDF MCQS WITH ANSWERS

A-Proportional unit
B-Modulus of elasticity
C-Stress/ stra i n
D-Ultimate tensile strength
Ans is B

162Which of the following is true of chlorhexidine?


• A. Adhere to tooth pellicle
• B. Short duration of action
• C. Inhibits bacterial cell wall biosynthesis
• D. Ineffective for candida
• E. Mainly effective for gram negative bacteria
Ans is C

163When preparing the Apical Zone, the use of the files sequentially from apex
to backwards (lower the size of instrument) - what is the best distance to
achieve good apical area preparation a 0.5mm
b.1.0mm
C. l.5mm
d 2.5mm
e. 3.5mm
Ans is C ?

164. Best material to use in the root caries and buccal caries in elderly patient.
AHybrid Composite
bAmalgam
C. Silicate cement
D. Zinc phosphate
E .RMGIC
Ans E
165 Six-year-old child with fluoridated water of 0.6ppm , what is the fluoride
supplement dose given

A.0.25 mg
B. 0.5 mg
C. 1 mg
Ans is B

166. A 90-year old gentleman


presented to clinic who is
edentulous and has
dentures upper and lower full
15 years old. Denture bit
uncomfortable, tooth
structures little bit worn out, freeway space 2-4mm, polished surfaces
satisfactory and occlusal wear minimal. How would you proceed?

A.Copy dentures

BY DR.ABDULRAHMAN ALMUALM
MJDF MCQS WITH ANSWERS

B.Hard reline
C.Soft reline Construction of new dentures
Ans is A = good polished surface, minimum occlusal wear

167. What is the least important factor to reduce radiation in dental radiographs?
A-Speed of film
B-Collimation
C-Filtration
D-Cone shape and length
E-Use of lead apron
Ans is E

168. A suddenly swollen upper lip that lasts for 48 hours or more is most likely
A.Haemangioma
B.Agioneurotic oedema
C.Mucocele
D.Cyst
Ans is B .

169. Medical condition of pt who reserved a seat in a dinner meating for his dead
wife
A-Mania
B-Anxiety
C-Depression
D-Schizophrenia
Ans is D

170) A Gracey curette is characterized by


A-The blade and the shank form a 902angle
B-Can be used on both sides
C-Can be used on any tooth surface
D-It is specific for each surface ofthe tooth
Ans is D

171. which of these disease that caused by positive lacto bacillus


A-Syphilis
B-TB
C-Angular cheilitis
D-Dental Caries
Ans is D

172. what type of cells that proliferate in later stage of her life?
A.Odontoblast
B.cemtoblast
C. rest of malazess
E.undifferentiated mesynchymal cells
Ans is E

BY DR.ABDULRAHMAN ALMUALM
MJDF MCQS WITH ANSWERS

173 Wrought metal is to be,


A-Marble
B-Quenched
C-Has undergone cold treatment during processing
Ans is c

174In minor oral surgery, what is TRUE in regard to antibiotics:


A-Amoxil satisfactorily covers the dental spectrum
B-Metronidazole and Amoxil have the same penetrating power
C-It is evident that it will reduce post operative swelling
D.There is convincing evidence that Prophylactic prescription of antibiotics will
reduce
postoperative infections
E.Most oral infections get anaerobic after 2 to 3 days
Ans is A

175What is true regarding pregnancy gingivitis?


A.lt is due to increased gingival microcirculation
B.Elevated estrogen and gestagen levels are directly responsible .;
C.Hormonal changes cause the growth of anaerobic bacteria (Prevotella intermedia
Ans is C

176.A patient comes with a firm, painless swelling of lower lobe of parotid which has
grown progressively for the past year. He complains of paresthesia for the past 2
weeks.
This is most likely to be:
A-Pleomorphic adenoma
B-Carcinoma of the parotid
C-Lymphoma of parotid
Ans is B ?

177. Swallowing will aid in the diagnosis of:


A-Branchial cyst
B-Thyroglossal duct cyst
C-Ranula
D-Retention cyst
E. Glossothyroid cyst | Ans is B
178. Characteristic of Squamous Cell Carcinoma of the tongue
A-more in white skinned people
B-more in alcohol drinking smoking males
C-associated with Plummer-Wilson-Syndrome
Ans is B

179. What is untrue about diabetes?


A.Hypoglycaemia is more common than hyperglycaemia
B.lnsulin-dependend patients are of more concern than non insulin-dependend
c. Adrenalin causes a decrease in the blood glucose level (adrenaline raises sugar)
Ans is C

BY DR.ABDULRAHMAN ALMUALM
MJDF MCQS WITH ANSWERS

180. Normal prothrombin time and elevated partial thromboblastin time is seen in
A.Factor VIII deficiancy
B.Thrombocytic pupura
C.Leukemia
D.Von Willebrand disease
Ans is A

181. Which is not true in sickle cell anaemia?


A.Deformed cells with less oxygen transport capacity
B.Higher infarction risk
C.Have wide bone marrow spaces with narrow trabeculae in the alveolar bone of
oral
cavity
D.Resistant to malaria parasites
E.More common in mediterranean people
Ans is E

182. A 58 year old male has been treated with radiation for carcinoma of tongue. The
patient complains of pain associated with
poor dentition. The dental management would be:
A-Immediate extraction of any poor teeth under local anaesthetic with antibiotic
coverage
B-Segmental dental clearance and closure to eliminate problems
C-No dental treatment may be due to neuronic of neoplasms
D. Clearance of poor dentition followed by hyperbaric oxygen treatment plus a
primary
closure of wounds under antibiotic
coverage
E. No extraction as radionecrosis is an important sequelae
Ans is D?

183. What effect has placing a sealant over pits and fissures on the progression of
caries?
A.Decreased new caries
B.lncreased new caries
C.Progression of existing caries
D. No effect on existing caries
Ans is A

184. 1 n a radiograph the roots of the upper teeth are too short because of:

BY DR.ABDULRAHMAN ALMUALM
MJDF MCQS WITH ANSWERS

Inadequate horizontal
angulation
A-Too high vertical
angulation
B-Too small vertical
angulation
C-Parallel
Ans is A

185. What is CORRECT in regard to the periodontal surface area in maxillary teeth:
A.central incisor> first premolar> second
premolar
B. Canine> first premolar> central incisor
C.Canine> lateral incisor> second premolar
D. Canine> central incisor> first premola
Ans is B

186. Which of the following is a feature of


Streptococcus mutans?
A.lt does not require a special environment to
grow
B.lt can be easily transported from one part of
the oral cavity to another
C.lt has the ability to re structure carbohydrates

Ans is C

187. patient attends with pain of four days duration in a carious upper molar tooth.
The
pain is constant and is not relieved by paracetemol. Sleep has been disturbed by the
pain.
The tooth is tender to percussion and gives a positive response to Ethyl Chloride.
What is
the most likely diagnosis?
A-Pericoronitis
B-Apical periodontitis
C-Marginal periodontitis
D-Reversible pulpitis
Ans is B

BY DR.ABDULRAHMAN ALMUALM
MJDF MCQS WITH ANSWERS

188. A 14 year old patient attends with a decayed and hypoplastic LL7. A radiograph
shows
the presence of an unerupted LL8 and the LL6 is sound. What would be the most
appropriate long-term treatment for this tooth?
A-Amalgam restoration
B-Antibiotics
D- Root canal therapy (best is to extract)
E-Sedative dressing
Ans is D since its hypoplastic decayed for longterm treatment best is to extract and
allow 8 to enter to the arch If applicable

189. EDTA (ethylene diamine tetra-acetate) has useful roles in certain situations in
clinical
dentistry. When would you use EDTA?
A-As a pulp capping agent
B-As a root canal irrigant
C-As a mouthwash
D-As a root end filling material
E-As a dentine bonding agent
Ans is B

190. You suspect that there is occlusal caries in the lower right first permanent molar
of a
10 year old child. You wish to confirm your suspicions. Which diagnostic test is most
commonly used in this situation?
A-Bitewing radiography
B-Electro-conductive caries monitors
C-Fibro-optic transillumination
D-Panoramic radiography
E-Visual examination of a dried tooth.
Ans is E// If proximal caries C or A

191. Which one of this restorative method will be LEAST compromised by a core,
A-Amalgam
B-Composite
C-GIC

BY DR.ABDULRAHMAN ALMUALM
MJDF MCQS WITH ANSWERS

D-Cast gold
Ans is D

192.child comes to your clinic with a fractured crown 3mm supra gingival how would
you treat this case
A. Formecorsol pulpotomy
b.calcuim hydroxide pulpotomy
C.pulpectomy
D. Direct pulp capping
E.indirect pulp capping
Ans is B
So its trauma and we can go with pulpotomy (child and trauma )
Formecresol isn't recommended due its carcinogenic concern .
The Caoh is good choice here
But I will give a recap for all materials used in pulpotomy
.ferric sulphate is best recommended by most
Formercrseol still used but recent concerns of mutagencity due to formalin
Caoh is good but time consuming but ferric sulphate is better
MTA show great success rate but expensive
Divitalizing paste (formaldehyde also containing and concern for use )

193.for caries to be able to spread through dentine,


A.enamel must contain glycoproteins
B. DIET MUST contain simple carbohydrate
C.diet must contain polysaccharides
D. Pulp must contain complement
Ans is B

194. patient in your dental chair shows chest pain, weak pulse and dysponea, what
is
your initial management,
A-Give a nitro-glycerine tablet and keep the patient up seated'
B-Put the patient in supine position

BY DR.ABDULRAHMAN ALMUALM
MJDF MCQS WITH ANSWERS

C-Do nothing and wait until the symptoms go


Ans is A

195. Developer contaminated with other chemical and was not mixed pro perlv. What
is
the effect on the X-ray film?
A-Too dark film
B-Light film
C-Foggy
Ans is C

196. Branchial Cleft cyst is located,


A-Medial to the neck
B-On anterior border of the Sternocleidomastoid muscle
C-Shows when
swallowing
Ans Is B

Source : new england


journal for pic

197. the most common way of oral carcinomas to


other tissues is/are
A-Lymphatic
B-Invasion and blood
C-Lymphatic and invasion
D-Blood and lymphatic
Ans is D
Metastasis of oral cancer is a complex process involving detachment of cells from
the tumor tissue, regulation of cell motility and invasion, proliferation and evasion
through the lymphatic system or blood vessels. ... The major determinant of the
prognosis of oral carcinoma is the risk of cervicalmetastasis.

198. The main purpose of periodontal treatment is,


A-Elimination of plaque and calculus
B-Elimination of periodontal pockets
C-Reformation of all the periodontal ligaments
D-The elimination of all occlusal trauma
Ans is A

199. The thermal and electric pulp tests will,

BY DR.ABDULRAHMAN ALMUALM
MJDF MCQS WITH ANSWERS

A-Give an accurate indications of the pulp status


B-The patient's response will be either pain or no pain
C-The patient can differentiate between cold or hot stimuli

Ans is B

200. Immediately after the extraction of lower molar the patient complains of post
operation bleeding and pain, how would
manage this,
A-Prescribe analgesics and ask the patient to follow a strict oral hygiene
B-Administer 5% Marcaine Local Anastatic, prescribe analgesics and pack the
socket
with alvogyl
C-Administer 5% Marcaine Local Anastatic, suture the socket and prescribe
analgesics
D-Suture and give pressure packs
Ans is C

201. 381. The removable partial denture requires relining what is would be the most
appropriate action,
A-take a new impression by asking the patient to occlude on it
B-Provide equal space between denture and gingival tissues.
C-Make sure the framework and retainers are seated in place before taking
impression
Ans is C

202.In regards to dentine strength, which is the right sequence,


A-Affected dentine> Sound dentine> Infected dentine
B-Sound dentine> Affected dentine> Infected dentine
Ans is B

203. After obturation and on X-ray you notice the obturation materials are lmm
beyond
apex. What is your first management?
A-Refill the canal
B-Pull the GP cone about lmm out and take a new X-ray
C-Leave it as it
Ans is C or B depend on time that you discovered there is overextention ! If
immediately after placing it you can pull it but if after you did coronal.restoration you
will wait ,,review it periodically to check any changes in preapical area

204. 2.21mg NaF contains,


A. lmg fluoride
B.2 mg
C. 0.5 mg
D- 3 mg
Ans is A

205.. Four years kid shows at your clinic with open bite as a result of thumb sucking,
you

BY DR.ABDULRAHMAN ALMUALM
MJDF MCQS WITH ANSWERS

notice a delayed speech ability what would


be your first management,
A-Refer to a speech therapist
B-Apply a removable habit inhibitor denture
C-Apply a removable habit inhibitor denture and educate the parents about it so the
kid
will not be taking it off so often
Ans is A

206. An 80 year old male presents at your surgery complaining of a sharp stabbing
pain of no
more than 2-3 minutes duration arising from his upper left pre-molar region. The pain
can be brought on by cold stimuli but also occurs spontaneously and has been
sufficiently severe to wake the patient from sleep. He has tried taking paracetemol
but this has been of no benefit. On examination the patient has a heavily restored
upper left 4, which is vital to electrical pulp testing and shows no radiographic
evidence of caries. Blowing cold air onto the tooth produces the pain but the pain
also occurs spontaneously when you are examining the patient. There is no
evidence of a crack or fracture in the tooth itself.
What is the most likely cause of the patient's pain?
A-Acute/reversible pulpitis
B-Dentine sensitivity
C-Chronic/irreversible pulpitis
D-Atypical odontalgia
E-Trigeminal neuralgia
Ans is C

207. A patient complains of a lower incisor which has been mobile for several
months. The
radiograph indicates a normal level of bony support although the periodontal space
has widened. The apical bone appears normal. The tooth is tender to pressure.
Which of the following tests and or examinations would be most likely to provide a
diagnosis?
A-Masticatory muscle palpation
B-Electric pulp test
C-Occlusal examination
D-Ethyl chloride test
E-Hot gutta percha application
Ans is C

208. 9 years old boy come to your clinic with cl 1 occlusion and normal overjet
grossly carious upper left 6 which is
not suitable for restoration. The upper left 7 is very near to eruption. Upper right 6,
lower right 6
and lower left 6 are sound and
fissure sealed. The oral hygiene is good. What is the most appropriate extraction
pattern for this
patient?
A-Extract all four 1st permanent molars
B-Extract upper left 6 and lower left 6

BY DR.ABDULRAHMAN ALMUALM
MJDF MCQS WITH ANSWERS

C-Extract upper left 6 and lower right 6


D-Extract upper right 6 and upper left 6
E-Extract upper left 6 only
Ans is E
209. Dentists are encouraged to carry out a thorough soft tissue examination and be
vigilant for
any lesions that might possibly be malignant or have malignant potential. Currently it
is not
recommended that dentists use a mucosal staining method for screening the general
population
for oral cancer. What is the most likely reason for this advice?
A-Unnecessary intervention where there is a false positive test result
B-False reassurance where there is a false negative test result
C-Low uptake of screening by those with low risk
D-Low uptake of screening by those with high risk
E-Negative result reinforcing existing bad habits
Ans is A

210. All the films which have been manually processed by an unsupervised new
trainee dental
nurse in your practice are extremely
dark. Which of the following errors could lead to a dark film?
Insufficient exposure
A-Films have been left in the developer for too long a period of time
B-Films have been in the fixer for too long a period of time
C-The concentration of the developer is too dilute
D-The concentration of the fixer is too dilute
Ans is A

211. A new patient attends your practice. At the initial examination, you carry out a
Basic
Periodontal Examination (BPE).
What does a BPE score of 1 signify?
A-Periodontal health
B-Probing depths greater than 3.5mm
C-Presence of overhanging restorations
D-Presence of supragingival calculus
E-Presence of bleeding on probing
Ans is E

BY DR.ABDULRAHMAN ALMUALM
MJDF MCQS WITH ANSWERS

212. Your colleague is having problems as


every panoramic film that he produces shows
extremely wide anterior teeth which are
also blurred. The film also invariably has the
images of the condyles cut off from the sides
of the
film. The most likely cause of this
A-The patient is incorrectly positioned too far
forward relative to the image layer
B-The patient is incorrectly positioned with
the chin too low
C-The patient is incorrectly positioned too far
back relative to the image layer
D- The patient is in a slumped position in the machine
E-The patients is incorrectly positioned with the head tilted in the machine
Ans is c

Tables next page for common errors in panorama

BY DR.ABDULRAHMAN ALMUALM
MJDF MCQS WITH ANSWERS

213.court asked you to testify against another dentist in this case you are :
A. Plaintiff
B. Witness
C.expert witness
D.defendant
E.hostile witness
Ans is c

214. Lignocaine (2%) is widely used in dental procedures. It is most often used in
combination
with epinephrine (1 in 80,000).
In which one of the following patients is the use of epinephrine containing local
analgesia
potentially hazardous?
A-Patient with severe hypertension
B-Patient on tricyclic antidepressants
C-Patient who is an alcoholic
D-Patient on monoamine oxidase inhibitors (MAOls)
E-Patient with Grand Mal epilepsy

BY DR.ABDULRAHMAN ALMUALM
MJDF MCQS WITH ANSWERS

Ans is B
Epinephrine (Primatene, Adrenalin, Ana-Kit, EpiPen, Marcaine) should not be used
with tricyclic antidepressants, since together they can cause severe high blood
pressure.

215.child mother heard that some foods may lead to caries but she isn’t sure what
should she avoid ,from the list below what should she avoid the most:
A.bread
B.cheese
C.ground almonds
D. Fruit juices
E. Vegetable and fruits
Ans is D (check pinkbook)

216. 29 year old man has a prosthetic (mechanical) aortic valve. He had bacterial
endocarditis
five years ago. He now requires
removal of his upper and lower right third molars. Which antibiotic prophylaxis is
recommended
prior to the procedure?
A-Amoxicillin and clindamycin
B-Metronidazole and cephalexin
C-Amoxicillin and gentamycin
D-Amoxycillin and vancomycin
E-Erythromycin and vancomycin
Ans is A ( Amoxicillin 3g for non allergic -clindamycin 600mg for allergic )

217. Pt comes with the career and they have an verbal argue and you have
noticed that caeer were aggressive towards client, what would you do;
A) ignore
B) report to the police
C) ask them politely to book next appointmen
D) report to local safeguarding services
E) report to care home
Ans is D ?
218. patient presents with a history of pain in the right pre-auricular region. There is
an
intermittent click during opening; when
the click is not present the patient can open to a normal range. On examination the
masticatory
muscles, including the lateral
pterygoid, on the RHS side are tender. What is the most likely diagnosis?

A-Myofascial pain
B-Disc Displacement with Reduction
C-Osteoarthrosis

BY DR.ABDULRAHMAN ALMUALM
MJDF MCQS WITH ANSWERS

D-Disc Displacement without Reduction


e. bruxism
Ans is B

219. An adult patient complains of her prominent upper front teeth and receding chin.
She says
she did not get 'braces' when she
was younger because the family had to move area a lot. Her oral care and health is
good, and
she has a 9 mm overjet.
Where is the most suitable place to refer her? "
A-A GDP friend who has a special interest in orthodontics and has been trained to
use fixed
appliances.
B-A specialist practitioner who uses a lot of functional
appliances.
C-A private specialist practitioner, because she's too old to
get NHS treatment now.
D-A hospital consultant, as it is likely she will need surgery
now to correct her problem
E-The nearest dental hospital, although it is 70 miles away.
Ans is D

220. 25 year old male attends for the first time complaining of sensitivity of a
number of teeth. On examination, the occlusal surfaces of all the teeth are
worn with obvious wear facets on the canines and premolars. Posterior
amalgam restorations are proud of the surrounding tooth. What would be the
first stage management?

A ● Take impressions for study models


B● Prescribe fluoride mouth rinse
C● Replace the amalgam restorations
D● Dietary analysis
E● Placement of resin sealant to sensitive teeth
Ans is D

221. 60 year old patient attends your surgery complaining of a sore mouth. He
has Type II diabetes well controlled by diet and metformin. On examination
white patches which cannot be removed are present on his buccal mucosa:
What is the most likely diagnosis?
A ● Frictional keratosis
B ● Leukoplakia
C● Lichen Planus
D● White sponge naevus

BY DR.ABDULRAHMAN ALMUALM
MJDF MCQS WITH ANSWERS

E. Candidosis
Ans is C

222. Cigarette smoking is considered to be the most important factor next to


microbial plaque in
periodontal disease progression. Which of the following is the most important factor
in the
disease progression in smokers?
A-Smokers have drier mouths than non-smokers .,
B-Smokers have poorer oral hygiene than non-smokers.
C-Nicotine will impair the chemotactic and phagocytic properties of PMNs.
D-The gingival blood flow is reduced in smokers.
E-Smokers alter the oral environment encouraging the growth of anaerobic bacteria
Ans is C

223. Which of the following is not a branch of the


facial nerve ?

A. Temporal nerve .
B.Orbital nerve
C.Zygomatic nerve
D. Buccal nerve
E.Cervical nerve
Ans is B

224. While you were removing amalgamm


restoration of lower right first
molar to replace with composite, a piece of Amalgam dislodges and disappear. If the
patient was sent to chest X-ray, which place is most likely to find
that piece?
1. Right lung
2. Left lung
3. Right bronchi

BY DR.ABDULRAHMAN ALMUALM
MJDF MCQS WITH ANSWERS

4. Left bronchi
5. alveoli
Ans is right bronchi

225. Any patient receiving treatment under IV sedation must have their blood
pressure recorded
as part of their assessment.
What is the maximum blood pressure that is generally regarded as being compatible
with safe
sedation in general dental practice?
A.160/95
B.140/95
C-160/90
D-170/100
E.120/80
Ans is A

226. A 13 year old boy with Down's syndrome attends for the first time. He is
cooperative and
has no relevant medical history. He
is caries free, apart from two small occlusal cavities in his lower second primary
molars. His
mother requests that these teeth are
restored.
Which of the following is the most appropriate management?
A-Arranging to extract these teeth at a future visit.
B-Arranging to place two simple restorations at a future visit.
C-Reassurance that these teeth can be left until they exfoliate naturally, with no
treatment being
necessary.
D-Recommending use of fluoride mouthwash.
E-Take OPG to check on unerupted teeth
Ans is E

227. You are designing a partial denture for a patient with several missing teeth in
the maxilla.
The reason for surveying the model prior to designing the denture is to:
A-Measure and mark out hard and soft tissue undercuts on the casts
B-Relate the intended position of the inter-papillary plane of the patient to the casts
C-Establish the position of the post dam
D-Relate the maxillary and mandibular casts
E-Aid setting up the prosthetic teeth prior to trial insertion
Ans is A

228. A child of 5 years attends with pain from a grossly decayed lower right D which
has a
discharging sinus. He is a hemophiliac.
Which of the following is the most appropriate treatment to relieve his pain?
A-Antibiotics

BY DR.ABDULRAHMAN ALMUALM
MJDF MCQS WITH ANSWERS

B-Extraction
C-Fluoride application
D-Non-vital pulpotomy
E-Vital pulpotomy
Ans is D

229. A 23 year old patient attends complaining of pain in an upper right molar and is
keen to
keep the tooth. The pain is typically
sharp in nature, is triggered by cold and persists after removal of the cold stimulus.
The tooth is
not tender to percussion; a
radiograph of the upper right first molar shows a large radiolucency extending to the
pulp horn
but no peri-radicular changes.
What treatment is most likely needed in this case?
A-Oral hygiene instruction and fluoride application
B-Excavation of caries and placement of a permanent restoration
C-Root-canal treatment
D-Indirect pulp cap and restoration
E-Direct pulp cap and restoration
Ans is C

230. An obese 40 year old who takes metformin is seen at the end of a morning
clinic. After
administration of local analgesia in thesupine position the patient complains
of feeling unwell. She is pale and sweating and is
confused. No pulse was detectable. She
was given oxygen and maintained in the supine with no improvement in
condition. What would
be the most appropriate drug to administer next?
A-Glucagon
B-Glucose
C-Adrenaline
D-Hydrocortisone
E-Chlorphenamine
Ans is A.patient is unconscious

231. There has been much debate regarding the effectiveness of fluoride in water for
preventing
tooth decay. A systematic review
of the evidence drew conclusions as to the reductions in decay which can be
expected.
Fluoridation of public water supplies:

BY DR.ABDULRAHMAN ALMUALM
MJDF MCQS WITH ANSWERS

A-Reduces tooth decay by 10%.


B-Reduces tooth decay by 20%.
C-. Reduces tooth decay by 30%.
D-Reduces tooth decay by 40%.
E-Reduces tooth decay by 50%
Ans is E

232. The biological process by which the architecture and function of the lost tissue
is completely
restored".
In Periodontology this is a definition of which of the following
A-New Attachment
B-Regeneration
C- Repair
D-Primary Healing
E-Reattachment
Ans is B

233. Randomised controlled trials enable us to establish cause and effect.


The main reason for using this study design is to:
A-Ensure that making a Type II error is minimised.
B-Ensure that test and control groups are similar.
C-Ensure that test and control groups are of equal size.
D-Help recruitment of patients into the trial.
E-Provide sufficient power for the study
Ans is E

234. An apical radiolucency (2mm) is noticed as an incidental radiographic finding


associated
with the apex of the mesiobuccal root of the lower right first molar. The tooth has
been root filled
but is 2mm short of the radiographic apex. There are no other clinical or radiographic
findings
and the patient is fit and well. What is the most appropriate course of action?
A.Extract the tooth.
b.Redo the root filling
c.Perform periapical surgery.
d.Advise the patient of the situation and monitor clinically and radiographically
e.Prescribe antibiotics then review
ans is D

235 A 23 year old male presents to your surgery. He lost his upper lateral incisors
some 10

BY DR.ABDULRAHMAN ALMUALM
MJDF MCQS WITH ANSWERS

years ago in a swimming pool accident. Since then he has been wearing a 'spoon'
denture
which he now feels in aesthetically unacceptable. He has sought an opinion on
dental implants but has been told that he would need bone grafting for this to be
successful and
he is not prepared to undergo this. His dentition is excellent with no restorations and
a Class I
occlusion. He wants some advice on what the best treatment might be.
Which option would you put first on your list of possibilities?
● Two fixed - fixed resin bonded bridges using the central and canine teeth
● Two cantilever resin bonded bridges from canines.
● Two conventional fixed - fixed bridges from the canine
● Conventional cantilever bridges from the canines
● Cobalt chrome partial denture
Ans is B

236. The principle clinical sign of active bruxism is:


● Head and / or neck pain
● Excessive tooth wear
● Temporomandibular joint clicking
● Sensitive teeth
● Cheek ridging and tongue scalloping
Ans is E

237. The patient wants removable partial denture to restore his missing teethfrom
second premolar
in both sideof lower mandible. What would be the best retention
for this case?
1. Mesial rest, I bar
2. Distal rest, I bar
3. Mesial & distal rest, Ibar
4. Circumferential clasp
5. C clasp withl bar.
Ans is A

238
You want to treat Pagetwith
medication. Which of the following
is theleast possibleto develop
MRONJ?
1. Denosumab
2. Risedronate
3. Calcitonin
4. Etidronate
5. ibandronate
Ans is 3

239. Patient on Renal dialysis due to chronic renal failure


suffered anemia. Which of the following
would you expect to decrease?

BY DR.ABDULRAHMAN ALMUALM
MJDF MCQS WITH ANSWERS

1. Vitamin D
2. Vitamin B12
3. Iron
4. Folate
5. Vitamin K ||||| Ans is 1 (Vit d)
240. You were asked to do help your Nurse about her CPD.
Which one of the following is the
minimum requirement?
1. 25 hours
2. 50 hours
3. 75 hours
4. 10 hours
5. 35 hours
Ans is 2

241. A 22 year old woman has acute gingival hypertrophy, spontaneous


bleeding from the gingiva and complains of weakness and anorexia. Her blood
analysis was as follows: HB=12gm, Neutrophils=90%, Monocytes=1%,
Platelets=250000, WBC=100000, Lymphocytes=9%,Eosinophils=0%The most
likely diagnosis is:

a. Myelogenous leukaemia
b. Infectious mononucleosis /glandular fever/
c. Thrombocytopenic purpura
d. Gingivitis of local aetiological origin
e. Pernicious anaemia /Vitamin B12 deficiency/
Ans is A

242. Symptoms free patient comes to you after four weeks of an endodontic
treatment and you
find on radiograph the canal is over filled with what it seems to be a cone of Gutta
Percha lmm
beyond the apex with a radiolucent small area. What is your initial
management?,
● Start apiectomy through a flap and surgery
● Obturate the root canal
● Ask for a recall and observe in three months time
● Seal the pulp chamber and keep it under observation
Ans is C

243. The patient has generalized chronic moderate


periodontitis and has history of diabetes and
smoking. You plan to treat with nonsurgical periodontal therapy
before surgical therapy. What
would check for follow up assess?
1. Color of gingival
2. Bleeding on probing
3. Reduced in pocket depth
4. Increased in bone level
5. Decrease in recession

BY DR.ABDULRAHMAN ALMUALM
MJDF MCQS WITH ANSWERS

Ans is3 //

244.
Which nerve is affected if a patient is unable to gaze laterally to the left?

-rt abducent
-rt trochlear
-lft abducent
-lft trochlear
Ans is c

245. What would u use if you experienced fire caused by


electricity in your clinic?
1. Water Extinguisher
2. CO2
3. Foam
4. Fire blanket
5. Sand
Ans is 2

246. According to Oxford center for evident base medicine


what is the level of
systematic review of
randomized control trial?
1. 1a
2. 1b
3. 2
4. 3
5. 4
Ans is 1

247. Given OPG X-ray of Fracture mandible in both left and


right para symphysis are and ask about
which investigation would do further?
1. PA of mandible
2. MRI
3. CT
4. No other investigation
5. Occipitomental
Ans is 1

248. If your nurse had Needle stick injury, who should she report first?

BY DR.ABDULRAHMAN ALMUALM
MJDF MCQS WITH ANSWERS

1. You, operator
2. Receptionist
3. Manager
Ans is 1

250. Incident of your junior dental practitioner extracted wrong teeth is


best described as?
A.Cumulative clinical errors
B. Inadequate pre-surgical checks
C. Never events
D. Poor clinical care
E. Poor structure for risk management

Ans is C

251. Which of the following is included in NHS?


A. Right to choose named consultant
B. Legally enforceable rights
C. Right to access drugs and treatment
recommended by doctor
regardless of cost
D. Right to have complaints properly investigated
and deal with
E. Right to have the required treatment/ service
locally
Ans is D?

252. Which age is necessary to check Mental


capacitiy?
1.16yrs
2.18yrs
3. 15yrs
4.21yrs
Ans is 1

253. the patients didn’t satisfy with your settlement, which he/she
could go to before going to
court?
1. Healthcare commissioner
2. Private inspector
3. Independent service
4. Public inspection service
5. GDC
Ans is 1
254. You want to put stainless
steel on 6yrs old child because
of severe gross caries and
he already

BY DR.ABDULRAHMAN ALMUALM
MJDF MCQS WITH ANSWERS

put GIC last 3 months. Which band would u claim?


1. Band 1
2. Band 2
3.Band 3
4. Band 4
5. None
Ans is 3

255. 25yrs old female patient whois quite anxious consult you for extraction of lower
impactedlast
molar. X-ray revealed that there is high risk of nerve injury but the tooth needto go.
You
concerned about patient anxiety and didn't explain about every detail about
complication.After surgery, patient develop paresthesia over lower lip. If she
wanted to sue you, onwhich ground she can?
1. Gross negligent
2. Consent negligent
3. Malpractice
4.civil assault
5.criminal assault
Ans is 2

256. Best way to record maxillary block?


1. Facebow
2. Fox bite
3 transfer articulator
4.Hinge articulator
5.semi adjustable articulator
Ans is 2

257. There is outbreak of diarrhea in hospital after


antibiotics given. What should we do about it?
1. sensible use of antibiotics
2. prophylactic antibiotics
3. give probiotics
Ans is 1

258. 28yrs old female who complain of sensitivity on


upper central incisor. Examination show that no
sign of trauma, normal occlusion and loss of enamel
on palatal surface. What would be your
treatment?
1.composite
2. GIC
3. RCT and filling
4.full crown
5.veener
Ans is 1

BY DR.ABDULRAHMAN ALMUALM
MJDF MCQS WITH ANSWERS

259. When you Air dry lower left first molar, you see white
lesion on teeth surface. What would that
be?
1. Enamel demineralization
2. Fluorosis
3. Enamel mineralization
Ans is 1

260. 61 yr old male patient complain of pain in


TMJ area. X-ray revealed Osteophyte formation on
TMJ. What would be your diagnosis?
1. osteoporosis
2.rheumatoid arthritis
3.osteoarthritis
4.anterior disc displacement
5.posterior disc displacement
Ans is 3

261. Proportion of the patients who test positive who


have disease is?
1. Positive predictive value
2. Sensitivity
3. Specificity
4. Negative predictive value
5. Prevalence
Ans is 2

262. You sent a patient for CT head and radiologist


report lump inside the sinus. What is the
likely diagnosis?
1. Polyps
2. Mucous extravasation cyst
3. Inverted mucocele
4. Pleomorphic adenoma
5. Monomorphic adenoma
Ans is 1

263. Clinical manifestation of isolated zygomatic arc fracture is?

1. Diplopia
2. Infraorbital paranesthesia
3. Ptosis
4. Subconjunctival hemorrhage
5. trismus
Ans is 5 /// all other signs are for zygomaticocomplex fracture

264.Pt with normal centric occlusion but have interferance in eccentric movement
what is your
treatment:
A-Grinding of lingually inclined maxillary teeth

BY DR.ABDULRAHMAN ALMUALM
MJDF MCQS WITH ANSWERS

B-Grinding of lingually inclined mandibular teeth


C-Grinding of maxillary incisors insically
D-Grinding of mandibular inscisors iinsically
Ans is A or B ??

265. At which angle to the external surface of proximal cavity walls in a class
II preparation for amalgam should be finished
A. An acute angle
B. An obtuse angle
C. A right angle
D. An angle of 45°
Ans is c

266. A 10½ year old boy with an uncrowded sound dentition attends your
practice.His permanent canines are unerupted and not palpable and primary
canines are retained. On radiographic examination, you find the canines are
mesially inclined and in the line of the arch. What is the most appropriate
management option?

A. Arrange surgical removal of permanent canines.


B. Extract primary canines.
C. Extract upper first premolars.
D. Grind primary canines.
E. Monitor occlusal development.
Ans is E

267. histopathological lesion closest to advancing edge in enamel caries:


a. dark zone
b. surface zone
c. translucent zone
Ans is C

268. The location and extent of subgingival calculus


is most accurately
determined clinically by
A. radiopaque solution used in conjunction with
radiographs.
B. disclosing solution.
C. probing with a fine instrument.
D. visual inspection
Ans is C : ref churchill book periodontology chapter

269. A characteristic sign of aggressive periodontitis in an


adolescent (juvenile periodontitis) is
A. marginal gingivitis.
B. painful, burning gingivae.
C. hyperplastic gingivitis.
D. drifting of the teeth.
Ans is D ref churchill book

BY DR.ABDULRAHMAN ALMUALM
MJDF MCQS WITH ANSWERS

270. A panoramic cassette was opened in the darkroom to remove and


process the exposed film. On opening the cassette, a piece of paper
was discovered on the surface of the intensifying screen. What kind of
artefact would the presence of paper in the cassette most likely
produce?
A. A black artefact
B. No artefact
C. A white artifact
D. Reticulation
E. Dichroic fog
Ans is C

271. 261. Probe pressure at the sulculus of pocket should be enough to:
a. Feel the top of the crestal bone
b. Balance the pressure between fulcrum and grasp
c. Define the location of the calculus deposit
d. Feel the coronal end of the attached tissues
e. Limit the lateral pressure
ans is D (note probe pressure is 25 g)

272. You are interested in finding out what the risk indicators are for a
rare form of oral cancer and decide to undertake a study to examine this.
What type of study would be the most appropriate for addressing this
issue?
A. Cohort
B. Prevalence study
C. Clinical trial
D. Case-control study
E. Case-series
Ans is a

Types of probes

Nabers probe

North
Carolina p

BY DR.ABDULRAHMAN ALMUALM
MJDF MCQS WITH ANSWERS

273. What enzyme assists microorganisms when they are causing dentine
caries?

a.Collagenase
b.Enolasec.
c. Lactoferrin
ans is B

274. Rank the following impressions materials according to their flexibility

A. Alginate> Polysulphide> Silicone> Zinc Oxide Eugenol


B. Silicone> Alginate> Polysulphide> Zinc Oxide Eugenol
C. Alginate> Polysulphide> Zinc Oxide Eugenol>Silicone
D. Alginate> Silicone> Polysulfide> Zinc Oxide Eugenol
E. Alginate> Zinc Oxide Eugenol> Silicone> Polysulphide
Ans is D

275. 3 year-old child, baby teeth to be extracted. You want to check


permanent germs. Which radiograph?
Panoramic,
periapical,
Bimolar,
occlusal
, lateral skull view
Ans is C Bimolar
276. In regards to carbide burs; the more number of cutting blades and low
speed will result in:
A. Less efficient cutting and a smoother surface

BY DR.ABDULRAHMAN ALMUALM
MJDF MCQS WITH ANSWERS

B. Less efficient cutting and a rougher surface


C. More efficient cutting and a smoother surface
D.More efficient cutting and a rougher surface
Ans is D /Number of cutting blades affects cutting efficiency.
Speed affects roughness/smoothness.

277. WHICH MUSCLE IS PIERCED IN INFIRIOR ALVEOLAR NERVE BLOCK


/?

●Medial pterygoid
●Superior constrictor and Buccinator
Ans is B

278. What is FIBRIN STABILIZING FACTOR ?


Factor Xlll

279. Which of the following ahs the highest sucrose content:


A. Ice cream
B. Canned juice
C. Cough syrups
D. Breakfast cereal
E. Sweet potato
Ans is B?

280After the age of 6 years, the greatest increase in the size of the mandible
occurs:
A. At the symphysis
B. Between canines
C. Distal to the first molar
Ans is C

281. Which of the following does state BEST the morphology of periodontal
ligament fibres:
A. Elastic
B. Striated
C. Non striated
D. Levity
E. Wavy
Ans is E

282. Patient whose hands fell warm and moist is MOST likely to be suffering
from:**

A. Anxiety
B. Congestive cardiac failure
C. Thyrotoxicosis
Ans is C

283. what is most likely to happen to an interproximal composite filling rather


than amalgam?

BY DR.ABDULRAHMAN ALMUALM
MJDF MCQS WITH ANSWERS

Overcontouring,
undercontouring ,
overhang.
Fracture
Ans is b (undercountring)

284. 20. You used N-type sterilizer for your instrument with proper packing. How
long can these instruments be stored?
A. 4 months
B. 8 months
C. 12 months
D. 15 months
E. 24 months
Ans is C// B type sterlizer don’t store

285. 26. You realize a problem calibrating delivery gas while treating patient with
Nitrous oxide. Who should you report to?A. Care quality commission
B. Clinical commissioning group
C. Device manufacturer
D. Health and safety executive
E. Medicine and healthcare product regulatory ageny
Ans is E

286. Which of the following has highest five year survival rate for class V
restoration?
A. Amalgam
B. Bonded amalgam
C. Composite (flowable)
D. Conventional GIC
E. Resin modified GIC
Ans is E

BY DR.ABDULRAHMAN ALMUALM
MJDF MCQS WITH ANSWERS

287. 7. Regarding quality adjusted life years (QALY),


which of the following is true?
A. Objective way of deciding the cost effective of
treatment intentions
B. Do not discriminating against certain patient group
C. Scrutinised by NICE
D. Used to compare improvement of quality of life
E. Used to consider the effectiveness of treatment
Ans is D

288. 5. You plan to do double blinded Randomised


controlled trial. Once you have finished design the
trial, what do you do next?
A. Advertise in dental research journal for participants
B. Allow patient to choose the treatment they wish to
receive
C. Ensure potential patient understand the method of
treatment allocation
D. Inform interested patient and get their consent
E. Seek approval from appropriate research ethics
committee
Ans is E

289. Which medication is prescribed to overcome acute anxiety prior to dental


treatment?
A. Barbiturates
B. Benzodiazepines
C. Monoamine oxidase inhibitors
D. Selective serotonin reuptake inhibitors
E. Tricyclic antidepressants
Ans is B

290.Which of the following is true regarding dabigatran?


A. Competitively inhibiting action of Vitamin K
B. Medication delivered enteraly (specifically inihibits clotting factors Xa)
C. Anticoagulant inhibits conversion of fibrinogen to fibrin X
D. Parental anticoagulant of choice for hospitalized patient
E. Prevent platelet aggregation in the early stages of haemostasis process
Ans is C
sold under the brand name Pradaxa among others, is an anticoagulant used to treat
and prevent blood clots and to prevent stroke in people with atrial
fibrillation.[1][2] Specifically it is used to prevent blood clots following hip or knee
replacement and in those with a history of prior clots.[1] It is used as an alternative
to warfarin and does not require monitoring by blood tests.[1] It is taken by mouth.[1]

BY DR.ABDULRAHMAN ALMUALM
MJDF MCQS WITH ANSWERS

291. Stephan's curve represents


a) change in pH of saliva with time
b) change in pH of plaque with time
c) change in pH of saliva with sugar intake.
Ans is “c

292.A 46 year old female presents with a slowly enlarging painless firm swelling in
the hard
palate to the left of the midline.
The most likely diagnosis is:
● A dental abscess
● Torus palatinus
● Osteoma
● Pleomorphic adenoma
● Canalicular adenoma
Ans is D pleomorphic adenoma

293. Which one is the most benign


tumor
A. Kaposi sarcoma
B. Adenolymphoma
C. Burkitt's Lymphoma
Ans is B

294. A curette may be inserted to the level of the attached gingiva with
minimal trauma to the tissues because of
A. Has a round base
B. Is easy to sharpen
C. Has rounded cutting edges
D. Provides good tactile sensitivity
E. Has two cutting edges
Ans is C

295. As far as localised alveolar osteitis is concerned; which one of the


following is true

BY DR.ABDULRAHMAN ALMUALM
MJDF MCQS WITH ANSWERS

A. The incidence in the mandible and maxilla


is similar
B. The prophylactic prescription of antibiotics
prior to extraction reduces
the incidence.
C. Excessive fibrinolysis is the likely aetiology
D. Purulent exudate must be seen for a
diagnosis and irrigation is
mandatory
E. Zinc oxide eugenol and alvogyl dressing
promote a rapid bone growth
Ans is C

296. 52. Which of the following anomalies occurs during the initiation and
proliferation stages of tooth development
A. Amelogenesis imperfecta - histodifferentiation
B. Dentinogenesis imperfecta- histodifferentiation
C. Enamel hypoplasia – apposition stage
D. Oligodontia – initiation stage ( absence of single or multiple teeth)
E. Ankylosis Hypocalcified/hypomineralised – normal organic matrix but
defective mineralization.
Ans is D

297. The management of patients taking corticosteroid for long time with
dental infection, abscess or high temperature, do we need double the
dose of antibiotic or double the dose of
cortisone
ANS double dose of cortisone to prevent
adrenal crisis

298. DOSAGE OF 8 YEAR OLD CHILD WITH


ANAPHYLACTIC REACTION?

Ans is 0.3 mg Adrenaline IM

BY DR.ABDULRAHMAN ALMUALM
MJDF MCQS WITH ANSWERS

299. about k files all are


true except
a. has more number of
flutes then reamer
b. is more flexible then
reamer
c. used to machine the
dentin
d. made up of
triangular/square

Ans is B

300. 100. What is the


extra cranial cause of
nerve palsy?
a. cerebral malignancy (Intracranial palsy)
b. stroke (Intracranial palsy)
c. MS (Intracranial palsy)
d. Ramsay hunt syndrome (Intracranial palsy)
e. Melkersson-Rosenthal syndrome

Ans is E

301. A young girl complains of swelling of


cheeks both sides. On doing
various tests the salivary glands were found
normal. What may be the
reason?
Ans is sialosis

302. A 43year old patient is missing on the


upper right first premolar and
molar. He has good oral hygiene and
requests a fixed replacement
for these teeth. The other teeth on the
same side are all
moderately restored with MOD amalgam
restorations and are vital,
except the canine, which has a very large
restoration and is root-
filled. He has group function. Radiographs show a large sinus
cavity and no peri-apical pathology. What would be the restoration of

BY DR.ABDULRAHMAN ALMUALM
MJDF MCQS WITH ANSWERS

choice for replacement of the missing teeth?


A. Implant supported crowns
B. A conventional fixed bridge using the 7 and 5 as abutments
C. Two conventional cantilevered bridges, using the 7 and 3 as
abutments
D. A resin-bonded bridge, using the 7 and 5 as abutments
E. A conventional fixed-moveable bridge using the 7 and 5 as abutments
Ans is E/

303. An 80 year old patient presents with an ulcer in the floor of the
mouth. This has been present for several months and has not
responded to conventional treatment. An incisional biopsy is taken.
Which of the following histological changes in the epithelium confirm a
diagnosis of squamous cellcarcinoma?
A. Hyperkeratosis
B. Acanthosis
C. Dysplasia
D. Invasion
E. Discontinuous epithelium
Ans is D

304. Cigarette smoking is considered to be the most important factor


next to microbial plaque in periodontal disease progression. Which of the
following is the most important factor in the disease progression in
smokers?
A. Smokers have drier mouths than non-smokers
B. Smokers have poorer oral hygiene than non-smokers

BY DR.ABDULRAHMAN ALMUALM
MJDF MCQS WITH ANSWERS

C. Nicotine will impair the chemotactic and phagocytic properties of


PMNs.
D. The gingival blood flow is reduced in smokers.
E. Smokers alter the oral environment encouraging the growth of
anaerobic bacteria.
Ans is C

305. A patient gives a history of rheumatic fever. Which of the following


procedures require prophylactic antibiotic cover.
a. scale and polish
b. extraction of tooth
c. inferior dental nerve block
d. impression for a new lower complete denture
e. placing a class 1 amalgam restoration
Ans is B /high risk patient with high risk procedure (bloodletting)

306. What make scaling easier and less exhausting?


A. length of instrument
B. breadth
C. grip
Ans is C

307.. .best method to detect sickle cell anemia:


a. full blood count – iron def
b.siderex - if results needed urgently
c. Hb electrophoresis – sickle cell anaemia.(on all afro Caribbean,
Mediterranean, middle easternand Indian)
Ans is C

308. 144. Last deciduous tooth to be replaced by permanent one:


a. max canine
b. max 2nd molar
c. mand canine
Ans is A

309 Elongation of pulp chamber seen in:


a.gemination
b.turner teeth
c.taurodontism
d.dilaceration
Ans is C

310.. Potassium sulfate is added to gypsum to


a. increase setting expansion
b.decrease working tTime
Ans is B

311. Hb e antigen means:


a. high risk infection
b. recent infection

BY DR.ABDULRAHMAN ALMUALM
MJDF MCQS WITH ANSWERS

c. immune to infection
Ans is A

312. 156. IgG and C3 are seen in:


a. lichen planus
b. phemphigoid
c. erythema multiforme
d. phemphigus vulgaris
ans is B

313.. actinomycosis is differentiated from osetomyelitis by the presence


of:
a. sequestrem
b. involucrum
c. sulfpur granules
d. pain
Ans is C

314. case scenarios where one central incisor is missing in a child who
has had a previous historyof trauma with an avulsed primary incisor, the
cause is
a.scar tissue
b.supernumerary teeth
c.tubercle
d.frenum
Ans is A /trauma lead to dilaceration to developing tooth

315. A mother brings her child to the out of hours clinic suffering
spontaneous excess bleeding from his gingival, what is the
expected diagnosis?
- acute leukemia
- injury
- stress
- scurvey
Ans is A

316. The most important feature to differentiate between an upper


neuron motor lesion and a lower neuron motor lesion is:
- eye involvement
- ear involvement
- forehead involvement
- anaesthesia of the facial nerve
Ans is C

317. A supernumerary tooth next to the maxillary lateral incisor is called


a:- conical tooth
- supplementary
- tuberculate
Ans is A

BY DR.ABDULRAHMAN ALMUALM
MJDF MCQS WITH ANSWERS

318. You suspect the patient suffers hepatitis B symptoms, after investigation,
the test show HBe Ag antibodies, what does that indicate?
- the patient is getting better
- the patient is in a transmitting state of the disease
Ans is B

319. Peptic ulcers are caused by the following type of bacteria:


- mycobacterium tuberculosis
sterptococus oralis
- helicobacter pylori
Ans is C

320. 186.According to the modified Anxiety Scale, a dental phobic patient is


on a scale of:
- 9-14
- 15-20
- 20-27
- 27-35
Ans is C
MDAS (Modified dental anxiety scale) is a scale of 5 to 25

5-11 --> low anxiety


12-14 --> moderate anxiety
15-19 --> high anxiety
20-25 --> extreme anxiety (Dentophobic).

321. A man has diet-controlled type 2 diabetes. He consumes


30 glasses of alcohol and smokes 40cigarettes per day. He is otherwise
healthy. What may be the reason of dry mouth in this person?
Sjogren's sydrome,
candidosis,
dehydration,
drug-induce
Ans is C

322. Laws and regulations governing different scenarios e.g Dental


Nurse wanting to take radiograph. Options included
IRMER,
Health and Safety,
COSHH.
Ans is A (IRMER)iniozing radiation medical exposure regulation

323. You arrive at a new practice and notice that almost every
radiograph in the patient's notes has turned brown. Your nurse confirms
that this is a widespread problem that no-one has ever remedied and
she also remarks that the films tend to get browner with age. What
corrective action will you take to remedy the problem?

BY DR.ABDULRAHMAN ALMUALM
MJDF MCQS WITH ANSWERS

A. Develop the films for the correct period of time


B. Heat the developer to a higher temperature
C. Ensure films are fixed for the correct period of time
D. Wash the films properly after fixing
E. Change the solutions more regularly
Ans is C ?

324. Which of the following organisms are pathognomonic of acute


necrotic ulcerative gingivitis?
a. Spirochaetes and fusobacterium SP
b. Spirochaetes and eikenella corrodes
c. Polymorphs and lymphocytes
d. Actinobacillus actinomycetes comitans oral capnocytophaga
e. Porphyromonas gingivalis and prevotella intermedia
Ans is A

325. The most accurate way to evaluate the effectiveness of root planning is
by:
a. Inspect the root surface with an instrument for root smoothness
b. Use air for visual inspection
c. Activate a curette against root surface and listen for a high pitched
sound which indicates as mooth, hard surface.
d. Evaluate the soft tissue at the end of the appointment for a
decrease oedema and bleeding
e. Evaluate the soft tissues 10 to 14 days later.
Ans is A

326. The most common type of injury caused by a Non accidental Injury is:
- burnt tip of tongue
- ulcer on the gingiva
- lacerated labial frenum
- neck bruise
Ans is d

327. Tetracycline hydrochloride conditioning of root surface in


periodontal surgery is to:
a. Sterilise the root surface
b. May enhance binding of fibronectin and fibroblast
c. Aids in re-mineralising the root surface
d. Assist the biding of lamina dura
e. Prevents post operative infections
Ans is B

328. A cold stimulus applied to a tooth will produce a hypersensitive


response if the tooth
A. is non vital
.B. has a periodontal pocket.
C. has a hyperemic pulp.
D. has chronic proliferative pulpitis
Ans is C

BY DR.ABDULRAHMAN ALMUALM
MJDF MCQS WITH ANSWERS

329. The location and extent of subgingival calculus is most accurately


determined clinically by
A. radiopaque solution used in conjunction with radiographs.
B. disclosing solution.
C. probing with a fine instrument.
D. visual inspection
Ans is D?

330. 267. Which treatment procedure is indicated for a patient with


asymptomatic age related gingival recession?
A. Connective tissue graft.
B. Gingivoplasty.
C. Lateral sliding flap.
D. Gingival graft.
E. No treatment. Ans is E
331. Which of the following is most effective de-sensitising agent?
A) A. Arginine
B) B. Glycerine
C) C. Hydrated silica
D) D. Na MFP
E) E. SnF2
Ans is E.

332. 8. Which of the following does not act by interfering with cyclooxygenase
pathway of arachidonic acid
A) A. Aspirin
B) B. Diclofenac
C) C. Ibuprofen
D) D. Dihydrocodeine
E) E. Paracetamol
Ans is D,?
333. 1. Which of the following is cyanotic congenital heart condition?
A) A. artrial septal defect
B) B. coarctation of aorta
C) C. floppy mitral valve
D) D. patent ductus arteriosus
E) E. transposition of great vessels
Ans is E

334. Which one is not a developmental line of teeth.

Von Ebner
Neonatal line
Perikymata
Non of the above
Ans is D

335. When treating a tooth with a non-vital pulp with a fistula presented;fistula
should be treated by

BY DR.ABDULRAHMAN ALMUALM
MJDF MCQS WITH ANSWERS

A. Surgical incision

B. Antibiotic coverage

C. The usual root canal procedures for non-vital teeth and no special
procedures for fistula
Debatable Question some answered A some as C

336. When primary molars are prepared for stainless steel crowns should the
depth for reduction of the proximal surface be similar to the depth of the
buccal and lingual surfaces?

A. Yes; reduction of all wall is similar for best retention


B. No, proximal reduction is greater to allow the crown to pass the contact
area
C. No, the buccal surfaces has the greatest reduction to remove the cervical
bulge
D. Yes, all undercuts are uniformly removed so that the steel crown can be
seated
E. No, because of lateral constriction, the lingual surface needs greatest
reduction
Ans is B

337. Which of these muscles may affect the borders of mandibular complete
denture,

A. Mentalis
B. Lateral pterygoid
C. Orbicularis oris
D. Levator angulioris
E. Temporal
Ans is B

338. Electrical pulp testing is least useful in /or does not detect in some
papers/

, A. Traumatised teeth
B. Just erupted teeth
C. Multi-rooted teeth
D. Capped teeth
E. Necrotic pulp
Ans is A

339. Maximum shrinkage after gingival curettage can be expected from


tissue that is
A. fibroedematous.
B. edematous.
C. fibrotic.
D. formed within an infrabony pocket.

BY DR.ABDULRAHMAN ALMUALM
MJDF MCQS WITH ANSWERS

E. associated with exudate formation.


Ans is B

340. The absence of adequate drainage in a periodontal pocket may


result in
A. cyst formation.
B. abscess formation.
C. epithelial hyperplasia.
D. increased calculus formation.
Ans is B

341. For an otherwise healthy patient, with an acute localized


periodontal abscess, initial treatmentmust include
A. scaling and root planing.
B. occlusal adjustment.
C. prescription of an antibiotic.
D. prescription of an analgesic.
Ans is A

342. You are interested in finding out what the risk indicators are for a
rare form of oral cancer and decide to undertake a study to examine this.
What type of study would be the most appropriate for addressing this
issue?
A. Cohort
B. Prevalence study
C. Clinical trial
D. Case-control study
E. Case-series
Ans is D (repeated Qs with different ans )

343. . Gingival pocket lining is formed from;


reduced E E,
hertwigs,
dental papilla
Ans is A

344. Membranes of expanded polytetraflourethylene have been


designed for periodontal regenerative techniques. Which of the following
defects will respond most predictably to regenerative therapy?
a shallow, wide 1-walled defects
b shallow 2- wall defect
c deep narrow 3- walled defect
d deep narrow 1- walled defect
e shallow narrow 2- walled defect
ans is C

345. What is the radiation exposure when taking an OPG


a 0.001mSv
B 0.01 mSv(preapica)

BY DR.ABDULRAHMAN ALMUALM
MJDF MCQS WITH ANSWERS

c.0.01mSv
d. 1.0mSv
e. 10 mSv
Ans is B

346. what is the optimal shape for collimator


a. square
b.rectangular
c. round
d. hexagonal
e. circular
ans B

347. what is the radiation dose in milli-Sieverts for a patient associated


with having an OPG??
A. 0.034
B 0.34
C 34
D 3.4
E 340
Ans is A (range from 0.01 to 0.03 mSv)

348. Which cells are found in hepatic stomatitis


A, lymphcytes
B, monocytes
C,neutrophils
D, macrophages
E, plasma cells
Ans is A

349. Which of the following drug is mast cell stabilizer?


a. salbutamol
b. adrenalinE
c. ipratropium bromide
Ans is A (B2 agonist )

350. Where the bone is lost the most in periodontal disease?


Buccal,
Lingual,
Mesial,
Distal,
Approximal
Ans is D .

351. What cement u use to glue a alumina core ceramic crown?- GIC-
RMGIC-
COMPOSITE RESIN CCEMENT

BY DR.ABDULRAHMAN ALMUALM
MJDF MCQS WITH ANSWERS

Ans is B

352. In which condition will direct immunoflourescence reveal binding of


autoantibodies to the intercellular substance of epithelial cells?-
Pemhigus ( IgG )-
Pemphigus/ benign mucous memb pemphigoid. -
erythema multiforme

353. How would you treat a patient with MRSA,TB,Hepatitis?> options:

single use instruments,


treat using standard conditions ,
send them to hospital
ans is B

354. An upper deciduous molar has a caries exposure and on X ray the
corresponding 2nd permanent premolar is absent. What treatment would you
do to the deciduous tooth:

A. Pulpotomy
B. Endodontic treatment
C. Pulp capping
Ans is B

355. How many pulp horns are presented in a typical mandibular deciduous
second molar: A. 2
B. 3
C. 4 .
D. 5
Ans is c

356. Who is more appropriate to give consent?-

Stepfather of a child- Grandmother of a child-


16 year old sister of an 8 yearold child
Mother of 4 year old child
Ans is c

357. which type of hypersensitivity reaction is seen in tuberculous lesion?


a) type1
b)type2
c)type3
d)type4
Ans is C

358. A 25-year-old patient presents with pain in a non-restored, lower posterior


tooth. Whatdiagnostic tests would be MOST EFFECTIVE in determining
cracked tooth syndrome?
1). EPT
2). Fiber optic light 3). Tooth slooth

BY DR.ABDULRAHMAN ALMUALM
MJDF MCQS WITH ANSWERS

4). Radiographs
5). Staining

A. (1) and (5)


B. (1), (2) and (5)
C. (2), (3), and (5)
D. (1), (3), (4) and (5)
E. (4) and (5)
Ans is C

359. If amalgam gets contaminated with moisture, the most uncommon result
is: **A. Blister formation
B. Post operative pain
C. Secondary caries D. Lower compressive strength
Ans is A
360.If the sealant of bonding agent is not placed on part of enamel that has
been etched by an acidsolution; you would expect:
A. Arrest of enamel carries by organic sulphides
B. The enamel is to return to normal within 7 days
C. Continued enamel declassification in the etched area
D. Slight attrition of the opposing tooth
Ans is C

361. A major difference between light cured and chemical cured composite is
that during setting or in function the light cures material tends to:

A. Seal the margins better and completely


B. Exhibit less wear on time
C. Undergo greater colour change
D. Shrink rapidly
E. Posses greater fracture toughness
Ans is D

362. The use of physical barriers in Guided Tissue Regeneration is to do


what?

A.Retard apical migration of epithelium.


B.Exclude gingival connective tissue from the healing wound
C.Both
D. Neither
Ans is A

363. What is the typical feature of Addisonian crisis?


A) A. Bradycardia
B) B. Hyperglycemia
C) C. Hypotension
D) D. Increase respiratory rate
E) E. Urinary incontinence
Ans is C

BY DR.ABDULRAHMAN ALMUALM
MJDF MCQS WITH ANSWERS

364. Which of the following is NOT tax deductible expenses?


A) A. Conference fee
B) B. Critical illness cover
C) C. GDC annual retention fee
D) D. Indemnity/ insurance
E) E. Surgery clothing
Ans is B

365. Citalopram is use to treat?


A) A. Depression
B) B. Management of focal seizure
C) C. Parkinson disease
D) D. Personality disorder
E) E. Schizoperia

Ans is A

366. 2.when a CD wearer says s n sh tip of tongue touches


a. hard palate
b.soft palate
c.max.canines
d.max.premolars
e.max.molars.
Ans is A

367.why should the lingual embrassure b/w upper 3 n upper 4 be enlarged


during mouth preparationfor maxillary partial denture??
a.to prevent denture slip mesially
b.to prevent denture slip distally
c.to provide adequate retention
d.to provide space for minor connector
Ans is D?

368. Which of the following materials has an alkenoic reaction:


A calcium hydroxide,
b. ZOE,
C.GIC.
Ans is C

369Which type of cells is seen in the early stage of chronic


periodontitis?
Basophils,
eosinophils,
lymphocytes,

BY DR.ABDULRAHMAN ALMUALM
MJDF MCQS WITH ANSWERS

neutrophils,
macrophages
ans is D

370.Which type of cells is seen in late stage of chronic


periodontitis?
Basophils,
eosinophils,
lymphocytes,
neutrophils,
macrophages
ans is C

371. type of restoration to be placed on upper 7 that is to be clasped buccally,


reciprocation palatally, and an occlusal rest mesially. A. 3/4 A.GC
B. FGC
C. NiCr
D. PJC 3/4 /full
Ans is B full gold.crown ?

372. If patient can't open after IDN block which muscle of mastication affected?
A. Medial pterygoid
B. Lateral pterygoid C. masseter
Ans is A.

373. Deep needle insertion during the standard inferior alveolar nerve block may
result in:
A. Premature bone contact
B. . A greater success rate.
C. Contact with the temporal crest of the ramus
. D. Penetration of the parotid capsule.
Ans is D

374. Which one of the following is most commonly used to bleach vital teeth:

A. . Ethyl chloride
B. Hydrogen chloride
C. Hydrogen peroxide
D. Sodium bicarbonate
E. Sodium hypochlorite
Ans is C

375. Patient with Lateral periodontal abscess ,what is the treatment of choice

A. Metronidazole
B. Amoxicillin
C. Tetracycline
D. Drainage of the pus and analgesic
Ans is D

BY DR.ABDULRAHMAN ALMUALM
MJDF MCQS WITH ANSWERS

376. picture of the gingival of a black person with melanin pigmentation


and a white line across the central incisors alone
1. What is the cause of the white line
a. Amelogenesis imperfecta
b.Dentinogenesis imperfecta
c.Hypo mineralization
d.Flourosis
Ans is D ???

377. Which of the following salivary gland tumours is most likely to occur
bilaterally?

a) pleomorphic adenoma
b)mucoepidermoid carcinoma
c)adenoid cystic carcinoma
d)acinic cell carcinoma
Ans warthin tumor

378. 206 Which sealant is more likely to cause tissue damage if extruded
– AH plus,
grossman's sealer,
tubliseal
Ans is C

379. A 35 year old male patient who admits to grinding his teeth at night has a
number of wedge-shaped cervical (Class V) lesions on his upper premolar teeth.
These are causing some sensitivity and are approximately 3mm deep. What is the
correct management option?

A. Provide tooth brushing instruction and fluoride


B. Restore the lesions with compomer
C. Restore the lesions with micro-filled composite D. Restore the lesions with a
hybrid composite
E. Restore the lesions with conventional glass-ionomer
Ans is E best is RMGI

380. Which of the following is the ideal treatment for a degree II furcation
involvement of a
mandibular molar?
● Tunnel preparation
● Root resection
● Furcation plasty
● Extraction
● Guided Tissue Regeneration
Ans is E for Class III furcation --> tunnel preparation
For Grade I --> furcationplasty

381. A 7 year old boy has previously had all primary molars restored and a
pulpotomy on upper

BY DR.ABDULRAHMAN ALMUALM
MJDF MCQS WITH ANSWERS

right E. He has an early mixed dentition with lower lateral incisors erupting. There is
a midline
diastema of 2 mm. The upper right E has become symptomatic and
requires extraction.The most likely long term effect of the extraction on the occlusion
is:
● Early eruption of the second premolar.
● Loss of upper central line.
● No significant effect.
● Overeruption of the lower right teeth.
E. Potential crowding in the upper right quadrant
Ans is E

382. Six months ago you saw a child patient, then aged 9 years. His upper right
maxillary
canine was palpable in the labial sulcus but the upper left was not. The situation is
now
unchanged, so you have taken two periapicals of the non-palpable tooth. They both
show that
there is some resorption of the CI root but the permanent canine appears somewhat
mesially
angled and is more mesial on the more mesially positioned film.
What is your the best course of action?
● Keep a careful watch on it and take another x-ray in 6 months.
● Refer to an oral surgeon for early exposure of the permanent canine.
● Refer to an oral surgeon for early removal of the permanent canine before it
damages
the lateral incisor.
● Refer to an orthodontist for a treatment
Ans debatable Qs some answered E some c

383. Which one of the following is true in regards t osseointegration implants in


dentistry?
● Fibrous tissues are formed and integrated directly between titanium and bone
● Following insertion, implants can be immediately loaded without problem
● The success of the implants is directly proportional to its area of contact with bone
● The success of the implants depends mostly on low torque preparation and
insertion
of the fixture
Ans is C

384. A 9 year old boy has a small white discolouration on his maxillary central
incisor. The
lesion is most probably,
● Hypocalcification due to trauma of the primary predecessor
● Hypoplasia due to acute systemic infection when 6-12 months old
● Defect during the histo differentiation stage of development
● Defect during the morho differentiation stage of development
Ans is A

385. Following extraction of the molar teeth

BY DR.ABDULRAHMAN ALMUALM
MJDF MCQS WITH ANSWERS

● The ridge height is lost more from the maxilla than from the mandible
● The maxillary ridge will get more bone lost from the palatal aspect than the buccal
● The mandibular arch is relatively narrower than the maxillary arch
● Compared with the pre-resorption state, the mandibular ridge will lose more bone
from the lingual aspect than the buccal one.
Some answered C some D

386. 178 In planning and construction of a cast metal partial denture the study cast
● facilitates the construction of custom trays
● minimizes the need for articulating
● provides only limited information about inter ridge distance,
Ans is A

387. A 50 years-old patient presents with pain from time to time on light cervical
abrasions.
What is your first management to help patient in preventing pain in the future?
A.Change diatary habits
B.Change brushing habits
C.GIC fillings ans is c and B
388. The most common cause of porosity in porcelain jacket crowns is,'
● Moisture contamination
● Excessive firing temperature
● Failure to anneal the platinum matrix
● Excessive condensation of the porcelain
● Inadequate condensation ofthe porcelain
Ans is e

389. Which of the following liquids is not suitable for prolonged immersion of cobalt
chrome
partial dentures:
A. Alkaline peroxidase
B. Sodium hypochlorite
C.Soap solutions
D.Water
Ans is b

390. On examination of a composite restoration you find a dark stain:


A. Replace the composite
B.Repair with unfilled resin
C.Apply topical fluoride at the margin

Ans is A /some answered B

391. The flexibility of the retentive clasp arm does not depend on:
● Length of the arm
● The cross section shape
● The material used
● Degree of taper
● The exerted force
Ans is e

BY DR.ABDULRAHMAN ALMUALM
MJDF MCQS WITH ANSWERS

392. Following calcium hydroxide pulpotomy, the dentist would expect dentine bridge
to form at,
● The exact level of amputation
● Level somewhere below the amputation
● Half way between amputation and apex
● At the apical region of the tooth
Ans is b

393. Where would you expect to find the mylohyoid muscle in relation to the
periphery of a full
lower denture:
A.Mandibular buccal in the midline
B.Mandibular lingual in the first premolar area
C.Mandibular lingual in the midline
D.Mandibular disto buccal area
Ans is B
394. In anxoius and psychologically stressed patients gingivitis is often more severe
because of
A.Stress causes histamine and serotonine release
B.stress causes catecholamine and corticosteroid release
c.Stressed people neglect their oral hygiene
Ans is b

395. What does not help in establishing the caries risk in children?
A.History of caries
B.Lactobacilius count
C.Dietary habits
D.Brushing habits
E.Genetic predisposition
Ans E

396 . 8 years old child who has sustained a fracture of maxillary


permanent central incisor in which 2mm of the pulp is exposed;
presents for treatment three hours after injury. Which of the
following should be considered
A. Remove the surface 1-2 mm of pulp tissue and place calcium
hydroxide
B. Place calcium hydroxide directly on the exposed pulp
C. Pulpotomy using formocresol
D. Pulpectomy and immediate root filling
E. Pulpectomy and apexification
Ans is A

397. The concentration of Fluoride in the topical NaF


A.2%
B.5%
C. 8%
D.10%
Ans A 2%

BY DR.ABDULRAHMAN ALMUALM
MJDF MCQS WITH ANSWERS

398. While removing the second primary molar of 9 years old child, the apical third of
the root
fracture and stay in the socket,
● You will just leave it and observe it
● You take surgically by a lingual flap
● You try to take out by using a root apex elevator
● You use a fine end forceps to take it out
Ans is A

399. 32-year-old man was treated for a fractured mandible and had post-operative
radiographs taken, which one of the following treatments has he received?

a)Closed reduction and inter-maxillary fixation


b)Intermaxillary fixation and conservative management of the fractures
c)Open reduction and internal fixation
d)Open reduction and internal fixation with temporary Intermaxillary fixation in the
form of eyelet wires
e)No surgical treatment
ans D

400. Which type of glass ionomer


cement is used for fissure sealants?
A.
Type I
B. Type II
C. Type III
D. Type IV
E. Type V
Ans D

TYPE l is for luting agent


Type ll for restoration
Type lll for base and liners
Type lV for fissure sealants
Type V for orthodontic cements
Type Vl for core build up

.401 In relation to resin-retained bridges the term retainer describes: The cement
attaching the wing to the tooth
A. The prosthetic tooth within the bridge
B. The tooth to which the bridge is secured
C. The wing portion of the bridge that is attached or cemented Jo the natural tooth
D. None of the above
Ans is c

BY DR.ABDULRAHMAN ALMUALM
MJDF MCQS WITH ANSWERS

402Which of the following is a common cause of dry socket?


a) Elderly patient
b) Forceps extraction
c) Medically compromised patient
d) Oral contraceptives
e) Previous treatment with bisphosphonates
ans is D

403Assuming that the distance between the gingival margin on the lingual su face of
the lower anterior teeth and the base of the lingual suIcus is 7 mm which of the
following is the most appropriate major connector to prescribe in this case?

A. Continuous plate
B. Dental bar
C. Lingual bar
D. Lingual plate
E. Sublingual bar
Ans is C

.404 Which one of the following statements best describes statistical power?
a) This is when a true null hypothesis gets rejected
b) This is when a false null hypothesis gets accepted
c) Refers to the probability that the observed statistical difference has
occurred by chance
d) It shows that there is no difference in outcome between two groups after an
intervention in one of the groups
e) The probability of correctly rejecting a false null hypothesis
Ans is E

. 405 In the construction of a full veneer gold crown, future recession of gingival
tissue can be
prevented or at least minimised by,
● Extension of the crown 1 mm under the gingival crevice
● Reproduction of normal tooth incline in the gingival one third of the crown
● Slight over contouring of the tooth in the gingival one fifth of the crown
● Slight under contouring of the tooth in the gingival one fifth of the crow
Ans is B

.406 patient had a superficial parotidectomy 6 weeks ago. They report that an
area of their cheek on the same side becomes red and sweats at mealtimes.
Which nerve has been damaged?
a) Auriculotemporal branch of the trigeminal nerve
b) Buccal branch of the facial nerve
c) Mandibular branch of the trigeminal nerve

BY DR.ABDULRAHMAN ALMUALM
MJDF MCQS WITH ANSWERS

d) Temporal branch of the facial nerve


e) Zygomatic branch of the facial nerve
ans is A ( Freyssyndrome)

.407 13 years old boy comes to you with excessive hyperplasia of the gingiva as a
result of
Phenytoin what is your management,
● Stop the medication
● Force a strict oral hygiene and surgical removal of excess gingival tissues
● Debridement and conservative approach
Ans is B ?

.408 What is the main purpose of using corticosteroids in pulpal obturation material?
● For their antibiotic action
● For their antiinflammatory action
● To relief pulp pressure
Ans is B

.409 which of the following is true regarding TMJ dysfunction,


● It is always due to arthritis, should be treated with NSAIDS before attempting
surgery
● Raising bite increases the space in the joint and should be attempted before
surgery
● It is mostly due to the medial movement of the condylar head over the glenoid
fossa
Ans is B ?
410
.To produce a stable correction of an upper labial segment in
lingual crossbite; it is essential to
A. Use fixed appliances
B. Have adequate overbite
C. Treat during growth
D. Use posterior capping
E. Increase vertical dimension
Ans is B to prevent relapse we must have adequate overbite

.411 Which one of the following is the guiding principle of utilitarianism?

a) It regards duties and rules


b) It regards doing the greatest good for the greatest number
c) The cultivation of virtue within oneself
d) All of the above
e) None of the above
Ans is D

.412 Which one of the following combinations produces the lowest stress within a
root when used as a post and core?
A. Cast metal post and core
B. Composite resin post and core
C. Composite resin core in combination with a glass fibre post

BY DR.ABDULRAHMAN ALMUALM
MJDF MCQS WITH ANSWERS

D. Parallel prefabricated metal post


E. Tapered prefabricated metal past
Ans is C / dentine and fiber glass got same modulus of elasticity

.413 When designing a partial denture, which one of the following steps comes first?.
A. Direct retention
B. Minor connectors
C. Support
D. Indirect retention
E. Major connectors
Ans is C

1. Support.
2. Direct retention.
3. Indirect retention.
4. Major connectors.
5. Minor connectors.

.414 Which of the following is


not a property of Fluoride ion
A. Crosses placental barrier
B. Deposits in bone
C. Excretes rapidly by kidney
D. Bacteria static
E. Produces extrinsic tooth stain
Ans is E

.415 Two conditions of enamel


facilitate post eruptive uptake of
fluoride element
A. Hyper mineralisation and
Surface dentine
B. Surface demineralisation and
hypo mineralisation
C. Dental fluorosis and enamel
Opacities

Ans is b

.416 5. Who is required to be


registerd with GDC?
• A) receptionist
• B) techincan
• C) cleaner
• D) manager
Ans is B

BY DR.ABDULRAHMAN ALMUALM
MJDF MCQS WITH ANSWERS

.417 According to European commission for guidelines on radiation protection in


dental radiology, lateral cephalometric radiograph is surely indicated in:
• A) class II/1
• B) class I
• C) class II/2
• D) orthognathic surgery
Ans is D

.418 The most common drug that causing ulcer resembling Squamous cell
carcinoma ulcer :
Ans is Nicorandil

.419 After chemotherapy pt is complaining of burning sensation in the mouth, what is


the most likely
Diagnosis
A) mucositis
• B) xerostomia
• C) anemia
Ans is A

.420 Pt are Complaining about the jaw pain two weeks after RCT, what is going be
your first management
• A) do RCT again
• B) reassure , sof diet and review in two weeks time
• C) soft splint
• D) remove the tooth
Ans is B

.421 How to know that non surgical periodontal therapy had effect?
• A) less than 15% bleeding on probing
• B) pockets getting bigger
• C) more than 15% bleeding on probing
• D)recession is controlled

Ans is A

422
BPE score is
324
3 2 2*
How to manage this case?
• A) refer to secondary care
• B) refer to GP
• C) refer to your colleague with special interest in perio
• D) refer to specialist in primary care
Ans is A
BPE 4 --> periodontist (secondary care)
BPE 3 --> GDP with special interest in perio

423BCC spread by

BY DR.ABDULRAHMAN ALMUALM
MJDF MCQS WITH ANSWERS

• A) locally
• B) lymphatically
• C) hematogenus
Ans is is A

.424 What is the most likely infection that can be caught by NSI in fully immunized
person? •
A)HepB
• B)HepC
C)HIV
Ans is B

.425 Patient had BMI=41. Which of the following is his ASA classification? •
A)ASAI
• B) ASA II
• C) ASA III
• D) ASA IV
Ans is C
BMI <30 --> ASA I (normal)
BMI 30-40 --> ASA II (mild systemic illness)
BMI >40 --> ASA III (severe systemic illness).

.426 Recent studies have shown that new drug therapy can avoid
osteoradionecrosis in the treatment of pt with bone metabolism disorder or cancer.
Which of the following does not cause MRONJ?
• A) alendronic acid
• B) calcitonin
• C) sunitamb
• D)denosumb
• E) zolendronic acid
Ans is B

.427 BPE score for the lower anterior teeth with pocket depth of 3mm with recession
of 3mm?
• A)0
• B)1
• C)2
• D)3
• E)4
Ans is A( recession part is not included )

.428 . Your associate has just found that he missed GDC renewal and not paid GDC
fee. What is your next action as his employer?
• A) suspend
• B) no action, it is associate’s problem
• C) inform the GDC
Ans is A suspend

BY DR.ABDULRAHMAN ALMUALM
MJDF MCQS WITH ANSWERS

.429 . A pt was referred to you for extraction of 5 by the ortho specialist in prepration
for ortho treatment. You mistakenly extracted the wrong tooth. What is the
consequence if you fail to report this incident?
• A) nothining if pt does not complain
• B) erasure from GDC
• C) criminal offence
• D) suspension
Ans is D

.430 Pt told the GP that she lost 4kg in last 4 months, have black stool, no
gastrointenstinal polyp, Lab results are; HB=80, low MCV, normal WBC, normal
palatal count.. What is your diagnosis?
• A) hypothyroidisam
• B ) B12 defficency
• C ) pernicious anemia
• D )chronic blood loss
Ans is D
Low MCV and MCHC with low Hb --> microcytic anaemia --> iron deficiency or
chronic blood loss
Dark stool is a sign of a GIT ulcer with bleeding.

.431 A83 year old pt with dementia comes to your practice with discharge pus from
parotide gland and does not have a family, also she does not have elected power of
attorney, who can give a consent for her?
• A ) her close relative
• B) her good friend
• C ) independent mental capacity advocate
• D) her GP
• E) her caregiver
Ans is C / best ans is do what is the best of interest of the patient

432. What is not found in the lining of Residual cyst:


• A) plasma cell
• B) stratified squamous epithelium
• C) stratified columnar epithelium
• D )fibrous tissue
• E ) cilliated columnar epithelium
Ans is E

433. You are going to employ clinical dental technician, which of the following
procedure you expected from him to do independently?
• A)diagnose and treatment planning of perio disease
• B) complete denture without the consultation with dentist (correct ans)
• C) administrate LA prior the ridge augmentation

BY DR.ABDULRAHMAN ALMUALM
MJDF MCQS WITH ANSWERS

.434You have given one dose of Prilocaine with


felipressin to the pt, but he is complaining of pain.
If we know that maximum dose is 300mg how many
of 2.2 ml ampules we can give to the same patient
safety?
• A)2.5
• B)5
• C)7
• D)11
Ans is B/

.435 A 11 year old girl with multiple caries, what would you advice to your
patient from following?
• A) use 5000ppm fluoride toothpaste
• B ) use 2800ppm F toothpaste
• C ) topical fluoride of 11600 ppm 2 or more times per year
Ans is B ,patient older than 15 ans is C

.436 4 years old girl comes with the mother, she has good hygiene and no caries,
living in area with fluoridated water, mother is asking about toothpaste and your
recommendation is:
• A) do not swallow toothpaste
• B ) use fluoride free toothpaste
• C) rinse with a water after brushing
• D) do not rinse after brushing, just spit all out and go to the bed
Ans is A (also D is right )

.437 Infection of which space can cause Stridor/


• A) massetreric
• B) sublingual
• C) submandibular
• D )retropharyngeal
Ans is D

.438 Definition of clinical governance – Framework through which NHS are


accountable to continuously improving the quality of their services and safeguarding
high standard of care.............etc. What does the statement indicates?
• A)CQC
• B) clinical audit
• C) clinical governance
• D) clinical operating services

BY DR.ABDULRAHMAN ALMUALM
MJDF MCQS WITH ANSWERS

Ans is C

.439 Pregnant pt complaining about swelling of her gums. Upon examination you
have noticed that is a pregnancy epulis, which of the following is correct?
• A) no treatment required, lesion will resolve by itself
• B) the presence of epulis is independent if pt oral hygiene
• C)surgical treatment mandatory after delivery
Ans is A

.440 A 6 years old child comes to you with fever, poor appetite, feeling of being
unwell, sore throat. 4 days later fever starts, painful rash, blister develop at the hand,
feet and buttocks. What is the causative agent?
• A) Herpes simplex
• B) paramyxovirus
• C) Coxackie virus
• D) HPV 16
Ans is C

.441 Kenedy class II classification, which of the following design is most suitable for
optimal function?
• A) distal rest and I bar
• B) occlusal approaching clasp
• C ) dental bar
• D) mesial rest and I bar
Ans is D

.442 A student did a study and found that her data skewed. Which of the following
data will be relevant/valuable in this scenario?
a. Mean, median
b. Median, upper and lower
c. Median, IQR
d. Mean and IQR ans is C (Skewed data --> non-normally distributed data -->
Median,and IQD so it's C.)
443(An OPG was given showing fracture line on the parasymphysis area)
What is the next radiograph will you order to further understand your findings?
a. SMV
b. OCM
c. PA Jaw
d. La Ceph
Ans is C PA mandible / SMV IS USED FOR ZYGOMA FRACTURE

444 Patient had a BMI of 41. Which of the following is her ASA Classification?
a. ASA I
b. ASA II
c. ASA III
d. ASA IV
Ans is C

BY DR.ABDULRAHMAN ALMUALM
MJDF MCQS WITH ANSWERS

.445. A couple came with heterozygous gene of FAM83H subtype of Amelogenesis


Impefecta. What is the percentage of their offspring to be born with Amelogenesis
Imperfecta?
a. 0%
b. 25%
c. 50%
d. 75%
e. 100%
Ans is D

.446. A patient came to you for GIC filling few months ago and now attend to you for
Stainless Steel Crown. Which of the following NHS Band can you claim?
a. GDP 21.60
b. GDP 59.10
c. GDP244.30
d. GDP299.90
ans is B

.447 Patient was diagnosed with cancer and she told her family it metastasis through
nerve. Which of the following is your diagnosis?
a. Adenoid Cystic Carcinoma
b. Pleomorphic Adenoma
c. Whartin’s Tumour
d. Sialolithiasis
e. Mucoepidermoid Carcinoma
Ans is A

448 You have been called in by the owner of the practice regarding you lack of
punctuality to work. Which of the following is the appropriate response?
a. My lack of punctuality does not affect patient flow
b. My lack of punctuality does not affect nurse staff flow
c. My lack of punctuality will be compensated with me staying longer after hours
d. My lack of punctuality can be better improved if the schedule was adjusted
ans is d

449What is the referral time for suspicious cancer lesion and for secondary care?
a. 1 week
b. 2 week, 20 weeks
c. 2 week, 18 weeks
Ans is c

450. 13. Patient had his history of epilepsy and presents to your practice with a
painful dental abscess. She is currently on Carbamazepine. Which of the following
antibiotic is contraindicated?
a. Amoxicilin
b. Erythromycin
c. Clindamycin
d. Metrodinazone
e. Tetracycline
Ans is B

BY DR.ABDULRAHMAN ALMUALM
MJDF MCQS WITH ANSWERS

.451. Your practice owner have just extracted a patients LL7. You were informed that
the patient has return a few visits for treatment of Dry socket. Today you see the
patient and decided to take another xray. The x-ray taken have shown the following
(an xray with fractured mandible w submerged tooth at the border of mandible)
What are your findings?
a. Fractured mandible
b. Fractured mandible with retained LL7
c. Displaced LL7
Ans is B

.452 . Patient request for his clinical records.


What is the protocol?
a. Prepare the records within 10 days, with
minimum fee of GDP10
b. Prepare the records within 40 days, with
minimum fee of GDP50
c. Prepare the records within 10 days, with
minimum fee of GDP10
d. Prepare the records within 40 days, with
minimum fee of GDP50
ans is 40 days and 10 GDP ,
453. A 7 year old patient presented with
multiple carious cervical lesion on the buccal
surface of her anterior. Which of the
following may be a possible cause?
a. Dry Fizzy Drinks
b. Aneroxia
c. Bullimia
d. Smoking Cannabis
Ans is A

454Which of the following is true regarding preparation of custom tray for


elastomeric impression:

A. Adhesive is preferred over perforation


B. Perforation provides adequate retention
C. Adhesive is applied immediately before procedure
D. Perforations are not made in the area over the prepared tooth
ANS B

. 455When restoring weakened cusps with dental amalgam you should


consider:
A. 2mm reduction while forming a flattened surface
B. 2mm reduction while following the original contour of the cusps
C. 4mm reduction while forming a flattened surface
D. 4mm reduction while following the original contour of the surface
Ans B

BY DR.ABDULRAHMAN ALMUALM
MJDF MCQS WITH ANSWERS

.456 Patient told her Gp she lost 3kg 4month have black stool with no
gastrointestinal polyp.
Lab results was:
- Low HB
- Low MCV
- Normal WBC
- Normal Platelet Count

Ans is Chronic blood loss

.457 Which of the following anesthetic agent do not require any anti-emetic?
a. Fentanyl
b. Ketamine
c. diamorphine
d. Pethidine

Ans is A

.458 Which of the following gland is richest in mucous secretion?


a. Submandibular
b. Sublingual
c. Parotid
d. Lacrimal
ans is A

.459 A student is carrying out a study on the effective use of antibiotics. Which of the
following do you would be relevant in this context?
a. Guardianship of antibiotics
b. Antibiotics resistance
c. Prophylactic antibiotics
ans is A

.460 With regards to burning mouth syndrome, which of the following is correct?
a. It disturbs sleep
b. It aggravates during eating
c. The intensity is bilateral
ans is B

.461 A 8 years old boy came to your dental clinic with the complain of impacted 11.
You manage to diagnose reason of delayed eruption of 11 and it was due to the
presence of a supernumerary teeth. Which of the following will not affect the eruption
of permanent 11.
a. Type of supernumerary
b. Number of supernumerary
c. Stage of root formation
ans is A ( number will affect but type ?? Any supernumery can affect the eruption of
secondary tooth)

462. You manage to complete a 11 year old orthodontic case. How long should you
keep her record?

BY DR.ABDULRAHMAN ALMUALM
MJDF MCQS WITH ANSWERS

a. 11 years
b. 14 years
c. 5 years
Ans is B( either 14 years or when patient reach to 25 years )

463 In the new NICE guidelines, antibiotic prophylaxis is no longer needed during
dental treatment for patients with risk of infective endocarditis. It was needed
previously due to the risk of which bacteria?
a. Streptococcus mutans
b. Streptococcus viridans
c. Staphylococcus Aureous (ans is B)
.464 1. Periodontal damage to abutment teeth of partial denture with
distal extension can best be avoided by
A. Applying Stressbreakers
B. Employing bar clasps on all abutment teeth
C. Maintaining tissue support of the distal extension
D. Clasping at least two teeth for each edentulous area
E. Maintaining the clasp arms on all abutment teeth at the ideal degree of
Tension

Ans is C / yes stress breakers are important but maintaining tissue support can
reduce forces over the abutment greatly

.465 Which of the following arrears CAN NOT be determined by


survey analysis of partially edentulous cast
A. Areas to be revealed as blocked out to properly loca?? Rigid parts of a
frame work
B. Areas to be shaped to properly loc?? Rigid parts of framework
C. Areas used for guideline planes
D. Areas used for retention
E. Areas used for support
F. Depth of rest seats
Ans is F

.466 What is the main purpose of using Stress breakers


A. To distribute the load between teeth and ridges
B. To distribute the load between the clasps and the face end of the
saddle
C. It relieves the abutment tooth of occlusal loads that may exceed their
physiologic strength
Ans is C

.467 What is the ideal length for a post in post-core in an endodontically


treated tooth:

A. 2/3 of the tooth length


B. ½ of the tooth length
C. 1.5 times that of the crown
D. Same as the anticipated crown
Ans is D

BY DR.ABDULRAHMAN ALMUALM
MJDF MCQS WITH ANSWERS

.468 17. In partial dentures the guidelines “Guiding Planes” serve to


A. Aids in balancing occlusion
B. Assure predictable clasp retention
C. Form right angle with the occlusal plane
D. Eliminate the necessity for precision attachment
E. Eliminate the necessity for a posterior clasp
Ans is B

.469 . You are going to employ clinical dental technician, which of the following
procedure you expected from him to do independently?
• A)diagnose and treatment planning of perio disease
• B) complete denture without the consultation with dentist
• C) administrate LA prior the ridge augmentation
Ans is B

.470 Oil or water on impression for treatment casts causes


A. An increase of the quality
B. No alteration
C. A decrease of the quality
D. Bubbles on the cast
E. None of the above
Ans is C ?

.471What is Path of Insertion


A. The movement of the appliance from the points of initial contacts to
path of final rest position
B. The movement of the appliance from the points of rest position until it
is not in contact with teeth

Ans is A

.472 5. Which of the following is a major disadvantage to immediate


complete denture therapy
A. Trauma to extraction site
B. Increased the potential of infection
C. Impossibility for anterior try in
D. Excessive resorption of residual ridg
Ans is C

BY DR.ABDULRAHMAN ALMUALM
MJDF MCQS WITH ANSWERS

.473 Fovea Palatini, is a landmark to


determine the posterior
bonds of upper denture
A. Post dam
B. Flanges
Ans is A

.474 . Gold clasp is more elastic than Cobalt


Chrome, but Co-
Chrome has high modulus of elasticity
A. The first statement is false the second is true
B. Both are true
C. The first is true the second is false
D. Both are false
Ans is B

.475 The position of cusps of maxillary first premolar during


setting of teeth and on occlusal view is positioned**
A. Distally
B. Mesially
C. Central buccolingually
Ans is B

.476 Antimicrobials should be prescribed for one of the following clinical scenarios:

a) After apicectomies
b) If there is a sinus present
c) After periodontal surgery
d) For surgical removal of mandibular third molars
e) After surgical removal of retained roots and teeth where there is a history of dry
sockets
ans is non of the above ..according to recent guidelines no antibiotic should
prescribed unless signs of systemic affection

.477 vicryl rapide loss 50% of tensile strength in how


many days?
A) 7 days.
B) 10 days.
C) 15 days.
D) 30 days.
Ans is A

.478)membrane needed during bone graft?


A)fibroblast not enter
B)fibroblast not escape
C)osteoclast not escape
D)osteoclast not enter
Ans is A

479Absolute contraindication in implant?

BY DR.ABDULRAHMAN ALMUALM
MJDF MCQS WITH ANSWERS

A)Active periodontitis
B)smoking
C)warfarin
Ans is A

.480 L)The most common cause of absent upper


central incisor with no history of trauma:
A)primary failure of eruption
B)supernumeray tooth
C)dilaceration
D)cyst
Ans is A

481)suitable time to extract first molar ?due to long term bad prognosis or
orthodontics
A)8_10 y
B)10_12 y
C)12_13 y
D)13_14y
Ans A

482)pt complain of swelling,crepitus,bony specule ,osteophyte


mostly ?
A)osteoarthritis
B)rehumatoid arthritis
Ans is A

483)after u give IAN block..pt feel numbness..but u are


not able to work because pt complain of pain what u
will do next?
A)intraligmentary injection
B)intrapulpal anaesthesia
Ans is A || in case of RCT = INTRAPULPAL INJ ||in case of extraction
intraligamentry

.484 on sialography u note area of stenosis and


diltation of parotid duct?
A)Obstructive parotitis
B)Sialolith
C)Sialadenitis
Ans is A

485) Metal framework in maryland bridge?


A)nickel chromium
B)cobalt chromium

BY DR.ABDULRAHMAN ALMUALM
MJDF MCQS WITH ANSWERS

Ans is A

486best criteria of NITI endodontic file?(flexibility


not in option)
A)shape memory
B)corrosion resistance
C)Durability
D)biocompatibility
Ans is A

487Patient with good oral hygiene low


caries risk you don't need any ttt recall for
?

A)6 months.
B)12 months
C)18 months
Ans C

488 Lateral Cephalometric radiograph :


A)Immediately proir to orthognathic
surgery
B)Used in IOTN more than 2
C)Detect unerupted upper incisor
D)Routinely used in orthodontic
assesment
Ans is A || in case of IOTN 4 OR 5
LATERAL cephalometric is important

.489) patient is unconsious,,, After shouting for help what


will u do?
A)start CPR
B)look, listen, feel breathing
C)chin lift
Ans is C ■ first check responsiveness, chin lift , listen feel breath if not start CPR

490radiopacity with radiolucent surround in lower


mandible adjacent to LL7... the LL8 is missing..?
A)cementoma
B)residual cyst
C)ameloblastoma
D)calcifying epithelial odontogenic cyst
Ans is A ( some selected D depending on the radiograph .if it was stick to the root its
cementoma .if it was away and on mandible angle it’s most likely CEOC

BY DR.ABDULRAHMAN ALMUALM
MJDF MCQS WITH ANSWERS

491)chrom cobalt rest seat preperation occlusaly?


A)1 mm
B)1.5 mm
C)2 mm
Ans is B

492)during u doing surgical extraction of lower 8


there was massive bleeding ,, extraction was
difficult ,..pt came to u with numbness in lower
lip & tongue nerve damge after surgical extraction
what u do?
A)reassurance and review 6 months later
B)refer to OMFs
C)prescribe dexamethadone & review later
Ans is B

493 What is the contraindications for taking MRI?


A)Allergic to contrast material
B)Cardiac pacemaker
C)Pt on ortho tx
D)Pt with dental implants
Ans is B

494Which one of the following would not be considered a plaque retentive


factor?
a) Subgingival calculus
b) Well fitting crown
c) Severe imbrication of the teeth
d) Supragingival calculus
e) Fixed orthodontic retention

Ans b

495A patient collapses in the dental practice surgery waiting room. They have
been fitting for 5 minutes, what is your next course of action?

a) Buccal midazolam
b) Chest compressions
c) Intramuscular midazolam
d) Rectal diazepam
e) Two rescue bbreath
Ans is A

496What is the reversal agent for a paracetamol overdose?


a) Alkaline diuresis

BY DR.ABDULRAHMAN ALMUALM
MJDF MCQS WITH ANSWERS

b) Atropine
c) Methionine
d) N-acetylcysteine
e) Naloxone
Ans is D

497Which clotting factors are inhibited by warfarin?


a) II, III, VII and X
b) II, VII, IX and X
c) II, VII, IX and XI
d) III, VII, IX and X
e) VII, IX, X and XI
ans is B

498Which one of the following is the greatest risk factor for periodontal
disease?
a) Plaque retentive factors
b) Down’s syndrome
c) Shortened dental arch
d) Smoking
e) Intravenous drug abuse
Ans is D

499Which one of the following drugs should not be given to asthmatic


patients? a) Metronidazole
b) Amoxicillin
c) Ibuprofen
d) Captopril
e) Paracetamol
Ans is C

500What is the correct dose of aspirin for prophylaxis of cardiovascular


events?
a) 7.5 mg
b) 75 mg
c) 150 mg
d) 300 mg
e) 750 mg
ans is B

501Which topically applied antibiotic can be applied to facial wounds to


prevent infection and reduce scarring?
a) Pencillin
b) Erythromycin
c) Chloramphenicol
d) Metronidazole
e) Fluconazole
ans is C

BY DR.ABDULRAHMAN ALMUALM
MJDF MCQS WITH ANSWERS

502Which one of the following interacts with warfarin to decrease the patient’s
INR?
a) Fluconazole
b) Vitamin K
c) Metronidazole
d) Erythromycin
e) Aspirin
Ans is B

.503 You are doing endo treatment for a diabetic patient. At which blood glucose
level you defer your treatment?

A. >7 mmol
B. >15MOL or <5mmol
C. <6mmol
D. >6 MOL
E. >20 MOL
Ans A

504IOPA showing nerve close to the third molar, What would be the next Xray you’ll
order?

A. Small beam CBCT


B. Large beam CBCT
C. OPG
Ans A

505Patient complaining of loose upper complete denture and you suspect a


remaining root underneath, the first choice of radiograph

A. Panoramic
B. Periapical
C. Bitewing
D. Occlusal
Ans B

506In the absence of oral hygiene measures, gingival inflammation is


detectable at:
a) 2 h
b) 24 h
c) 48 h
d) 2 weeks
e) 2 months
ans C

507Which one of the following statements about the periochip is correct?


a) The periochip is used in the treatment of periodontitis
b) The periochip is completely synthetic
c) Use is confined to pockets less than 5 mm
d) Is associated with an increase in pocket probing depths

BY DR.ABDULRAHMAN ALMUALM
MJDF MCQS WITH ANSWERS

e) Is an topical antibiotics
ans is A : periochip.is consist of gelatin within it chlorhexidine gluconate 2.5mg

508Which one of the following is associated with the greatest increase in risk
of periodontal disease?
a) Malocclusion
b) Pregnancy
c) Poor diet
d) Type 1 diabetes mellitus

Ans d

509Chlorhexidine is a chemical antiseptic. Which one of the following


statements is false?
a) It is most effective immediately after the use of toothpaste
b) Can be present in gels and rinses
c) Its mechanism of action is membrane disruption
d) Is bactericidal as well as bacteriostatic e) Activity is reduced in the presence
of blood, soap and pus
Ans is A

510Which one of the following statements regarding the platform switching


concept in implant dentistry is FALSE?
a) The platform switching concept uses an increased diameter of abutment
compared to the implant size
b) The platform switching concept uses a reduced diameter of abutment
compared to the implant size
c) The platform switching concept is associated with a reduction in crestal bone
resorption
d) The platform switching concept can be used in both submerged and
unsubmerged surgical techniques
e) The platform switching concept provides a horizontal platform to offset the
biological width surrounding a dental implant
Ans A

511Which one of the following statements regarding bisphosphonates is true?


a) The use of oral bisphosphonates contraindicates the use of dental
implants
b) Bisphosphonates increase the osteoblastic activity
c) Bisphosphonates can only be given intravenously
d) Bisphosphonates are contraindicated in osteoporosis
e) Bisphosphonates encourage osteoclasts to undergo apoptosis
Ans E

512Which one of the following statements regarding Aggregatibacter


actinomycetemcomitans (AA) is false?
a) AA is a gram negative non motile rod

BY DR.ABDULRAHMAN ALMUALM
MJDF MCQS WITH ANSWERS

b) AA is a leucotoxin killing polymorphonuclear leukocytes (PMNs) and


monocytes
c) AA does not invade tissues
d) AA is particularly associated with the onset of localized aggressive
periodontitis
e) AA may be present as part of the biofilm in periodontally involved teeth
Ans C
non motile anaerobic facultative

513Which one of the following is the correct definition for a xenograft?


a) Tissue transplanted from the same individual, same site b) Tissue
transplanted from the same individual, different site .
c) Tissue transplanted from a different individual, same species
d) Tissue transplanted from a different species
e) Synthetically constructed tissue
ans is D

514Which one of the following statements about the Florida probe is


incorrect?
a) Reduces inter- and intra-operator error by using a constant force
b) Records the pocket depth and not attachment loss
c) The tip is placed at the bottom of the sulcus d) The sleeve is positioned at
the gingival margin
e) The correct measurement is visually assessed
ANs E / digital measurement

515Buccal recession associated with implants is most commonly associated


with which one of the following?
a) Greater than 2mm of buccal plate
b) Immediately placed implants
c) Thick gingival biotype
d) Platform switched implants
e) All ceramic restorations
Ans B

516Which one of the following is the most reliable indicator of active


periodontal disease?
a) Bleeding on probing
b) Presence of subgingival calculus
c) Probing depths of over 7 mm
d) DNA presence of periodontal pathogens e) Plaque
ans A

517year-25 old man attends your clinic who has not been seen by a dentist
for the last 10 years. He is complaining of mobile upper central incisors.
These have probing depths of 9 mm and are bleeding on probing. There is
bleeding on probing in all other sextants but no probing depths above 3mm.
Which one of the following is the most likely diagnosis?
a) Juvenile periodontitis
b) Refractory periodontitis

BY DR.ABDULRAHMAN ALMUALM
MJDF MCQS WITH ANSWERS

c) Localized aggressive periodontitis


d) Generalized periodontitis
e) Necrotising ulcerative periodontitis
Ans C

518Which one of the following treatments would not be considered to be in


the initial phase of perimplantitis treatment?
a) Occlusal evaluation and equilibration
b) Anti-smoking advice
c) Subgingival irrigation with 0.12% chlorhexidine
d) Non-surgical debridement
e) Surgical debridement
Ans E

519Which one of the following statements is incorrect regarding surgical


regenerative treatment of peri-implantitis lesions?
a) A three-walled defect is more successfully treated than a single walled defect
b) The use of bone regenerative materials without barrier membranes show
good clinical improvement
c) Impeccable oral hygiene is a prerequisite for successful treatment d) Should
follow a course of non-surgical treatment
e) The use of bone regenerative materials with barrier membranes show good
clinical improvements
ans B

520Which one of the following examinations may be used as a screen to


identify patients for further investigations?
a) Community Periodontal Index of Treatment Need (CPITN)
b) 6-point pocket chart
c) Gingival index
d) Plaque index
e) Bleeding index
Ans A

521Which one of the following is the correct pressure applied when using the
BPE?
a) 5g
b) 10-15g
c) 20-25g
d) 30g
e) 35g
Ans C
.522
●When performing a Basic Periodontal Examination (BPE) for a new patient,
you record the following scores:
3/1/3
3/1/3
● As this is a new patient, you then decide that radiographs are indicated.
Which are the most appropriate radiographs to prescribe for this patient?
a) Bitewings (horizontal or vertical)

BY DR.ABDULRAHMAN ALMUALM
MJDF MCQS WITH ANSWERS

b) Intraoral periapical radiographs (IOPAs) of upper and lower posterior teeth


c) Lateral cephalogram
d) OPG (with no condyles)
e) Upper anterior occlusal
Ans is A

523Which one of the following associated with dentifrices does not offer
advantages in controlling gingivitis?
a) Amine fluoride/stannous fluoride
b) Chlorhexidine
c) Fluoride
d) Stannous fluoride
e) Triclosan
Ans c

524Which of the following statements about the association between diabetes


mellitus and periodontal status is correct?

a) Diabetic patients who control their condition by diet alone are still likely to
develop advanced periodontal destruction
b) Poorly controlled diabetic patients experience a similar degree of periodontal
destruction to non-diabetic patients
c) Responsiveness to periodontal treatment is not correlated with overall diabetic
control
d) There is no difference in neutrophil function between diabetic and non-
diabetic patients
e) There is evidence to suggest the existence of a bidirectional association
between diabetes mellitus and periodontal disease (ie where glycaemic control
can affect periodontal destruction, and the level of periodontal inflammation can
affect glycaemic control)
ans E

525Which one of the following is classed as a code 2 on the Silness and Loe
plaque index?
a) Film of plaque visible only by removal on probe or by disclosing
b) Heavy accumulation of soft material filling the niche between the gingival
margin and the tooth surface; the interdental region is filled with debris
c) Moderate accumulation of plaque that can be seen with the naked eye
d) Soft debris covering more than one-third but not more than two-thirds of
the tooth surface e) Soft debris covering not more than one-third of the tooth
surface

Ans C
0 --> no plaque
1 --> thin film of plaque that cannot be visible by naked eyes and can be visible by
disclosing agents.
2 --> moderate accummulation.
3 --> heavy

BY DR.ABDULRAHMAN ALMUALM
MJDF MCQS WITH ANSWERS

526What treatment is indicated from the following basic periodontal


examination (BPE)?
313
223
a) Oral hygiene instruction with supragingival scale
b) Oral hygiene instruction, 6 point pocket chart, supra and subgingival scaling
c) Supragingival scaling
d) Vertical bitewings, oral hygiene instruction, 6 point pocket chart
e) 6 point pocket chart, supra and subgingival scale
527Whats the first line drug which can be used as an alternate in pencillin allergic
patients ?
A) doxycycline
B) erythromycin
C) clindamycin
Ans C

528Which pin system has proven to be the most retentive

A. Self tapping threaded pin


B. Friction peak pin
C. Cemented pin
Ans is A

529●54-year-old man attends for an examination. He is a smoker. Intra-orally


calculus deposits are present. Radiographic examination reveals approximately
70% bone loss affecting his lower molar teeth, and approximately 20% bone
loss elsewhere throughout his dentition. What is his diagnosis?

a) Chronic generalised mild periodontitis, with localised advanced areas around


the lower molars
b) Chronic moderate periodontitis, with localised moderate periodontitis around
the lower molars
c) Generalised chronic periodontitis, with localised aggressive periodontitis
around the lower molars
d) Localised aggressive periodontitis around the lower molars
e) Necrotising ulcerative gingivitis
ans is A

BY DR.ABDULRAHMAN ALMUALM
MJDF MCQS WITH ANSWERS

530Which one of the following is associated with periodontal disease in the


permanent dentition?
a) Apert syndrome
b) Chediak–Higashi syndrome
c) Gorlin–Goltz syndrome
d) Papillon–Levefre syndrome
e) Ramsay Hunt syndrome
Ans B

531A 62-year-old diabetic man who was started on diclofenac for post-
operative pain develops abdominal pain and dark stools. On examination, he
has melaena. By what mechanism has this drug caused his symptoms?
a) Type-1 hypersensitivity reaction
b) Cytotoxicity
c) Induction of cyclo-oxygenase pathway
d) Inhibition of PGE2 and PGI2 action of
mucosae
e) Induction of thromboxane
Ans is D

532A 56-year-old woman attends for a


routine appointment. She tells you she is
undergoing chemotherapy for metastatic
breast carcinoma and has noticed white
patches on her tongue and inside of her
mouth for the last 4 days. She does not
complain of a sore throat and is otherwise well in herself. Which would be
the best therapeutic agent?
a) Co-amoxiclav
b) Flucloxacillin
c) Fluconazole
d) Caspofungin
e) Aciclovir
ans C

533A 70-year-old gentleman attends for an emergency appointment


complaining of severe, throbbing toothache. He says he cannot eat on the
affected tooth and on examination, you see a periapical abscess. You decide
to drain the pus away but want to cover this gentleman with antibiotics as the
surrounding gum is very inflamed. On questioning, he tells you that when he
last had penicillin he has an anaphylactic reaction and needed intensive care
unit (ITU) treatment. Which would be the most appropriate antibiotic to use in
this case?
a) Co-amoxiclav
b) Metronidazole
c) Amoxicillin and metronidazole
d) Flucloxacillin
e) Clindamycin
ans is E??

BY DR.ABDULRAHMAN ALMUALM
MJDF MCQS WITH ANSWERS

534□55-year-old obese, Asian gentleman complains of increasingly severe


central, crushing chest pain and shortness of breath while he sits in the
waiting room. He tells you that he had a heart attack 1 year ago and this
feels very similar and that he took his 75 mg aspirin this morning but missed
his cholesterol lowering tablet last night. You suspect this gentleman is having
a myocardial infarction (MI) and call an ambulance. Which of the following
medications are known to decrease mortality during an acute MI?

a) Aspirin 300 mg
b) Aspirin 75 mg
c) Simvastatin 40 mg
d) Warfarin 5 mg
e) Dipyridamole 200 mg
Ans A ||MONA = Morphine, Oxygen ,nitroglycerin, Aspirin

□535Oral steroids are often used in the treatment of chronic illnesses such as
chronic obstructive airways disease and rheumatological conditions. Which of
the following concurrent illnesses can be exacerbated by steroids?
a) Asthma
b) Diabetes
c) Psoriasis.
d) Inflammatory bowel disease
e) Systemic lupus erythaematous
ans is B steroids increase glucose level

536You are suturing a laceration on an 81 kg man under local anaesthesia.


You have lidocaine available without adrenaline. What is the maximum safe
dose you can use in this patient subcutaneously?
a) 81 mg lidocaine
b) 162 mg lidocaine
c) 243 mg lidocaine
d) 324 mg lidocaine
e) 567 mg lidocaine
Ans is D (According to 4.4mg/kg but in pastest it was answered as C 3mg /kg)

537diabetic patient on your operating list for dental extraction under local
anaesthesia complains of feeling sweaty and unwell. He has been mistakenly
instructed to fast overnight and took his evening insulin as usual. You ask the
nurse to check the blood sugar, it reads as 1.8 mmol. If the patient was to
become unconscious, what is the most appropriate immediate pharmacological
option?
a) Hypostop
b) Administer a sugary drink
c) 10 ml of 50% dextrose solution
d) 1 mg glucagon iv
e) 1 mg glucagon im
ANS E (case if he was unconscious what will be the treatment )

BY DR.ABDULRAHMAN ALMUALM
MJDF MCQS WITH ANSWERS

538In the assessment of a patient with severe anaphylactic shock, which part
of assessment is most crucially affected by the administration of intramuscular
adrenaline?
a) Airway
b) Breathing
c) Circulation.
d) Disability
e) Exposure
ans is A

539During a cardiac arrest, you are asked to administer adrenaline


intravenously between the second and third cycle of CPR. What is the correct
IV dose?

a) 0.5 mg 1:1000 adrenaline


b) 1 mg 1:10 000 adrenaline
c) 0.5 mg 1:10 000 adrenaline
d) 5 mg 1:10 000 adrenaline
e) 50 mg 1:10 000 adrenaline
ans B

540Which of the following systems is affected by local anaesthetic toxicity


earliest?
a) Cardiovascular
b) Respiratory
c) Neurological
d) Endocrine
e) Gastrointestinal
ans C

541You are taking a medical history from a 55-year-old woman who tells you
she is on ramipril. What class of drug is ramipril?
a) B-blocker
b) Thiazide diuretic
c) Loop diuretic
d) Angiotensin II blocker
e) Angiotensin-converting enzyme (ACE) inhibitor
Ans E

542Sulphonamides are synergistic bacteriostatic agents because in bacteria


they:
a) Interfere with folinic acid production

b) Inhibit RNA synthesis c) Inhibit DNA synthesis


d) Inhibit cell wall synthesis
e) Interfere with folate synthesis
Ans A

543Which one of the following drugs should not be given to asthmatic


patients? a) Metronidazole

BY DR.ABDULRAHMAN ALMUALM
MJDF MCQS WITH ANSWERS

b) Amoxicillin
c) Ibuprofen
d) Captopril
e) Paracetamol
Ans C

545 Which one of the following commonly prescribed antibiotics produces a


disulfiramlike reaction on ingestion of alcohol?
a) Amoxicillin
b) Erythromycin
c) Clindamycin
d) Metronidazole
e) Chloramphenicol
Ans D

546The gingivae of child is diagnosed on the basis of all of these except of


A. Contour of gingival papilla
B. Sulcus depth
C. Contour of Nasmyth
membrane
D. Tight filling of gingival collar
Ans C

547Brown skin pigmentation does


not occur in
A. Hyperparathyroidism
B. Von Willebrand’s syndrome
Ans A

548In line with the mental capacity


act, a person is regarded as being
unable to give
consent or make decisions if he is
unable to do [4 steps to assess the mental capacityl
a) Understand, retain, weight and communicate the information
b) Understand, retain, weight and repeat the information
c)Understand, retain, weight and write the information
d Understand, retain, weight the information and write his name
e) Understand, retain, weight the information and state his address
Ans A

549cleido-cranial dysplasia is not associated with [in the head and neck]:
a) Supernumerary teeth
b) delayed closure of fontanels-hypodontia
c)prominent clavicles
d) posing of forehead
Ans C

BY DR.ABDULRAHMAN ALMUALM
MJDF MCQS WITH ANSWERS

550--35 y old women came with fracture


mandible ,,change occlusion,, from her partner
while was drunk?
A) Advice her to inform police
B) Keep the infomation confidential
C)Tell her GP
D)social services
Ans D

551What is the least probable consequence in thumb-sucking?


A.Reclining of lower incisors
B.Protrusion of upper incisors
C.Formation of deep palate with big overbite
Ans is A

552. The main purpose of periodontal treatment is,


A-Elimination of plaque and calculus
B-Elimination of periodontal pockets
C-Reformation of all the periodontal ligaments
D-The elimination of all occlusal trauma

Ans A

553you are examining a patient whom u suspect as a cracked tooth.


which of the following signs & symptoms and diagnostic test help
confirm ur diagnosis
a)pt has pain when they bite on something
b)pt has pain when they release their bite
c)tooth is tender to percussion
d)applying orthodontic band to tooth will result in reduction of the pain
e)transillumunation shows that the light travels thru the toot
Ans B

554The use of latex gloves does has the following effect when a polyvinyl
siloxaneimpression is taken
a. retards the set of the impression material
b. enhances the set of the impression material
c. results in porosities in the impression material
d. latex gloves stick to the polyvinyl siloxane impression material
ans A

555Attached gingiva is the thinnest in the mouth in the following region of


themouth?
a. Maxillary anterior buccal region
b. Maxillary molar buccal region
c. Mandibular anterior buccal region
d. Mandibular anterior lingual region

BY DR.ABDULRAHMAN ALMUALM
MJDF MCQS WITH ANSWERS

e. Mandibular molar lingual region

556Afteryou had finished scrubbing, glove and other PPE, you accidentlytouch the
chair. What
would bethe appropriate management?
1. Remove glove, wash again and put glove on
2. Remove glove but no need to wash because didn’t touch the
patient yet
3. Remove all PPE, wash again and wear PPE again
4. Remove glove, alcohol gel and glove on
5. Carry out the procedure because it isn’t infectious
Ans is 1 / A

557If your patient has history of Haemophilia father and normal


mother, risk of your patient who is
male is?
.1 100%
.2 0%
%3.25
.4 50%
.5 75%
Ans is 2

55821 yrs old female patient who give you the history of
autoimmune disease which turn out to be
Pernicious anemia. What you think would be primarily deficiency?

.1 Intrinsic factor
.2 Iron
.3 Vitamin B12
.4 Folate
.5 Vitamin D
Ans 1

55935 yrs old athlete who come to see you in your


clinic. You took an x-ray and see Root fracture
with no other pathology. How long you will splint this tooth?
.1 Flexible splint 4 wks
.2 Flexible splint 6 wks
.3 Hard splint 4 wks
.4 Hard splint 6 wks
.5 Flexible splint 2 wk

Ans 1

560yrs old patient complaint of pain in his mouth.


On examination, you see Erythematous lesion
on uvula, palate & fauces but not on gingiva. What would be
your probable diagnosis?
.1Apthous ulcer

BY DR.ABDULRAHMAN ALMUALM
MJDF MCQS WITH ANSWERS

.2Herpes stomatitis
.3Herpangina
.4Hand foot & mouth disease

Ans is 3

561Patient with Cleidocranial dysostosis consult you for his implant


placement. When you took X-
ray, what would you expect to find?
.1 Dentigerous cyst
.2 Supernumerary teeth
.3 Dermoid cyst
.4 Osteoma
.5 cementosis

Ans is 2

.562 While you are doing RCT on 21 yrs old female


patient, suddenly her respiratory rate increase,
dizzy, sweating and tremors. What is the mechanisms behind
this process?
1.decrease calcium, respiratory alkalosis and metabolic acidosis
.2 decrease calcium, respiratory acidosis, metabolic alkalosis

.3 Increase oxygen, respiratory acidosis and metabolic


alkalosis
.4 increase calcium respiratory acidosis and metabolic alkalosis

.5 decrease carbon dioxide, respiratory alkalosis, metabolic


acidosis

Ans is 5

563TMN staging for CT of the patient with 3 cm


in diameter tumor, nodal involvement of single 4
cm in diameter same side as the tumor and no distant
metasis noted.
1.T1 N1 M0
2. T2 N2a Mo
3. T2 N2b Mo
4. T2 N2c Mo
5. T2 N1 Mo
Ans 2

BY DR.ABDULRAHMAN ALMUALM
MJDF MCQS WITH ANSWERS

564. An autistic pt comes to your clinic for follow up dental treatment, on his previous
dental visit another doctor prescribed fissure sealant, mother is nervous and called
the clinic asking about the procedure for her autistic son. What would you provide?
• A) non invasive treatment
• B) prophylactic treatment
• C )filling
• D) familiarization visit
Ans D

565A 58 year old female patient complain of the pain in TMJ area, OPG shows flat
condylar procecus, pain is more intense in the morning, no muscle tenderness, what
would be diagnosis?
• A) osteoporosis
• B)reumathoid artritis
• C) ostheoartritis
• D )anteriror disc displacement
• E)posterior disc displacement
Ans c

566. Reason for failure of canine eruption without early loss of deciduous teeth?
• A) supernumary
• B) ectopic
• C) crypt
• D)supplementary
• E) primary failure of eruption
Ans B

567Manifestation of Coelic disease in the mouth:


• A) mucosal tags
• B) pigmentation
• C) attrition
• D )abrasion
• E) hypodontia
Ans A
568. Picture of lower central incisors, fractured on the gum level, there is no
pathological changes in
the bone but pt is on biposphonates for 5 years..
• A)remove the teeth
• B)leave as it is
• C) provide an overdenture on the state how it is now
• D) provide an overdenture after you did RCT
Ans D

569What is the level of evidence for expert opinion


• A)1
• B)2b •
C)5
• D)4
Ans C

BY DR.ABDULRAHMAN ALMUALM
MJDF MCQS WITH ANSWERS

570PA radiograph was given which was not that good but
acceptable. According to guideline which
one of the following best describe
this situation.
1. 10% cone cutting
2. 10% elongation
3. 20% elongation
4. 30% elongation
5.40% cone cutting
Ans 1

571Which of the following has highest five year survival rate for class V restoration?
• A. Amalgam
• B. Bonded amalgam
• C. Composite (flowable)
• D. Conventional GIC
• E. Resin modified GIC
And e

572You had 8 years old female patient now, till how


long u need to keep her record?
1. 5yrs
2. 10 yrs
3.8yrs
4. 17yrs
5. 11yrs
Ans 4

57321 yrs old male patient who give family history of X-linked
amelogenesis Imperfecta of his
father. What is the likely chance of him having this disease?

1.25%
2.75%
3.50%
4.0%
5.100%
Ans 4

574
Which of the following is NOT tax deductible expenses?
• A. Conference fee
• B. Critical illness cover
• C. GDC annual retention fee
• D. Indemnity/ insurance
• E. Surgery clothing

BY DR.ABDULRAHMAN ALMUALM
MJDF MCQS WITH ANSWERS

Ans B

575. Which of the following is included in NHS?


• A. Right to choose named consultant
• B. Legally enforceable rights
• C. Right to access drugs and treatment recommended by doctor regardless of cost
• D. Right to have complaints properly investigated and deal with
• E. Right to have the required treatment/ service locally
Ans is D

576Which probe do you use for detailed periodontal examination?


• A. CPITN probe
• B. Dental probe No 6
• C. Nabers probe
• D. University of North Carolina probe
• E. Williams 14 W probe
Ans E ??

577What is the minimum Factor VIII level required to avoid significant post extraction
haemorrhage in Haemophilic patient?
• A. 30%
• B. 50%
• C. 60%
• D. 70%
• E. 75%
Ans is A

578What is the value of gift that must be recorded and kept at dental practice?
• A. All gifts regardless of value
• B. No obligation to record
• C.>20GDP
• D.>50GDP
• E. >100 GDP
Ans is E

579.Which of the following most likely to present at an advanced T stage after


squamous cell carcinoma of Head and neck?
• Buccal mucosa
• Lateral of tongue
• Hard palate
• Lip
• Maxillary antrum
ANs E

580Patient has generalized probing 5-8mm, bone loss 30%-80%. What is the stage
and grade of his periodontitis?
• Stage III Grade B
• Stage III Grade C

BY DR.ABDULRAHMAN ALMUALM
MJDF MCQS WITH ANSWERS

• Stage IV Grade A
• Stage IV Grade B
• Stage IV Grade C
Ans is E (stage lV grade c )

581. Pharmaceutical company is doing randomized control trail in


two groups for drug. What would
be the appropriate test to get lethal dose in both groups?
1. chi-square test
2. pair t-test
3. Unpaired t-test
Ans 3

582. Basal cell carcinoma – spread?


1.local
2.perineural
3.blood vessels
Ans 1

583
28 yrs old female patient comes with pain in lower jaw
and while you took x-ray, finds
fractured mandible. Then she tells that her partner hits her
when he drunk. She has capacity.
What would you do?
1. Report to police
2. Safeguarding to social service
3. Respect her confidentiality
4. Inform GP
5. Advise her to consult
Ans is 2
If the patient has the capacity --> adviser to consult safeguarding social services.
If the patient has no capacity (Minor, <16 years old) --> consult safeguarding social
services after taking consent from the patient's parents. If you suspect that the child's
life might in danger or sexual abuse then you have to contact safeguarding social
services immediately.

584. The main purpose of finishing the enamel walls is,


A-Remove loose enamel rods
B-Provide a better surface for the adoption of restorative material

Ans A

585. The initial priority in treatment of horizontal fractures is:


Preservation of pulp
A-Immobilisation
B-Root canal treatment
C-Calcium hydroxide treatment

BY DR.ABDULRAHMAN ALMUALM
MJDF MCQS WITH ANSWERS

Ans A

586. Which of the following? is classed as pharmacy medicine


a) Fluoride mouthwash 2% (En-de-Kay Fluorinse
b) Corsodyl mouthwash 0.12%
C) Fluoride mouthwash 0.05% (Fluorigard
D) Toothpaste containing < 1500 ppm fluoride
Ans is A

587. NSAIDs do not interact with


a) Phenytoin
b) Antihypertensive drugs
c) Methotrexate
d) Systemic Corticosteroids
d) Fluoxetine
Ans is A

588. Which antibiotic associated with infectious mononucleosis or glandular fever


a) Penicillin
B) Ampicillin
C) Tetracycline
ans B

589. What is the flavoured topical anasthesia


a) Lidocain wth adrenaline
b) Prilocain wth fellypressen
c) Bupivacaine
d) Articaine
e) Benzocaine
ans E

590) Maximum Percentage of NO come out from Machine during Sedation is:
a) 10%
b) 40%
c) 50%
d) 30%
e) 70%
ans is e

591) The artificial teeth in rpd are made of?


a) Porcelain
b) cross linked Methyl-Methacrylate
c) Ethyl ethacrylate
d) Acrylic
Ans D

BY DR.ABDULRAHMAN ALMUALM
MJDF MCQS WITH ANSWERS

592.Which one can not a dentist alter?


a) Incisal angle
b) Condylar angle
c) Condylar guidance
Ans is C

593. 91) Best cement for minimal preparation metal crown?


a) GIC
b) Zinc Phodphate
c) RMGIC
d) Zinc polucarboxylate
e) Resin Cement
Ans E

594. Which of following score will indicate asterix*


a) Recession of 6mm
b) 4mm recession plus 3mm pocket
c) 12mm pocket

Ans is c???12 mm will.lead to expose furcal ?

Asterix is furcal involvement

595. While you were trying to remove UR6, one of the roots of UR6 fractured. You
were trying to remove
the retained root, however it suddenly disappears and was displaced into the
maxillary sinus. What is
your first management?
a. Antibiotics and review
b. Refer to the oral surgeon department
c. Attempt removal

Ans B ??

596. (An X ray showing a UL8 being displaced into the maxillary sinus)
While you are performing a UL8 removal, the tooth was displaced and went into the
maxillary sinus. And
a Posterior anterior radiograph was taken. Which x ray would you take next?
a. CBCT
b. CT
c. Upper occlusal
d. Periapical
e. OPG

597. Occasional sensitivity in a shallow class I amalgam restoration after two


days would be managed by:

A. Replace old filing immediately


B. Oxide Zinc and eugenol
C. Using thicker mix of cements

BY DR.ABDULRAHMAN ALMUALM
MJDF MCQS WITH ANSWERS

D. Tell patient the discomfort will disappear after 4 t o6 weeks


E. Ledermix

Ans D

598On examination of a composite restoration you find a dark attain:

A. Replace the composite


B. Repair with unfilled resin
C. Apply topical fluoride at the margins
Ans A

599Which is NOT characteristics of canal filing materials “obturation material”


A. Tacky adhesive to walls
B. Radio opaque
C. Not irritating
D. Quick in setting
Ans D

600Full mouth x ray survey at


birth reveals:
A. Ten teeth are present
B. Twenty teeth are present
C. Twenty four teeth are
present
D. Twelve teeth are present
Ans C

BY DR.ABDULRAHMAN ALMUALM
MJDF MCQS WITH ANSWERS

BY DR.ABDULRAHMAN ALMUALM
MJDF MCQS WITH ANSWERS

BY DR.ABDULRAHMAN ALMUALM
MJDF MCQS WITH ANSWERS

601. Cervical lines and dark lines on central incisors are more likely from:
• A) childhood illness
• B) amelogenesis
• C)dentinogenesis
• D )flourosisi
Ans is A

602. 87. patient on clopidrogrel and warfarin inform you that


recently his GP change his medication.
What medication would the patient have?
1. Aspirin
2. Dabigatran
3. Denosumab
4. Argatroban
5. bivalirudin
Ans is 2

603. Q10. Stainless steel crown placed on 7 yr old 2 months after MO GIC filling LLE
What NHS band is treatment
A.1
b.2
C. 3
d. urgent
ans B (no lab work needed )

604. Autoclave maintenance according to regs 2000

a. 6
b. 12
C. 18

BY DR.ABDULRAHMAN ALMUALM
MJDF MCQS WITH ANSWERS

d. 24
e. 36 montHs
Ans B

605. Pt attends with non bacterial TsL he is taking medication -which is likely meds

a. cimetidine
b. omeprazole
C. aspirin
d. insulin
ans B

606. Which LA may cause neurological disorders following block administration

a. Articane
b. Lignocaine
C. Bupivocaine
d. Prilocaine
ans A

607. Titanium is used in dentistry


A. ln a very pure form in implants
B. ln an alloy with aluminium in casting for crowns and bridges
C. ln an alloy with nickel in orthodontic wires
D. A and B
E. A, B and C
Ans is E

608. 50.Which of the following is a toxoid vaccinine

Diphtheria
HepatitisA
C. Hepatitis B
D. Measles
Ans is A (diphtheria)

609. A fifteen year old patient asks for bleaching of teeth. What is the first line of
management?
Discuss options with both patient and parent
Discuss option with both patient and parent, once you have obtained consent from
patient
Discuss only with patient
Discuss only with parent
Ans is B

610. Which one of the following is not a complications of Pagets' disease2


a) Osteosarcoma
b) Low output congestive cardiac failure

BY DR.ABDULRAHMAN ALMUALM
MJDF MCQS WITH ANSWERS

C)Cranial nerve defect


d) Pathological fractures
e) Bone pain
Ans is B

611. child avulsed permanent tooth came with her aunt,


what will you
do?
Do not do anything
Do not do treatment as aunt is not legal guardian of the
child
Do not do treatmentas child's mother is not there to give
consent
Re-implant and splint it with aunt's consent
Ans is D but bear in mind that no one can consent on
the patient if he >16 years
So the ans is reimplant it for the best interest of the patient )emergency case )

612 . Ipsilateral rash malar, nose, upper lip, gingiva (


A.struge webber,
B.SLE
ANS IS A

613 Aortic root defect,eye problems , high palate suggest what condition
Ans Marfan syndrome

614. Sanitation is what level of infection control??


Ans low level

615. 8 year old, all first permanent molars carious, which toothpaste you prescribe
Ans is 1450 -1500

BY DR.ABDULRAHMAN ALMUALM
MJDF MCQS WITH ANSWERS

.1nasopalatine cyst
2 stafne bone cyst
3residual cyst
4dentigerous cyst
5odotnogenic Keratocyst tumor
6infected mandibular cyst
7simple bone cyst
8periodontal cyst
9preapical cyst

616.dental hygienist allowed to


1 / prescribe radiographs
2/ do a cancer screen
3/ give inferior alveolar anaesthesia
4record probe depth
5.all of the above

Ans is 5

617 procedures may be undertaken by a registered dental


nurse provided that they have been appropriately trained?

A Record dental charting and oral tissue assessment carried out by other registrants
B Process dental radiographs

BY DR.ABDULRAHMAN ALMUALM
MJDF MCQS WITH ANSWERS

C.Take intra- and extraoral photographs


D Take shades
E Take radiographs
F all of the above

Ans F
618.Which of the following protocols must be included in the written practice
protocols'?
A Disposal of hazardous waste
B Disposal of sharps
C Annual leave entitlements
D Radiation protection
E Autoclaving
F A+B+D+E ( Ans is E)
619. In juvenile periodontitis, which bacterium is the recognised aetiological agent=
A Porphyromonas gingivalis
B Prevotella intermedia
C Actinobacillus actinomycetemcomitans
D Borrelia burgdorferi
E Fusobacterium intermedium

Ans C

.620 Which teeth are most commonly affected in juvenile periodontitis?


A Incisors and first permanent molars
BIncisors and canines
C.Canines and premolars
D.premolar and first permanent molars
Ans A

.621 Six months ago you saw a child patient, then aged 9 years. His upper
right maxillary canine was palpable in the labial sulcus but the upper left was
not. The situation is now unchanged, so you have taken two periapicals of the
non-palpable tooth. They both show that there is some resorption of the CI root
but the permanent canine appears somewhat mesially angled and is more
mesial on the more mesially positioned film. What is your the best course of
action?
A● Keep a careful watch on it and take another x-ray in 6 months.
b● Refer to an oral surgeon for early exposure of the permanent canine.
C● Refer to an oral surgeon for early removal of the permanent canine before it
damages the lateral incisor.
D ● Refer to an orthodontist for a treatment plan.
E● Wait and watch' until the child is 11.
Ans is D

.622 pt with chronic periodontitis ..his BPE IS :. .


BPE
313
4* 1 3
what cause his condition to progress?

BY DR.ABDULRAHMAN ALMUALM
MJDF MCQS WITH ANSWERS

A)enamel pearl
B)root groove
C)overhang restoration

ANS IS A (enamel pearls lead to involve of furcal area )

623A child of 5 years attends with pain from a grossly decayed lower right D
which has a discharging sinus. He is a hemophiliac. Which of the following is the
most appropriate treatment to relieve his pain?
● Antibiotics
● Extraction
● Fluoride application
● Non-vital pulpotomy
● Vital pulpotomy
Ans D
.624 48 year old woman complains of a sore area on the right buccal mucosa
adjacent to a restored tooth. The lesion has a lichenoid appearance and this is
confirmed histopathologically following a biopsy. Which of the following
restorative materials is most frequently associated with *lichenoid* changes.?
● Gold
● Amalgam
● Porcelain
● composite
● Glass ionomer cement
Ans is B (Amalgam)
.
.625 Which of the following about acrylic is true:
A) can be stained and designed
B) has higher strength compared to Co-Cr
C) has less hypersensitivity than Co-Cr
D) less surface area required for tissue cOverage

Ans is A

.626 Unilateral rush on one side of the face, pt is feeling well what is the most
common features you
will find in the mouth?
A) vesicular rash on the palate
B) ulcers on border side of the tongue
C) something on the cheek

Ans is B

627which Fluoride may aggravate the ANUG :


Ans duraphat

BY DR.ABDULRAHMAN ALMUALM
MJDF MCQS WITH ANSWERS

.628 Fluoride application


Which of the following statements about 2.2% fluoride (F) varnish application
are correct?

A F varnish is contraindicated in patients who have ulcerative gingivitis


B F varnish is contraindicated in children who have caries-free deciduous teeth
C F varnish is contraindicated in children who use toothpaste containing 1500 parts
per
million (ppm)
D F varnish is contraindicated in children who have contact dermatitis to iodine
Ans is A

.629 Landmark for oropharyngeal opening


A.angle of mouth to angle of mandible
B.angle of mandible to maxillary incisors,
C angle of mandible to mandibular incisors
Ans is B

630. Which of the following is not predisposing factor to DM?


a) Steroids
b) Acromegaly
c) Dwarfism disease
d) Cushing disease
e) Gigantism
Ans is C

.631 12 yr old with missing UR 1st permanent molar. What is the most right
diagnosis
(A.hypodontia
B macrodontia
C failure of primary eruption
D, early tooth loss)
Ans A
632 15year old with retained upper primary canine . contralateral permanent canine
is erupted 1.5 years ago. Reason is ?
(A.hypodontia,
b.macrodontia,
c.early tooth loss)
D.ectopic canine
Ans is D ( please note this Qs was adjusted, it was without D option ,I added it
because it’s more logical answer , hypodontia in permanent canine is very rare /but if
you faced the Qs without D option the most close answer will be A )

633. 16 year old boy with missing UR2 . What is your diagnosis? (
A.ectopic,
B. hypodontia,
c. primary failure in eruption,
D.deciduous tooth loss
Ans here is B

BY DR.ABDULRAHMAN ALMUALM
MJDF MCQS WITH ANSWERS

634. Appliance you will give to 12 yr old boy who complains his upper teeth are too
outward and children in school call him boy with no chin
A(fixed appliance,
B.functional appliance,
C.headgear)

Ans here B

.635 The professional conduct committee can suspend the dentist registration for a
period not exceeding 12 months, or erase his name?
True
False

Ans True

.636 Which one of the following statements regarding the platform switching concept
in Implent
dentistry is FALSE?

o The platform switching concept uses an increased diameter of abutment compered


to the implant size
o The platform switching concept uses a reduced diameter of abutment compared to
the implant sze
o The platform switching concept is associated with a reducton in crestal bone
resorption
oThe platform switching concept can be used in both submerged and unsubmerged
surgical techniques
○The platform switching concept provides a horizontal platform to offset the
biological width surrounding
a dental implant

Ans A

.637 After prescribing a dose of post-operative antibiotics, you are called to the ward
to review the
patient who Is dyspnoeic with audible wheeze, has facial swelling and ls covered in a
red rash. Her
pulse is 120/min regular and blood pressure is 86/40 mmHg. Oxygen saturations on
151 of oxygen
are 100%. You quickly diagnose anaphylactic shock. Which is the most important
immediate
treatment?

o 0.5 mg 11000 adrenaline given intravenously

BY DR.ABDULRAHMAN ALMUALM
MJDF MCQS WITH ANSWERS

o 0.5mg 11000 adrenaline given intramuscularly


o 1.0 mg 1 10,000 adrenaline given intravenously
o 0.5 mg 1 1000 adrenaline given subcutaneously
o 0.5 mg 1.1000 adrenaline given by sublingual route
Ans B

.638 Which one of the following has the best 10-year survival rate, when replacinga
single upper centra
incisor?

o Cantilever adhesive bridge from the lateral incisor


o Double-winged adhesive bridge with wings on the lateral incisor and corresponding
central incisor
o Cantilever conventional bridge, central incisor abutment
o Fixed movable bridge
o Osseointeograted dental implant
ans E

.639 Which one of the following does not cause a unitateral lower motor neurone
palsy of the facial
nerve?

oGuillin-Barré syndrome
o Diabetes
o Ramsey Hunt Syndrome
oBell's palsy
o Acoustic neuroma
Ans is A

.640 what does contra indicate distal wedge in molars are


A.Distal fluting
B.long attached gingiva
C. sharply ascending ramus that limits space distal to molarss
D.supra bony pockets distal to molars

Ans is C?

.641 The official name of a drug is generally used inter-changeably with the:
(a) proprietary name
(b) chemical name
(c) brand name
(d) trade name
(e) generic name
Ans is E

.642 The mode of action of a drug may be defined as:


(a) where the drug has its effect
(b) how it produces its action
(c) what effects the drug has

BY DR.ABDULRAHMAN ALMUALM
MJDF MCQS WITH ANSWERS

(d) how the drug is distributed


Ans is B

.643 The Therapeutic Index is a measure of a drug's:


(a) mean effective dosage
(b) effectiveness against disease
(c) usual therapeutic dose
(d) margin of safety
(e) potential for adverse side effects

.644 The Therapeutic Index is a measure of a drug's:


(a) mean effective dosage
(b) effectiveness against disease
(c) usual therapeutic dose
(d) margin of safety
(e) potential for adverse side effects
Ans is D The Therapeutic Index, a ratio de-fined as the dose necessary to pro-duce
toxic effects divided by the dose that produces the therapeutic effect. is a measure of
the margin of safety.

645 The time until the patient notes the therapeutic ef-fect of an administered drug is
termed its:
(a) onset of action
(b) duration of action
(c) pharmacological action
(d) latency
(e) individual variation
Ans is D
latency
From the administration of drug to the onsent of action

.646 A dental patient becomes sleepy following the ad-ministration of a narcotic


analgesic for relief of pain from an acute dentoalveolar abscess. This effect would be
termed a:
(a) toxic effect
(b) hyperreaction
(c) contraindication
(d) idiosyncracy
(e) side effect
Ans is E

.647 A patient exhibits very little effect following ad-ministration of a relatively large
dose of a pharmaco-logically active drug. This is best described as:
(a) idiosyncrasy
(b) high therapeutic index

BY DR.ABDULRAHMAN ALMUALM
MJDF MCQS WITH ANSWERS

(c) hyporeactlon
(d) hyperreaction
(e dependence
Ans C

.648 Administering double the usual dosage of a drug to a patient would most likely
double the: f
a) therapeutic effectiveness
b) mcidence of side effects
c) therapeutic index l.
d) potential for toxicity (
e duration of action
ans is D

.649 A drug that has an affinity for a receptor site and c-roduces the intrinsic activity
of the receptor cell or e1zyme system is known as: (a) a) an agonist
(b) a competitive antagonist
(c) a noncompetitive antagonist
(d) any of the above (
e) none of the above

Ans A

650 A drug that binds to an acceptor site would be ex-pected to:


(a) alter the permeability of the cell
(b) stimulate or depress the physiological activity of the cell
(c) inhibit the mitotic process of the cell
(d) release mediating substances from the cell
(e) be inactive
Ans is E
any drug binds to acceptor site or storage site wont produce action it wont be
avaliable for excretion or metabolism until its released from it

651. The most serious disadvantage to the parenteral route of administration,


compared with the peroral route. is that it: (a) is less convenient to a patient
(b) is more expensive to a patient
(c) has greater severity of systemic reactions
(d) has less rapid, predictable absorption
(e) is painful on administration
Ans is C

.652 Drugs are best absorbed when they are: Choose from (a). (b), (c), (d), and (e).

1. of high molecular weight


2 water soluble .
3. unionized
4. highly polar
5. lipid soluble

(a) 1 and 2.

BY DR.ABDULRAHMAN ALMUALM
MJDF MCQS WITH ANSWERS

(b) 2 and 3
(c) 2 and 4
(d) 3 and 4
(e) 3 and 5
Ans is E
# Drugs best absorbed when they are low molecular weight ,unionized, and lipid
soluble

653.Which one of the following statements is true as defined in endodontics?


A. Overfilling refers to the incomplete filling of the canal system with a
surplus of material extruding beyond the apical foramen.
B. Underfilling refers to the complete filling of the canal space, but leaving
voids in the pulp chamber for possible recontamination or infection.
C. Overextension refers to the extrusion of filling material through an
accessory canal.
D. Underextension refers to the vertical extent of the filling material regardless
of its completeness of obturation.
E. All of the above.
Ans D

.654 Special attention is given to matrix adaptation for the insertion of


amalgam in a MO cavity in a maxillary first premolar because of the
A. concavity in the cervical third of the mesial surface of the crown.
B. restoration being in the esthetic zone.
C. unusual position of the contact area.
D. buccolingual width of the tooth's mesial marginal ridge.
E. size of the interproximal gingival embrasure.
Ans A

.655 After a drug has been absorbed, an equilibrium is usually established at all of
the following possible sites of distribution except· is:
(a) nephrons
(b) hepatic microsomal enzymes
(c) urine
(d) plasma and ttssue fluids
(e) fat binding and protein binding sites
Ans is C

.656 A drug that becomes plasma-protein bound ts: (a) available for
biotransformation
(b) easily excreted
(c) more readily transported across cell membranes
(d) pharmacologically active
(e) pharmacologically inactive
Ans is E

.657 Trichloroacetic acid, a strong acid, has been used by dentists for chemical
cautery of hypertrophic tissue and of aphthous ulcers. Its mechanism of action is:
(a) thermodynamic action
(b) activatton of tissue enzymes

BY DR.ABDULRAHMAN ALMUALM
MJDF MCQS WITH ANSWERS

(c) osmotic pressure


(d) protein precipitation
(e) neutralization
Ans is D

658 Full-denture patients with inflammation of the en-tire tissue-bearing area should
probably be treated with:
(a) neomycin
(b) polymyxin B
(c) amphotericin B
(d) ampicillin
(e) chloramphenicol
Ans is C

659 Permanent auditory nerve deafness is frequently associated with use of:
(a) the streptomycins
(b) chloramphenicol
(c) the penicillins
(d) the tetracyclines
(e) polymyxin B
Ans is A

660 fracture in an all-ceramic crown may be caused by


1. inadequate ceramic thickness.
2 sharp line angles in the tooth preparation.
3 excessive occlusal load.
4 use of an inappropriate luting material.
A. (1) (2) (3)
B. (1) and (3)
C. (2) and (4)
D. (4) only
E. All of the above. .
Ans is E

.661 In which of the following conditions is there a risk of malignant change


after radiotherapy?
A. Fibrous dysplasia.
B. Mucocele.
C. Lymphangioma.
D. Torus palatinus.
Ans is A
#also Mccune Albright syndrome

662 A protective mechanism of the dental pulp to external irritation or caries is the
formation of
A. pulp stones.
B. tertiary dentin.
C. secondary cementum.
D. primary dentin.

BY DR.ABDULRAHMAN ALMUALM
MJDF MCQS WITH ANSWERS

Ans is B

663Which of the following is NOT a component of a dental cartridge containing 2%


lidocaine with 1:100,000 epinephrine? A.
A.Methylparaben.
B. Water.
C. Sodium metabisulphite.
D. Sodium chloride.

Ans A

664 The finding of “acid-fast” microorganisms


in sputum suggests the presence of
A. Mycobacterium tuberculosis.
B. Diplococcus pneumoniae.
C. Streptococcus pyogenes.
D. Neisseria gonorrhoeae.
Ans A

665 In an infrabony pocket, the epithelial


attachment is located
A. within basal bone.
B. coronal to alveolar bone crest.
C. apical to alveolar bone crest.
Ans is C

666 .Which dental tissue is 45 percent mineralised an d resorts slower than


bone?
Enamel,
Dentin ,
Cementum
Ans C

Behavioral management :
Remember Techniques for behaviour
management
Tell, show, do Self-explanatory, but use
language the child will understand.
Desensitization Used for child with pre-
existing fears or phobias. Involves helping
patient to relaxin dental environment, then constructing a hierarchy of fearful
stimuli for that patient. These areintroduced to the child gradually, with
progression on to the next stimulus only when the child isable to cope with
previous situation.

Modelling Useful for children with little previous dental experience who are
apprehensive. Encourage child to watch other children of similar age or
siblings receiving dental Rx happily.

BY DR.ABDULRAHMAN ALMUALM
MJDF MCQS WITH ANSWERS

Behaviour shaping The aim of this is to guide and modify the child's
responses, selectively re-inforcing appropriate behaviour, whilst
discouraging/ignoring inappropriate behaviour.

Reinforcement This is the strengthening of patterns of behaviour, usually by


rewarding good behaviour with approval and praise. If a child protests and is
uncooperative during Rx, do not immediately abandon session and return them
to the consolation of their parent, as this could inadvertently reinforce the
undesirable behaviour. It is better to try and ensure that some phase of the
Rx is completed, e.g. placing a dressing.

667. Most appropriate situation to use hand gel to clean hands?


1 After eating and drinking
2 after seeing patient
3 before seeing patient
Ans is 3

.668 A boy 14 years old came to ur clinic with avulsed tooth not accompanied
by parents, most appropriate in this scenario ?
1- carry out treatment
2- ask the boy to place tooth in storage media
3- contact parents
Ans is 1

669Which cells are defective in diabetes mellitus?


Neutrophils
Monocytes
Lymphocyte
Acidophils

Ans A
#to remember :
# Cephalometrics Most commonly used cephalometric points :
S = Sella: mid-point of sella turcica.
N = Nasion: most anterior point on fronto-nasal suture.
Or = Orbitale: most inferior anterior point on margin of orbit (take average of
twoimages).
Po = Porion: uppermost outermost point on bony external auditory meatus.

.670 The light emitted by the polymerization lamp has to be checked from
time to time. The meter used for this only measures light in the range of:
A. 100-199 nm
B. 200-299 nm
C. 300-399 nm
D. 400-499 nm

BY DR.ABDULRAHMAN ALMUALM
MJDF MCQS WITH ANSWERS

Ans D

671. Patient with xerostomia, will benefit much more from


1. Scaling polishing
2. Flouride rinse

Ans 2

.672 Which is least likely to be unilocular?


A ameloblastoma
B odontogenic keratocyst
C aneurysmal bone cyst
D myxoma
E osteoporosis
Ans A

673 Trauma to tooth with closed apex. In which one the tooth is more likely
to maintain vitality?
Concussion
Subluxation
Intrusive luxation
Avulsion Luxation
Ans is A

674 Which odontogenic pathology would contain clear straw coloured fluid?
Options are
1. Odontomes
2. Solitary bone cyst
3. Ameloblastoma
4. Dentigerous cyst
5. Keratocyst.
Ans is 2

675 Which of the following types of tooth movement can be achieved with a
removable appliance?
a) Bodily movement
b) Correction of rotation
c) Rapid maxillary expansion
d) Tipping
e) Torque
Ans is D

.676 Which of the following is a component of removable appliances?


a) Active self-ligating system
b) Adam’s clasp
c) Bracket Module
d) Passive self-ligating system
ans B

BY DR.ABDULRAHMAN ALMUALM
MJDF MCQS WITH ANSWERS

677 Which active component is commonly used to move a tooth mesio-distally within
the arch?
a) Palatal finger spring
b) T spring
c) Robert’s retractor
d) Z spring
e) Labial bow
Ans A

678Which appliance would you use to treat an 8-year-old child with an anterior
crossbite associated with an upper central incisor?
a) URA with anterior bite plane and a midline screw
b) URA with a midline screw and a T spring
c) URA with posterior bite planes and a midline screw
d) URA with an anterior bite plane and a T spring
e) URA with posterior bite planes and a T spring
Ans e

.679 Which of the following statements about rapid maxillary expansion is true?
a) It can be used in the mandible
b) It can only be carried out in adults
c) It can only be carried out in the deciduous dentition in It involves expansion of the
mandibular symphysis
d) It is a form of distraction osteogenesis
ans is D

680 Which of the following features are most commonly associated with a digit
sucking habit?
a) An increased overjet with a deep Overbite and no crossbites
b) A Unilateral Crossbites with a decreased overjet and a deep overbite
c) An increased overjet with a deep Overbite and a unilateral crossbite
d) A Unilateral Crossbites with a reduced Overjet and an incomplete bite
e) A unilateral crossbite with an increased overjet and an incomplete overbite
Ans is E

681 Which of the following cases is an ideal case for functional appliance therapy?

a) A 16-year-old girl with a moderate class II malocclusion and uncrowded arches


b) A 12-year-old girl with a moderate class III malocclusion and uncrowded arches
c) A 16-year-oId girl with a moderate class III malocclusion and crowded arches
d) A 12-year-oId girl with a moderate class II malocclusion and uncrowded arches
e) A 12-year-old girl with a class I malocclusion and crowded arches

Ans is D age 12 under the growth spurt

.682 What is the definition of anchorage?


a) The resistance to unwanted tooth movements
b) The use of inter-maxillary elastics
c) The resistance to treating a malocclusion

BY DR.ABDULRAHMAN ALMUALM
MJDF MCQS WITH ANSWERS

d) The resistance to favourable tooth movement


t e) The use of extra-oral traction

Ans A

683 Fixed appliances can achieve tooth movement in all three spatial planes.
What type of tooth movement is achieved with a second order bend?
a) Bucco-lingual movement of the root apex to correct the inclination of teeth
b) Vertical movement of the crown to correct discrepancies in crown height
c) Bucco-lingual movement of the crown to correct the inclination of teeth
d) Movement in the horizontal plane to account for differences in bucco-lingual
thickness of teeth
e) Movement in the vertical plane to correct the mesio-distal angulation of
teeth
Ans is E

684 An adult patient complains of her prominent upper front teeth and receding chin.
She says she did not get 'braces' when she was younger because the family had to
move area a lot. Her oral care and health is good, and she has a 9 mm overjet.
Where is the most suitable place to refer her? "
a) A GDP friend who has a special interest in orthodontics and has been trained to
use fixed appliances.
b) A specialist practitioner who uses a lot of functional appliances.
c) A private specialist practitioner, because she's too old to get NHS treatment now.
d) A hospital consultant, as it is likely she will need surgery now to correct her
problem
e) The nearest dental hospital, although it is 70 miles aaway
Ans D?

685 pt. presents with an increased overjet with 7 mm, an anterior open bite of 5 mm,
and a lower left second permanent molar partially erupted & impacted against the
first perm. Molar. Which IOTN category? a) None
b) Little need
c) Moderate need
d) Great need
e) Very great need

Ans c

686 ANB angle in class III?


a) ANB 2-4
b) ANB < 2
c) ANB > 4
ans is B?
.687 Which part of the cranium is considered as the most stable area?
a) Frankfort plane
b) Occlusal plane
c) Anterior cranial plane
d) Anterior nasal to gnathion.
Ans is C

BY DR.ABDULRAHMAN ALMUALM
MJDF MCQS WITH ANSWERS

688 The Frankfort plane is defined by which anatomical landmarks,


a) Porion, orbitale
b) Sella, orbitale
c) Nasion, Tragus

Ans is A

689 Frankfort plane extends from


a) horizontally from Sella to nasion
b) Sagittal from
c) Horizontally from point on superior aspect of external auditory meatus to orbitale
ans is C

. 690 Which sagittal relationship described below is not represented via a lateral
cephalogram?
a) Jaws to the skull base
b) Maxilla to mandible
c) Teeth to each jaw
d) Soft tissues to hard tissues
e) Teeth in the maxilla to the teeth in the mandible

D?

691 An orthodontic patient requires imaging for treatment planning, which radiograph
listed below is standardized and reproducible for all patients? a) Dental
Orthopantomogram (OPG)
b) Lateral cephalogram
c) Lower standard occIusaI
d) Upper standard occIusaI
e) Lateral skull radiograph

Ans A??

692 You are meeting a three year old patient for the first time. His mother explains
that he had a unilateral cleft lip and palate that was repaired in infancy. She has
heard that orthodontics will usually be required when he is older. She asks you what
is the commonest orthodontic problem that occurs with a repaired cleft palate. What
is the most appropriate answer?
a) A contracted maxillary arc
b) An anterior open bite
c) An increased overjet
d) An expanded arch
Ans A

693 What percentage of the population has ectopic maxillary canines?


a) 5%
b) 10%
c) 20%
Ans is A?

BY DR.ABDULRAHMAN ALMUALM
MJDF MCQS WITH ANSWERS

694 Patient with a cleft palate often have skeletal relationship?


a) Class I
b) Class II division 1
C) Class II division 2
d) Class II division 1 + class II division 2
e) Class III
Ans E??

695 In embryology, when does fusion of the secondary palate occur?


a) At 2—3 weeks in utero
b) At 3--4 weeks in utero
c) At 4—5 weeks in utero
d) At 5---6 weeks in utero
e) At 7—-8 weeks in utero
Ans D??

696 What is the prevalence of non-syndromic cleft lip and palate in the UK?
a) 1 in 20
b) 1 in 300
c) 1 in 400
d) 1 in 700
e) 1 in 1000

697. Which one of the following is the correct incidence of cleft lip in the UK?
a) 1 in 200
b) 1 in 500
c) 1 in 900
d) 1 in 1800
e) 1 in 2500
Ans is B?? Its 1/750

698 Which one of the following is the classical extraction pattern for the serial
extraction technique?
a) Deciduous canines, first premolars then first deciduous molars
b) Deciduous canines, first permanent molars then first premolars
c) Deciduous canines, second deciduous molars then second premolars
d) Deciduous canines, second molars then second deciduous molars
e) Deciduous canines, first deciduous molars then first premolars
Ans E

699 nine year old boy presents with a class I occlusion with no crowding or overjet
with a grossly carious upper left 6 which is not suitable for restoration. The upper left
7 is very near to eruption. Upper right 6, lower right 6 and lower left 6 are sound and
fissure sealed. The oral hygiene is good. What is the most appropriate extraction
pattern for this patient?

a) Extract all four 1st permanent molars


b) Extract upper left 6 and lower left 6
c) Extract upper left 6 and lower right 6

BY DR.ABDULRAHMAN ALMUALM
MJDF MCQS WITH ANSWERS

d) Extract upper right 6 and upper left 6


e) Extract upper left 6 only
ans is e ?

700 n facial growth, at the age of 5 years, what percentage of growth has been
completed?

a) Cranium 96%, maxilla 45%, mandible 45%


b) Cranium 85%, maxilla 4%, mandible 65%
c) Cranium 85%, maxilla 45%, mandible 45%
d) Cranium 4%, maxilla 85%, mandible 45%
e) Cranium 85%, maxilla 65%, mandible 4%
ans is C

.701 Which of the following statements about the Leeway space is correct?

a) It is always greater in the upper arch than in the lower arch


b) It is usually equal in the upper and lower arches
c) It relates to the total space taken up by the combined mesiodistal widths of the
deciduous canine, first and second premolar teeth
d) It relates to the difference between the combined mesiodistal widths of the
permanent canine, first and second premolar teeth that is greater than that of their
deciduous precursors
e) It relates to the difference between the combined mesiodistal widths of the
permanent canine, first and second premolar teeth that is less than that of their
deciduous precursors
Ans is E

702 8-year-old patient attends with a painless lower right 1st permanent molar which
requires extraction. The patient has a Class I occlusion with no crowding. What
feature will determine the ideal time to extract the tooth to maximize space closure?

a) Clinically, the lower 2nd molar is fully erupted


b) Clinically, the lower right 2nd premolar is fully erupted
c) Radiographically, none of the root of the lower right 2nd molar has formed
d) Radiographically, the bifurcation of the lower right 2nd molar has formed
e) Radiographically, the roots of lower right 2nd molar are fully formed
Ans D

703 Early loss of which upper teeth is the most likely cause of impacted upper
second premolar teeth?
a) Both first and second deciduous molars
b) First permanent molars
c) Second deciduous molars
d) First deciduous molars
e) Both first and second permanent molars
Ans C

704 If a patient has a full unit II molar relationship then:

BY DR.ABDULRAHMAN ALMUALM
MJDF MCQS WITH ANSWERS

a) The mesiobuccal cusp of the upper first permanent molar occludes with the
mesiobuccal groove of the Iower first permanent molar
b) The distobuccal cusp of the upper first permanent molar occludes with the
mesiobuccal groove of the Iower first permanent molar
c) The mesiobuccal cusp of the upper first permanent molar occludes with the
distobuccal groove of the Iower first permanent molar
d) The mesiobuccal cusp of the upper first permanent molar occludes with the
distobuccal cusp of the Iower first permanent molar
e) The mesiobuccal grove of the upper first permanent molar occludes with the
distobuccal cusp of the Iower first permanent molar
Ans is B

705 What is the perfect time to extract all four molar?

a) early dentin calcification at bifurcation of lower molar


b) early dentin calcification of bifurcation of upper molar
c) early calcification of dentin at coronal part of root
d) 8-10 yo
e) 10-12 yo
Ans A

706 year old boy has previously had all primary molars restored and a pulpotomy on
upper right E. He has an early mixed dentition with lower lateral incisors erupting.
There is a midline diastema of 2 mm. The upper right E has become symptomatic
and requires extraction. The most likely long term effect of the extraction on the
occlusion is:
a) Early eruption of the second premolar.
b) Loss of upper central line.
c) No significant effect.
d) Overeruption of the lower right teeth.
e) Potential crowding in the upper right quadrant
Ans e

707 retained lower primary incisor causes the permanent incisor to


a) Erupt buccally
b) Erupt lingually
c) ankylose
Ans B

708 Which of the following is a typical consequence of dental crowding, assuming no


primary tooth has been lost prematurely?

a) Overlapping of lower incisors


b) Palatal displacement of upper canines
c) Impaction of 15 and 25 between first premolars and first molars
d) Mesial tipping of 16 and 26
e) Rotation of 16 and 26

BY DR.ABDULRAHMAN ALMUALM
MJDF MCQS WITH ANSWERS

Ans A

.709 Which of the following is not potential consequence of submerged deciduous


teeth
a) Over eruption
b) Centreline shift
C) Tipping of adjacent
d) Premature exfoliation
ans is D

710 -failure of eruption:

a. primary failure of eruption


b. abnormal position of tooth crypt
c. premature loss of deciduous
d. supernumerary tooth
e. dilaceration
f. congenitally absent,

1) crowding in premolar region

2) palatally displaced max. Canine

3) failure of eruption of upper central incisor in


10 years child with a history of trauma
4) absent lower first perm. Molar
5) absent upper lateral incisor

AnS
1 C
2 B
3 E
4 A
5 F

711 What teeth should a 9 year old child have a given quadrant?
a) 12CDE6
b) 1BCDE6
c) 12CDE
d) ABCDE6
e) 1234E6

Ans A

712 At which age you should try to locate upper canine?


a) 6 y
b) 8 y
c) 9-10 y
d) 13 y

BY DR.ABDULRAHMAN ALMUALM
MJDF MCQS WITH ANSWERS

e) 16 y
Ans D

713 At which age you should try to locate upper canine?


a) 6 y
b) 8 y
c) 9-10 y
d) 13 y
e) 16 y

AnsC

.714 Which one of the following is not a cause of median diastema?


a) Normal development
b) Microdontia
c) Hypodontia
d) Lingual frenum
e) Midline supernumerary
ans D

715 Which of the following statements about fluoride is correct?

a) in All children under the age of 5 years should use toothpaste with a fluoride
concentration of 500 ppm
b) Fluoride supplements are recommended for all children
c) It exerts its ant cariogenic effects mainly pre eruptively
d) It is more effective in preventing pit and fissure caries than in preventing caries in
smooth surfaces
e) The certain lethal dosage is 32—-64 mg/kg
ans is E

.716 Which one of the following immunoglobulins is found in saliva?


a) lgA
b) lgD
c) lgE
d) IgM
Ans is A

717 Chlorhexidine is a chemical antiseptic. Which one of the following statements is


false?
a) It is most effective immediately after the use of toothpaste
b) Can be present in gels and rinses
c) Its mechanism of action is membrane disruption
d) Is bactericidal as well as bacteriostatic
e) Activity is reduced in the presence of blood, soap
ans is A

BY DR.ABDULRAHMAN ALMUALM
MJDF MCQS WITH ANSWERS

718 Which one of the following relates to saliva and dental caries?

a) The buffering power of saliva depends mainly on its bicarbonate content and is
increased at high rates of flow
b) The buffering power of saliva is independent of the bicarbonate content
c) The buffering power of saliva depends on mainly its bicarbonate content and is
decreased at low rates of flow
d) The buffering power of saliva depends on its IgA content and is increased at high
rates of flow
e) The buffering powers of saliva depends on its IgA content and is decreased at
high rates of flow
Ans A

719 Which one of the following is the most common cause of dental pain and loss of
teeth, especially in the younger population?
a) Fracture of a crown/cusp
b) Thermal or chemical irritation
c) Dental caries
d) Traumatic pulpal exposure
e) ‘Cracked tooth syndrome’
Ans C?

720 Which one of the following is not a factor in the etiology of dental caries?
a) Diet
b) Oral flora
c) Age
d) Time
e) Susceptible surface
Ans C

721 Which one of the following inter-dental aids is most effective at removing inter-
dental plaque?
a) Dental floss
b) Electrical toothbrushes
c) Inter—dental brushes
d) Triangular toothpicks
e) Dental tape
ans C (pas)

722 18 months of age, what is fluoride supplement for child living in 0.25 ppm
fluoridated water?
a) 0.25mg
b) 0.5 mg
c) 1 mg
Ans A

BY DR.ABDULRAHMAN ALMUALM
MJDF MCQS WITH ANSWERS

723 There has been much debate regarding the effectiveness of fluoride in water for
preventing tooth decay. A systematic review of the evidence drew conclusions as to
the reductions in decay which can be expected. Fluoridation of public water supplies:

a) Reduces tooth decay by 10%.


b) Reduces tooth decay by 20%.
c) Reduces tooth decay by 30%.
d) Reduces tooth decay by 40%.
e) Reduces tooth decay by 50%
Ans E

724 Epidemiological studies have shown that dental decay is normally greater
amongst northerners and those in socially deprived circumstances. Which of the
following would be most effective in reducing caries in a high risk population?

a) Brushing with fluoride toothpaste


b) Publicity campaign
c) Fluoridation of the water supply
d) Dietary advice via schools
e) Fissure sealant provision
ans is C

725 Duraphat in high caries risk group children is applied every


a) 2 months
b) 4monttls
c) 6 months
d) 8 months
e) One-yearly
ans c

726 3 year old with fluoride concentration of 0.3 ppm fluoridated water, what is the
fluoride supplement dose given
a) 0.25 mg
b) 0.5mg
c) 1mg

Ans B?

727 The concentration of Fluoride in the topical NaF


a) 2%
b) 5%
c) 8%
d) 10%
Ans A

728 Child accidentally consumed 5mg/kg fluoride what the first line treatment?
a) Give child salty drink
b) Give child sugary drink
c) Give child milk
ans C

BY DR.ABDULRAHMAN ALMUALM
MJDF MCQS WITH ANSWERS

729 Mother of a 3 years old child came to your clinic asking for advice on what do to
after brushing her child’s teeth , you would tell her :
a) Spit out after brushing and rinse
b) Spit out after brushing and don’t rinse
c) Spit out and wait for 10 minutes then rinse
d) Spit out and wait 30 minutes then rinse
Ans B

730Patient needs a minimal preparation metal crown for his LL6 the best cement is
a) GIC
b) zinc phosphates
c)RMGIC
d) zinc polycarboxylate cement
e) resin cement
ans is E

731Best abutment(s) for resin bonded replacement of upper lateral is:


a) upper central only
b) upper canine & 1st premolar
c)upper canine only
d) upper canine & central
ans A

732What is the type of impression used for recording of upp dentur pa well fit stock
tray
for a patient having a hypersensitive gag reflex
a) Alginate
b Zo/E
c) Addition silicon
d) Polyether
e) Polysulphide
Ans is C

733What is the best type of restoratiom. storing buccal root caries in the lower
mandibular molar in patient:
a) Glass ionom
b) Amalgam
c) Composite
d) porcelain veneer crown
Ans A

734. 5)patient with compelete denture complaining difficulty in F &V sounds most
probably:
a) Lack on retention
b )palatal valut too high
c.high behind incisors

BY DR.ABDULRAHMAN ALMUALM
MJDF MCQS WITH ANSWERS

D.Palatal arè too thick


E.Incisor far palataly
Ans E

735The best way to determine hinge axis during prosthetic construction


a) Face bow
b) Fox bite
c) Articulator
d) Wax rim
e) Photos and study models
ans A

736The liquid which is present in the adhesive dentin systems to wet the dentin is:
a) Water
b) Acetone
c) Polymers liquid
d) Ketones
Ans B

737During the try-in stage of complete Denture you need to do slight grinding of
back teeth
Which teeth cusps should be reduced
a) Reduce teeth randomly
b) Upper buccal and lower lingual
c)Upper palatal and Jower buccal
d) Upper palatal and lower lingual
e) Upper buccal and lower buccal

Ans B

738. the obturation technique which gives the best hermetic seal for gutta-percha in
endodontitreatment
a)warm lateral compaction
b)warm verticalcompaction
c)thermoplasticised gutta-percha technique
d) coldlateral coropaction
e)chlorpercha technique
Ans is C

739The use of latex gloves does has the following effect when a polyvinyl siloxane
impression is taken
a) Retard the set of the impression material
b) Enhance the set of the impression material
c)Result in porosities in the impression material
d) Has no effect on impression
e) Latex gloves stick to the polyvinyl siloxane impression material
Ans A

740After crown replacement in how many months should you review it


radiogaphically?

BY DR.ABDULRAHMAN ALMUALM
MJDF MCQS WITH ANSWERS

a) 1 month
b) 3 months
c) 6 months
d) 1 year
e) Only if become symptomatic
Ans is D

741The metal framework of metallic dentures is formed of


a) Nickel chromium
b Cobalt chromium
Stainless steel
d) gold
ans B

742The metal framework of Ceramo-metallic erowns is formed of


a) Nickel chromium
b) Cobalt chromium
c) Stainless steel
d) gold
e) iron
ans A

743. First line treatment for patient with NCTL in palatal surfaces of upper front teeth:
a) Direct Composite palatal veneers
b) Full coverage crowns
c)Gold veneers
d) Indirect composite palatal veneers
e) Indirect porcelain veneers
ans A

744. Regarding Using balanced force technique in endodontic treatment

a) Decrease incidence of perforations and fractures in root


b) Ensure better apical seal
c)Ensure moving the irrigation solution coronally
d) Ensure better shaping especially in curved canals
e) Ensure widening of canals coronally to make filling the canal easy

Ans is D

745A 35 year old female patient requires replacement of 6 pér anterior


crowns with a confirmative occlusal scheme. What would the most
appropriate way to articulate study and working cast ?

a) Average value articulator with incisal guidance table


b) Fully adjustable articulator
c) Plane line articulator.
d) Semi adjustable articulator with facebow transfer and custom incisal
guidance table.

BY DR.ABDULRAHMAN ALMUALM
MJDF MCQS WITH ANSWERS

e) Semi adjustable articlator with facebow transfer.

Ans D

746 MMPA value is: •maxillary mandibular plane angle


A) 111 •
B) 79 •
C) 2-3 •
D) 27
Ans is D

747According to European commission for guidelines on radiation protection in


dental radiology, lateral cephalometric radiograph is surely indicated in: •
A) class II/1 •
B) class I •
C) class II/2 •
D) orthognathic surgery
Ans D

748Child pt who has severely broken down of all four permanent molars, they
have poor prognosis so you have decided to remove them. What would you
check to determine the right time for extraction? •

A) root formation of lower 2nd molar reach bifurcation •


B) root formation of lower 2nd molar one third complete •
C) root formation of upper 2nd molar reach bifurcation •
D) root formation of upper 2nd molar remain one third •
E) root formation of lower 2nd molar remain one third
Ans A

749Your pt is on Warfarin because of cardiac problem, which of the following


AB would the worst to potentiate effect of warfarin? •
A) amoxicillin •
B) metronidazole •
C) clindamycin •
D) erythromycin
Ans B
750 7year- old woman attends for a routine checkup with her gen.
eral dental practitioner, having been discharged rom her ortho
donust following comprehensive orthodontic treatrment. she reports
that she has lost the instructions for wearing her vacuum-Tormed re
diners. Vwhat is the single most appropriate advice to give in
this scenario?
A Wear the retainers full-time
BWear the retainers only during the day and remove 1or eating/
drinking
C Wear the retainers for 2-3 hours per day
D Wear the retainers only at night
E Wear the retainers only during the day and keep them in if eating/
Drinking

BY DR.ABDULRAHMAN ALMUALM
MJDF MCQS WITH ANSWERS

Ans D

751 Radiograph taken on wrong patient

A.Inform medical devices agency


B. Inform radiation protection advisor
C.Inform radiation protection supervisor
D. keep records for 50 years
E. Inform H&s executive
Ans B

752. Pt is taking rivoroxaban drug, what postoperative complication you can


expect? •
A) prolonged bleeding time •
B) bone necrosis •
C) osteomyelitis •
D )dry socket
Ans A

753. Midazolam – most common route of administration in dental office?


IV Triturated for sedation
For emergency Buccal

754. Wrong tooth extracted is under which protocol?


Never events

755Tonsil Ca gives metastasis to which lymph nodes?


Cervical lymph nodes /jugular lymph nodes ‘

756How to reduce radiation exposure:


A) speed D film
• B) lead apron
• C) speed E film
Ans is C

757.Severe case of Down Sy require surgical removal of one tooth, who can
give a consent ?
Before 16 > parents
After 16 > no one only patient according to his mental.capacity to understand retain
and communicate or (best interest of the patient )

758After chemotherapy pt is complaining of burning sensation in the mouth,


what is the most likely diagnosis:
A) mucositis •
B) xerostomia •
C) anemia

Ans is A

BY DR.ABDULRAHMAN ALMUALM
MJDF MCQS WITH ANSWERS

759Pt are compaling about the jaw pain two weeks after RCT, what is gonna
be your first management •
A) do RCT again •
B) reassure , sof diet and review in two weeks time •
C) soft splint •.
D) remove the tooth
Ans is B

760Absolute contraindication for MRI •


A) hip replacement •
B) pacemaker •
C) implant •
D) fixed ortho appliance

Ans us B

761How to know that non surgical periodontal therapy had effect? •


A) less than 15% bleeding on probing •
B) pockets getting bigger •
C) more than 15% bleeding on probing •
D)recession is controlled
Ans is A

762. Maximum temperature for washing (decontamination)instruments: •


A) 60C •.
B) 95C •
C)45C •
D) 30C
Ans C

763 BPE score is


324
3 2 2*
How to manage this case? •
A) refer to secondary care •
B) refer to GP •
C) refer to your colleuge with special interest in perio •
D) refer to specialist in primary care
ANS D ?

764. Which cells biposphonates act on •


A) osteoblast •
B)osteoclast •
C)fibroblast
Ans B

BY DR.ABDULRAHMAN ALMUALM
MJDF MCQS WITH ANSWERS

765. BCC spread by •


A) locally •
B) lymphatically •
C) hematogenus
Ans A

766. Pt comes to your practice and on OPG you have noticed fracture of
angle of mandible, then she said she was hit by her husband. Pt has
capacity, how to manage this situation/ •
A) report to the police •
B) suggested her to report case to the police •
C) contact local safeguarding services •
D) ignore and start treating injuries
Ans C

767. What is the most likely infection that can be caught by NSI in fully
immunized person? •
A) Hep B •
B) Hep C •
C) HIV
Ans is B

768. Patient had BMI=41. Which of the following is his ASA classification? •
A) ASA I •
B) ASA II •.
C) ASA III •
D) ASA IV
Ans C

769. A double blind RCT was carried out to study xxxx at University. In this
case who is not informed of which group were given new drugs and which
group is given placebo? •
A) nurse only •
B) dentist only
C) participants and dentist •
D) participants only
Ans C

770Which of the following defines prognosis of SCC? •


A) aneuploidy •
B) P53 •
C) differenciation •
D) size of the tumor
Ans C

BY DR.ABDULRAHMAN ALMUALM
MJDF MCQS WITH ANSWERS

771. PICTURE OF LEAF FIBROMA and question what is the most likely
diagnosis? ( did not
rememeber was it leaf or
fibroepithelial fibroma)

772. If the pt get Anaphylactic


shock what is your
management? •
A) Adults; IV 500mg Child<6
IM150mg, Child 6-12 300mg
IM, Child>12 500mg IM
Adrenaline •

B) Adults IM 500mg, Child<6 IM


150mg, Child 6-12, IM 300mg,
Child >12 IM 500 mg
Adrenaline

C) Adults IM 500mg, Child<6 IM


150mg, Child 6-12, IM 300mg, Child >12 IM 500 mg Adrenaline •

D) Adults IV 500mg, Child<6 IM 150mg, Child 6-12, IM 150mg, Child >12 IM


500 mg Adrenaline

Ans B

773. BPE score for the lower anterior teeth with pocket depth of 3mm with
recession of 3mm? •
A) 0
• B) 1 •.
C) 2 •
D) 3 •
E) 4
Ans is A

BY DR.ABDULRAHMAN ALMUALM
MJDF MCQS WITH ANSWERS

774Your associate has just found that he missed GDC renewal and not paid
GDC fee. What is your next action as hid employer? •
A) suspend •
B) no action, it is associate’s problem •
C) inform the GDC

Ans is A ?

775. . A 83 year old pt with dementia comes to your practice with discharge
pus from parotide gland and does not have a family, also she does not have
elected power of attorney, who can give a consent for her? •
A ) her close relative •..
B) her good friend •
C ) independent mental capacity advocate •
D) her GP •
E) her caregiver
Ans is C

776 A pt was referred to you for extraction of 5 by the ortho specialist in


prepration for ortho treatment. You mistakenly extracted the wrong tooth. What
is the consequence if you fail to report this incident? •
A) nothining if pt does not complain • .
B) erasure from GDC •
C) criminal offence •
D) suspension (// Ans is D ?)

777. Question about basic stands of Montgomery Consent..

778How many CPD hours for clinical dental technician? •


A) 50 • .
B )75 • .
C) 100 •

BY DR.ABDULRAHMAN ALMUALM
MJDF MCQS WITH ANSWERS

D) 150
Ans B

779What is not found in the lining of Residual cyst: •


A) plasma cell •
B) stratified squamous epithelium •
C) stratified columnar epithelium •
D )fibrous tissue •
E ) cilliated columnar epithelium
Ans E

780You are going to employ clinical dental technician, which of the following
procedure you expected from him to do independently? •

A)diagnose and treatment planning of perio disease •


B) complete denture without the consultation with dentist •
C) administrate LA prior the ridge augmentation
Ans is B

781. Calculation for sensitivity, you will be given the number in form of true
positive =40, true negative=10, false positive=5, false negative=20 ect and
need to remember formula to get a result!

782. Pt has an anaphylactic attack during the dental procedure, what type of
hypersensitivity it is? • A) type I

BY DR.ABDULRAHMAN ALMUALM
MJDF MCQS WITH ANSWERS

• B) type II •..
C) type III •
D) type IV
Ans A

783. You have given one dose of Prilocaine with felipressin to the pt, but he
is complaining of pain. If we know that maximum dose is 300mg how many
of 2.2 ml ampules we can give to the same patient safety? •
A ) 2.5 •
B )5 •
C )7 •
D)11
Ans B

784. 11 year old girl with multiple caries, what would you advice to your
patient from following? •
A) use 5000ppm fluoride toothpaste •
B ) use 2800ppm F toothpaste •
C ) topical fluoride of 11600 ppm 2 or more times per year

785. 11 year old girl with multiple caries, what would you advice to your
patient from following? •
A) use 5000ppm fluoride toothpaste •
B ) use 2800ppm F toothpaste •
C ) topical fluoride of 11600 ppm 2 or more times per year
Ans B

786. years old girl comes with the mother, she has good hygiene and no
caries, living in area with fluoridated water, mother is asking about toothpaste
and your recommendation is: •
A) do not swallow toothpaste •
B ) use fluoride free toothpaste •
C) rinse with a water after brushing •
D) do not rinse after brushing, just spit all
out and go to the bed
Ans is D

787. Biological width of periodontal


attachment : • ..

A) from most coronal part of JE to crest


of alveolar bine •
B) from gingival margin to crest of
alveolar bone • ..
C) from CEJ to JE

Ans is A

BY DR.ABDULRAHMAN ALMUALM
MJDF MCQS WITH ANSWERS

788. Definition of clinical governance – Framework through which NHS are


accountable to continuously improving the quality of their services and
safeguarding high standard of care………….etc. What does the statement
indicates? •
A) CQC •
B) clinical audit •
C) clinical governance
D) clinical operating services
Ans is C

789. Question about blue light ……… •


A)460nm
B)470nm •
C)480nm • .
D)490nm •
E)500nm
Ans D

790. Pregnant pt complaining about swelling of her gums. Upon examination


you have noticed that is a pregnancy epulis, which of the following is correct?

A) no treatment required, lesion will resolve by itself •


B) the presence of epulis is independent if pt oral hygiene •
C)surgical treatment mandatory after delivery
Ans is A

791. male pt comes for regular dental check up, but OPG shows OKC and
he said that he is previously was diagnosed with BCC. What condition it is? •
A) Gardener syndrome •
B) Gorlin-Gotz Sy •
C) Apert sy •
D)Frey’s Syndrome •
E) Marfan Syndrome
Ans B

792. A 6 years old child comes to you with fever, poor appetite, feeling of
being unwell, sore throat. 4 days later fever starts, painful rash, blister develop
at the hand, feet and buttocks. What is the causative agent? •
A) Herpes simplex •
B) paramyxovirus •
C) Coxackie virus •.
D) HPV 16

Ans C

793. Kenedy class II classification, which of the following design is most


suitable for optimal function? •

A) distal rest and I bar • .

BY DR.ABDULRAHMAN ALMUALM
MJDF MCQS WITH ANSWERS

B) occlusal approaching clasp •


C ) dental bar •
D) mesial rest and I bar
Ans is D

794. Which of the following about acrylic is true: • .


A) can be stained and designed • .
B) has higher strength compared to Co-Cr
C) has less hypersensitivity than Co-Cr •
D) less surface area required for tissue coverage
Ans Is A

795. An autistic pt comes to your clinic for follow up dental treatment, on his
previous dental visit another doctor prescribed fissure sealant, mother is nervous
and called the clinic asking about the procedure for her autistic son. What
would you provide? •

A) non invasive treatment •


B) prophylactic treatment •
C )filling
D) familiarization visit
Ans is D

796. A 80 year old pt with swelling, does not have capacity nor relative to give
a consent. What would be an appropriate management? •

A) do a treatment as long as she did nit refuse


B) do it in the best interest of the patient •
C) ask consent from her carer
D)ask colleague about this .

Ans B

797. 58 year old female patient complain of the pain in TMJ area, OPG
shows flat condylar procecus, pain is more intense in the morning, no muscle
tenderness, what would be diagnosis? • .

A) osteoporosis •
B)reumathoid artritis •
C) ostheoartritis •
D )anteriror disc displacement •
E)posterior disc displacement

798. Reason for failure of canine eruption without early loss of deciduous
teeth? •

BY DR.ABDULRAHMAN ALMUALM
MJDF MCQS WITH ANSWERS

A) supernumary •
B) ectopic •
C) crypt •
D)supplementary •
E) primary failure of eruption
Ans is C

799 Manifestation of Coelic disease in the mouth: •


A) mucosal tags •
B) pigmentation •
C) attrition •
D )abrasion •
E) hypodontia

Ans is A

800Picture of fractured condylar process of mandible

801. Picture of lower central incisors, fractured on the gum level, there is no
pathological changes in the bone but pt is on biposphonates for 5 years.. •
A)remove the teeth •
B)leave as it is •
C) provide an overdenture on the state how it is now •
D) provide an overdenture after you did RCT
Ans is D *

BY DR.ABDULRAHMAN ALMUALM
MJDF MCQS WITH ANSWERS

802. What is the level of evidence for expert opinion •


A) 1 •
B)2b •
C)5 •
D)4
Ans is C

803. You plan to do double blinded Randomised controlled trial. Once you
have finished design the trial, what do you do next? •
A. Advertise in dental research journal for participants •
B. Allow patient to choose the treatment they wish to receive •
C. Ensure potential patient understand the method of treatment allocation •
D. Inform interested patient and get their consent •
E. Seek approval from appropriate research ethics committee
Ans is E

804. Which medication is prescribed to overcome acute anxiety prior to dental


treatment? •
A. Barbiturates •
B. Benzodiazepines •
C. Monoamine oxidase inhibitors •
D. Selective serotonin reuptake inhibitors •
E. Tricyclic antidepressants

Ans is B

805. Which of the following is included in NHS? •


A. Right to choose named consultant •
B. Legally enforceable rights •
C. Right to access drugs and treatment recommended by doctor regardless of
cost •
D. Right to have complaints properly investigated and deal with
• E. Right to have the required treatment/ service locally

Ans is D

806. Which is best described suspected internal resorption for maxillary central
incisor that had trauma? •
A. Asymmetric, central lesion with discontinued canal • B. Asymmetric, peripheral
lesion with intact canal •
C. Round, central lesion with discontinued canal •
D. Round, central lesion with intact canal •
E. Round, peripheral lesion with discontinued canal
Ans C

807. Which probe do you use for detailed periodontal examination? •


A. CPITN probe •
B. Dental probe No 6 •
C. Nabers probe •

BY DR.ABDULRAHMAN ALMUALM
MJDF MCQS WITH ANSWERS

D. University of North Carolina probe


• E. Williams 14 W probe

Ans is E ?

808. What is the value of gift that must be recorded and kept at dental
practice? •
A. All gifts regardless of value •
B. No obligation to record •
C. > 20 GDP •
D. > 50 GDP •
E. >100 GDP

Ans is E

809. Which of the following acts by reversibly competing with vitamin K for
binding site with an enzyme essential for formation of clotting factors II, VII, IX,
X by liver? •
A. Aspirin •
B. Clopidogrel •
C. Dabigatran •
D. Heparin
• E. Wafarin
Ans is E

810.One more question about ALL but they gave figure of lymphocites,
leukocites, hemoglobin ect.. Make sure you know normal blood count range
figures….(ALL=ACUTE lymphoid leukaemia)

BY DR.ABDULRAHMAN ALMUALM
MJDF MCQS WITH ANSWERS

BY DR.ABDULRAHMAN ALMUALM
MJDF MCQS WITH ANSWERS

811. comes with the career and they have an verbal argue and you have
noticed that caeer were aggressive towards client, what would you do; •
A) ignore •
B) report to the police •
C) ask them politely to book next appointment •
D) report to local safeguarding services •
E) report to care home
Ans is D??

812. Placing canula for iv sedation, patient say pinky finger pain, what structure
injured??
#!

813. To ensure high quality of health care that is provided in line with standards the
NHS recommend:
a) clinical governance
b) clinical effectiveness
c) clinical audit
d) weekly peer review
Ans is C

814. framework through which UK National Health Service (NHS) organisations and
their staff are Accountable for continuously improving the quality of patient care
a) clinical governance
b) clinical effectiveness
c) clinical audit
d) weekly peer review
ans is A

815. Patient with history of removal of multiple intestinal polyps has multiple
supernumerary teeth is going to do a panorama so what do you expect:
a) Multiple cysts
b) Multiple osteoma
c) Multiple abscess
d) Multiple bony defects
e) Multiple exposed teeth
ans is B

816. Patient is presented with wear facets due to bruxism and you want to make an
appliance for him. how would he pay?
a) 21.6 pounds
b) 59.10 pounds.
c) 256.5 pounds
d) No claim
ans is B??

817. Patient needs to make sports guard, how would he pay?


a) 21.6 pounds
b) 59.10 pounds

BY DR.ABDULRAHMAN ALMUALM
MJDF MCQS WITH ANSWERS

c) 256.5 pounds
d) No claim

818.most aggressive Carcinoma of the mouth :


A floor of the mouth
B. Lip
C.palate
Ans A??

819. To ensure high quality service, the general dental practice should audit:
a) Weekly checks for emergency medical equipment and drugs
b) Daily checks for emergency equipment and drugs
c) Monthly checks for emergency equipment and drugs
d) Check the equipment annually and the drugs in its expire date
e) Follow the manufacturer instructions of every single items
Ans is A

820. 55 years old female patient with history of hip fracture and osteoporosis taking
oral bisphosphonates from about 2 years, no other medications. She want to extract
a tooth:
a) Extract with no alteration but advice to keep her mouth clean
b) Stop the drug 3 days before extraction after consultation with GP
c) Consider 3 months drug holiday after consultation with GP
d) Never to extract, RCT and crown amputation
e) Extract under antibiotic cover
ans is A??

821. The antibiotic of choice for MRSA infection is


: a) Clindamycin
b) Erythromycin
c) Vancomycin
d) Antiviral drug
e) Gentamycin
Ans is C

822. Pregnant and lactating mother with swelling in the mandibular region &
submandibular lymphadenopathy she is allergic to penicillin, what type of antibiotic
should you prescribe
a) Clindamycin
b) Co-amoxiclav
c) Erythromycin
d) Metronidazole
e) Metronidazole and co-amoxclav
ans C

BY DR.ABDULRAHMAN ALMUALM
MJDF MCQS WITH ANSWERS

821. Regular attending patient who is epileptic and usually have fits in the clinic what
is the medication that must be present?
a) Phenytoin
b) Midazolam
c) Diazepam
d) Carbamazepine
e) Sodium valproate
ans B

822. What is the next step after calling for help (999) in BLS
: a) 30 chest compressions
b) 2 rescue breaths
c) 5 rescue breaths
d) 20 chest compressions
e) Shout for anybody help

Ans is A

823. Patient with candidiasis and taking warfarin so the antifungal of choice is:
a) Nystatin
b) Fluconazole
c) Amoxycilin
d) Metronidazole
e) Miconazole
Ans A

824The concept of staff appraisal is to provide

a) Good communication between patients and staff


b) Improve the performance and increase patient outcome
c) Accepting new employee after detailed interview
d) Rejecting new employee after interview
e) Enhancing patient communication and decreasing treatment expenses
Ans A

825. The best time for staff appraisal is


a) 3 Months
b) 6 Months
c) 12 Months.
d) 18 Months
e) 24 Months

Ans is C

826. dose of miconazole as antifungal

a) 100 mg once daily


b) 150 mg twice daily
c) 250 mg once daily
d) 200 mg twice daily

BY DR.ABDULRAHMAN ALMUALM
MJDF MCQS WITH ANSWERS

e) 50 mg once daily
Ans is E

827. Patient with high arched palate, aortic aneurism and tall, slim legs

a) Treacher Collin syndrome


b) Marfan syndrome
c) Gardner syndrome
d) Peutz jhegers syndrome
e) Plummer venson syndrome
ans is B

828. After avulsion trauma for 10 years old child, the survival and prognosis of
periodontal tissue is dependent mostly on: a) Stage of root development
b) Contamination of root surface
C.Extra-alveolar dry time
d) Immunization of the child
e) Open or close apex
Ans is C

829. patient asks for his records what is the best action:
a) pay fees and give records in maximum 20 days
b) pay fees and give records in maximum 40 days
c) no fees and give records in maximum 20 days
d) no fees and give records in maximum 40 days
ans is B

830. The required CPD points for dentist to be completed every 5 years:
a) 50 CPD
b) 75 CPD
c) 100 CPD
d) 150 CPD
e) 200 CPD

Ans is B

831. The minimum CPD points a dentist can provide to the GDC for renewal over 2
years: a) 10 CPD
b) 20 CPD
c) 50 CPD
d) 5 CPD
e) zero

832. Patient with liver disease and dental abscess, antibiotic of choice is:
a. clindamycin
b. doxycycline
c. ceftriaxone
d. erythromycin
e. ciprofloxacin
Ans is C

BY DR.ABDULRAHMAN ALMUALM
MJDF MCQS WITH ANSWERS

833. Which of the following drugs interacts with warfarin and increases the patients
INR?
a) Fluconazole
b) Cephalosporin
c) Tetracycline
d) Nifipedine
e) Conaz
ans A

834. Patient faint on the dental chair after finishing long dental procedure the most
likely diagnosis is
a) Vasovagal attack
b) Epilepsy
c) Cardiac arrest
d) Diabetic coma
e) Anaphylactic shock
ans A

835. Patient taking apixaban/Dabigatran needs extraction do which test will be


suitable: a) INR
b) Bleeding time
c) APTT
d) PT
e) Non-needed
ans A

836. Patient taking apixaban/Dabigatran have done extraction but complains of


continuous bleeding. What is the reversal agent that may be suitable:
a) Vitamin K
b) Protamine sulphate
c) Ca gluconate
d) Flumazenil
e) No reversal
Ans is E

837. Dental nurse can work on the patient without prescription :


a) Take radiographs
b) Place rubber dam
c) Remove sutures
d) Shade taking
e) Take impression
ans D

838. In line with the mental capacity act, a person is regarded as being unable to
give consent or make decisions if he is unable to do [4 steps to assess the mental
capacity]

a) Understand, retain, weight and communicate the information


b) Understand, retain, weight and repeat the information

BY DR.ABDULRAHMAN ALMUALM
MJDF MCQS WITH ANSWERS

c) Understand, retain, weight and write the information


d) Understand, retain, weight the information and write his name
e) Understand, retain, weight the information and state his address
Ans A

839. cleido-cranial dysplasia is not associated with [in the head and neck]:
a) Supernumerary teeth
B) delayed closure of fontanels-hypodontia
c) prominent clavicles
d) posing of forehead
e) dentigerous cyst formation
ans is C

840. according to the mental capacity act, advance decisions can be made by
people who lack mental capacity who are:
a) above 16
b) above 18
c) below 16
d) below 18
e) doesn’t matter
ans B?

841. During clinical audit which of the following may lead to direct dismissal without
warning
a) No enough informed consent
b) No enough confidentiality
c) No enough staff available
d) incorrect hiring of staff member
e) Wrong extraction
ans B

842. Which of the following is considered “never events” that may lead to series
incidents which may lead to closing the practice permanently?
a) Extraction of wrong tooth [wrong side surgery]
b) Causing temporary blindness
c) Causing perforation
d) Causing temporary paralysis
Ans A

843. Patient with multiple missing teeth and sparse hair


a) Cleidocranial dysplasia
b) Ectodermal dysplasia
c) Gardner syndrome
d) Crouzon syndrome

Ans B

844. According to the recent recommendations for infection control the floor of the
practice should be :
a) Wipe-able

BY DR.ABDULRAHMAN ALMUALM
MJDF MCQS WITH ANSWERS

b) Shiny
c) Wooden
d) At 90 degrees to the walls
e) Impervious
Ans is E

845. After doing pulpectomy, you performed a readymade stainlesteal crown and
space maintainer So under which band you should treat this child:
a) Band1
b) Band 2
c) Band 3
d) Band 4
e) No claim
ans C

846. nurse caught using nitrous oxide in the clinic


a) verbal warning
b) written warning
c) suspense, investigate and if proven dismiss for gross misconduct
d) suspense, investigate and if proven send warning letter
e) dismiss directly
ans C?

845. Your assistant called you Friday morning and said that she just realized that her
GDC license has expired and she forget to pay for renewal. What should be the most
appropriate action?
a) Immediately inform the GDC
b) Tell her to work under supervision until she pay
c) Tell her that it is her responsibility
d) Pay for her
ans C??

846. 70 years old incapacitated patient with dementia suffering from parotid gland
infection and pus oozing who needs facial surgery. He is unable to consent and lost
power of attorney so according to mental capacity who will give consent:
a) Patient young carer
b) 1st degree relative
c) Solicitor
d) Independent mental incapacity advocate
e) Patient’s friend
Ans C

847. According to the recent infection control recommendations, the partially charged
local anesthetic cartridge should be disposed via: a) Clinical wastes container
b) Sharp hazardous wastes container
c) Amalgam container
d) Fixer container

BY DR.ABDULRAHMAN ALMUALM
MJDF MCQS WITH ANSWERS

e) Non clinical waste


Ans B

848. Oral steroids are commonly used for treatment of chronic disease like COPD,
which of the following conditions may be exacerbated by the use of steroids
a) Asthma
b) Psoriasis
c) Diabetes
d) Inflammatory bowel disease
e) Systemic lupus
ans C

849. During decontamination stage; the temperature of the used water shouldn’t
exceed
a) 25°
b) 35°
c) 45°
d) 55 °
e) 65°
Ans C

850. The antibiotic of choice after traumatic avulsion


: a) Amoxicillin
b) Doxiclyine
c) Clindamycin
d) Metronidazole combined with amoxicillin
e) Erythromycin
Ans is B

851. Who of the following should be registered with GDC other than dentist?
a) Practice manager
b) Laboratory technician
c) Laboratory technician assistant
d) RPA
e) Dental receptionist
ans B

852. the least numbers of bacterial colonies in DUWLs


a) less than 20
b) less than 200
c) less than 2000
d) less than 20000
e) less than 200000
Ans B

853. According to RIDDOR if an accident causes an employee to remain absent


from work this must be notified to the Health and Safety executive within seven days.
What’s the period of absence that necessitates this action
a) One day
b) Two days .

BY DR.ABDULRAHMAN ALMUALM
MJDF MCQS WITH ANSWERS

c) Three days
d) seven days
e) Five days
Ans D

854. Matrix band should be disposed in:


a) Clinical waste
b) Sharp waste
c) Hazardous waste
d) Trade waste
e) Nonclinical waste

Ans B

855. Which of the following fire extinguishers should be present in the dental clinic to
control electrical fire:
a) Carbon dioxide powder
B) Wet chemical agents
C) Dry powder
D) Water supply
E) Foam
Ans A

856. Extracted teeth for educational purpose are stored in :

a) Formalin
b) Water
c) Sodium hypochlorite 10% diluted
d) Distilled water
e) Glutaraldehyde
Ans C

857. Extracted teeth with amalgam restoration for educational purpose are best
stored in:
a) Formalin
b) Water
c) Sodium hypochlorite 10% diluted
d) Distilled water
e) Glutaraldehyde
Ans is A

858. Patient has TIA [transient ischemic heart disease], you have planned extraction
and the patient is on aspirin, what is the best way to proceed
a) Delay the extraction
b) Refer to GP
c) Refer to oral surgeon
d) Go ahead with extraction but with local measures

BY DR.ABDULRAHMAN ALMUALM
MJDF MCQS WITH ANSWERS

e) Stop aspirin for 10 days then extract normally


Ans D

859. First thing to do if patient faint on the dental chair during treatment on supine
position :
a) Give Adrenalin .05 ml of 1:1000 intra-muscular
b) Call the ambulance
c) Shout for help
d) Check airway, pulse rate and Start ABCD approach
e) Open airway
ans D

860. Gingival enlargement is caused by:


a) Aspirin .
b) NSAIDs
c) Corticosteroids
d) Benidipine
e) Azathioprine
Ans D

861. What will be the best action for a patient with prolonged epileptic seizures :
a) Give him o2
b) Give him midazolam IV
c) Give him midazolam IM
d) Give him midazolam Bucally
e) Give him diazepam
ans D

862. 28 years old female patient complaining of pain in lower jaw, you took xray to
find a fractured mandible. She then admitted that her partner hit her while he was
drunk, she has good capacity

a) report to police
b) safeguard her to social service
c) refer to community service
d) refer to hospital
e) respect her confidentiality
#! Either e or B ..????

863. Which of the following lap investigation is important for a patient taking aspirin
a) PT
b) PPT
c) Bleeding time
d) Clotting time
e) INR
ANS C

864. You realized that the patient is oversedated during IV sedation with midazolam
a) Flumazenil

BY DR.ABDULRAHMAN ALMUALM
MJDF MCQS WITH ANSWERS

b) O2
c) Diazepam
d) Glycril nitrates
e) Adrenalin IM
ans A

865. Patient entered to automated external ventricular defibrillator, what is your next
step
a) Administer adrenaline
b) Dc shock 150/200
c) Assess the rhythm
d) Don’t continue or do
anything
e) Start CPR immediately
Ans C

866. 14 years old girl had


a sick leave from school
claiming that she had to
visit her dentist for
treatment, the school
secretary called the dentist
to confirm that she has
gone to dental practice,
what is the best action the
dentist can do

a) call the girl or her


parents
b) refuse to disclose the
patient confidential
information
c) gives a clear answer
d) ask a colleague for
advice
Ans B?

867. Patient who can’t


speak English came to
you with his relative to
translate to him, they
argue with each other, he
was making a simple
filling, what will you do?

a) ask the relative to


continue to translate and you continue your work
b) ask a colleague to translate and you continue your work
c) you proceed to work without translation
d) you ask them to end the appointment and come after fight end

BY DR.ABDULRAHMAN ALMUALM
MJDF MCQS WITH ANSWERS

e) cancel the appointment to hire formal interpreter


ans is E

868. 8 years old child has extracted his permanent 1st molar, how long should the
dentist keep the records
a) 11 years
b) 25 years
c) 8 years
d) 17 years
e) 50 years
Ans is D

869.
For a patient with ulcer, the most common area in which the ulcer may chan
ge to malignancy is:
A)Upper eye lid
B)Lower eye lip
C)Lower lip
Ans C

870. on sialography u note area of stenosis and diltation of parotid duct?

A)Obstructive parotitis
B)Sialolith
C)Sialadenitis

Ans A

871. titurated dose midazolam given as:

A)bolus 1 mg midazolm
B)bolus 2 mg midazolam
C)no midazolam
Ans B

872. pt recieve ttt in private where to complain?

BY DR.ABDULRAHMAN ALMUALM
MJDF MCQS WITH ANSWERS

A) Dental Complaints Service

873. Female with visual and jaw claudcaiton ?


A.Gaint cell arteritis

874. Dosemeters used to calculate?

A)Effective dose.
B)Absorbed dose.
Ans B

875. clopidegrel act


A.ADP
B)ATP.
C)Cyclooxgenase
Ans A

876. autoclave checking?


A)6 months
B)8 months
C.12 months
D)18 months
E)30 months

Ans C

877. what if the solely caused by lignocaine toxiity:


A)Dizzines
B)tremor
C)twitching
Ans C

878.patient 27 years
old good oral hygiene low caries risk don't need any ttt recall is every ?
A)6 months.
B)12 months
C)18 months
Ans C

879. pt with crusted lip.,,intraoral ulcer..gingiva spare?


Erythemia multiforme

880. Que about splinting midroot fracture?


A)flexible splint for 2 week
B)flexible splint for 4 week
C)fixed 2 week
D)fixed 4 week
e)24 months
ans B

BY DR.ABDULRAHMAN ALMUALM
MJDF MCQS WITH ANSWERS

881
if u know that the risk of cancer from panoramic radiograph is 24 then the ri
sk from CBCT is how much times multiple than panorama?
A)0.5_1 times
B)2_42 times C)200_300
#!#8
882. percentage of smoker develop cancer more than non smoker?
A)34
B)7-10
C)14-17
Ans B

883. about direct retention:


A)it is well provided by extracoronal retainer
B)it is divided into primary and secondary retention. .

Ans A??

884. pt unconsious,,, After shouting for help what will u do?


A)start CPR
B)look, listen, feel breathing
C)chin lift
Ans C

885
during u doing surgical extraction of lower 8 there was massive bleeding ,, ex
traction was difficult ,..pt came to u with numbness in lower lip & tongue ner
ve damge after surgical extraction what u do?
A)reassurance and review 6 months later
B)refetr to OMFs
C)prescribe dexamethadone & review later
Ans B

886. prolonged use of Steroid cause? (thining of skin not in option)


A)weight loss
B)Diabetic type 2
Ans B

887. Pts who are under conscious sedation are:

A)Conscious, deactivated protective reflex, respond to verbal command.


B)Conscious, activated protective reflex, respond to verbal command
C)Conscious, deactivated protective reflex, doesn’t respond to verbal command
D)Unconscious, activated protective reflex, unresponsive to verbal command
E)Unconscious, dactivated protective reflex,responsive to verbal command
Ans B

888. radiographic x ray equipment check should be done every?


A)Annually

BY DR.ABDULRAHMAN ALMUALM
MJDF MCQS WITH ANSWERS

B)3 year)
C)5 years
Ans B???

889. dental practices should audit their decontamination processes?


A)6 months
B)12 months
Ans A?

890. Missed lower centrals teeth , What is the bridge of choice:


A)convention Fixedfixed bridge
B)RBB on LL2 and LR2
C)Cantilever convential bridge on LL2 and LL3
D)Separated 2 RBBs on LL2 and LR2
Ans A

891.
lignocaine stored out of refrigator by how months u will reduce the shelf life
of the drug?
A)6 months
B)1 year
C)3 years
Ans A

892.
Patient with long term dietary erosion, most probable clinical sign would be :

A)wear facet on labial surface


B)stained teeth
C)teeth sensitivity
D)matching wear on occlusal surfaces
E)fractured cusps
Ans A ? “c also right “

893.
calcification of seconed permenant molar to ensure maximum closure after extr
action of first permenant molar :
A)early dentin calcification at bifurcation of lower molar
B)early calcification of dentin at coronal part of root
C) mid root calcification
Ans A

894. what cause oral pigmentation?is minocycline


True
False
True

895. child 5 years , pale and weak came with spontaneous gum bleeding?
A. ALL
B. AML

BY DR.ABDULRAHMAN ALMUALM
MJDF MCQS WITH ANSWERS

C. Vit k deficiency
D. Von willebrand disease
E. All of the above
Ans A

896. During clinic audit, what can caused dismission before warning?
A)breach in confidentiality
B)Uncorrect hiring of staff
C)Not enough staff
Ans is A

897. child had 5mg/kg flouride its potential lethal dose whats the first line treatment?
Give child salty drink
Give child sugary drink
Give child injection i cant remember the name of the drug

Ans is A ??
(Antidotes: < 5mg F/kg body weight= large volumes of milk.
>5mg F/kg = refer to hospital for gastric lavage.
If any delay = IV Calcium Gluconate + An Emetic )

.898 A Dentist did pulpotomy of primary molar and then after few moths he did
stainless steel crown. What band he gets to claim.
Band 1
Band 2
Band 3
Band 4
No Claim
Ans B

.899 After doing mesio-angular impaction of lower third molar, which suture
material is best used to close mucoperiosteal flap A.3-0 Silk cutting B.needle
3-0 Vicryl C.cutting needle
Catgut
Ans B

.900 Patient has TIA (Transient ischemic attack three months ago, you have
planned extraction and patient is on aspirin. What is the best way to proceed?
A.Delay the extraction for three months
B. Go ahead with extraction, following appropriate local measures
C.Refer to Oral surgery
DStop aspirin
Ans B

901Patient has an over jet of above 9mm which UDA band it will be
Band 1
Band 2
Band 3
Band 4
Band 5

BY DR.ABDULRAHMAN ALMUALM
MJDF MCQS WITH ANSWERS

Ans C

902Patient has an over jet of above 9mm which UDA band it will be
Band 1
Band 2
Band 3
Band 4
Band 5

903Over jet of 4.5 mm criteria which UDA band it will be


Band 1
Band 2
Band 3
Band 4
Band 5
Ans A
only score 4 and 5 iotn will be nhs treatment

904Autoclave temperatures (at what temperature the sterilization is


achieved)

121 degrees centigrade for 15 minutes


121 degrees centigrade for 3 minutes
131 degrees centigrade for 15 minutes
Ans A

905A patient with irritable bowel syndrome what do you see.


Mucosal blistering
Mucosal tags
Herpitiform ulcers
Ans C

.906 Patient had pain in the chest after prolonged dental session, with an
extraction procedure, what is the best medicine to give
GTN (nitroglycerin)
ASPIRIN
Ans A

.907 Patient is insulin dependent diabetic, complains of faint after prolonged


dental session, what is best to give
Oral Glucose
IV Glucagon
Insulin
1M Glucagon
Ans D =if unconscious

BY DR.ABDULRAHMAN ALMUALM
MJDF MCQS WITH ANSWERS

Treatment If conscious, give glucose orally in any available form. Repeat if


necessary every 10–15min. Get help. If impaired consciousness, give buccal
glucose gel and/or1mg of IM glucagon. If unconscious, protect airway, place in
recovery position. If the correct kit is available, establish IV access and give up
to 50mL of 20–50% glucose.

908Patient is waiting in waiting room and collapse and faints. Patient is cold,
clammy, but pulse is good. What do you give?
IM Glucagon
Oral Glucose
GTN
Hydrocortisone
Diazepam

Ans B (patient is conscious)

909Patient is suffering myocardial infarction in dental chair, which position is


best for patient

Upright sitting position


Laying flat
Recovery position ➔ after epileptic seizures place patient in recovery position
Lay the pt flat and raising legs
Ans A

910A 90-year old gentleman presented to clinic who is edentulous and


has dentures upper and lower full 15 years old. Denture bit uncomfortable,
tooth structures little bit worn out, freeway space 2-4mm, polished
surfaces satisfactory
and occlusal wear
minimal. How would you
proceed?
Copy dentures
Hard reline
Soft reline
Construction of new
dentures
Ans is A

911When do not extract


impacted 8's
a.Extraction of 8's
recommended by
orthodontist
B.Extraction of 8's
recommended in patient
undergoing renal
transplant -
immunosuppresent

BY DR.ABDULRAHMAN ALMUALM
MJDF MCQS WITH ANSWERS

C.47years old with moderate periodontitis D.Second permanent molar undergoing


internal resorption due to third molar
Ans C

912Non dental origin, No clinical or radiological findings. Patient complains


of pain maxillary sinus and ear
deafness, unilateral ulcer on
face and forehead
Herpes Simplex
Herpes Zoster
Paramyxo virus
Orthomyxo virus
Ep-stein bar virus
Ans B

.913 How do you check maxillary


plane

A Fox-bite-plane
B OCcluding the upper and lower denture
Ans A

.914 What sort of matrix is best for restoring disto-occlusal restoration of 7?

A.Sectional matrix to protect neighbouring tooth


B.Auto matrix
C. Tofflomeir matrix to apply material
Ans A

.915 How do you get incisal


guidance for setting incisal
teeth?
A.Study casts
B.Unsupported/supported labial
fullness
Ans B

BY DR.ABDULRAHMAN ALMUALM
MJDF MCQS WITH ANSWERS

BY DR.ABDULRAHMAN ALMUALM
MJDF MCQS WITH ANSWERS

916Burning mouth syndrome which part of the oral cavity is more painful

#Tongue

917Lichenoid reaction seen


associated with
Atenolol
Ramipril
Ans A

918GTN in Angina
a.Relaxes heart muscle
b.Decreases blood Pressure
C. Increase positive ionotropic
charge
D. Dilation of arterioles and
venules
Ans D

.919 had trauma - Nasal- midface area and


has watery discharge and blood
discharge from nose

BY DR.ABDULRAHMAN ALMUALM
MJDF MCQS WITH ANSWERS

A.Orbital fracture
b.Zygomatic fracture
c.Lefort-l
D. Lefort-ll
Ans D

920Patient with only mid-root fracture of


front tooth, what is apropriate splinting
time
1 week
2 week
4 week
3 months
Ans is C

921Patient with lower four central incisors


with alveolar fracture, you splinted and
immobilized successfully, and then what is
the next thing to do.
A. Soft diet and review

.922 Patient had trauma with minimal mobility of front teeth, no symptoms or
pain or anything, what is the best approach

Splinting one week


Splinting two weeks
Splinting 4 weeks
Splinting 6-8 weeks
Soft diet and review ~
Ans E

.923 Patient had trauma 8 days ago, upper central incisor palatal luxation mild,
not interfering with occlusion but tooth non vital. What do you do?

A Pulp extirpation +CaoH2 dressing


B.Reposition
C.Reposition +Splinting ~
Ans A

924Best material for using vital pulpotomy ..

,Ferric sulphate
Formocresol
Ans A

925Patient had trauma on 14 (vital pulp), only thin buccal cusp remaining, best
treatment option
MOD-Onlay

BY DR.ABDULRAHMAN ALMUALM
MJDF MCQS WITH ANSWERS

inlay

Direct composite MODL

Extract and options for replacement


Ans A

92618 months of age, what is fluoride supplement for child living in 0.25 ppm
fluoridated water?
0.25mg
0.5 mg
1 mg
Ans A

927Six-year-old - Non fluoridated water, what is the fluoride supplement dose


0.25 mg
0.5 mg
1 mg
Ans B

928Six-year-old child with fluoridated water of 0.6ppm , what is the fluoride


supplement dose given
0.25 mg
0.5 mg
1 mg
Ans A

9293-year old with fluoride concentration of 0.3 ppm fluoridated water, what is
the fluoride supplement dose given
0.25 mg
0.5mg
Ans B
930Duraphat in high caries risk group children is applied every
2 months
4monttls ~.
6 months
8 months
One-yearly
Ans B ?

931Reason for failure of eruption of lower permanent molar

A Canine palatally place


B Crowding in premolar area
C Failure of eruption of central incisor
d.Absence of showing lateral incisor
Ans B

932Failure of normal path of eruption

BY DR.ABDULRAHMAN ALMUALM
MJDF MCQS WITH ANSWERS

Abnormal position of crypt


Early loss of deciduous teeth
Supernumerary tooth
Ana A

933Cervical line and dark lines on Central incisors, molars and incisal edges,
most possible reason could be
Ameologensis
Dentinogenesis
Childhood illness
Hypoplasia
Ans C

934An edentulous patient presents with ulcer in retro molar pad area first line
of management would be

Adjusting the denture


Incisional biopsy
Periapical xray
Ans A

935Possible infection that would occur from needle stick injury in a person
who is appropriately or properly immunized
a-Hepatitis B
b-Hepatitis C
c-Hepatitis A
d-HIV
Ans B
30% chances for HBV incase of non immunised person exposed to infected blood,
3% HCV and 0.3% HIV

936Got a call from mother of a 3-year old child, who had trauma and avulsed
the front tooth. What instruction you will give to

A. Attend Surgery as soon as possible with no special consideration to the


tooth
B.Place tooth in milk and attend surgery as soon as possible
C.Place in the salt water and attend surgery as soon as possible
D.Place the tooth in its position back and attend surgery as soon as possible
Ans A

937A patient was diagnosed with salivary gland malignancy which has infiltrated
through the
nerves quickly:
a) Muco-epedrmoid carcinoma
b) Adencystic carcinoma
c) Pleomorphic adenoma
d) Warthon’s tumour
Ans B

BY DR.ABDULRAHMAN ALMUALM
MJDF MCQS WITH ANSWERS

938A mother has a newly born baby with cleft palate, what to tell her about the
condition?
a) It affects 1:140 of population
b) It is due to genetic factors
c) It is inherited condition
d) It is more common in males than females
e) Multifactorial due to genetic and environmental causes

Ans E

93913 year old patient with non-crowded arch and good occlusion except for
palatally
displaced upper lateral incisor:
a) Leave it until puberty
b) Fixed appliance orthodontic therapy
c) Removable orthodontic appliance
d) Correct with crowns
e) Surgical intervention for repositioning
ans C

94013-year-old patient with non-crowded arch and good occlusion except for
palatally
displaced upper lateral incisor in crossbite, which component will you use in your
orthodontic appliance?
a) Z spring
b) T spring
c) Z spring with posterior bite-plan [posterior capping]
d) Howley appliance with labial bow
e) Expansion screw
ans C

941For a patient, only lower first premolar didn’t have BOP, there are 4 teeth in that
sextant. What is the BPE score for this sextant :
a) BPE 0
b) BPE 1
c) BPE 2
d) BPE 3
e) BPE 4
Ans B

.942 What is the referral time for suspicious cancer lesion :


A) 2 weeks
B) 3 weeks
C) 1 week
D) 10 weeks
E) 12 weeks
Ans A

943Patient presented with hemorrhagic erythematous ulcers and crusted lips, the
diagnosis may be?

BY DR.ABDULRAHMAN ALMUALM
MJDF MCQS WITH ANSWERS

a) Erythema multiform
b) Pemphigus vulgaris
c) Mucous membrane pemphigoid
d) Systemic lupus erythematosus
e) Liver cirrhosis
Ans A

944) The most common type of supernumerary is:


a) Tuberculate tooth
b) Conical tooth
c) Supplemental tooth
d) Odontoma
Ans B

945Which of the following is the right sequence of primary teeth eruption?


a) ABCDE
b) ABDCE
c) ABDEC
d) ABEDC
Ans B

9468 years old child has been


diagnosed with delayed eruption of
upper central incisor
due to supernumerary tooth, which of
the following factors will not affect the
eruption
process of the permanent incisor:
a) Type of supernumerary
b) Number of supernumerary
c) Stage of root formation
d) Presence of crowding
ans C

947Patient diagnosed with mucous membrane pemphigoid, what do you expect to


find
intraorally?
a) Mucosal tags
b) Wickham’s stria
c) Symplyphron and desquamative gingivitis
d) +ve nikolosky sign
e) Itching marks
ans C?

948Graft which is bovine based is called


a) Allograft
b) Alloplast
c) Xenograft
d) Autograft
Ans C

BY DR.ABDULRAHMAN ALMUALM
MJDF MCQS WITH ANSWERS

949the feature best supporting inhalation sedation over the oral sedation is
a) can be titrated according to patient response
b) can be titrated according to painful stimuli
c) can be titrated according to patient age
d) doesn’t require anesthesia
e) can be reversed in case of over-sedation
Ans A

950
what best describes sedation?
a) patient conscious, protective reflexes retained, can respond to verbal
b) patient unconscious, protective reflexes lost, cannot respond to verbal
c) patient conscious, protective reflexes lost, can respond to verbal
d) patient conscious, protective reflexes lost, cannot respond to verbal
e) patient conscious, protective reflexes retained, cannot respond to verba
ans A

951. Nabers probe is used for:


a) Measuring BPE
b) Measuring probing depth in full periodontal charting
c) Measuring bleeding amount
d) Measuring furcation involvement
e) Measuring plaque scor
ans D

.952 The 1st colored band in the nabors probe used for measuring furcation:
a) 3-6 mm
b) 3.5-5.5 mm
c) 2-4 mm
d) 4-6 mm
e) 8-11 mm
Ans B??

953The treatment of choice for a patient with BPE 4* is:


a) OHI and scaling
b) Flap surgery
c) OHI, scaling and root planning
d) OHI, scaling, surgical root planning and GTR
e) Scaling followed by tunnel surgery under strong cover of antibiotic
Ans D

954A 35 years old patient complains of swollen gums, present for several years this
is:
a) Cyclosporine associated enlargement
b) Chronic gingivitis
c) Vitamin c deficiency
d) Atenolol therapy
e) Pregnancy gingivitis
Ans B

BY DR.ABDULRAHMAN ALMUALM
MJDF MCQS WITH ANSWERS

955Dentists start serial extraction process when the child at:


a) 8 ½ to 9½ years
b) 10 years
c) 12 years
d) 7 years
e) 6 to 7 years
Ans A

.956
The treatment of choice for a patient with BPE 3 should be:
a) OHI and scaling
b) OHI, scaling and root planning
c) Flap surgery
d) OHI, scaling and restore faulty restorations
e) Refer to specialist
Ans B

957A child of 8 years old come suffering from impaction of upper 1st molar and the
upper
E is still in place so the correct action is:
a) Extract E
b) OHI and review for self-correct
c) Refer to orthodontic for disimpaction
d) Extract E and remove 6 as well
e) Put crown on E and review
Ans B???

958A 3-year-old boy avulsed 61, and mother re-implanted it in its socket and
clot is holding the primary tooth. IOPA shows permanent tooth present, what
would you do
A. Leave it and review
B.Splinting for one week
C. Remove and space maintenance
Ans C

.959 Patient has painful symptoms at the time of eating and also has ear pain,
what x-ray would you consider. (The option of occlusal x-rays was not given in
the choices)

OPG
Oblique Lateral
Lateral chepalograph Postero-anterior view
Ans B

960A child has grossly broken down upper and lower molars, carious tooth,
which x-ray would you consider
IOPA

BY DR.ABDULRAHMAN ALMUALM
MJDF MCQS WITH ANSWERS

Bitewings_ .
Vertical Bitewings
DPT
Ans is D

961A dental practitioner can be


1-Radiologyadviser+physicst+operator
2-Superviso r + adviser+physicst
3-practioner+refer+operator
4-Practioner + physicst +supervisor
5-Physicst + supervisor + 0perator
Ans 3

962Risk of cancer in dental Radiography periapical


● 1:20,000
● 1:200,000 .
● 1:2,000,000 in essential
● 1:20,000,000 in modern
Ans is D

963When preparing the Apical Zone, the use of the files sequentially from apex
to backwards (lower the size of instrument) - what is the best distance to
achieve good apical area preparation
0.5mm
1.0mm
l.5mm
2.5mm
3.5mm
Ans is B

964Best material to use in the root caries and buccal caries in elderly patient.

Hybrid Composite
Amalgam
Silicate cement
Zinc phosphate
Ans is C

965A diabetic patient complains of pain on a root canal treated tooth. Root
canal is done 2 years ago. On examination the restoration on the tooth seems
leaking. Reasons for pain and failure
A.No proper obturation
b.Medical condition
C.Lack of coronal seal Inadequate
D.biomechanical preparation
Ans C

966Best success of root canal is achieved by

Straight canals

BY DR.ABDULRAHMAN ALMUALM
MJDF MCQS WITH ANSWERS

Straight line access


Widening of aperture of root canals
Proper irrigation all the times under presence of rubber dam
Ans is D

967Prolonged use of corticosteroids in adults may result in or can cause


Alopecia
Insulin - diabetes
Libidos
Thinning of skin
Less arousal
Ans D If its topical steroids-then thinning of skin If systemic steroids-then drug
induced hyperglycemic state (diabetes)

968A child patient is brought to the dental clinic, parents complain of child having
trauma and has fallen down, hurting his teeth. You suspect NAI's. Which is the most
appropriate finding which would give suspension of NAI'S?

Red eye
Neck bruise
Child agitated and distressed
Injuries to shin and knee areas
Ans B

969What is the bpe score if gingival recession of 2mm and band completely
disappears
1
2
3
4
4* due loss of attachment >7
Ans E

970What is the bpe score if Gingival recession 2mm +5.5mm


1
2
3
4
Ans D

971Patient has damaged infra-orbital nerve - orbital floor fracture, patient


presents to you after two hours later. What symptoms you would find, choose
most appropriate one

Peri-orbital edema
Loss of visual acuity
Loss of sensation or paresthesia over cheek
Ans is C

972For retention of post which is not important

BY DR.ABDULRAHMAN ALMUALM
MJDF MCQS WITH ANSWERS

Diameter
Size
Luting cement
Type of post
Ans C

973RCT treated tooth, post placed, which is the best option to restore the
tooth. Nayyar technique used, good tooth structure 50%

Gold crown
Gold inlay
PJC
Gold Onlay
Ans A

974The movement of tongue on protrusion is affected to lateral side -which


nerve is involved. Patient recently had surgery and radiation close to
submandibular gland

Lingual
Glossopharyngeal
Hypoglossal
Chordatympani
Ans is C

975When patient complaints about a treatment and unhappy about the


procedure and gives a complaint, in how many days you have to acknowledge
the complaint
1 day
2 days
3 days
10 days
20 days
Ans C

976Patient has cardiac arrhythmias on warffarin and yesterday had INR of 3,


today needs extraction. What do you do?
A.Deferre the extractions
BConsult physician c.Alter the dose of warfarin
D. Do the extraction and control post operative bleeding
Ans D

Patients requiring minor dental procedures (including extractions) who have an INR
below 4 may continue warfarin without dose adjustment’. INR should be checked
within 72 hours of the procedure starting. If multiple extractions are required, then a
single extraction should be carried out first. Subsequent extractions of two to three
teeth at a time may be carried out if recovery is uneventful.

BY DR.ABDULRAHMAN ALMUALM
MJDF MCQS WITH ANSWERS

977Secondary impressions for edentulous patient close fit tray with hyper gap
reflex. Which material you use for taking an impression
A Alginate
B. Silicone rubber based material
C. Plaster of Paris
D. Zinc oxide eugenol
Ans D???

978Palatally impacted upper maxillary canines. How would you know? Whether
they are impacted palatally or buccally and which of the following x-rays are
best option

A. Paralleling technique
B. Bisecting technique Bitewings
C. OPG
D Taking two x-rays, with paralleling technique
Ans D

979Defective horizontal angulations while taking bitewings results in?


Overlapping of contact points
Dark radiograph
Light radiograph
Ans A

980A patient came after two days of extraction, complains he had pain for two
days, examination showed localized swelling, no lymphadenopathy. What is the
first line of treatment?
Possible dry socket

A -irrigation and placement of sedative medicament Irrigation -


B. curettage of socket, antibiotics and analgesics
C. Irrigation and antibiotics
D. Possible root piece or bone piece take IOPA x-ray and analgesics
Ans D

981Patient getting treatment done under IV Sedation - suddenly not responding


has decreased blood pressure and decreased breathing

Oversedation
Asthma
Ans A

.982 Patient is having epileptic seizures continuously for 10 minutes in dental


surgery - first line of management is Place in recovery position

Give diazepam
Oxygen
Intranasal or buccal midolazam

BY DR.ABDULRAHMAN ALMUALM
MJDF MCQS WITH ANSWERS

Ans is C

983Sudden unilateral onset of facial swelling and pain over parotid area,
which is not yet fluctuant:
Bacterial saliadenitis
Salivary duct infection
Saliorrhea .
Viral siliadenitis
Ans D

984What do you see in irritable bowel disease?


Mucosal blistering
Herpitiform ulcers
Mucosal tags
Macular patches on buccal mucosa
Ans C

985A fifteen year old patient asks for bleaching of teeth. What is the first line
of management?
Discuss options with both patient and parent
Discuss option with both patient and parent, once you have obtained consent
from patient
Discuss only with patient
Discuss only with parent
Ans is B ?

986child avulsed permanent tooth came with her aunt, what will you do?
Do not do anything Do not do treatment as aunt is not legal guardian of the
child
Do not do treatment as child's mother is not there to give consent
Re-implant and splint it with aunt's consent
Do the best interest for the patient
Ans is E

987Best ways to reduce radiation for patient who is going to have IOPA X-rays
Lead apron
Parallel
Rectangular collimation
D-Film
Ans is C

988
46 and 44 present 45 missing, 44 minimally tilted, 46 heavily filled both are
tipped 46mesial and 44 distal>Fixed movable

989What is the best cement used to cement minimally done inlay restoration
(MOD)
GIC

BY DR.ABDULRAHMAN ALMUALM
MJDF MCQS WITH ANSWERS

ZNO
Zinc phosphate
Zinc poly carboxylate
Resin cement
Ans e

990What sort of matrix is best for restoring disto-occlusal restoration of 7?


Sectional matrix
Auto matrix
Ans A

991Patient had trauma with minimal mobility of front teeth, no symptoms or


pain or anything, what is the best approach

Splinting one week


Splinting two weeks
Splinting 4 weeks
Soft diet and review ~
Ans D

992 Patient had trauma 8 days ago, upper central incisor palatal luxation mild,
not interfering with occlusion but tooth non vital ,What do you do?
Pulp extirpation +CaoH2 dressing
Reposition
Reposition and splinting
Ans A

993Deciduous tooth had intrusion 61, what is most likely to occur for
permanent tooth

Impaction
Uneruption
Hypoplasia
Dilaceration
Ans D
The traumatically induced type is caused by intrusion of the 1° incisor, resulting in
displacement of the developing 2° incisor tooth germ. The effects depend upon the
developmental stage at the time of injury

994Splinting time for Avulsion


2 weeks
1 week
7-10 days
Ans C

995Which test is the most reliable test to indicate the presence of active hepatitis
H bs(surface }antigen
Hbc(core)antigen
Hbe antigen
Antibody to HBs antigen

BY DR.ABDULRAHMAN ALMUALM
MJDF MCQS WITH ANSWERS

Ans C

996First line treatment to needle stick injury

Wash the area under running water


Scrape the area
Refer to infectious disease specialist
Ans A

997Medical condition of pt who reserved a seat in a dinner meating for his dead wife
Mania .
Anxiety
Depression
Schizphrenia
Ans D

998Lady in 40s got TMJ pain for about few months, she got divorced recently what is
the first line treatment

a.amitrptyline B.physiotherapy c.occlusal splint //Ans A


999. Drug of choice to acute asthma
Salbutamol
Steroids
Ans A

1000The term abrasion best describes:

A.Loss of substance by chemical agent (erosion)


B.Loss of substance by external agent (abrasion)
C.Loss of substance by the movement of tooth against tooth (attrition)
D.The rapid loss of substance that is seen in the movement of porcelain
crowns aganist natural teeth (attrition)
Ans B

1001Some days after preparation and filling of a shallow class I amalgam


cavity the patient complains of pain on biting. You would:
A.Perform vitality test
B.Replace filling
C.Check for premature contacts
D.Remove all occlusal contacts from this filling
E.Tell the patient to wait 2-4 weeks, the pain will go away
Ans C

1002The placement of a retentive pin in the proximal regions of posterior teeth


would MOST likely result in periodontal ligament perforation in the
A. mesial of a mandibular first premolar.
B. distal of a mandibular first premolar.
C. distal of a mandibular first molar.
D. mesial of a mandibular first molar.
Ans D

BY DR.ABDULRAHMAN ALMUALM
MJDF MCQS WITH ANSWERS

1003For a cast gold restoration, a gingival bevel is used instead of a shoulder


because a bevel
1 protects the enamel.
2 increases retention.
3 improves marginal adaptation.
4 increases the thickness of gold.
A. (1) (2) (3)
B. (1) and (3)
C. (2) and (4)
D. (4) only
A. E. All of the above.
Ans A

1004Which of the following may affect the results of electric pulp testing?
A. Emotional factors.
B. Pain threshold.
C. Analgesics.
D. Recent trauma.
E. All of the above.
Ans E.

1005 In a composite filling, the matrix band is for


A.Help shaping and contouring the filling
B.Prevent material to be pushed under the gingival margin
Ans is A

1006In a class 11.2 malocclusion, which bridge design would be contraindicated


for a missing lateral upper incisor?
A.Cantilever bridge
B.Maryland bridge
Ans B

1007Which is the best cantilever bridge design for missing maxillary canine?
Abutment on
A.Both premolars
B.Lateral and central incisor
C.Lateral incisor
D.First premolar
Ans B

1008A 9 years-old child who has sustained a fracture of a maxillary permanent


central incisor in which 2 mm of the pulp is exposed, presents for treatment 30
minutes after injury. Which of the following should be considered?
A.Remove 1-2 mm of the pulp tissue surface, place calcium hydroxide and fill
with resin
B.Remove 1-2 mm of the pulp tissue surface and cover with ledermix
C.Place calcium hydroxide directly on the exposed pulp
D.Pulpotomy using formocresol
E.Pulpectomy and immediate root filling

BY DR.ABDULRAHMAN ALMUALM
MJDF MCQS WITH ANSWERS

Ans A

1009 In a flouridated toothpaste with 0.304% sodium fluoride the amount of


flouride ions is
A.400 ppm
B.l000 ppm
C.1500 ppm
D.4000 pppm
Ans C

1010The most important diatary habit for caries development is


A.Amount of sugar intake
B.Frequency of sugar intake
C.Form of sugar intake
Ans B

1011The normal unstimulated salivary flow rate is


A.0.02 ml/min
B.0.2 ml/min
C. 2 ml/min
Stimulated- 1-2 ml/min
Unstimulated- 0.2- 0.5
Xerostomia- less than 0.1
Ans is B

1012Titanium is used in dentistry


A. ln a very pure form in implants
B. ln an alloy with aluminium in casting for crowns and bridges
C. ln an alloy with nickel in orthodontic wires
D. A and B.
E. A, B and C
Ans E

1013What is the reason for a tooth to develop pulpitis several years after
setting of a full veneer gold crown?
A. Bacterial microleakage

1014How is the regeneration process after damage by injury to odontoblasts


working?
A.Proliferation of the remaining odontoblasts
B.Differentiation from fibroblasts
C.Regeneration from undifferentiated mesenchyme I cells
D.Histodifferentiation from ectodermal cells
E.Differentiation from the inflammation cells
Ans C

BY DR.ABDULRAHMAN ALMUALM
MJDF MCQS WITH ANSWERS

1015In construction of full dentures, what does the term "too low vertical
dimension" refer to?
A.A situation in which there is too much interocclusal space between upper and
lower artificial teeth when the mandible is in rest position.

1016Reversible pulpitis is characterized by


A.Pain lasts longer on hot or cold stimulus than normally
B.Patient can't localize pain
C.Will have periapical involvement in radiograph
Ans B

1017Irreversible pulpitis is characterized by


A.There is often a history of spontanous pain
B.Sudden throbbing pain
C.Pain can't be localised when it reaches the periapical area
D.There is pain which lingers for a short duration after romoval of stimulus

1018What are the symptoms of internal resorption?


A.very painful
B.Symptom-free or only mild pain
Ans B

1019What kind of root fracture in a tooth has the best prognosis? A fracture at
the
A.Apical third
B.Coronal third
C.Middle third
D.Vertical fracture
Ans A

1020which of these disease that caused by positive lacto bacillus


Syphilis>Treponema pallidum
TB> Mycobacterium tuberculosis
Angular cheilitis > combined staphylococcal, β-haemolytic streptococci and
candidal infection

Lactobacillus species are also acidophilic and have been implicated in fissure
caries

Lactobacillus Secondary colonizer in caries. Very acidogenic. Often found in


dentine caries.

1021Child with sore throat and feeling unwell,he got macular rash on his
cheek?
A.measles
b.chicken pox
c.herpes simplex
d. Infectious mononucleosis (glandular fever)
Ans A

BY DR.ABDULRAHMAN ALMUALM
MJDF MCQS WITH ANSWERS

1022Pt worried of getting cancer what the advice that should u give? Regular
check up
5 fruit/veg a day

1023Pt got recently metallic taste sensation after taking oral medication?
A.burning mouth syndrome
b.oral dysthesia
c.oral dysgeusia
ans C

1024Reasons for crowding


A early loss of deciduous teeth
b.delayed eruption of lower first molar
c primary failure of eruption
Ans A

1025Reason for palatal position of upper canine


a early loss of deciduous teeth
b.delayed eruption of lower first molar
c primary failure of eruption
d abnormal position of crypt
ans A

1026A patient in your dental chair shows chest pain, weak pulse and dysponea,
what is your initial management,
● Administer nitro-glycerine and keep the patient up seated
● Put the patient in supine position
● Wait until the symptoms go away
Ans A

1027Developer was contaminated with other chemical and was not mixed
properly. What is the effect on the X-ray film?
● Too dark film
● Light film
● Foggy
And C

1028Branchial Cleft cyst is located


● In front of the neck
● On anterior border of the Sternocleidomastoid muscle
● Shows when swallowing
Ans B

1029What is the approximate unstimulated salivary flow rate,


● 2 ml/min
● 0.2 ml/min
● c. 0.02 ml/min
● D. 20 ml/min
Ans B

BY DR.ABDULRAHMAN ALMUALM
MJDF MCQS WITH ANSWERS

1030Some hours after the extraction of a lower molar the patient complains of
prolonged post operation bleeding and pain, how would manage this,
● Prescribe analgesics and ask the patient to follow a strict oral hygiene
● Administer 5% Marcaine Local Anaesthetic, prescribe analgesics and pack
the socket with alvogyl
● Administer 5% Marcaine Local Anaesthetic, suture the socket and prescribe
analgesics
● Suture and give pressure packs
Ans C

1031A Gracey curette(area specific) is characterized by


● The blade and the shank form a 902angle
● Can be used on both sides
● Can be used on any tooth surface
● It is specific for each surface ofthe tooth
Ans D

1032 A patient with no positive history came along for scaling. The moment
you pick up your anterior scaler you punch your finger, what should you do?
● Complete the procedure as if nothing has happened
● Check dentist's blood for Hepatitis B antibody HBsAb
● Check dentist's blood for Hepatitis B antigen HBsAg
● Check patient's blood for Hepatitis B antibody HBsAb and HIV antigen HIVAg
● Check patient's blood for Hepatitis B antigen HBsAg and HIV antibody HIVAb
● Dentist should go and take a HBsAb vaccine
Ans A
1033A patient on the dental chair has cardiac arrest. What is INCORRECT,
● Observing the vital signs and check that the air way is clear is at high
importance
● Expired air has 15% 02 only, and cardiac compressions achieve 30-40% of
cardiac output
● Intermittent positive pressure at the rate of 40/min will reduce the chances
of cerebral hypoxia
● Intermittent positive pressure is better than mouth to mouth when it has
been given at the same rate.
● You check the pulse and respiration of the patient before starting any
resuscitation
Ans C

1034Treatment for small carious cavity in co operative child


Pulp capping
Pulpotomy
Pulpectomy
Simple restoration

Ans D

1035Treatment for pinpoint exposure in symptomless tooth


Pulp capping
Pulpotomy

BY DR.ABDULRAHMAN ALMUALM
MJDF MCQS WITH ANSWERS

Pulpectomy
Ans a

1036Pt with labially impacted upper canine, how does u know clinically?
Swelling on the labial area
Swelling on the palatal side
Ans is A

1037First line management of dry socket


a.irrigation.x ray.metrondiazole 200
b.irrigation xray metrodiazole 400
c.irrigation and x ray only
ans is C

1038First line management of pt with palatal dietary erosion


a.crown.,
b.palatal veener
c.direct composite

1039Preferred bridges for missing UR2


A Minmal prepartion adhesive bridge
B Conventional adhesive bridge
C Fixed-fixed bridge
D Minimal preparation fixed fixed
E Simple cantilever
F Spring cantilever
Ans A

1040Preferred bridges for missing ULl


A Minmal prepartion adhesive bridge
B Conventional adhesive bridge
C Fixed-fixed bridge
D Minimal preparation fixed fixed
E Simple cantilever
F Spring cantilever
Ans A

1041Preferred bridges for missing UR3


A Minmal prepartion adhesive bridge
B Conventional bridge
C Fixed-fixed bridge
D Minimal preparation fixed fixed
E Simple cantilever
F Spring cantilever
Ans B or E

1042Preferred bridge for missing 1st molar


A Minmal prepartion adhesive bridge
B Conventional adhesive bridge
C Fixed-fixed bridge

BY DR.ABDULRAHMAN ALMUALM
MJDF MCQS WITH ANSWERS

D Minimal preparation fixed fixed


E Simple cantilever
F Spring cantilever
Ans C

1043Preferred bridge for missing 1stpremolar


A Minmal prepartion adhesive bridge
B Conventional adhesive bridge
C Fixed-fixed bridge
D Minimal preparation fixed fixed
E Simple cantilever
F Spring cantilever
Ans E

1044Preferred local anesthesia option for a normal fit pt?


A Lidocaine + adrenaline
B Prilocaine+felypressine
C Bupivcaine
D Benzocaine
E Articaine
Ans A

1045LA with prolonged duration of action?


A Lidocaine + adrenaline
B Prilocaine+felypressine
C Bupivcaine
D Benzocaine
E Articaine
Ans C

1046Preferred option of la for pt with heart disease?


A Lidocaine + adrenaline
B Prilocaine+felypressine
C Bupivcaine
D Benzocaine
E Articaine
Ans is A
In patients with severe hypertension or unstable cardiac rhythm, mepivacaine
without adrenaline may be used. Alternatively, prilocaine with or without felypressin
can be used but there is no evidence that it is any safer. Felypressin can cause
coronary vasoconstriction when used at high doses; limit dose in patients with
coronary artery disease'

1047Flavoured topical anesthesia?


A Lidocaine + adrenaline
B Prilocaine+felypressine

BY DR.ABDULRAHMAN ALMUALM
MJDF MCQS WITH ANSWERS

C Bupivcaine
D Benzocaine
E Articaine
Ans is B

1048Pt on warfarin, what is the accepted inr to do simple extraction

A2
B3
C4
D5
Ans is C

1049which of this pt has risk of dry socket


a.smoker
b.pt on warfarin
c.elderly pt

Ans A

1050what is the most reasonable indication for third molar removal


A First episode of pericorinitis
B Second episode
C Periapical infection with erupted wisdom tooth
Ans B

1051Dentist did root canal treatment previously to child pt and now he do a


crown for the same pt what band the dentist should claim?
Band 1
Band 2
Band 3
No calim
Ans B

.1052 what ur advice to childs mum whos her child s teeth knocked out?
a.Reimplant it
b.keep it in the milk and come as soon as possible correct
c.keep it in the water
if permenent tooth // ans is B ??
1053What is the most correct flap design in apecictomy?
Apical re position
Apex is broader than the base
Base is broader than the apex
Apex and base are equal
Ans is C

1054Common irrigation solution for endodontices?


A sodium hypochlorite
B Saline
C Chlorhexidine

BY DR.ABDULRAHMAN ALMUALM
MJDF MCQS WITH ANSWERS

1055What will develop after prophylaxis?


A.Aceliular pellicle is formed immediately after
B.Celiular pellicle is formed immediately after
C.Acellelar pellicle is formed after 48 hours
D.Cellular pellicle is formed after 48 hours.
Ans A

.1056 What is the purpose of making a record of protrusive relation and what
function does it serve after it is made?
A.To register the condylar path and to adjust the inclination of the incisal
guidance.
B.To aid in determining the freeway space and to adjust the inclination of the
incisal guidance.
C.To register the condylar path and to adjust the condylar guides of the
articulator so that they are equivalent to the condylar paths of the patient.
D. To aid in establishing the occlusal vertical dimension and to adjust the
condylar guides of the articulator so that they are equivalent to the condylar
paths of the patient.
Ans C

1057A 50 years-old patient presents with pain from time to time on light
cervical abrasions. What is your first management to help patient in preventing
pain in the future?
A.Change diatary habits
B.Change brushing habits
C.GIC fillings
Ans B

1058planning and construction of a cast metal partial denture the study cast
● facilitates the construction of custom trays
● minimizes the need for articulating
● provides only limited information about inter ridge distance, which is best
assessed clinically
● can be used as a working cast when duplicating facilities are not available
Ans A

1059What are the artificial teeth in removable dentures made of?


A.Porcelain
B.Cross-linked methyl-methacrylate
C.Ethyl-methacrylate
D.Acrylic
Ans D

1060Following extraction of the molar teeth


● The ridge height is lost more from the maxilla than from the mandible
● The maxillary ridge will get more bone lost from the palatal aspect than the
buccal
● The mandibular arch is relatively narrower than the maxillary arch

BY DR.ABDULRAHMAN ALMUALM
MJDF MCQS WITH ANSWERS

● Compared with the pre-resorption state, the mandibular ridge will lose more
bone from the lingual aspect than the buccal one.

Ans is D?

1061Which anatomical landmark is important to include in impressions for lower


full dentures
A.Mylohyoid ridge
b.Lower incisive papilla
Ans A

1062Wrought metal is to be,


● Marble
● Quenched > casting alloy
● Has undergone cold treatment during processing
Ans is C

1063Which ofthe following is ONE indication for indirect pulp capping?


A.When further excavation would lead to pulp exposure
B.Excavation of a very deep caries
Ans A

1064 How does fluoridation work in theory? A. Fluoride ions are integrated by
changing Hydoxylapatite to Fluorapatite

1065A major difference between light cured and chemical cured composite is
that during setting or in function the light cured materials tend to:
● Seal the margins better and completely
● Exhibit less wear on time
● Undergo greater colour change
● Shrink more rapidly
● Posses greater fracture toughness
Ans D

1066What consideration is important in deciding if a bridge for upper missing


incisors should be made in pontic design or with gingiva imitation?
A.Wishes of patient
B.Bone resorption in edentoulos span

Ans B

1067The most common cause of porosity in porcelain jacket crowns is,'


● Moisture contamination
● Excessive firing temperature
● Failure to anneal the platinum matrix
● Excessive condensation of the porcelain
● Inadequate condensation ofthe porcelain

BY DR.ABDULRAHMAN ALMUALM
MJDF MCQS WITH ANSWERS

Ans E

1068How should the occlusion in partial removable dentures be designed?


A.Artificial teeth should be out of occlusion
B.Artificial teeth should not interfere with the incisal guidance established by
the remainig natural teeth..
Ans B

1069 The minimal labial tooth reduction for satisfactory aesthetics with
porcelain fused to metal crown is,
A.1mm
B.The full thickness of enamel
C.1.5 mm
D.2.5mm
E. One third of the dentine thickness
Ans C

1070removable partial denture, the principle of an indirect retainer is to:


● Stabilise against lateral movement
● Prevent settling of major connectors
● Restrict tissue movement at the distal extension base of the partial denture
● Minimise movement of the base away from the supporting tissue
Ans C

1071 When a removable partial denture is terminally seated the retentive clasps
tips should:
A. Apply retentive force into the body of the teeth
b.Exert no force
c.Be invisible
d.Resist torque through the long axis of the teeth
Ans is B

1072 Glass lonomer Cement sets because of,


● Acid-Base reaction
● Addition polymerisation reaction
● Growth of glass crystals
● Slip plane locking
● Solvent evaporation
Ans is A

1073The reflex in gagging patients is caused by:


● Trigeminal nerve
● Glossopharyngeal
● Facial nerve
● Recurrent laryngeaI
Ans is B
IX Glossopharyngeal. Supplies sensation and taste from the posterior 1⁄3 of the
tongue, motor to stylopharyngeus, and secretomotor to the parotid. Lesions impair
the gag reflex in conjunction with X.

BY DR.ABDULRAHMAN ALMUALM
MJDF MCQS WITH ANSWERS

X Vagus. Has a motor input to the palatal, pharyngeal, and laryngeal muscles.
Impaired gag reflex, hoarseness, and deviation of the soft palate to the unaffected
side are seen if damaged. The vagus has a huge parasympathetic output to the
viscera of the thorax and abdomen

1074The use of nickel chromium in base plate should be judiciously considered


because:
● A significant number of females are allergic to nickel
● A significant number of females are allergic to chromium .,
● A significant number of males are allergic to nickel
Ans is A

1075 Which of the following liquids is not suitable for prolonged immersion of
cobalt chrome partial dentures:
A. Alkaline peroxidase
B. Sodium hypochlorite
C.Soap solutions
D.Water
Ans is B

1076In complete dentures, cheek biting is most likely a result of:


● Reduced Overjet of posterior teeth
● To high vertical dimension
● Teeth have large cusp inclines
Ans is A

1077The most common cause of fracture at the isthmus of a class II dental


amalgam restoration is:
A.lnsufficient condesation
B.Fracture line developing from pulpal-axial angle of the cavity
C.Underconturing of the isthmus area
D.Moisture contamination of the amalgam during placement
E.lnadequate bulk of amalgam at pulpo-axialline angle
Ans is E

1078 Why do you overpack amalgam fillings?


A. To remove excess mercury
B To prevent microleakage
Ans is A

1088What is CORRECT in regard to the periodontal surface area in maxillary


teeth:

BY DR.ABDULRAHMAN ALMUALM
MJDF MCQS WITH ANSWERS

● central incisor> first premolar> second premolar


● Canine> first premolar> central incisor
● Canine> lateral incisor> second premolar
● Canine> central incisor> first premolar
Ans is B

1089When restoring with composite resins, why do we do the cavo-surface


bevelling:
A.Aesthetic
B.To open enamel rods for acid attack
C.To smooth preparation
D.A and B
E.AII of the above
Ans is D

1090A well constructed complete denture:


● Needs little maintenance
● Needs less than a week for adjustment and total success
● Has adverse effects and decreases taste sensations
Ans is C
New denture even if constucted well is always a greatest cause for max complains
from patient in first few weeks. As new, they cant adapt to it. Their automated
chewing reflex and crushing food takes 4 to 6 months to fully get used to. New
denture usually causes ulcers If upper , Covers palate and blocks taste buds.

1091On examination of a composite restoration you find a dark stain:


A. Replace the composite
B.Repair with unfilled resin
C.Apply topical fluoride at the margin
Ans is A/ one of Qs that got more than one answer (some answered as B)

1092patient complains of sensitivity, on examination you find a composite filling


restoring a good cavity preparation without any secondary caries; what is your
next step:
● Extirpate the pulp that is obviously inflamed
● Place ZOE dressing to sedate the pulp
● Ask patient to come back in six months
● Repeat restoration
Ans is C

1093What is the best way to cement a Maryland bridge,


A.GIC
B.Resin cement
C.Composite resin
D.Zinc Phosphate cement
E.Oxide Zinc and eugenol
Ans is B?
1094The ideal length of a post in the fabrication of crown and core of
endodontica!ly treated tooth is:

BY DR.ABDULRAHMAN ALMUALM
MJDF MCQS WITH ANSWERS

A.2/3 of tooth
B.the tooth length
C.1.5 times that of the crown
D.Is root length
E.The length of the crown
Ans is E

1095Buccal cross-bite means:


a) The buccal cusps of the lower teeth occlude buccal to the buccal cusps of the
upper
teeth.
b) The buccal cusps of the lower teeth occlude lingual to the lingual cusps of the
upper
teeth
c) Buccal cusps of the upper teeth occlude buccal to the buccal cusps of the lower
teeth
d) Anterior teeth occlude with severe overjet
e) Anterior teeth occlude with reverse overjet
ans is A

1096submandibular salivary gland Calculi may appear as radio-opaque in the


radiograph ,
the percentage of this to happen is:
a) 10-20%
b) 10-30%
c) 30-40%
d) 50-60%
e) 60-70%

109740 years old patient presented to your clinic complaining of salty taste and signs
of
unilateral inflammation affecting the check near to the upper 1st molar, what will be
the
1
st examination to do to reach diagnosis
a) Stimulated saliva sialometry
b) Un-stimulated saliva sialometry
c) Sialograph
d) MRI

1098what is true regarding burning mouth syndrome:


a) common bilateral
b) common unilateral
c) presented as pain on eating
d) presented as acute sudden condition
e) in males more than females

Ans A

BY DR.ABDULRAHMAN ALMUALM
MJDF MCQS WITH ANSWERS

109930 years old patient came to your clinic for treatment under IV midazolam
sedation so
how can you achieve optimum sedation?
a) Bolus injection of 1 mg and 1 mg to be titrated
b) Bolus injection of 2 mg and 1 mg to be titrated
c) Bolus injection of 3 mg and 1 mg to be titrated
d) Bolus injection of 1 mg and 2 mg to be titrated
e) Bolus injection of 1 mg and 3 mg to be titrated
ans B?

1100Patient with crowding and displacement of 3 mm the IOTN score is


a) 2 d
b) 3 d
c) 4 d
d) 1 d
e) 5 d
Ans B
1101Patient with impacted canine:
a) IOTN 5i
b) IOTN 4i
c) IOTN 5P
d) IOTN 4X
Ans A

1102Patient with 9mm overjet


a) IOTN 4A
b) IOTN 5A
c) IOTN 4B
d) IOTN 5B

Ans A

1103patient presented with unilateral rashes/ulcers on chin and mandible, what do


you
expect intraorally:
a) ulcers on the tongue
b) macules on the check
c) red patches on lips and check
d) desquamative gingivitis
e) lipoma on tongue
ans A

1104patient has round chronic lesion on the dorsum surface of the tongue
a) geographic tongue
b) migratory glossitis
c) median rhomboid glossitis
d) lichen planus

BY DR.ABDULRAHMAN ALMUALM
MJDF MCQS WITH ANSWERS

e) psoriasis
Ans C

110510 years old boy, developed small greyish white lesions surrounded by red
halos in the
soft palate, and small vesicles are presented at the tonsillar pillar as well. He is
feverish
and reporting pain with swallowing. What is the most likely?
a) Herpangina
b) Herpes simplex
c) Post herpetic neuralgia
d) Glosso-pharyngeal neuralgia
ans A

1106squamous cell carcinoma has the tendency to metastasis through


a) blood stream
b) lymphatic system
c) direct contact
d) none of the above
Ans B

1107In pregnant woman , cleft lip and palate can be prevented by


a) Folic acid supplement
b) B12 supplement
c) High protein diet
d) Fresh fruit frequently
ans A

1108Normal sulcular epithelium in man is 1. nonkeratinized.


2. squamous.
3. stratified.
4. nonpermeable.
A. (1) (2) (3)
B. (1) and (3)
C. (2) and (4)
D. (4) only
E. All of the above.
Ans A

1109The arrangement and character of the principal fibres of the periodontal


ligament are modified by
A. tooth morphology.
B. occlusal function.
C. cementum.
D. alveolar bone.
Ans B

1110Which of the following will produce hemostasis when applied topically?


A. Oxidized cellulose.
B. Absorbable gelatin sponge.

BY DR.ABDULRAHMAN ALMUALM
MJDF MCQS WITH ANSWERS

C. Microfibrillar collagen.
D. Topical bovine thrombin.
E. All of the above
Ans E

1111 . Extreme resorption of an edentulous mandible can bring the alveolar


ridge to the level of the attachment of the

A. buccinator, styloglossus and geniohyoid muscles.


B. mylohyoid, buccinator and styloglossus muscles.
C. superior constrictor, mylohyoid and buccinator muscles.
D. mylohyoid, buccinator and genioglossus muscles.

1112. Habitual thumbsucking continued after the age of six causes


A. anterior open bite.
B. skeletal malocclusion.
C. mouth breathing.
D. sinusitis.
Ans A

1113A 8-year old child has an 8mm central diastema. The etiology could include
1. frenum.
2. cyst.
3. mesiodens.
4. normal development.
A. (1) (2) (3)
B. (1) and (3)
C. (2) and (4)
D. (4) only
E. All of the above.
Ans A

1114While you finish a class I cavity, the enamel is sound but you notice a
thin brown line ln the dentine and on the dentino-enamel junction, what is your
response,
● You leave it and complete the final restoration
● You extend your preparation and clean it
● You apply a cover of varnish
Ans A
1115Dental caries ofthe proximal surfaces usually starts at,
● Somewhere between the ridge and the contact area
● Just gingival to contact areas
● Just above the gingival margin
● At the contact point

BY DR.ABDULRAHMAN ALMUALM
MJDF MCQS WITH ANSWERS

Ans B

1116The Frankfort plane is defined by which anatomical landmarks,


● Porion, orbitale
● Sella, orbitale
Ans A//
Frankfort plane Line joining porion (superior aspect of external auditory
meatus) with orbitale (lowermost point of bony orbit).

1117The difference between normal stone and the die stone is,
● In the size and shape of the particles
● The mixing
Ans A

1118The advantage of the silicone in soft relining material over hard plastic
acrylic materials is,
● Capability to flow
● Prevents the colonization of Candida albicans
● Resilient in long run
● Better bond strength
Ans C

1119A female patient comes to you complaining of persistent pain in a heavily


restored central incisor; you suspect irreversible pulpitis and you have been told
that she is in transit leaving by plane next day. Your treatment will be,
● Remove filling and place a sedative dressing
● Pulpectomy and Ledermix dressing
● Pulpectomy and calcium hydroxide dressing
● Prescribe analgesics and systemic antibiotic
Ans B

1120The flexibility of the retentive clasp arm does not depend on:
● Length of the arm
● The cross section shape
● The material used
● Degree of taper
● The exerted force
Ans E

1121Following calcium hydroxide pulpotomy, the dentist would expect dentine


bridge to form at,
● The exact level of amputation
● Level somewhere below the amputation
● Half way between amputation and apex
● At the apical region of the tooth
Ans B

1122the construction of a full veneer gold crown, future recession of gingival


tissue can be prevented or at least minimised by,
● Extension of the crown 1 mm under the gingival crevice

BY DR.ABDULRAHMAN ALMUALM
MJDF MCQS WITH ANSWERS

● Reproduction of normal tooth incline in the gingival one third of the crown
● Slight over contouring of the tooth in the gingival one fifth of the crown
● Slight under contouring of the tooth in the gingival one fifth of the crown
Ans is B

1123What is correct in regard to high copper amalgam,


● Reacts and strengthens the amalgam by its dispersion properties
● Reacts to form copper-tin phase thereby eliminating the tin-mercury phase
● Reacts to form copper-silver phase thereby eliminating the silver mercury
phase
● Reacts and strengthens the amalgam by its grain diffusion ||| Ans isB

1124The removable partial denture requires relining,what would be the most


appropriate action,
● take an impression by asking the patient to occlude on it
● Provide equal space between denture and gingival tissues.
● Make sure the framework and retainers are seated in place before taking
impression
Ans is C

1125Stiffness of material is measured by


● Proportional unit
● Modulus of elasticity
● Stress/ strain
● Ultimate tensile strength
Ans B

1126Two central incisors on a radiograph are showing with what looks like eye
drop radiolucency. You decided to start endodontic treatment on these teeth but
when you tried to open access to the root canal you find clearly closed orifices
with what look like secondary dentine. What is your initial management?
● Leave as it is and start a permanent restoration.
● Start systemic antibiotic
● Try to ream and file canals
Ans C

1127After the initial development stage and in the absence of pathology, the
size of the pulp chamber has been reduced by, ● Deposition of primary dentine
● Deposition of secondary dentine
● Reparative dentine
● Pulp fibrosis
● Deposition of reparative dentine
Ans B

1128Denture stomatitis is commonly associated with,


● The continuous wearing of removable orthodontic appliances in otherwise
healthy patient
● The proliferation of hypertrophic tissue at the denture periphery
● The overgrowth of some constituents of oral normal microflora
● Allergy to denture base material

BY DR.ABDULRAHMAN ALMUALM
MJDF MCQS WITH ANSWERS

Ans C

1129 The light emitted by the polymerization lamp has to be checked from time
to time. The meter used for this only measures light in the range of:
● 100-199 nm
● 200-299 nm
● 300-399 nm
● 400-499 nm
Ans is D

1130Which is correct in regard to shade selection of crowns:


A. It should be selected before starting preparation
b.Chroma is the lightness/darkness of colours
c.Value is the colour itself
d.Hue is the concentration of colours
ans A

1131Where would you expect to find the mylohyoid muscle in relation to the
periphery of a full lower denture:
A.Mandibular buccal in the midline
B.Mandibular lingual in the first premolar area
C.Mandibular lingual in the midline
D.Mandibular disto buccal area

Ans B

1132After reimplantation of an avulsed tooth the prognosis may be poor


because of
A.External resorption
B.lnternal resorption
Ans A

1133 |||2.2 mg of NaF contains how many mg of fluoride ions?


A.O.5mg
B.1.0 mg
C.1.5mg
D.10mg
Ans B
Each ReNaf Fluoride Chewable Tablets 1 mg contain 1 mg fluoride ion (F-)
from 2.2 mg sodium fluoride (NaF). Each ReNaf Fluoride Chewable Tablets 0.5
mg contains 0.5 mg F- from 1.1 mg NaF. Each ReNaf ReNaf Fluoride
Chewable Tablets 0.25 mg contains 0.25mg F- from 0.55 mg NaF.

1134How would you treat denture stomatitis?


A.Nystatin
B.Tell the patient to leave the denture out for some days
Ans A

1135What is true about third molar surgery?


A.Swelling is maximum after 24 - 48 hours

BY DR.ABDULRAHMAN ALMUALM
MJDF MCQS WITH ANSWERS

B.Mental paraesthesia indicates careless technique

1136 1n anaesthetizing a 70 kg healthy man with Lignocaine 2% with 1:100,000


vasoconstrictor.what is correct?

A.The toxic threshold is 22 ml


B.2.2 ml is the maximum you can give in one session
C.Lignocaine has the same anaesthetizing capacity as Benzocaine
D.Lignocaine is 5 times less potent than Bupivacaine
ANS A

1137A suddenly swollen upper lip that lasts for 48 hours or more is most likely
A.Haemangioma
B.Agioneurotic oedema
C.Mucocele
D.Cyst
Ans A

1138What is the most important factor to reduce radiation in dental radiographs?


● Speed of film
● Collimation
● Filtration
● Cone shape and length
● Use of lead apron
Ans B

.1139 What is the best way for a permanent decline in caries of a population?
A.Change diatary habits
B.Topic and water fluoridation
C.Awareness of dental health matters
D.Better tooth brushing
Ans is B

1140Loss of sensation/paraesthesia in the lower lip may be produced by,


● Bell's palsy
● Traumatic bone cyst
● Trigeminal neuralgia
● Osteomyelitis
● Ludwig's angina
Ans D

1141In anxoius and psychologically stressed patients gingivitis is often more


severe because of
A.Stress causes histamine and serotonine release
B.stress causes catecholamine and corticosteroid release
C.Stressed people neglect their oral hygiene
Ans B

1142A retained lower primary incisor causes the permanent incisor to


A.erupt buccally

BY DR.ABDULRAHMAN ALMUALM
MJDF MCQS WITH ANSWERS

B.erupt lingually
C.ankylose
Ans B

1143What is the least probable consequence in thumb-sucking?


A.Reclining of lower incisors
B.Protrusion of upper incisors
C.Formation of deep palate with big overbite
Ans C

1144Ankylosis of teeth is often found after changes in the continuity of the


occlusal plane. These changes are caused by
A.Overeruption of opposing teeth
B.Localised growth inhibition of the alveolar bone
C.Sinking of ankylosed tooth into the bone
Ans C

1145What is not important in obduration materials for primary teeth?


A.Good apical seal
B.Radioopacity
C.Antibacterial
D.Resorbable
Ans A

.1146 What is your first consideration in the treatment of dry socket?


A.Prevention of osteomyelitis
B.Pain relief
ANS B

1147patient presents with pain in the upper left segment. On inspection you
find a localized alveolar abscess distal 27. What will be you management? -
A.Drainage
B.Extraction of tooth ||
Ans is A
1148What does not help in establishing the caries risk in children?
A.History of caries
B.Lactobacilius count
C.Dietary habits
D.Brushing habits
E.Genetic predisposition
Ans is E?

1149 Opioid drugs are similar to which endogenous substances?


● Bradykinins
● Peptides
● Prostaglandins
● Serotonins
● Enkephalins
Ans is E

BY DR.ABDULRAHMAN ALMUALM
MJDF MCQS WITH ANSWERS

1150Which of the following have a tendency to recur if not treated correctly?


● Giant cell granuloma
● Lipoma
● Fibrous epulis
● Haematoma
● Pulp polyps

Ans A

1151When no radiation shield is available, the operator should stand out of the
primary x-ray beam at a distance from the patient's head of at LEAST:
A.0.5 metres
B.1 metre
C.1.5 metres
D.2 metres
E. 3 metres
Ans is D
Exposure switches should be positioned so that the operator can remain outside the
controlled area and at least 2m from the X-ray tube and patient the controlled
area>1.5 m in any direction from the patient and tubehead and anywhere in the line
of the main beam until it is attenuated by a solid wall.

1152Which of the following is a typical consequence of dental crowding,


assuming no primary tooth has been lost prematurely? ● ●Overlapping of lower
incisors
● Palatal displacement of upper canines
● Impaction of 15 and 25 between first premolars and first molars
● Mesial tipping of 16 and 26
● Rotation of 16 and 26
Ans A

1153What is the dominant microflora in acute necrotic ulcerative gingivitis


(ANUG)?
● Spirochaetes and fusobacterium SP
● Spirochaetes and eikenella corrodes
● Polymorphs and lymphocytes
● Actinobacillus actinomycetes comitans oral capnocytophaga
Ans is A

1154Which of the following is true regarding gingivosis (Desquamative gingivitis)


● It is caused by hormononal imbalance
● Is seen only at or after menopause
● Is frequently caused by lichen planus
● Is a variant pregnancy gingivitis
● Is related to nutritional disturbance
Ans C

1155What are the points that determine the facial line in cephalometric points
● Nasion, pronasale, pogonion.
● Sella, nasion, pogonion

BY DR.ABDULRAHMAN ALMUALM
MJDF MCQS WITH ANSWERS

Ans A

1156A 10 year old boy presents with small greyish white lesion surrounded by a
red halos on the soft palate and tonsillar pillars, small vesicles are found. He
has fever and pain in the ear and won't eat. The MOST probable diagnosis is?
A.Herpangina
B.Measles
C.Primary herpetic stomatitis
Ans is A

Herpangina Febrile illness with sore throat due to ulcers on soft palate and
throat. Usually lasts about 3–5 days. Rx: soft diet.

1157A 12 years-old child presents with symptoms of widespread gingivitis with


!bleeding and general malaise for several weeks. How would you manage this
patient?
● Prescribe Metronidazole 100mg
● Locally debride, give oral hygiene instruction and prescribe H202 mouth
wash.
● Give a prophylaxis with ultra sonic scaling
● Refer for haematological screening
● Advise for bed rest with supportive and palliative treatment
Ans D

1158What is the effect of office dental prophylaxis of regular six month intervals
on children's oral health?
● Reduced caries incidence by approximately 30%
● Provide a long term improvement in oral hygiene
● Provide a short term improvement in oral hygiene
● Prevent gingivitis
● Reduce the need for patient cooperation
Ans is C?? /

1159What is the most frequent cause of pain which occurs several days after
obturation?
● Entrapped Bacteria in the periapical region
● Underfilling the root canal system
● Overfilled root canal
Ans A
The following factors need to be considered should pain occur following sealing of
the root canal system. High restoration Overfilling Underfilling Root fracture Once
obturation of the root canal space has been completed, restoration of the rest of the
tooth can be carried out. The occlusion must be checked for interferences, to avoid
an apical periodontitis, or worse, a fractured tooth. Root fillings that are apparently
overfilled do not as a rule cause more than mild discomfort after completion. The

BY DR.ABDULRAHMAN ALMUALM
MJDF MCQS WITH ANSWERS

most likely cause of pain following obturation of the root canal space is the presence
of infected material in the periapical region. The significance of an underfilled root
canal is whether the canal has been properly cleaned and prepared in the first
instance, and infected debris is still present in the canal. Post-endodontic pain in
these circumstances may well be due to inadequate debridement of the canal.
Removal of an overextended root filling is rarely completely successful and the
options left are as follows: Prescription of analgesics and, if the pain is more severe
and infection is present, antibiotics. An attempt at removal of the root filling and
repreparation of the root canal. Periradicular surgery and apicectomy

1160diabetic patient with moist skin, moist mouth and weak pulse; what would
you do:
● Give glucose orally
● Administer 02
● Administer adrenaline
● Inject insulin
Ans A

1161patient has developed a sever chest pain and difficulties in breathing while
in the dental chair. Your initial response is:
● Administer glycerine trinitrate and monitor patient in upright position
● Patient has an acute episode of angina as demonstrated by curve in ECG
● No treatment is required until confirmed as MI by ECG
● Patient has myocardial infarction as confirmed by ECG
Ans is A

1162In the case of malignant melanoma occurring intraorally, which of the


following is true:
● Uncommon on the palate
● Should not be biopsied, as this will increase metasis
● The 5 years survival rate is 20%
● The incidence of oral melanoma is the same as those on the skin
Ans is C

1163What is NOT TRUE in relation to the use of diazepam for sedation?


● Patient commonly complain of post operative headache >F
● An acceptable level of anxiolytic action is obtained when the drug is given
one hour preoperatively >T
● There is a profound amnesic action and no side effects >T
● Active metabolites can give a level of sedation up to 8 hours post
operatively >T
● Can be used safely for children but might hyperactive or ineffective
Ans A

1164Patient on anti-coagulant therapy requires an extraction to be performed.


Which of the following is NOT true:
● Minor bleeding can be reduced somehow by using tranexamic acid
● Prothrombin value above 2.5 is required to perform extraction
● It takes up to 12 hours for Vitamin K reverse effects of warfarin
● Heparin can be administered sub-cutaneous and acts rapidly

BY DR.ABDULRAHMAN ALMUALM
MJDF MCQS WITH ANSWERS

Ans is C

1165A physician refers a nine year old boy to you to confirm diagnosis. The
boy has a fever of 40°C and coughing. When you focus your light into his
eyes he turns away. Intra-orally there are white spots surrounded by red
margins. The disease and lesions are:
● Measles and the spots are Koplik's spots
● AHGS vesicles
● Rubella and the spots are Fordyce's spots
Ans A

1166What is true in TMJ dysfunction therapy?


A.Should be treated surgically
B.Appliances that raise the bite usually relief the symptoms and are used prior
to any surgery
Ans us B

1167What is true regarding pregnancy gingivitis?


A.lt is due to increased gingival microcirculation
B.Elevated oestrogen and gestagen levels are directly responsible .;
C.Hormonal changes cause the growth of anaerobic bacteria (Prevotella
interrnedia)
Ans is C

1168 5 mm probing depth means:


A.Patient has periodontitis
B.Probe is 5 mm beyond gingival margin
C.Probe is 5 mm beyond dentino-enamel junction
Ans is B

1169middle aged woman gives a history of intermittent unilateral pain in the


submandibular region, most probable cause is
, ● Calculus in the salivary duct resulting in sialolithiasis.
● Ranula >typically painless
● Cyst
● Mucocele >mainly affect the minor salivary glands
Ans A

1170By which of the following mechanism reduces Aspirin pain:


● It is anti inflammatory by the release of histamine
● It blocks the cyclo-oxygenase pathway.

BY DR.ABDULRAHMAN ALMUALM
MJDF MCQS WITH ANSWERS

Ans is B

1171In minor oral surgery, what is TRUE in regard to antibiotics:


A.Amoxil satisfactorily covers the dental spectrum
B.Metronidazole and Amoxil have the same penetrating power
C.It is evident that it will reduce post operative swelling
D.There is convincing evidence that Prophylactic prescription of antibiotics will
reduce postoperative infections
E.Most oral infections get anaerobic after 2 to 3 days
Ans is A?

1172 A patient comes with a firm, painless swelling of lower lobe of parotid
which has grown progressively for the past year. He complains of paresthesia
for the past 2 weeks. This is most likely to be:
● Pleomorphic adenoma
● Carcinoma of the parotid
● Lymphoma of parotid
Ans B

1173 What is true in treating a patient with secondary herpes simplex:


● Acyclovir inhibits viral transcription when applied in the prodromal phase
● Idoxuridine is better than acyclovir when applied topically
● Antivirals are contraindicated in immuno-compromised patient
Ans is A

1174During extraction of a maxillary third molar the tuberosity is fractured;


however, it remains in place attached to the mucoperiosteum. Which of the
following procedures should be employed:
● Remove the tuberosity and suture
● Leave the tuberosity and stabilize if required
● Remove the tuberosity and fill the defect with Gelfoam then suture.
● If fractured tuberosity is greater than 2 ern, leave in place and suture
Ans is B

1175 An incision biopsy of an ulcerated and indurated clinically suspicious lesion


in a 50year old female reveals chronic inflammation; you would:
● Inform the patient and her physician of your findings and instruct the patient
to return in six months
● Surgically excise the entire lesion since you know it is not malignant

BY DR.ABDULRAHMAN ALMUALM
MJDF MCQS WITH ANSWERS

● Dismiss the patient with instructions for warm saline rinses and re-
examination
● Repeat the biopsy
Ans D

1176 Reducing the size of the focal spot will:


● increase sharpness
● increase density
Ans is A

1177 The initial priority in treatment of horizontal fractures is:


● Preservation of pulp
● Immobilisation
● Root canal treatment
● Calcium hydroxide treatment
Ans is B

1178 Which of the following has proven to be the MOST important in


community preventive program:
● Dental awareness of the community
● Institution of oral hygiene measures
● Water fluoridation

Ans is C

1179 What effect has placing a sealant over pits and fissures on the
progression of caries?
l.Decreased new caries
2.lncreased new caries
3.Progression of exististing caries
4 No effect on existing caries
Ans is 1

1180 In advanced periodontitis with marked mobility, teeth may be splinted:


A.To improve comfort for the patient
B.splinting helps in transmitting the force to the adjucent teeth to reduce the
load on the involved teeth
■ Both answers are right ,we splint to reduce discomfort to the patient and also to
reduce occlusal forces but closet ans is B
Ans is B/A

1181 Swallowing will aid in the diagnosis of:


● Branchial cyst
● Thyroglossal duct cyst
● Ranula
● Retention cyst
Ans B

1182 Which of the following will increase sharpness:


● Larger focal spot

BY DR.ABDULRAHMAN ALMUALM
MJDF MCQS WITH ANSWERS

● Smaller focal spot


● Increase object-film distance
● Decrease patient-source distance
Ans B
what increase sharpness :1- increase source object distance 2- decrease object
film distance 3- smaller focal spot

1183 In severe periodontitis the probe will eventually be:


● prevented to go deeper by calculus
● beyond connective tissues in the junctional epithelium
● at the end of the junctional epithelium
● Touching the middle of junctional epithelium
● Touching sulculuar epithelium
Ans is B

.1184 58 year old male has been treated with radiation for carcinoma of tongue.
The patient complains of pain associated with poor dentition. The dental
management would be:
A.Immediate extraction of any poor teeth under local anaesthetic with antibiotic
coverage
B.Segmental dental clearance and closure to eliminate problems
C.No dental treatment may be due to neuronic of neoplasms
D. Clearance of poor dentition followed by hyperbaric oxygen treatment plus a
primary closure of wounds under antibiotic coverage E. No extraction as
radionecrosis is an important sequelae
Ans is D ?

1185 Which of the following is NOT true about anticoagulation therapy?


● INR of 3 is enough to start any extraction
● Affects extrinsic system and increases prothrombin time
● Heparin can be given subcutaneously and acts rapidly
● It takes at least 12 hours for Vitamin K to reverse the effects of coumarin
Ans is D

1186 in a radiograph the roots of the upper teeth are too short because of:
● Inadequate horizontal angulation
● Too high vertical angulation
● Too small vertical angulation
Ans is B

1187 Characteristic of Squamous Cell Carcinoma of the tongue


● more in white skinned people
● more in alcohol drinking smoking males
● associated with Plummer-Wilson-Syndrome
Ans is B|

1188 Characteristic of Squamous Cell Carcinoma of the lips


● It reacts far simply to radiotherapy
● metastizes mainly by blood
● relatively rare in Australia

BY DR.ABDULRAHMAN ALMUALM
MJDF MCQS WITH ANSWERS

Ans A

1189 Which type of dentin is not formed due to pulp pathology?


A.Reparative dentin
B.Secondary dentin
C.Primary dentin
D.Reaction dentin
E.Tertiary dentin

ANS C /

1190 Which is not true in sickle cell anaemia?


A.Deformed cells with less oxygen transport capacity
B.Higher infarction risk
C.Have wide bone marrow spaces with narrow trabeculae in the alveolar
bone of oral cavity
D Resistant to malaria parasites
E.More common in mediterranean people
ANS E

1191 Normal prothrombin time and elevated partial thromboblastin time is seen
in
A.Factor VIII deficiancy (Haemophilia)
B.Thrombocytic pupura
C.Leukemia
D.Von Willebrand disease
ANS A

1192 Which of the following describes best a 9 years-old child permanent


dentition?
612CDE
6123DE

1193 What is the best reason to promote tooth brushing to the public?
A.Less fissure caries
B.Less gingivitis
C.Gingival massage
Ans is B

1194 What is untrue about diabetes?


A.Hypoglycaemia is more common than hyperglycaemia> true
B.lnsulin-dependend patients are of more concern than non insulin-dependend>
true
C.Adrenalin causes a decrease in the blood glucose level
Ans c
Only Insulin and Somatostatin decreasing glucose level !! Keep it in your mind
always

1195 Which of the following is a feature of Streptococcus mutans?


A.lt does not require a special environment to grow

BY DR.ABDULRAHMAN ALMUALM
MJDF MCQS WITH ANSWERS

B.lt can be easily transported from one part of the oral cavity to another
C.lt has the ability to restructure carbohydrates
Ans C

1196 What does the term "caries prevalence" mean?


A.The total number of carious areas affected and any present caries
B.The individual risk for a patient to acquire caries
Ans is A

1197 Which of the following are features of herpetic gingivostomatitis?


● Irritability
● Fever
● Occurs in teenagers
● Vesicles occur only on buccal mucosa and tongue
Ans is B

1198 A patient in your dental chair suddenly becomes agitated with shallow
breathing, full pulse and a blood pressure of 150/80. You would
A.Give oxygen
B.Give insulin
C.Give glucose
D.Place patient in supine position
Ans C

1199 The principle clinical sign of active bruxism is:


● Head and / or neck pain
● Excessive tooth wear
● Temporomandibular joint clicking
● Sensitive teeth
● Cheek ridging and tongue scalloping
Ans is E

1200 An adult patient attends your practice complaining of pain-and swelling


aSsOCiated with a previously restored upper first premolar tooth. The pain has
been present for a number of days and is no longer responding to analgesics.
His dentition is otherwise well maintained and his periodontal health is good.
What is the most appropriate approach to treatment?
● Antibiotics and analgesics.
● Extract the tooth
● Carry out a pulpotomy and temporary dressing.
● Carry out a pulpectomy. Temporary dressing
● open Drainage
Ans is D

1201 A patient reports that his post crown has fallen out. This crown had been
present for many years. You note that there appears to be a hairline vertical
fracture of the root. The tooth is symptomless. What is the most sensible
approach to treatment? A.Replace the post crown using a resin-reinforced glass
ionomer material

BY DR.ABDULRAHMAN ALMUALM
MJDF MCQS WITH ANSWERS

B.Replace the post crown using a polycarboxylate cement


C.Replace the post crown using a dentine bonding agent and a resin-reinforced
glass ionomer material
D Replace the post crown using a resin composite luting agent
E. Arrange to extract the tooth
ANS IS E

1202A patient says that he does not like the appearance of his previously root
filled upper central incisor tooth. His dentition is otherwise well maintained and
his periodontal health is good. The tooth appears to be darker than the adjacent
teeth. What is the most appropriate approach to treatment?
A.Provision of a post crown
B.Provision of an all ceramic crown
C.Provision of a metal bonded to ceramic crown
D Non vital bleaching
E. Provision of a porcelain veneer
ANS IS D

120325 year old male attends for the first time complaining of sensitivity of a
number of teeth. On examination, the occlusal surfaces of all the teeth are
worn with obvious wear facets on the canines and premolars. Posterior
amalgam restorations are proud of the surrounding tooth. What would be the
first stage management?
● Take impressions for study models
● Prescribe fluoride mouth rinse
● Replace the amalgam restorations
● Dietary analysis
Ans is D

120435 year old male patient who admits to grinding his teeth at night has a
number of wedge-shaped cervical (Class V) lesions on his upper premolar teeth.
These are causing sensitivity and are approximately 3mm deep. What is the
correct management option? ● Provide tooth brushing instruction and fluoride
● Restore the lesions with compomer
● Restore the lesions with micro-filled composite
● Restore the lesions with a hybrid composite
● Restore the lesions with conventional glass-ionomer
Ans is C

1205patient attends with pain of four days duration in a carious upper molar
tooth. The pain is constant and is not relieved by paracetemol. Sleep has been
disturbed by the pain. The tooth is tender to percussion and gives a positive
response to Ethyl Chloride. What is the most likely diagnosis?
● Pericoronitis
● Apical periodontitis
● Marginal periodontitis
● Reversible pulpitis
Ans is B?

BY DR.ABDULRAHMAN ALMUALM
MJDF MCQS WITH ANSWERS

1206A 14 year old patient attends with a decayed and hypoplastic LL7. A
radiograph shows the presence of an unerupted LL8 and the LL6 is sound.
What would be the most appropriate long-term treatment for this tooth?
● Amalgam restoration ● Antibiotics ● Extraction
Ans is C
120730 yr-old patient attends complaining of pain from the lower left quadrant.
Clinical examination reveals an extensively restored dentition with generally good
oral hygiene. There is no significant periodontal pocketing other:.than an
isolated defect in the region of the furcation of lower left first molar which is
non-mobile. The gingival tissue in thiS area appears erythematous and slightly
hyperplastic with a purulent exudate on probing. From the list below, which is
the most appropriate next step?

● Obtain a radiograph
● Biopsy the gingival tissue
● Remove the restoration
● Vitality testing
● Prescribe antibiotics
Ans is D
Vitality testing. We want to reach a diagnosis as to whether the lesion is endodontic
or periodontal. The radiograph may show a radioloucency in the furcal area in both
cases (Primary endo lesions can cause furcal or periapical radiolucency. Furcation
involvement also causes furcal radiolucency) but will still not give us a definite
diagnosis. We initially need to know whether the tooth is vital or not, then carry on
with radiographic investigation.

1208A 40 yr old patient had root-canal treatment to his upper first molar. This
was performed 6 months ago using contemporary techniques under rubber dam
and was crowned after completion of treatment. He attends complaining of
continued discomfort from this tooth. Radiographic examination shows each of
the three roots to be obturated with a well-condensed filling to the full working
length though there is no evidence of in-fill of the periapical lesion when
compared to the pre-op view. What is the most likely cause of the continued
problem?
● Extra-radicular infection
● Contamination of canal(s) with E.faecalis
● Uninstrumented canal >MB 2
● VerticaL root fracture
● Perio-endo problem
Ans is C

120921 year old female presents for the first time to your practice. She is very
upset with the appearance of her upper left central incisor. On examination you
find healthy oral hard and soft tissues and excellent oral hygiene. On close
examination you can see that the upper left central incisor is slightly greyer than
the upper right central incisor and has a composite restoration placed palatally.
What is the most appropriate form of treatment given the information you have?
● Bleaching with carbamide peroxide in custom formed trays of upper and
lower arches
● A bonded crown

BY DR.ABDULRAHMAN ALMUALM
MJDF MCQS WITH ANSWERS

● A composite veneer
● A porcelain veneer
● E. Non-vital bleaching with carbamide peroxide
Ans E

1210EDTA (ethylene diamine tetra-acetate) has useful roles in certain situations


in clinical dentistry. When would you use EDTA? A.As a root end filling material
B.As a pulp capping agent
C. As root canal chelating agent
D. As a mouthwash
E. As a dentine bonding agent
Ans is C

1211patient presents with a history of clicking from their temporomandibular joint.


This click occurs mid way through the opening cycle and is consistent. There is
some pre-auricular pain and the lateral pterygoid muscle on the affected side is
tender to resisted movement test. There is no trismus and the click is not present
when the patient opens from an incisor edge to edge relationship, instead of her
normal Class I occlusion. The patient would like treatment. The most appropriate
occlusal splint for this patient would be:
●Stabilisation splint
●Localised Occlusal Interference Splint
●Bite Raiser
●Soft Bite Guard
●Anterior Repositioner Splint
Ans E

The patient has anterior disc displacement with reduction. that's y he has midway
click .. when the condyle tries to translate.. an anterior repositioner splint will put the
condyle in a downword and anterior position allowing the disc to get back to original
position and hea A.Stabilization Splint: mainly used for the correction of the
occlsuion or creating a perfect occlusion for patient with TMD reflected from occlusal
interference, till the muscles return to their normal tension and lengh, then restorative
or selective grinding work will b done B. Localized occusal splints: used for clenching
and bruxism, focusing the occlusion on less number of teeth, making their
propioseptive response sensetive for biting, so the brain avoid excessive biting and
then decrease the Bruxism
C.Bite Raiser: used for the raisong of the bite to relive muscle stress mainly
D. Soft Bite Guard: first line of treatment in symptomatic bruxer, act to decrease
muscle spasm and decrease the habbit. E. Anterior repositioning splint: it is done as
we say [to catch up the disc] as the patient doesnt experience clicking when he
opens from Edge to Edge, thats means that first stage of translation is executed in
this edge to edge movement where the condyle and the disc are aligned as in
normal position. keeping this realtion for a long time allows the retrodiscal tissues
[where pain is moderate ] to heal on this level and the pterigoid spasm will b
decreased. THAT`S THE ANSWER TO THE CASE BEST REGARDS EBTISAM
Combination muscle and disc disorders are identified by joints that click or pop, and
muscle symptoms are also present. These disorders tend to be more chronic in
nature (unless there has been an acute exacerbation), and are associated with more
damage. Stabilization splints are the treatment of choice, as they provide long-term

BY DR.ABDULRAHMAN ALMUALM
MJDF MCQS WITH ANSWERS

wear that is usually needed. They also cover the entire dental arch, ensuring that the
covered teeth do not move. They must be worn continually for 24 hours (except
when eating) for as long as required to eliminate muscle, disc, ligament, and tooth
symptoms. Three to 6 months of wear is often required. These disorders may be
reversible if detected relatively early and treated appropriately.

.1212 A patient presents with a history of a post-crown having fallen out. The
post-crown was originally placed fifteen years ago and had been successful up
until four months ago since when it has come out and been recemented four
times. At recementation there was no evidence of any caries. The patient had
been a regular attender and not needed any restorative treatment for the last
eight years. Which of the following is the most likely cause for the failure of
this crown?
● The post was to narrow
● The post was to short.
● The root canal treatment was failing.
● A vertical root fracture was present.
● There were excessive occlusal loads on the tooth
Ans D

1213A 23 year old patient attends complaining of pain in an upper right molar
and is keen to keep the tooth. The pain is typically sharp in nature, is triggered
by cold and persists after removal of the cold stimulus. The tooth is not tender
to percussion; a radiograph of the upper right first molar shows a large
radiolucency extending to the pulp horn but no peri-radicular changes. What
treatment is most likely needed in this case?
● Oral hygiene instruction and fluoride application
● Excavation of caries and placement of a permanent restoration
● Root -canaI treatment?
● Indirect pulp cap and restoration?
● Direct pulp cap and restoration
Ans C

1214A patient attends your surgery complaining of severe pain, swelling and
mobility associated with a lower first molar tooth in which there is a broken
filling. A periapical radiograph indicates
that the tooth has not been root filled and there is loss of apical lamina dura
associated with the distal root and at the bifurcation. The periodontal bone
support is good. There is no significant pocketing. What is the likely diagnosis?
● Chronic periapical periodontitis
● An acute periodontal-endodontic periodontitis
● A chronic periodontal-endodontic periodontitis
● An acute periapical periodontitis
● Pararadicular periodontitis
Ans is D??

1215 Which substance most widely varies in radiographic appearance

BY DR.ABDULRAHMAN ALMUALM
MJDF MCQS WITH ANSWERS

Acrylic
Soft tissue
Amalgam
Composite

Ans is B

1216 An upper incisor in a 16 year old patient has suffered trauma and the
coronal tissue has been lost. The tooth has been endodontically treated. How is
the tooth best restored?
● With a fibre post, direct core and crown.
● With a direct core and crown.
● With composite
● With an indirect post-core from a pre-fabricated pattern and a crown
● With a custom indirect post-core and crown
Ans E

1217 50 year old male patient has a Class III jaw relationship with an anterior
open bite. It is planned to restore his lower right second molar, which has
suffered tooth wear and fracture, with an indirect restoration. This tooth has
approximately 2mm of coronal height. What would be the most suitable
approach to restore this tooth?
● Provide an adhesively retained gold onlay
● Provide a conventional full crown
● Increase the vertical dimension and provide a full crown
● Surgically crown lengthen and provide a gold crown
● Provide an adhesively retained ceramic onlay
Ans A

1218 patient complains of a lower incisor which has been mobile for several
months. The radiograph indicates a normal level of bony support although the
periodontal space has widened. The apical bone appears normal. The tooth is
tender to pressure. Which of the following tests and or examinations would be
most likely to provide a diagnosis?
● Masticatory muscle palpation
● Electric pulp test
● Occlusal examination
● Ethyl chloride test
● Hot gutta percha application
Ans is C

BY DR.ABDULRAHMAN ALMUALM
MJDF MCQS WITH ANSWERS

1219A 23 year old male presents to your surgery. He lost his upper lateral
incisors some 10 years ago in a swimming pool accident. Since then he has
been wearing a 'spoon' denture which he now feels in aesthetically unacceptable.
He has sought an opinion on dental implants but has been told that he would
need bone grafting for this to be successful and he is not prepared to undergo
this. His dentition is excellent with no restorations and a Class I occlusion. He
wants some advice on what the best treatment might be. Which option would
you put first on your list of possibilities?
● Two fixed - fixed resin bonded bridges using the central and canine teeth
● Two cantilever resin bonded bridges from canines.
● Two conventional fixed - fixed bridges from the canine
● Conventional cantilever bridges from the canines
● Cobalt chrome partial denture
Ans is B

1220 A stabilisation splint [Michegan splint] is commonly indicated in patients


needing advanced restorative dentistry and patients suffering from some
Temporomandibular Disorders Other than upper and lower impressions, which of
the following records will be needed to construct this splint?
● Facebow and a protrusive wax record
● Facebow and an "Oclusal Sketch"
● Centric Relation [Retruded Contact Position] record
● Facebow and a Centric Occlusion [Incuspation Position] record
● Facebow and Centric Relation [Retruded Contact Position] record _
Ans E

1221 55 year old female patient is missing her upper right second premolar
and upper right first molar and also is missing the upper left second molar. The
upper right second molar is functional and has an amalgam restoration (MOD
and buccal wall) that requires replacing. The patient has no functional or
aesthetic concerns. What would be the treatment of choice in this situation? ●
Provide an upper removable partial denture
● Replace the amalgam in the upper right 7 only >bcs patient has no funct and
esth concerns
● Provide a full coverage crown in the upper right 7
● Provide a fixed bridge in the upper right quadrant
● Provide a full coverage crown in the upper right 7 with guide planes and
occlusal rests
Ans A

1222 Endodontic treatment has failed on an upper first molar; the patient is
keen to retain the tooth. There are persistent symptoms from the tooth.
Radiographically there is evidence of periapical radiolucency although the three
canals are obturated with good length and compaction. What is the best course
of action?
● Extract the tooth
● Re-treatment with an iodine solution as irrigant
● Re-treatment with hypochlorite solution as irrigant
● Re-treat the tooth, looking particularly for additional canals
● apicectomy

BY DR.ABDULRAHMAN ALMUALM
MJDF MCQS WITH ANSWERS

Ans is D

1223 You suspect that there is occlusal caries in the lower right first permanent
molar of a 10 year old child. You wish to confirm your suspicions. Which
diagnostic test is most commonly used in this situation?
● Bitewing radiography
● Electro-conductive caries monitors
● Fibro-optic transillumination
● Panoramic radiography
● Visual examination of a dried tooth.
Ans E

1224 You notice that a 20 year old patient has marked tooth surface loss
associated with the labial and palatal aspects and incisal edges of the upper
anterior teeth. They are sensitive to hot and cold. The remainder of the dentition
is mainly unaffected What is the likely diagnosis?
● Attrition
● Active erosion >sensitivity
● Passive erosion
● Abfractions
● Abrasion
Ans is B

1225 The best radiograph for investigating the maxillary sinus is,
● Periapical radiograph
● Panoramic view
● Lateral cephaloghraph
● Occipitomental view
● Reverse Towne's view
Ans is D /waters view 30°

1226 1 In the preparation of Premolar class I cavity what is the best way of
getting retention,
● Slightly done undercut of the mesial and distal walls
● Slightly done undercut ofthe buccal and lingual walls
● The convergence of the cavity walls

Ans is C

1227 In respect to Class V


● it occurs on the buccal groove (fissure)
● it occurs on the lingual groove (fissure)

BY DR.ABDULRAHMAN ALMUALM
MJDF MCQS WITH ANSWERS

●C. it is a result of bad oral hygiene


Ans is C

1228 After the placement of a class I amalgam the patient comes back to you
complaining of pain on masticating and biting; what is the first thing you would look
at,
● Occlusal height
● Contacts areas
Ans A

1229 Child comes to your clinic with a fractured incisor 3 mm super-gingival,


how would you treat the case,
● Formocretasol pulpotomy
● Calcium hydroxide pulpotomy >cveck
● Pulpectomy
● Direct capping
● Indirect capping
Ans is B

1230 What is Ante's Law about,


● The relation between the span of the bridge and the poetics
● The periodontal area of the abutment teeth
● The relation between the length of the root and the abutment.
Ans is B

1231 What is the best way to cement Maryland bridge,


●.GIC
● Resin
● High compression restorative resin
● Zinc Phosphate cement
● Oxide Zinc and eugenol
Ans B

1232 The ideal length of core in the fabrication of crown and core of
endodontically treated tooth is,
● 1.5 of crown length > core not post
● The length of the crown
● 2/3 tooth/root length
● D. 1/2, root length
Ans A

1233 If aesthetic is not a concern what is the first thing to do to treat soreness
under dentures,
● Take the denture off for a week
● Rinse the denture in nystatin
● Apply tissue conditioner
Ans A

BY DR.ABDULRAHMAN ALMUALM
MJDF MCQS WITH ANSWERS

1234 While you finishing a class I cavity, the enamel is sound but you noticed
in the dentine and on the Dento-enamel junction a brown line, what is your
response,
A. You leave it and complete the final restoration
B.You extend you preparation and clean it
C.You apply a cover of varnish
Ans A

1235 Dental caries of the proximal surfaces are usually start at,
● Somewhere between the ridge and the contact area
● Just gingival to contact areas
● Just about the gingival margin
Ans is B

1236 What is the property of high copper amalgam


● Reduced physical creep
● Higher retention
Ans A

1237In regards to colours what is Chroma stands for,


● Degree of saturation of hue
● Brightness
● Value
● Contrast

.1238 Frankfort plane extends from,


● horizontally from Sella to nasion
● Sagittal from
● Horizontally from point on superior aspect of external auditory meatus to
orbitale
Ans C

1239 Which of the following local anaesthetic is indicated in case of the need
to long acting one after a surgical operation, /~
● Mepivacaine
● Bupivacaine (Marcaine)
Ans is B

1240 In respect to Lidocaine 2% with 1:100000 vasoconstrictor,


● The toxic threshold is 22ml
● 8.8 ml is the maximum you can give in one session
Ans is A

BY DR.ABDULRAHMAN ALMUALM
MJDF MCQS WITH ANSWERS

1241Which one of this restorative method will be LEAST compromised by a


core,
● Amalgam
● Composite
● GIC
● Cast gold
Ans is D

1242 In preparing a very small proximal amalgam cavity on a molar tooth what
would consider,
● Extend the cavity to the gingival margin
● Extend the cavity beyond the contacts areas
● Achieve at least 2mm in dentine
● Extend cavity just beyond dento enamel junction
Ans is B

1243 What is true about partial dentures,


● They cause an immediate changes in the oral plaque behaviour
● Night wearing of dentures reduces plaque accumulation
● Relieving the gingival area reduces the gingival enlargement.
Ans A

1244 Compound is,


● Very accurate compression material
● Thermoplastic material
Ans is B

1245 The different between normal stone and the dye stone is,
● In the particles size
● The amount of water
Ans A

1246 A patient comes to you with medium pain of tooth filled with Composite
resin as a result of cold or hot drinks, what your initial management will be
, ● Remove the restorative material and start an Endontic treatment
● Remove the restorative material and place a sedative temporary material
● Place a coat of bonding material on the old composite
Ans C?

1247 Throbbing pain increases with heat and cold stimuli, the MOST probable
diagnosis is,
● Cyst
● Occlusal trauma
● Advanced pulpitis
Ans C

1248 In making your custom trays which of the following is true,


● A uniform thickness is required
● Perforation is better
● Only adhesive is better than perforation

BY DR.ABDULRAHMAN ALMUALM
MJDF MCQS WITH ANSWERS

Ans A

1249 The most common cause of caries in children is,


● Soft diet
● High intake of carbohydrate
● Poor oral hygiene
Ans B

1250 The best storage media for avulsed tooth is,


● Saline
● Milk
● Water
● Saliva
Ans B
The Qs of what is the best medium

You have two references

1.PINK BOOKS IS SAYING --> saline >milk> water> air ...

2. IADT is saying
Milk is better then saliva (which is not recommended for children since they could
swallow the tooth )
or HSSb if it's there..

Based on those you have the right to select what you think is the most correct

.1251 An occlusal approaching clasp TIP,


● Should occupy a predetermined undercut
● Contact the tooth under the survey line
Ans is B

1252 In the construction of partial denture the surveyor is not used to,
● Contour the wax as part of the fabrication of the working cast
● Locate the guide planes
● Determine the location of indirect retainers
● Identify any undesirable undercuts
Ans Is C

Surveyor can -outline desirable and undesirable undercuts -establish guide planes
-contour wax -block out undercuts -establish path of insertion -check parallelism

.1253 The main purpose of finishing the enamel walls is,


● Remove loose enamel rods
● Provide a better surface for the adoption of restorative material
Ans A

1254 The location of Class V is in,


● The bucca I pit /fissu re

BY DR.ABDULRAHMAN ALMUALM
MJDF MCQS WITH ANSWERS

● The occlusal surface


●Cervical third
Ans is C

1255 Occlusal cavity with extension of the buccal fissure is classified as,
● Class II
● Class III
● Class I
Ans C

1256 Which of the following does not affect the elasticity of retentive clasp?
● Length of the arm
● The cross section shape
● The material used
● The undercut area
Ans is D

1257 A partial denture that seats on the master cast but fails to seat correctly
in the mouth is a result of,
● Contraction of the metal framework during casting
● Insufficient expansion of the investment material
● Distortion of impression
● Failure to block out unwanted undercuts
Ans is C

1258 Which of the following muscles may affect the borders of mandibular
complete denture?
● Mentalis
● Lateral pterygoid
. ●. Orbicularis oris
● Levator oris
● Temporalis
Ans A

1259 What is the main purpose of using corticosteroids in pulpal obturation


material?
● For their antibiotic action
● For their antiinflammatory action
● To relief pulp pressure
Ans B

1260
Which of the following statements is incorrect regarding Smoker's Keratosis?
● Typically affects the hard palate
● Minor mucous glands are swollen with red orifices
● There is a little regression if smoking is stopped
Ans is C

1270 13 years old boy comes to you with excessive hyperplasia of the gingiva
as a result of Phenytoin what is your management,

BY DR.ABDULRAHMAN ALMUALM
MJDF MCQS WITH ANSWERS

● Stop the medication


● Force a strict oral hygiene and surgical removal of excess gingival tissues
● Debridement and conservative approach
AnS B

1271 White man 56 years old comes to you with a brown spot on his gingiva
and another one on his oral mucosa, when taking the history he mentioned a
weight and memory lost. He as well complains of headaches. What is your
most probable diagnosis,
● Addison's disease
● Hyperthyroidism
Ans A

1272While removing the second primary molar of 9 years old child, the apical
third of the root fracture and stay in the socket,
● You will just leave it and observe it
● You take surgically by a lingual flap
● You try to take out by using a root apex elevator
● You use a fine end forceps to take it out
Ans is A

1273 What is the most important factor to reduces dental irradiation,


● Speed of film
● Collimation
● Filtration
●. Cone shape and length
Ans B

1274 With view to Nitrous Oxide what is the major pharmacological problem?
● Contraindicated in pregnancy
● Contra indicated in cardiac dysrhythmias > reletive contraindaction
Ans A

1275 Which of the following is an expansile lesion of the oral mucosa,


● Keratocyte
● Radicular cyst
● Cementoma
Ans is C

1276 The concentration of Fluoride in the topical NaF


A.2%
B.5%
C. 8%
D.10%
Ans B

1277 patient in your dental chair shows chest pain, weak pulse and dysponea,
what is your initial management,
● Give a nitro-glycerine tablet and keep the patient up seated'
● Put the patient in supine position

BY DR.ABDULRAHMAN ALMUALM
MJDF MCQS WITH ANSWERS

● Do nothing and wait until the symptoms go


Ans is A

1278 What are two teeth connected at the cementum called,


A. Concrescence
B. Dilaceration
C. Gemination
D. Fusion
Ans A

1279 Developer contaminated with other chemical and was not mixed pro
perlv. What is the effect on the X-ray film?
● Too dark film -
● Light film
● Foggy
Ans is C !!

1280 Which part of the cranium is considered as the most stable area,
● Frankfort plane
● Occlusal plane
● Anterior cranial plane
● Anterior nasal to gnathion.
Ans is C

1289 The difficulty of placing matrices on deciduous dentition is a result of,


● The small mouth of kids which result in problem keeping the matrices in
their mouths
● The occlusal convergence of the deciduous teeth
Ans B

1290 the most common way of oral carcinomas to other tissues is/are
● Lymphatic
● Invasion and blood
● Lymphatic and invasion
● Blood and lymphatic
Ans C

1291 The main purpose of periodontal treatment is,


● Elimination of plaque and calculus
● Elimination of periodontal pockets
● Reformation of all the periodontal ligaments
● The elimination of all occlusal trauma
Ans is a

1292 The thermal and electric pulp tests will,


● Give an accurate indications of the pulp status
● The patient's response will be either pain or no pain
● The patient can differentiate between cold or hot stimuli
Ans is B

BY DR.ABDULRAHMAN ALMUALM
MJDF MCQS WITH ANSWERS

1293 The removable partial denture requires relining what is would be the most
appropriate action,
● take a new impression by asking the patient to occlude on it
● Provide equal space between denture and gingival tissues.
● Make sure the framework and retainers are seated in place before taking
impression
Ans is C

1294 In regards to dentine strength, which is the right sequence


, ● Affected dentine> Sound dentine> Infected dentine
● Sound dentine> Affected dentine> Infected dentine
Ans B

1295 Symptoms free patient comes to you after four weeks of an endodontic
treatment and you find on radiograph the canal is over filled with what it seems
to be a cone of Gutta Percha lmm beyond the apex with a radiolucent small
area. What is your initial management?,
● Start apiectomy through a flap and surgery
● Obturate the root canal
● Ask for a recall and observe in three months time
● Seal the pulp chamber and keep it under observation
Ans C

1296 After obturation and on X-ray you notice the obturation materials are lmm
beyond apex. What is your first management?
● Refill the canal
● Pull the GP cone about lmm out and take a new X-ray
● Leave it as it
Ans C

1297 Four years kid shows at your clinic with open bite as a result of thumb
sucking, you notice a delayed speech ability what would be your first
management,
● Refer to a speech therapist
● Apply a removable habit inhibitor denture
● Apply a removable habit inhibitor denture and educate the parents about it
so the kid will not be taking it off so often
Ans is A

1298 After the initial development stage and in the absence of pathology, the
size of the pulp chamber has been reduced by,
● Deposition of primary dentine
● Deposition of secondary dentine
● Reparative dentine
● Pulp fibrosis
Ans A

1299The most desirable outcome of endodontic treatment is,


● The healing of the alveolar bone

BY DR.ABDULRAHMAN ALMUALM
MJDF MCQS WITH ANSWERS

● The deposition of cementum at the apex


● Formation of fibrous capsule around the apex
Ans A

1300What is NOT related to the normal aging process?


● Progressive bone loss
● Reduced elasticity of muscles
● Decreased elasticity of the skin
● Lower pain threshold
Ans D*

1301The most accurate finding of pulpal pathology


● Radiolucency on the apical region
● Pain on hot or cold drinks
● The absence of response to pulp
Ans is B

1302For dental caries to progress in dentine,


● enamel must contain glycoproteins
● diet must contain simple carbohydrate
● diet must contain polysaccharides
● pulp must contain complement
● the dentine must contain soluble collagen
Ans is B

1303year old boy has a small white discolouration on his maxillary central
incisor. The lesion is most probably,
● Hypocalcification due to trauma of the primary predecessor
● Hypoplasia due to acute systemic infection when 6-12 months old
● Defect during the histo differentiation stage of development
● Defect during the morho differentiation stage of development
Ans A

In hypocalcification of successor due to trauma of predecessor you will see "whitish


discoloration" (white spots) - due to hypocalcification. & In hypoplasia due to acute
early childhood illness or stressfull birth , you obseve "greyish whitish bands" (An
attenuation of lines of retzius)

1304The best method to take X-ray of the maxillary sinus is,


● Periapical radiograph
● Panoramic view
● Lateral cephaloghraph
● Occipitomental view
● E. Reverse Towne's view
Ans D

1305Which of the following is not a part of the fully formed enamel organ,
● Outer enamel epithelium
● Inner enamel epithelium
Ans is B

BY DR.ABDULRAHMAN ALMUALM
MJDF MCQS WITH ANSWERS

1306.18 years old female her weight is 52Kg and she is 163cm tall. On dental
examination erosion of teeth on the most of her lingual surfaces is clearly
showing. Dietary history revealed a daily rate of 5000 to 7000 Kcal/day. What is
most probable would be her case?
● Alcoholism
● Drug abuse
● Bulimia
● Excessive smoking
● Diabetic mellitus type I
Ans is C

1307Which one of the following is true in regards t osseointegration implants in


dentistry?
● Fibrous tissues are formed and integrated directly between titanium and bone
● Following insertion, implants can be immediately loaded without problem
● The success of the implants is directly proportional to its area of contact
with bone
● The success of the implants depends mostly on low torque preparation and
insertion of the fixture
Ans C

1308Generalised lost of tooth structure by chemical means called,


● Erosion

1309On X-ray, the buccal roots of 16 is considerably elongated; this is a result


of,
● Too great vertical angulation
● Inadequate vertical angulation
● Excessive object film distance
Ans is B

1310The principle muscle responsible for the opening of the mouth is,
● Mylohyoid
● Anterior temporal
● Posterior temporal
● I. Anterior belly of digastric
Ans D

1311Denture stomatitis is commonly associated with


● The continuous wearing of removable orthodontic appliances in otherwise
healthy patient
● The proliferation of hypertrophic tissue at the denture periphery
● The overgrowth of some constituents of oral normal microflora
● Allergy to denture base material
Ans C

1312Which of the following is NOT characteristic of Down's syndrome?


● Decreased neutrophil function
● Macroglossia

BY DR.ABDULRAHMAN ALMUALM
MJDF MCQS WITH ANSWERS

● Macrodontia
● An increased susceptibility to periodontal disease
● Congenitally missing teeth

Ans C

1313Which of the following is the best evidence that a previous periodontal


treatment is successful?
● The patient keeps a 3 month recall appointment
● There is no extrinsic stain
● The patient demonstrates good understanding of brushing and flossing
techniques
● There is no bleeding on probing
Ans D

131465 year old female presents at your surgery complaining of an extremely


sharp pain of a few seconds duration which arises whenever she touches an area
of skin above the right hand side of her upper lip adjacent to the angle of her
mouth. The patient is fit and well and is not taking any medication. You make a
diagnosis of trigeminal neuralgia. What is the drug of choice for treating
trigeminal neuralgia in such a patient?
● Carbamazepine
● Oxcarbamazepine
● Gabapentin
●Phenytoin
Ans is A

1315patient on examination was found to have swollen gingiva around a crown


that had been present for several years. The papillae were particularly enlarged.
What is the most important feature of a crown that may be responsible for
this?
● Material of the Crown
● The occlusion
● proximal Contour
● Labial Contour
● Surface finish
Ans C

131613 year old girl presents with an unerupted permanent canine and a
retained primary canine. You cannot palpate the unerupted canine in the buccal
sulcus and you are uncertain as to whether it is displaced palatally or in the
line of the arch. Which single radiographic view would be most helpful in
locating the unerupted tooth?
A. Bitewing
B.DPT
● Lateral oblique
● Single periapical
● Upper anterior occlusal
Ans E

BY DR.ABDULRAHMAN ALMUALM
MJDF MCQS WITH ANSWERS

131730 year old man with unknown allergy to latex goes into anaphylactic
shock whilst being treated in the dental surgery. Which drug and route of
administration is of most benefit in this situation?
● Hydrocortisone - orally
● Chlorphenamine - intramuscularly
● Chlorphenamine - orally
● Epinephrine - intravenously
● Epinephrine – intramuscularly
Ans E

131862 year old female presents at your surgery complaining of a persistent,


dull ache affecting her upper left 4. The pain is present all the time but varies
in its severity although the patient cannot think of any exacerbating factors. It is
not relieved by analgesics. Over the past 18 months the patient has had several
teeth extracted from the upper left quadrant. Each extraction brings about
temporary relief of her symptoms only for them to recur in an adjacent tooth.
What is the most likely cause of the patient's pain? ● Acute/reversible pulpitis
● Dentine sensitivity
● Chronic/irreversible pulpitis
● Atypical odontalgia
● E. Trigeminal neuralgia
Ans is D

131925 year old patient attends your surgery complaining of a swelling at the
angle of the mandible. A radiograph shows a uni-locular radiolucency associated
with the crown of an unerupted wisdom tooth. Which of the following is the most
likely diagnosis? ● Radicular cyst
● Dentigerous cyst
● Lateral periodontal cyst
● Ameloblastoma
● Odontogenic keratocyst
Ans is B

Ameloblastoma and keratocyst are usually multilocular.asscoiated with diplacement


of tooth/roots. When its unilocular surrounding unerupted crown -always
dentigerous).

1320A 70 year old female, who suffers with persistent looseness of her lower
complete denture, is considered for implants. She will require radiological
evaluation of the potential implant sites. Which would be the most appropriate
radiological investigation at this stage?
● MRI scan of the mandible
● CT scan of the mandible > first choice is cbct then ct
● Periapical radiographs of the mandibular anterior region
● True lower anterior occlusal view
● Panoramic radiograph
Ans is B

BY DR.ABDULRAHMAN ALMUALM
MJDF MCQS WITH ANSWERS

1321Six months ago you saw a child patient, then aged 9 years. His upper
right maxillary canine was palpable in the labial sulcus but the upper left was
not. The situation is now unchanged, so you have taken two periapicals of the
non-palpable tooth. They both show that there is some resorption of the CI root
but the permanent canine appears somewhat mesially angled and is more
mesial on the more mesially positioned film. What is your the best course of
action?
● Keep a careful watch on it and take another x-ray in 6 months.
● Refer to an oral surgeon for early exposure of the permanent canine.
● Refer to an oral surgeon for early removal of the permanent canine before it
damages the lateral incisor.
● Refer to an orthodontist for a treatment plan.
● Wait and watch' until the child is 11. .
Ans D

1322An apical radiolucency (2mm) is noticed as an incidental radiographic


finding associated with the apex of the mesiobuccal root of the lower right first
molar. The tooth has been root filled but is 2mm short of the radiographic apex.
There are no other clinical or radiographic findings and the patient is fit and well.
What is the most appropriate course of action?
A.Extract the tooth.
b.Redo the root filling
c.Perform periapical surgery.
d.Advise the patient of the situation and monitor clinically and radiographically
E .Prescribe antibiotics then review
ANS D

132343year old patient is missing on the upper right the first premolar and
molar. He has good oral hygiene and requests a fixed replacement for these
teeth. The other teeth on the same side are all moderately restored with MOD
amalgam restorations and are vital, except the canine, which has a very large
restoration and is root-filled. He has group function. Radiographs show a large
sinus cavity and no peri-apical pathology. What would be the restoration of
choice for replacement of the missing teeth?
● Implant supported crowns
● A conventional fixed bridge using the 7 and 5 as abutments
● Two conventional cantilevered bridges, using the 7 and 3 as abutments
● A resin-bonded bridge, using the 7 and 5 as abutments
● A conventional fixed-moveable bridge using the 7 and 5 as abutments
Ans A.

1324Epidemiological studies have shown that dental decay is normally greater


amongst northerners and those in socially deprived circumstances. Which of the
following would be most effective in reducing cariesJn a high risk population?
● Brushing with fluoride toothpaste
● Publicity campaign
● Fluoridation of the water supply
● Dietary advice via schools
● Fissure sealant provision
Ans C

BY DR.ABDULRAHMAN ALMUALM
MJDF MCQS WITH ANSWERS

132558 year old male presents at your surgery complaining of a sharp pain of
no more than 30 minutes duration arising from his upper left molar region. The
pain is brought on by cold stimuli but persists after-the stimulus is removed. It
does not seem to occur spontaneously. He has tried taking paracetemol and
this does temporarily stop the pain from recurring. The upper left 6' reacted to
a lower current on electronic pulp testing than the upper right 6, upper left ~
or the lower left molars. What is the most likely cause of the patient's pain? -
● Acute/reversible pulpitis
● Dentine sensitivity
● Chronic/irreversible pulpitis
● Periapical periodontitis
● Trigeminal neuralgia
1326 .A 58 year old male presents at your surgery complaining of a sharp pain
of no more than 30 minutes duration arising from his upper left molar region.
The pain is brought on by cold stimuli but persists after-the stimulus is
removed. It does not seem to occur spontaneously. He has tried taking
paracetemol and this does temporarily stop the pain from recurring. The upper
left 6' reacted to a lower current on electronic pulp testing than the upper right
6, upper left ~ or the lower left molars. What is the most likely cause of the
patient's pain? -
● Acute/reversible pulpitis
● Dentine sensitivity
● Chronic/irreversible pulpitis
● Periapical periodontitis
● Trigeminal neuralgia
Ans C

1327.60 year old patient attends your surgery complaining of a sore mouth. He
has Type II diabetes well controlled by diet and metformin. On examination
white patches which cannot be removed are present on his buccal mucosa:
What is the most likely diagnosis?
● Frictional keratosis
● Leukoplakia
● Lichen Planus
● White sponge naevus
● Candidosis

1328An 80 year old patient presents with an ulcer in the floor of the mouth.
This has been present for several months and has not responded to
conventional treatment. An incisional biopsy is taken. Which of the following
histological changes in the epithelium confirm a diagnosis of squamous cell
carcinoma?
● Hyperkeratosis
● Acanthosis

BY DR.ABDULRAHMAN ALMUALM
MJDF MCQS WITH ANSWERS

● Dysplasia
● Invasion
● Discontinuous epithelium
Ans D

1329 . A 35 year old patient complains of swollen gums. This has been present
for several years. What is the commonest cause of this complaint?
● Cyclosporin therapy
● Vitamin C deficiency
● Chronic Gingivitis
● Atenolol therapy
● E. Pregnancy

Ans C

1330A 45 year old patient attends the dental clinic complaining of a clicking
jaw. Examination reveals a reproducible click of the right TMJ when opening
wide. Upon asking the patient to open wide, close with incisors edge-to-edge
and then open and close to this position, the click is absent. From the options
below, which one is the most likely diagnosis?
● Myofascial pain
● Disc displacement with reduction
● Disc displacement without reduction
● TMJ osteoarthritis
● Arthralgia
Ans B

1331Cigarette smoking is considered to be the most important factor next to


microbial plaque in periodontal disease progression. Which of the following is the
most important factor in the disease progression in smokers?
● Smokers have drier mouths than non-smokers .
, ● Smokers have poorer oral hygiene than non-smokers.
● Nicotine will impair the chemotactic and phagocytic properties of PMNs.
● The gingival blood flow is reduced in smokers.
● Smokers alter the oral environment encouraging the growth of anaerobic
bacteria
Ans C

1332An adult patient attends your practice complaining of pain and swelling
associated with a previously restored upper first premolar tooth. The pain has
been present for a number of days and is no longer responding to analgesics.
His dentition is otherwise well maintained and his periodontal health is good.
What is the most appropriate approach to treatment?
● Antibiotics and analgesics.
● Extract the tooth
● Carry out a pulpotomy. Temporary dressing.
● Carry out a pulpectomy. Temporary dressing
● E. Establish open drainage
Ans D

BY DR.ABDULRAHMAN ALMUALM
MJDF MCQS WITH ANSWERS

1333patient says that he does not like the appearance of his previously root
filled upper central incisor tooth. His dentition is otherwise well maintained and
his periodontal health is good. The tooth appears to be darker than the adjacent
teeth. What is the most appropriate approach to treatment?
● Provision of a post crown
● Provision of an all ceramic crown
● Provision of a metal bonded to ceramic crown
● Carry out a non vital bleaching procedure
● Provision of a porcelain veneer
Ans D

133420 year old patient attends your surgery for the first time. You suspect
that he may have proximal caries as he has a frequent sugar intake. Which of
the following is the most accurate method of diagnosing proximal caries in a
lower molar tooth in this young adult? A. Clinical history
● Periapical radiograph
● Bitewing radiograph X
● Digital image
● Electronic resistance measurements
Ans B

133525 year old male attends for the first time complaining of sensitivity of a
number of teeth. On examination, the occlusal surfaces of all the teeth are
worn with obvious wear facets on the canines and premolars. Posterior
amalgam restorations are proud of the surrounding tooth. What would be the
first stage management?
● Take impressions for study models
● Prescribe fluoride mouth rinse
● Replace the amalgam restorations
● Dietary analysis
● Placement of resin sealant to sensitive teeth
Ans is D //☆ some drs answered E

133635 year old male patient who admits to grinding his teeth at night has a
number of wedge- shaped cervical (Class V) lesions on his upper premolar
teeth. These are causing some sensitivity and are approximately 3mm deep.
What is the correct management option?
● Provide tooth brushing instruction and fluoride
● Restore the lesions with compomer
● Restore the lesions with micro-filled composite
● Restore the lesions with a hybrid composite
E. Restore the lesions with conventional glass-ionomer
Ans C /* some answered B

1337patient attends with pain of four days duration in a carious upper molar
tooth. The pain is constant and is not relieved by paracetemol. Sleep has been
disturbed by the pain. The tooth is tender to percussion and gives a positive
response to Ethyl Chloride. What is the most likely diagnosis? .,
● Pericoronitis

BY DR.ABDULRAHMAN ALMUALM
MJDF MCQS WITH ANSWERS

● Apical periodontitis
● Marginal periodontitis
● Reversible pulpitis
● Irreversible pulpitis
Ans B?

1338You are trying in a partial chrome denture framework which fails to seat
properlv. It fits the master cast. What is the most likely cause of this problem?
● Insufficient expansion of the investment material
● Distortion of the impression
● Contraction of the metal framework during casting
● Failure to block out unwanted undercuts
● Complex denture design
Ans B

1339Bitewing radiography is the main special text used to help in diagnosis of


proximal caries. The performance (accuracy) of a diagnostic test like bitewing
radiography can be expressed in terms of sensitivity and specificity. Which of the
following is a reasonable summary of the diagnostic accuracy of bitewing
radiography for proximal caries diagnosis?
● Moderate sensitivity and low specificity
● Moderate sensitivity and moderate specificity
● Moderate sensitivity and high specificity
● D. High sensitivity and moderate specificity
● E. High sensitivity and high specificity
Ans D

1340You are interested in finding out what the risk indicators are for a rare
form of oral cancer and decide to undertake a study to examine this. What
type of study would be the most appropriate for addressing this issue?
● Cohort
● Prevalence study
● Clinical trial
● Case-control study
Ans D

1341You take a panoramic radiograph of a patient and discover a well-defined,


corticated radiolucent area below the inferior dental canal just anterior to the
mandibular angle. What is the most likely diagnosis?
● Radicular cyst
● Stafne bone cavity

BY DR.ABDULRAHMAN ALMUALM
MJDF MCQS WITH ANSWERS

● Metastatic carcinoma of the breast


● Adenomatoid odontogenic tumour
● Complex odontome

Ans B

1342You take a panoramic radiograph out of a patient's records but you find
that the film has a low density and poor contrast. Which of the following errors
could lead to low density and poor contrast?
● Too long a development time > poor contrast dark film or high density
● Developer temperature too high >poor contrast dark film
● Developer temperature too low> poor contrast pale or low density film
● Inadequate fixation > poor contrast
● poor storage > poor contrast
Ans C

1343You want to evaluate the effectiveness of using tetracycline as an adjunct


to scaling and root planing for the treatment of chronic periodontitis. What type
of primary study design would be most appropriate for addressing this topic?
● Cohort study
● Non-randomised controlled trial
● Randomised controlled trial
● Case-control study
E. Case-series
Ans C,

1344Radiation protection of patients is partly dependent upon equipment


factors (x- ray set and film or digital system). The different factors that can be
changed vary in financial cost to the dentist and in their effectiveness in cutting
x-ray dose. Which of the following provides the most cost-effective means of
minimizing patient radiation dose in dental intraoral radiography?
● Constant potential ('DC) x-ray set
●Rare earth filtration
● Lead apron
● D speed film
● F speed film
Ans E

BY DR.ABDULRAHMAN ALMUALM
MJDF MCQS WITH ANSWERS

134514 year old patient attends with a decayed and extensive hypoplastic LL7:
He is a very irregular attender with poor oral health habits. A radiograph shows
the presence of an unerupted LL8 and the LL6 is sound. What would be the
most appropriate long-term treatment for this tooth?
● Amalgam restoration
● Antibiotics
● Extraction
● Root canal therapy
● Sedative dressing
Ans C

13467 year old boy has previously had all primary molars restored and a
pulpotomy on upper right E. He has an early mixed dentition with lower lateral
incisors erupting. There is a midline diastema of 2 mm. The upper right E has
become symptomatic and requires extraction.The most likely long term effect of
the extraction on the occlusion is:
● Early eruption of the second premolar.
● Loss of upper central line.
● No significant effect.
● Overeruption of the lower right teeth.
●potential crowding of the right upper quadrant

.1347 Which of the following is the ideal treatment for a degree II furcation
involvement of a mandibular molar?
● Tunnel preparation
● Root resection
● Furcation plasty
● Extraction
● Guided Tissue Regeneration
Ans is E

1348You examine a patlent and find BPE code 4 in all sextants. Radiographs
show generalised horizontal bone loss with a minimum of 50% of bone support
remaining on all teeth. Which of the following is the most important factor when
considering the prognosis for the teeth?
● Age of the patient
● The Oral Hygiene Status
● Bleeding on Probing Score
● Mobility
● Gingival Recession

1349A mother is concerned that her child's adult upper front teeth have not
erupted and asks your advice. What is the usual age in years for the upper
permanent central incisors to erupt?
● 5 years.
● 6 years.
● 7 years.
● 8 years.
● 9 years.
Ans is C

BY DR.ABDULRAHMAN ALMUALM
MJDF MCQS WITH ANSWERS

1350You decide to refer an eight year old child to the oral surgery department
in your local hospital for extractions under a general anaesthetic. What key
reason for asking for a general anaesthetic would you put in the referral letter?
● Parents request GA.
● Failed to complete treatment under inhalation sedation.
● Child would not accept local analgesia
● Parents think hospital more convenient.
● Not one of my regular patients.
Ans B
1351 An eight-year-old boy presents with pain of three days duration that has
kept him awake. On examination you see a grossly carious lower left 6 and
some associated buccal swelling. Which of the following is the most appropriate
to give immediate relief of his pain?
● Extract the LL6.
● Gently excavate the. caries and obtain drainage.
● Give antibiotics.
● Incise any swelling
● Refer for-general anaesthetic
Ans is B

135210 year old girl tripped and-knocked out her upper incisor 10 minutes ago.
She is holding it in her hand. What is your most effective immediate treatment?
● Accept the tooth is lost and do nothing.
● Refer to specialist.
● Re-plant immediately.
● Root fill, clean and re-plant.
● Sterilise root in alcohol and re-implant.
Ans C

1353You are treating a patient who has a long history of recurrent episodes of
oral candidosis. His mouth has recently become sore once again and you want
to prescribe some antifungals for him. You check his medical history and find
that he suffers from atrial fibrillation and is taking warfarin. Which of the
following drugs would be most appropriate to treat this patient's candidosis? ●
●Metronidazole
● Nystatin
● Fluconazole
● Miconazole
● Amoxycillin
Ans is B

1354dental company has claimed that ozone is better than conventional


methods for treating decay in peoples' mouths. Which of the following would
provide the best evidence to support these claims?

BY DR.ABDULRAHMAN ALMUALM
MJDF MCQS WITH ANSWERS

● Systematic review of Randomised Controlled Trials


● Randomised controlled trial
● Cohort study
● Case/control study
Ans is A

1355A GDP in Manchester randomly selects 5000 patients, aged between 45


and 55 years, who are free from periodontitis. He determines that 300 of them
are smokers. He follows all 5000 patients for 10 years, by which time 150 had
developed periodontitis. Of the patients with periodontitis, 60 had previously
been identified as smokers and 90 as non-smokers. What type of study is this?
● Case-series
● Prevalence study
● Clinical trial
● Case-control study
● Cohort
Ans is E

1356Randomised controlled trials enable us to establish cause and effect. The


main reason for using this study design is to:
● Ensure that making a Type II error is minimised.
● Ensure that test and control groups are similar.
● Ensure that test and control groups are of equal size.
● Help recruitment of patients into the trial.
● Provide sufficient power for the study
Ans is E

1357 53 year old male patient presents with an asymptomatic white patch on
the ventral surface of his tongue. He has smoked 35 cigarettes a day since he
was 17. An incisional biopsy of the white patch is likely to show which
histological feature:
● Basal cell liquefaction
● Hyperkeratosis
● Saw tooth rete ridges
● Acantholysis
E. Basal cell hypoplasia
Ans B

135846 year old female presents withi a slowly enlarging painless firm swelling
in the hard palate to the left of the midline. The most likely diagnosis is:
● A dental abscess
● Torus palatinus
● Osteoma
● Pleomorphic adenoma
● Canalicular adenoma

BY DR.ABDULRAHMAN ALMUALM
MJDF MCQS WITH ANSWERS

Ans D

135924 year old man presents with a loculated cystic radiolucency in the lower
third molar area approximately 3cm in diameter. nThere is no tooth associated
with the cyst and there is no bucco-lingual expansion of the mandible. There
is no resorption of the roots of the overlying second molar tooth. How would this
be treated?
● Marsupialisation
● Enucleation and ethyl chloride
● Block resection
● Enucleation and Carnoy's solution >keratocyst
● Enucleation and formalin solution
Ans D

1360year old female presents for the first time to your practice. She is very
upset with the appearance of her upper left central incisor. On examination you
find healthy oral hard and soft tissues and excellent oral hygiene. On close
examination you can see that the upper left central incisor is slightly greyer than
the upper right central incisor and has a composite restoration placed palatally.
What is the most appropriate form of treatment given the information you
have? ● Bleaching with carbamide peroxide in custom formed trays of upper
and lower arches
● A bonded crown
● A composite veneer
● A porcelain veneer
● Non-vital bleaching with carbamide peroxide

1361EDTA (ethylene diamine tetra-acetate) has useful roles in certain


situations in clinical dentistry. When would you use EDTA? A.As a root end filling
material
B. As a pulp capping agent
C. . As a root canal chelating agent
D. As a mouthwash
E. As a dentine bonding agent
Ans is C

1362Glass ionomer cements are used in restoring Class V cavities. Which of


the following constituents are most likely to be present in glass
ionomer'cements?
A Alumino-silicate glass and phosphoric acid
B. Phosphoric acid and zinc oxide
C.. Polyacrylic acid and zinc oxide
D.Aluminosilicate glass and polyacrylic acid
E.Itaconic acid and zinc oxide
Ans D

1363n dental epidemiology, indices are used to measure the oral health of a
population. The DMF index is commonly used to measure the prevalence and
severity of dental caries. What is the main limitation of this approach?
● It does not allow statistical analysis.

BY DR.ABDULRAHMAN ALMUALM
MJDF MCQS WITH ANSWERS

● It gives equal weight to decayed, missing and filled teeth.


● e. It is difficult to calibrate examiners.
● It is reversible.
● There is no gold standard
Ans is B

1364The biological process by which the architecture and function of the lost
tissue is completely restored". In Periodontology this is a definition of which of
the following
● New Attachment
● Regeneration
● Repair
● Primary Healing
● Reattachment
Ans B

1365There has been much debate regarding the effectiveness of fluoride in


water for preventing tooth decay. A systematic review of the evidence drew
conclusions as to the reductions in decay which can be expected. Fluoridation of
public water supplies:
● Reduces tooth decay by 10%.
● Reduces tooth decay by 20%.
● e. Reduces tooth decay by 30%.
● Reduces tooth decay by 40%.
● Reduces tooth decay by 50%.
Ans is E

1366An obese 40 year old who takes metformin is seen at the end of a
morning clinic. After administration of local analgesia in the supine position the
patient complains of feeling unwell. She is pale and sweating and is confused.
No pulse was detectable. She was given oxygen and maintained in the supine
with no improvement in condition. What would be the most appropriate drug to
administer next?
● Glucagon
● Glucose
● Adrenaline
● Hydrocortisone
● Chlorphenamine
Ans B

136712 year old boy is brought in by his mother who is concerned about the
appearance of his central and lateral maxillary incisors. She says they look like
they might be decayed. They are a new family to the practice having recently
moved to the area from Birmingham. On examination you notice a generalised
whitening of the incisors which seems to worse at the incisal edges. After you

BY DR.ABDULRAHMAN ALMUALM
MJDF MCQS WITH ANSWERS

have dried the teeth you notice that the whiteness appears to be linear running
from mesial to distal. There is a similar pattern on the lower incisors. What is
the most likely explanation for the presentation of the boy's incisors?
● Tetracycline staining
● Early caries (white spots)
● Fluorosis
● Demarcated Enamel Opacities
E. Previous treatment with composites
Ans C

1368A 23 year old patient attends


complaining of pain in an upper
right molar and is keen to keep the
tooth. The pain is typically sharp in
nature, is triggered by cold and
persists after removal of the cold
stimulus. The tooth is not tender to
percussion; a radiograph of the upper
right first molar shows a large
radiolucency extending to the pulp
horn but no peri-radicular changes. What treatment is most likely needed in this
case?
● Oral hygiene instruction and fluoride application
● Excavation of caries and placement of a permanent restoration
● Root-canal treatment
● Indirect pulp cap and restoration
● Direct pulp cap and restoration
Ans C

1369A child of 5 years attends with pain from a grossly decayed lower right D
which has a discharging sinus. He is a hemophiliac. Which of the following is the
most appropriate treatment to relieve his pain?
● Antibiotics
● Extraction
● Fluoride application
● Non-vital pulpotomy
● Vital pulpotomy
Ans is D (its pulpectomy #!)

1370You are designing a partial denture for a patient with several missing teeth
in the maxilla. The reason for surveying the model prior to designing the
denture is to:
● Measure and mark out hard and soft tissue undercuts on the casts
● Relate the intended position of the inter-papillary plane of the patient to the
casts
● Establish the position of the post dam
● Relate the maxillary and mandibular casts
● Aid setting up the prosthetic teeth prior to trial insertion
Ans A

BY DR.ABDULRAHMAN ALMUALM
MJDF MCQS WITH ANSWERS

1371The parotid gland is one of the major salivary glands that supply the oral
cavity. Where is the orifice of the duct of the parotid gland located?
● At the hamular notch
● In proximity to the incisive papilla
● On the buccal mucosa near the maxillary second molar
● Slightly posterior to the mandibular central incisors
● Distal to the maxillary third molars on the palatal side
Ans C

1372Any patient receiving treatment under IV sedation must have their blood
pressure recorded as part of their assessment. What is the maximum blood
pressure that is generally regarded as being compatible with safe sedation in
general dental practice?
A.160/95
B.140/95
● 160/90
● 170/100
● E.120/80
Ans D

1373A 13 year old boy with


Down's syndrome attends
for the first time. He is
cooperative and has no
relevant medical history. He
is caries free, apart from
two small occlusal cavities
in his lower second primary
molars. His mother requests
that these teeth are
restored. Which of the
following is the most
appropriate management?
● Arranging to extract these
teeth at a future visit.
● Arranging to place two
simple restorations at a
future visit
. ● Reassurance that these
teeth can be left until they exfoliate naturally, with no treatment being
necessary.
● Recommending use of fluoride mouthwash. ● Take OPT to check on
unerupted teeth.

Ans is E

BY DR.ABDULRAHMAN ALMUALM
MJDF MCQS WITH ANSWERS

13743 year old child attends, showing evidence of bottle caries and also decay
affecting primary molars. The child's mother has heard that some sugars are
damaging her child's teeth. She is not sure which foods to avoid. From the
following, which are you most likely to suggest she avoids?
● Bread
● Cheese
● Ground almonds
● Pure fruit juices
● Whole fruits and vegetables
Ans D

1375An adult patient complains of her prominent upper front teeth and
receding chin. She says she did not get 'braces' when she was younger
because the family had to move area a lot. Her oral care and health is good,
and she has a 9 mm overjet. Where is the most suitable place to refer her? "
● A GDP friend who has a special interest in orthodontics and has been
trained to use fixed appliances.
● A specialist practitioner who uses a lot of functional appliances
● A private specialist practitioner, because she's too old to get NHS treatment
now.
● A hospital consultant, as it is likely she will need surgery now to correct her
problem.
● The nearest dental hospital, although it is 70 miles away.
Ans D

1376You are meeting a three year old patient for the first time. His mother
explains that he had a unilateral cleft lip and palate that was repaired in
infancy. She has heard that orthodontics will usually be required when he is
older. She asks you what is the commonest orthodontic problem that occurs
with a repaired cleft palate. What.:.is the most appropriate answer?
● A contracted maxillary arch -
● An anterior open bite.
● An elongated maxillary arch.
● An expanded maxillary arch
. ● An increased overjet
Ans is A

1377Chronic periodontitis is a disease of the periodontium initiated and


sustained by microbial plaque. Which of the following is the main diagnostic
feature of the disease?
● Tooth Mobility
● Loss of attachment
● Pain
● Gingival recession
● Drifting
Ans is B

BY DR.ABDULRAHMAN ALMUALM
MJDF MCQS WITH ANSWERS

1378Gingivitis and periodontitis are associated with the loss of normal tissue.
Which of the following is the principal cause of the tissue loss
● Bacterial enzymes such as collagenase and hyaluronidase
● Release of lysosomal enzymes and oxygen free radicals by PMNs and
macrophages
● Cytotoxic metabolic by products such as ammonia, hydrogen sulphide and
toxic amides
● Complement activation
● Release of bacterial endotoxins and exotoxins.
Ans D

1379Localised gingival recession is recession of the gingiva confined to one


surface of the tooth. Which of the following is the most important factor leading
to the development of localised gingival recession?
● Habit activity.
● Class 2 division 2 malocclusion with complete overbite
● Excessive toothbrushing
● Bony dehiscence
● Fenestration
Ans is C

138048 year old woman complains of a sore area on the right buccal mucosa
adjacent to a restored tooth. The lesion has a lichenoid appearance and this is
confirmed histopathologically following a biopsy. Which of the following
restorative materials is most frequently associated with lichenoid changes.?
● Gold
● Amalgam
● Porcelain
● composite
● Glass ionomer cement
Ans is B

1381patient attends your surgery complaining of severe pain, swelling and


mobility associated with a lower first molar tooth in which there is a broken
filling. A periapical radiograph indicates that the tooth has not been root filled
and there is loss of apical lamina dura associated with the distal root and at
the bifurcation. The periodontal bone support is good. There is no significant
pocketing. What is the likely diagnosis?
● Chronic periapical periodontitis
● An acute periodontal-endodontic periodontitis
● A chronic periodontal-endodontic periodontitis
● An acute periapical periodontitis
● Para radicular periodontitis
Ans D

138255 year old female presents at your surgery complaining of a sharp pain
of no more than a few minutes duration arising from her lower incisors. The
pain only occurs when she eats or drinks cold or sweet foods and only lasts for

BY DR.ABDULRAHMAN ALMUALM
MJDF MCQS WITH ANSWERS

as long as the stimulus is present. A previous dentist applied a varnish to the


teeth affected which seemed to help. The teeth indicated by the patient appear
healthy but when you blow air onto them the patient experiences the pain.
What is the most likely cause of the patient's pain?
● Acute/reversible pulpitis
● Dentine sensitivity
● Chronic/irreversible pulpitis
● Atypical odontalgia
● Trigeminal neuralgia
Ana is B

1383As a newly qualified dentist you are offered a contract in which


remuneration is on capitation basis. What is the most important and well-
recognised feature of this method of remuneration?
● Tends to encourage under-prescribing
● Tends to encourage high technical quality of work
● Tends to encourage high output of procedures
● Tends to encourage over-prescribing
● Tends to encourage low technical quality of work

Ans is B

1384A 50 year old male patient has a Class III jaw relationship with an anterior
open bite. It is planned to restore his lower right second molar, which has
suffered tooth wear and fracture, with an indirect restoration. This tooth has
approximately 2mm of coronal height. What would be the most suitable
approach to restore this tooth?
● Provide an adhesively retained gold onlay
● Provide a conventional full crown
● Increase the vertical dimension and provide a full crown
● Surgically crown lengthen and provide a gold crown
● Provide an adhesively retained ceramic onlay
Ans is A

1385Gracey curettes are specially designed for subgingival debridement and


are site specific. Which curette is recommended for use on the lingual surface
of a lower second molar?
A.Gracey Curette number 1 and 2.
B.Gracey Currette number 3 and 4
CGracey Currette number 7 and 8.
D. Gracey Currette number 11 and 12.
E. Gracey Currette number 13 and 14.
Ans is C

BY DR.ABDULRAHMAN ALMUALM
MJDF MCQS WITH ANSWERS

1386Lignocaine (2%) is widely used in dental procedures. It is most often used


in combination with epinephrine (1 in 80,000). In which one of the following patients
is the use of epinephrine containing local analgesia potentially hazardous?
● Patient with severe hypertension
● Patient on tricyclic antidepressants
● Patient who is an alcoholic
● Patient on monoamine oxidase inhibitors (MAOls)
● Patient with Grand Mal epilepsy
Ans is A

1387An 80 year old male presents at your surgery complaining of a sharp


stabbing pain of no more than 2-3 minutes duration arising from his upper left
pre-molar region. The pain can be brought on by cold stimuli but also occurs
spontaneously and has been sufficiently severe to wake the patient from sleep.
He has tried taking paracetemol but this has been of no benefit. On
examination the patient has a heavily restored upper left 4, which is vital to
electrical pulp testing and shows no radiographic evidence of caries. Blowing
cold air onto the tooth produces the pain but the pain also occurs spontaneously
when you are examining the patient. There is no evidence of a crack or fracture
in the tooth itself. What is the most likely cause of the patient's pain?
● Acute/reversible pulpitis
● Dentine sensitivity
● Chronic/irreversible pulpitis
● Atypical odontalgia
● Trigeminal neuralgia
Ans C

1388patient complains of a lower incisor which has been mobile for several
months. The radiograph indicates a normal level of bony support although the
periodontal space has widened. The apical bone appears normal. The tooth is
tender to pressure. Which of the following tests and or examinations would be
most likely to provide a diagnosis?
● Masticatory muscle palpation
● Electric pulp test
● Occlusal examination >occlusal trauma
● Ethyl chloride test
● Hot gutta percha application
Ans is C

1389nine year old boy presents with a class I occlusion with no crowding or
overjet with a grossly carious upper left 6 which is not suitable for restoration.
The upper left 7 is very near to eruption. Upper right 6, lower right 6 and lower
left 6 are sound and fissure sealed. The oral hygiene is good. What is the most
appropriate extraction pattern for this patient?
● Extract all four 1st permanent molars
● Extract upper left 6 and lower left 6
● Extract upper left 6 and lower right 6
● Extract upper right 6 and upper left 6
● E. Extract upper left 6 only

BY DR.ABDULRAHMAN ALMUALM
MJDF MCQS WITH ANSWERS

Ans E

1390 new patient attends your practice. At the initial examination, you carry out
a Basic Periodontal Examination (BPE). What does a BPE score of 1 signify?
● Periodontal health
● Probing depths greater than 3.5mm
● Presence of overhanging restorations
● Presence of supragingival calculus
● Presence of bleeding on probing
Ans is E

1391patient presents with a history of pain in the right pre-auricular region.


There is an intermittent click during opening; when the click is not present the
patient can open to a normal range. On examination the masticatory muscles,
including the lateral pterygoid, on the RHS side are tender. What is the most
likely diagnosis?
● Bruxism
● Myofascial pain (or Pain dysfunction syndrome)
● Disc Displacement with Reduction
● Osteoarthrosis
● Disc Displacement without Reduction
Ans is C

1392All the films which have been manually processed by an unsupervised new
trainee dental nurse in your practice are extremely dark. Which of the following
errors could lead to a dark film?
● Insufficient exposure
● Films have been left in the developer for too long a period of time ● Films
have been in the fixer for too long a period of time
● The concentration of the developer is too dilute
● The concentration of the fixer is too dilute
Ans is B
1393your colleague is having problem as every panoramic film that he produce
shows extremely wide anterior teeth which are blurred. The film also invariably
has the images of the condyles cut off from the sides of the film. The most
likely cause of this?
● The patient is incorrectly positioned too far forward relative to the image layer
● The patient is incorrectly positioned with the chin too low
● The patient is incorrectly positioned too far back relative to the image layer
● D. The patient is in a slumped position in the machine
Ans C

OPG fault - patient too far back Wide


, blurred anterior teeth, loss of apices.Ghosting of rami, spread-out turbinates, ears,
and nose in image, condyles off lateral edges of film.

When patient too far back- anterior wide teeth plus lateral side of condyle cut off.

When patient chin slumped down -condyle HEADS are cut off

BY DR.ABDULRAHMAN ALMUALM
MJDF MCQS WITH ANSWERS

1394 When an individual is exposed to radiation a certain amount of radiation


is needed before clinical signs of damage to somatic cells appear. For these
effects to occur a minimum radiation dose has to be exceeded and this is
known as:
● Background radiation dose
● Threshold dose
● Equivalent dose
● Absorbed dose
● Effective dose
Ans B

1395Dentists are encouraged to carry out a thorough soft tissue examination


and be vigilant for any lesions that might possibly be malignant or have
malignant potential. Currently it is not recommended that dentists use a
mucosal staining method for screening the general population for oral cancer.
What is the most likely reason for this advice?
● Unnecessary intervention where there is a false positive test result
● False reassurance where there is a false negative test result
● Low uptake of screening by those with low risk
● Low uptake of screening by those with high risk
● Negative result reinforcing existing bad habits
Ans A

1396A patient presents with a chief complaint of “severe pain in my right ear” which
began when eating, three hours ago. An examination reveals tenderness over the
right preauricular region, maximum interincisal opening of 21mm with deflection to
the right, right lateral excursion of 9mm and left lateral excursion of 2mm. The most
likely diagnosis is

A. left anterior disc displacement with reduction.


B. right anterior disc displacement with reduction.
C. left anterior disc displacement without reduction.
D. right anterior disc displacement without reduction
Ans D

1397What’s most common salivary malignant tumor?


a) Pleomorphic Adenoma
b) Warthin tumor
c) Mucoepidermoid carcinoma
d) Adinocarcinomas
e) Adenoid cystic Carcinoma
Ans C?

1398What is the maximum dose of lignocaine 2% without epinephrine used to


extract a tooth
in 5 years old girl who weight 20 kg ?
a) 1.1 ml
b) 2.2 ml
c) 3.3ml

BY DR.ABDULRAHMAN ALMUALM
MJDF MCQS WITH ANSWERS

d) 4.4 ml
e) 5.5 ml
ANs D

1399The maximum dose of lidocaine with epinephrine is:


a) 500 mg
b) 300 mg
c) 400 mg
d) 150 mg
e) 200 mg
A

1400The maximum dose of lidocaine with epinephrine is:


a) 5 cartidges
b) 7 cartidges
c) 9 cartidges
d) 10 cartidges
e) 3 cartidges
D?

1401The maximum recommended dose of lidocaine with epinephrine for a patient


whose
weight is 30 kg:
a) 210 mg
b) 300 mg
c) 420 mg
d) 140 mg
e) 200 mg

1402The maximum dose of prilocaine is:


a) 500 mg
b) 300 mg
c) 400 mg
d) 150 mg
e) 200 mg

1403The maximum recommended dose of prilocaine for a patient whose weight is


10 kg:
a) 10 mg
b) 60 mg
c) 120 mg
d) 240 mg
e) 200 mg

1404The maximum dose of Bubivicaine is:


a) 500 mg
b) 300 mg
c) 400 mg

BY DR.ABDULRAHMAN ALMUALM
MJDF MCQS WITH ANSWERS

d) 150 mg
e) 90 mg

1405The maximum recommended dose of bupivicaine for a patient whose weight is


10 kg:
a) 13 mg
b) 60 mg
c) 120 mg
d) 240 mg
e) 200 mg

1406A patient with BPE 4 due to poor filling; the best action is to:
a) Refer him to specialist
b) Treat him in the clinic
c) Refer him to a college who has special interests in periodontal treatment
d) Give him antibiotics
e) BPE4 necessitate extraction

140716 years old female patient presented with 4.5 mm open bite, 4 mm reverse
overjet
and impacted upper canine, his IOTN is:
a) 4e
b) 4b
c) 4a
d) 5i
e) 4t

1408The radiograph of choice for a new adult patient with BPE 4* in all sextants and
proximal decay in lower 1st molars is:
a) Full mouth vertical bitewings
b) Panorama with selected periapicals
c) Panorama with full mouth periapicals
d) Panorama with full mouth periapicals and selected bitewings
e) Full mouth periapicals

1409Basal cell carcinoma spreads by


a) Via blood stream
b) Via lymphatic drainage
c) Via seeding
d) Never spread
e) Via sweat glands

1410Kaposi sarcoma is caused by which type of virus:


a) HHV8
b) HHV3
c) HBV
d) HIV
e) HCV

BY DR.ABDULRAHMAN ALMUALM
MJDF MCQS WITH ANSWERS

1411the term symblephron describes unusual sign of the eye and it is mostly
associated with
:
a) Behcet’s syndrome
b) MAGIC syndrome
c) Mucous membrane pemphegoid
d) Marfan syndrome
e) Pemphigus vulgaris

1412Patient presented with flaccid bulla affecting wide area of his back and his
mouth, he
also complaints of eye manifestations:
a) Pemphigus vulgaris
b) Mucous membrane pemphegoid
c) Bullous pemphigoid
d) Cicatricle pemphegoid
e) Behcet syndrome

1413For a patient with ulcer, the most common area in which the ulcer may change
to
malignancy is
a) Upper eye lid
b) Lower eye lid
c) Upper lip
d) Lower lip
e) Nose

1414Which of the following is mostly related to the strength of the the local anaesthia
effect
a) lipid solubility
b) weight of the patient
c) volume of LA that has been given
d) PKa must be less than 7
e) adrenalin content

1415Burkett’s lymphoma is caused by :


a) Epstein bar virus
b) Cytomegalo virus
c) HHV8
d) Paramyxo virus
e) Bacteria tubercular

1416female patient was very angry and has a very painful tooth necessitate
extraction; she
insist on extraction under general anesthesia
a) extract it under general anesthesia

BY DR.ABDULRAHMAN ALMUALM
MJDF MCQS WITH ANSWERS

b) refer to a specialist or hospital


c) refer for 2nd opinion
d) refer to a college who can do extraction under IV midazolam sedation
e) give her strong analgesics and antibiotics

1417Sudden acute unilateral swelling of the parotid gland which is not fluctuant:
a) Sialosis
b) Mumps
c) Acute sialedinitis
d) Pleomorphic adenoma
e) Salivary stone

14189 years old child presented with lymphadenopathy and petechae on the junction
between hard and soft palate. most likely causative virus is:
a) EBV
b) Herpes zoster
c) Paramyxovirus
d) Coxacki A
e) HSV

1419Patient with INR 3 is going to have an operation under general anaesthesia so


which of
the following is contraindication for one day GA.:
a) Body mass index 36
b) Patient blood pressure below 100
c) Myocardial infarction 8 months ago
d) Diabetic patient
e) Patient with ASA 2

1420Eruption date of upper canine:


a) 9-10
b) 11-12
c) 12-13
d) 13-14
e) 8-10

1421Eruption date of lower canine:


a) 9-10
b) 11-12
c) 12-13
d) 13-14
e) 8-10

1422elderly man with ulcer on the lateral surface of the tongue, the patient is heavy
smoker and heavy drinker; clinically he has loose lower partial denture; the 1st action
:
a) reline the denture
b) refer to the oral surgery clinic

BY DR.ABDULRAHMAN ALMUALM
MJDF MCQS WITH ANSWERS

c) ease the denture and review the patient after one week
d) take a biopsy to exclude malignancy
e) reassure him and give antifungal cream

1423Patient who have night blindness :


a) Vit C deficiency
b) Vit A deficiency
c) Vit d deficiency
d) Vit B12 deficiency
e) Folic acid deficiency

1424Treatment of overjet in class II division I on skeletal base, patient is adult with


overjet
more than 9mm
a) Extraction then fixed appliance
b) Functional appliance followed by fixed
c) Orthognathic surgery
d) Wait until puberty to commence fixed applaince therapy
e) Camouflage treatment

1425The most commonly used material in oral sedation in UK :


a) nitrozepam
b) Tiazepam
c) diazepam
d) midazolam
e) flumazenil

1426The escort's most important role for patient receiving IV sedation will be :
a) Drive the patient home at end of the operation
b) Stay 24 hour with him for observation
c) Prevent him from smoking or drinking alcohol
d) Talk to do doctors and take a written cons

1427Patient with severe pain, non-clinical nor radiographical finding; he complains of


sinus
pain and ear deafness; he also has unilateral ulcer on forehead….
a) HSV1
b) HSV5
c) HSV3
d) HSV4
e) HSV5

1428Got a call from a mother of 3 years old child who had trauma and avulsed his
front
tooth, what instruction will you give to the mother?
a) Attend as soon as possible with the tooth
b) Place the tooth in milk and attend as soon as possible
c) Place the tooth in salty water and attend as soon as possible
d) Put the tooth back in its position and attend as soon as possible
e) Come at the end of the day

BY DR.ABDULRAHMAN ALMUALM
MJDF MCQS WITH ANSWERS

14291st sign of periodontitis


a) Loss of attachment
b) Furcation involvement
c) Gingival Recession
d) Deep pocket
e) Bone loss

1428daily alcohol units for males shouldn’t exceed:


a) 1-2
b) 2-3
c) 3-4
d) 4-5

1429tooth identified as grade 2 mobility index so the tooth move:


a) < 1mm in horizontal buccolingual direction
b) > 1mm in horizontal buccolingual direction
c) > 2mm in horizontal buccolingual direction
d) < 1mm in vertical direction
e) > 1 mm in vertical direction

1430Tooth identified as grade 2 furcation involvement:


a) The probe pass more than one 1/3 of the buccolingual width
b) The probe pass less than one 1/3 of the buccolingual width
c) The probe pass through all the buccolingual width radiographically only
d) The probe pass through all the buccolingual width radiographically and clinically
e) The probe pass more than one 1/3 and less than two 1/3 of the buccolingual width

1431The antibiotic of choice for patient with aggressive periodontitis who is allergic
to
penicillin should be:
a) Metronidazole 200 mg for 7 days
b) Doxycycline 100 mg for 21 days
c) Doxycycline 100 mg for 7 days
d) Clindamycin 300 mg for 7 days
e) Doxycycline 100 mg + metronidazole 200 mg for 7 days

1432functions of barrier membrane in sinus lifting is:


a. Prevent osteoblast from leaving graft site
b. Prevent infection in graft site
c. Prevent fibroblast from entering graft site
d. attract fibroblast to enter graft site
e. attract osteoblast to enter graft site

1433Common complication of sjogren syndrome is:

BY DR.ABDULRAHMAN ALMUALM
MJDF MCQS WITH ANSWERS

a. Malignant lymphoma
b. Pleomorphic adenoma
c. Lymphangioma
d. Sialosis
e. mumps

1434Patient with retroclined central incisor, what is the thickness of Z spring to


procline
this central incisor?
a. 0.5 mm
b. 0.6 mm
c. 0.7 mm
d. 0.8 mm
e. 0.9 mm

1435patient with unilateral parotid gland swelling and weak facial nerve, mostly has
a) Bell’s palsy
b) Pleomorphic adenoma
c) Mucoepedirmoid carcinoma
d) Bacterial sialdenitis
e) Mumps

1436For lower premolars, the purpose of inclining the handpiece lingually is to


A. Avoid buccal pulp horn
B. Avoid lingual pulp horn
C. Remove unsupported enamel
D. Conserve lingual dentine
Ans A

1437For an amalgam Restoration of weakened cusp you should


A. Reduce cusp by 2mm on a flat base for more resistance
B. Reduce cusp by 2mm following the outline of the cusp
C. Reduce 2mm for retention form
Ans B

1438Before filling a class V abrasion cavity with GIC you should


A. Clean with pumice, rubber cup, water and weak acid
B. Dry the cavity thoroughly before doing anything
C. Acid itch cavity then dry thoroughly
Ans A?

1439Which of the following statement about the defective margins of amalgam


restoration is true?
A. The larger the breakdown, the greater the chance of decay.

BY DR.ABDULRAHMAN ALMUALM
MJDF MCQS WITH ANSWERS

1440The retention Pin in an amalgam restoration should be placed


A. Parallel to the outer wall
B. Parallel to the long axis of tooth

.1441 The most common cause of failure of the IDN “Inferior Dental Nerve”
block is
A. Injecting too low
B. Injecting too high

1442Which one of the following are not used in water fluoridation


A. SnF2
B. 1.23% APF
C. H2SiF2
D. CaSiF2 .
E. 8% Stannous fluoride

1443The best way to clean cavity before the placement of GIC is


A. H2O2
B. Phosphoric Acid
C. Polyacrylic acid

1444The most mineralised part of dentine is


A. Peritubular dentin

1445A 45 year old patient awoke with swollen face, puffiness around the
eyes, and oedema of the upper lip with redness and dryness. When he went
to bed he had the swelling, pain or dental complaints. Examination shows
several deep silicate restorations in the anterior teeth but examination is
negative for caries, thermal tests, percussion, palpation, pain, and periapical
area of rarefaction. The patient’s temperature is normal. The day before he
had a series of gastrointestinal xrays at the local hospital and was given a
clean bill of health. The condition is
A. Acute periapical abscess
B. Angioneurotic oedema
C. Infectious mononucleosis
D. Acute maxillary sinusitis
E. Acute apical periodontitis

1446Internal resorption is
A. Radiolucency over unaltered canal
B. Usually in a response to trauma
C. Radiopacity over unaltered canal

1447On replantation of an avulsed tooth could see


A. Surface resorption, external resorption
B. Internal resorption
C. Inflammatory resorption
D. Replacement resorption
E. A, C and D
F. All of the above

BY DR.ABDULRAHMAN ALMUALM
MJDF MCQS WITH ANSWERS

1448The percentage of total dentine surface dentinal tubules make in 0.5mm


away from pulp is
A. 20%
B. 50%

1449The junction between primary and secondary dentine is


A. A reversal line
B. Sharp curvature
C. A resting line
D. reduction in the number of tubules

1450What is the correct sequence of events 3


A. Differentiation of odontoblast, elongation of enamel epithelium, dentine
formation then enamel formation.
B. Differentiation of odontoblast, dentine formation then enamel formation,
elongation of enamel epithelium.
C. Elongation of enamel epithelium, differentiation of odontoblast, dentine
formation then enamel formation.

1451What is the sequence from superficial to the deepest in dentine caries


A. Zone of bacterial penetration, demineralisation, sclerosis, reparative dentine
B. Zone of bacterial penetration, reparative dentine, demineralisation, sclerosis.
C. Zone of bacterial penetration, sclerosis, reparative dentine,
demineralisation.

1452The nerve supply of the pulp is composed of which type of nerve fibres
A. Afferent & sympathetic

1453In which direction does the palatal root of the upper first molar usually
curve towards
A. Facial / buccal/
B. Lingual
C. Mesial
D. Distal

1454What is the common appearance of vertical tooth fracture


A. Perio abscess like appearance
B. Displacement of fragments

1455Which of the following would be ONE possible indication for indirect pulp
capping

BY DR.ABDULRAHMAN ALMUALM
MJDF MCQS WITH ANSWERS

A. Where any further excavation of dentine would result in pulp exposure.


B. Removal of caries has exposed the pulp
C. When carious lesion has just penetrated DEJ

1456Following trauma to tooth, the next day there was no response to pulp
tests you should
A. Review again later
B. Start endodontic treatment
C. Extraction of tooth

1457What is the main purpose of performing pulp test on a recently


traumatised tooth
A. Obtain baseline response
B. Obtain accurate indication about pulp vitality

1458What is the main function of EDTA in endodontics


A. Decalcification of dentine
B. Cleaning debris from root canal

1459Which is NOT TRUE in relation to the prescription of 5mg or 10mg of


diazepam for sedation
A. Patient commonly complain of post operative headache
B. An acceptable level of anxiolytic action is obtained when the drug is
given one hour preoperatively
C. There is a profound amnesic action and no side affects
D. Active metabolites can give a level of sedation up to 8 hours post
operatively
E. As Benzodiazepine the action can be reversed with Flumazepil

1460Which of the following is TRUE in regards to high risk patient


A. 0.1ml of blood from Hepatitis B carrier is less infective than 0.1ml of
blood from HIV patient
B. 0.1ml of blood from Hepatitis B carrier is more infective than 0.1ml of
blood from HIV patient
C. Level of virus are similar in the blood and saliva of HIV patient
D. Level of virus in the saliva is not significant for Hepatitis B patient
E. The presence of Hepatitis B core Antigen in the blood means that active
disease is not present

1461Your employer in an attempt to update office sterilization procedures;


what would you recommend as the BEST method to verify that sterilization
has occurred**
A. Use spore test daily
B. Use indicator strips in each load and color change tape on each package
C. Use indicator strips daily and spore test weekly
D. Use color change tape daily and spore test monthly
E. Use color change tape in each load and spore tests weekly

BY DR.ABDULRAHMAN ALMUALM
MJDF MCQS WITH ANSWERS

146265 year old woman arrived for dental therapy. The answered
questionnaire shows that she is suffering from severe cirrhosis.
The problem that can be anticipated in the routine dental therapy is
A. Extreme susceptibility to pain
B. Tendency towards prolonged haemorrhage
C. Recurring oral infection
D. Increased tendency to syncope
E. Difficulty in achieving adequate local anaesthesia

1463Loss of sensation in the lower lip may be produced by


A. Bell’s palsy
B. Traumatic bone cyst
C. Trigeminal neuralgia
D. Fracture in the mandible first molar region
E. Ludwig’s angina

1464Patient received heavy blow to the right body of the mandible sustaining
a fracture there. You should suspect a second fracture is most likely to be
present in
A. Symphysis region
B. Left body of the mandible
C. Left sub-condylar region
D. Right sub-condylar region
E. sub-condylar region

1465Signs and symptoms that commonly suggest cardiac failure in a patient


being assessed for oral surgery are
A. Elevated temperature and nausea
B. Palpitations and malaise
C. Ankle oedema and dyspnoea
D. Erythema and pain
E. Pallor and tremor

1466A cyst at the apex of an upper central incisor measuring 1 cm in


diameter is visualized in radiograph and confirmed by aspiration biopsy; which
method of treatment would you consider**
A. Extraction of the central incisor and retrieving the cyst through the socket
B. Exteriorizing the cyst through the buccal bone and mucosa
C. Making a mucoperiosteal flap and removing the cyst through an opening
made in the alveolar bone, followed by tooth removal.
D. Making a mucoperiosteal flap and removing the cyst through an opening
made in the alveolar bone, followed by endodontic treatment.
E. Routine orthograde endodontic treatment followed by observation.

1467A persistent oroantral fistula for a 12 weeks period following the


extraction of a maxillary first permanent molar is best treated by

BY DR.ABDULRAHMAN ALMUALM
MJDF MCQS WITH ANSWERS

A. Further review and reassurance since it will most probably heal


spontaneously
B. Antibiotic therapy and nasal decongestants
C. Curettage and dressing of the defect
D. Excision of the fistula and surgical closure
E. Maxillary antral wash out and nasal antrostomy.

1468The most significant finding in clinical evaluation of parotid mass may be


accompanying
A. Lympha adenopathy
B. Nodular consistency
C. Facial paralysis
D. Slow progressive enlargement
E. Xerostomia

1469As far as surgical removal of wisdom teeth is concerned which of the


following is true**
A. Prophylactic prescription of antibiotic reduces dramatically the chances of
infection
B. Raising a lingual flap will increases the incidence of neurapraxia but will
reduce the incidence of neurotmesis with respect to the lingual nerve
C. Prophylactic prescription of dexamethasone will dramatically reduces post
operative swelling
D. Inferior dental nerve injury is unlikely since the nerve passes medial to
the wisdom tooth root
E. The use of vasoconstrictors in local anaesthetics will increase the chances
of infection.

1470Endogenous morphine like substance which can control pain is known


as**
A. Bradykinins
B. Peptides
C. Prostaglandins
D. Serotonins
E. Enkephalins

1471Platelets play an important role in haemostasis; which of the following


describes this role?
A. They convert fibrinogen to fibrin 7
B. They agglutinate and plug small, ruptured vessels
C. They initiate fibrinolysis in thrombosis
D. They supply fibrin stabilizing factors
E. They supply proconvertin for thromboplastin activation

1472Suppuration is mainly the result of the combined action of four factors;


which of the following is not one of these factors A. Necrosis
B. Presence of lymphocytes
C. Collection of neutrophils
D. Accumulation of tissue fluid
E. Autolysis by proteolytic enzymes

BY DR.ABDULRAHMAN ALMUALM
MJDF MCQS WITH ANSWERS

1473Which of the following lesions CANNOT BE classified as an intra-


epithelial lesion**
A. Herpes simplex infections
B. Pemphigus vulgaris
C. Herpangina
D. Lichen planus
E. Hand, foot and mouth disease

1474In regards to HIV infection, which of the following is the earliest finding
A. Kaposi sarcoma on the palate
B. Reduced haemoglobin
C. Infection with pneumocystic carinii
D. Reduction in white cells count
E. B cell lymphoma

1475Which of the following is NOT CHARACTERISTIC of trigeminal


neuralgia**
A. The pain usually last for few seconds up to a minute in the early stages
of the disease
B. The pain is usually unilateral
C. Patient characteristically have sites on the skin that when stimulated
precipitate an attack of pain
D. An attack of pain is usually preceded by sweating in the region of the
forehead
E. It is a paroxysmal in nature and may respond to the treatment with
Carbamazepine

1476Benign migratory glossitis or Geographic Tongue, manifests itself in the


oral cavity as

A. Irregularly outlined areas of hyperkeratosis of the dorsal surface of the


tongue
B. Furrows outlined the dorsal surface radiating out from a central groove in
the centre of the tongue
C. Loss (atrophy) of filiform papillae in multiple irregularly outlined areas
D. Irregularly outlined erythematous area of hyper trophic fungiform
E. A fibrinous exudate on the dorsal surface F. Grooves (fissures) radiating
from a central fissure
G. Irregular area in the midline of the tongue

1477Which one of the following is true about oral hairy leukoplakia


A. Associated with HIV virus infection and is commonly seen on the dorsal
of the tongue
B. Associated with HIV virus infection and is commonly seen on the lateral
side of the tongue
C. Usually caused by Candida species

BY DR.ABDULRAHMAN ALMUALM
MJDF MCQS WITH ANSWERS

D. Always associated with trauma to the lateral side of the tongue


E. Always associated with pernicious anaemia

1478Which of the following have a tendency to recur if not treated


A. Giant cell granuloma
B. Lipoma
C. Fibrous epulis
D. Haematoma
E. Pulp polyps

1479Which of the following have a tendency to recur if not treated


A. Giant cell granuloma
B. Lipoma
C. Fibrous epulis
D. Haematoma
E. Pulp polyps

1480Basal cell carcinoma is characterised by


A. Rapid growth and metastasis
B. Local cutaneous invasion
C. Inability to invade bone
D. Poor prognosis
E. Radiation resistance
F. Can not metastasise to the bone

1481Carcinoma of the tongue has a predilection for which of the following


sites**
A. Lateral border anteriorly
B. Anterior dorsal surface
C. Posterior dorsal surface
D. Lateral border posteriorly
E. No preferred location

1482A patient presents complaining of a stomach upset 48 hours after starting


a course of antibiotic for oral infection, this is an example of
A. Type I allergic reaction
B. Nervous disorder
C. Side effect of the drug
D. Type IV hypersensitivity reaction
E. Pyloric stenosis

1483Trichloroacetic acid, a strong acid, has been used by dentists for


chemical cautery of hypertrophic tissue and aphthous ulcers; its mechanism of
action is
A. Thermodynamic action
B. Activation of tissue enzymes
C. Osmotic pressure
D. Protein precipitation PPT
E. Neutralization

BY DR.ABDULRAHMAN ALMUALM
MJDF MCQS WITH ANSWERS

1484Which of the following adverse reaction of oral contraceptives is the most


common and the most serious
A. Hypotension
B. Hepatotoxicity
C. Uterine neoplasia
D. Thromboembolism disorder
E. Decreased resistance to infection

1485A patient who has been taking quantities of aspirin might show increased
post operative bleeding because aspirin inhibits** A. Synthesis of thromboxane
A2 and prevents platelet aggregation
B. Synthesis of prostacyclin and prevents platelet aggregation
C. Synthesis of prostaglandin and prevents production of blood platelets
D. Thrombin and prevents formation of the fibrin network
E. Increase the absorption of vitamin K and prevents synthesis of blood
clotting factors

1486A patient who recently had a calculus removed from the kidney
presented with radiolucent area in the left maxilla with clinical evidence of
swelling. The disease that you would immediately suggest is
A. Diabetes
B. Thyrotoxicosis
C. Hyperparathyroidism
D. Osteoporosis
E. Adrenal insufficiency

1487Typical features of Down’s syndrome (Mongolism) do not include


A. A multiple immunodeficiencies
B. Sever caries but minimal periodontal disease
C. Susceptibility to infections
D. Multiple missing teeth and malocclusion
E. Hepatitis B carriage in institutionalised patients

1488The patient whom you are about to treat, states that he has Von
Willbrand’s disease. Which one of the following preoperative hematological
analysis may reflect this disease
A. Bleeding time and factor VIII level
B. Bleeding time and factor IX level
C. Bleeding time and factor X level
D. Platelet count
E. Thromboplastin generation time

1489A 22 year old woman has acute gingival hypertrophy, spontaneous


bleeding from the gingiva and complains of weakness and anorexia. Her
blood analysis was as follows: HB=12gm, Neutrophils=90%, Monocytes=1%,

BY DR.ABDULRAHMAN ALMUALM
MJDF MCQS WITH ANSWERS

Platelets=250000, WBC=100000, Lymphocytes=9%, Eosinophils=0% The most


likely diagnosis is
A. Myelogenous leukaemia
B. Infectious mononucleosis /glandular fever/
C. Thrombocytopenic purpura
D. Gingivitis of local aetiological origin
E. Pernicious anaemia /Vitamin B12 deficiency/

1490The tonsillar lymph node is situated at the level of


A. Angle of the mandible
B. C6 vertebrae
C. Jugulodigastric crossing
D. Clavicle
E. Jugulo-omohyoid crossing

1491Exposure of the patient to ionising radiation when taking a radiograph is


NOT REDUCED by
A. The use of fast film
B. The addition of filtration
C. Collimation of the beam
D. The use of an open and lead lined cone
E. Decreasing the kilovoltage KvP

1492X-ray films have an emulsion on one or both side of a support material.


The emulsion contains particles of
A. Silver nitrate crystal
B. Metallic silver in gelatine
C. Silver bromide in gelatine
D. Silver nitrate in gelatine
E. Potassium bromide in gelatine

1493The inverse Square Law is concerned with intensity of radiation using


type D film of 200mm target to film distance, the exposure time was 0.25s.
What would be the exposure for the same situation with 400mm target to film
distance
A. 0.5s
B. 1.0s
C. 2.0s
D. 0.25s
E. 0.125s

1494You wish to purchase a dental X ray machine and have the choice
between 60kVp and 70kVp machines. With single change from 60kVp to
70kVp what would the approximate affects on exposure time
A. No effect
B. Half the time
C. Double
D. Quarter
E. Triple the time

BY DR.ABDULRAHMAN ALMUALM
MJDF MCQS WITH ANSWERS

1495You wish to purchase a dental X ray machine and have the choice
between 60kVp and 70kVp machines. With single change from 60kVp to
70kVp what would the approximate affects on exposure time
A. No effect
B. Half the time
C. Double
D. Quarter
E. Triple the time

1496When no radiation shield is available, the operator should stand out of


the primary x ray beam and a distance from the patient’s head of at LEAST
A. 0.5 metres
B. 1 metre
C. 1.5 metres
D. 2 metres
E. 3 metres

1497The obturating material of choice for primary teeth following complete


pulpectomy is

A. Zn phosphate cement and formcresol combination paste


B. Quick setting hydroxide cement
C. Zinc oxide and eugenol cement
D. Gutta-percha
E. Polycarboxylate cement

14988 years old child who has sustained a fracture of maxillary permanent
central incisor in which 2mm of the pulp is exposed; presents for treatment
three hours after injury. Which of the following should be considered
A. Remove the surface 1-2 mm of pulp tissue and place calcium hydroxide
B. Place calcium hydroxide directly on the exposed pulp
C. Pulpotomy using formocresol
D. Pulpectomy and immediate root filling
E. Pulpectomy and apexification

1499When primary molars are prepared for stainless steel crowns should the
depth for reduction of the proximal surface be similar to the depth of the
buccal and lingual surfaces
A. Yes; reduction of all wall is similar for best retention
B. No, proximal reduction is greater to allow the crown to pass the contact
area
C. No, the buccal surfaces has the greatest reduction to remove the cervical
bulge
D. Yes, all undercuts are uniformly removed so that the steel crown can be
seated
E. No, because of lateral constriction, the lingual surface needs greatest
reduction

BY DR.ABDULRAHMAN ALMUALM
MJDF MCQS WITH ANSWERS

1500Which primary teeth are LEAST affected with the nursing bottle syndrome
A. Maxillary molars
B. Maxillary and mandibular canines
C. Mandibular incisors
D. Maxillary incisors
E. Mandibular molars

1501Which of the following anomalies occurs during the initiation and


proliferation stages of tooth development
A. Amelogenesis imperfecta
B. Dentinogenesis imperfecta
C. Enamel hypoplasia
D. Oligodontia
E. Ankylosis

1502Which is the right sequence of the histological stages of tooth


development
A. Initiation, proliferation, histodifferentiation, morphodifferentiation,
mineralization
B. Proliferation, initiation, histodifferentiation, morphodifferentiation,
mineralization
C. Proliferation, morphodifferentiation, histodifferentiation, mineralization
D. Initiation, proliferation, morphodifferentiation, histodifferentiation,
mineralization

1503A health 6 year old child presents with carious maxillary second primary
molar with a necrotic pulp. Which treatment would be preferred
A. Extraction
B. Indirect pulp treatment
C. Pulpotomy
D. Pulpectomy
E. Antibiotic coverage

1504To produce a stable correction of an upper labial segment in lingual


crossbite; it is essential to
A. Use fixed appliances
B. Have adequate overbite
C. Treat during growth
D. Use posterior capping
E. Increase vertical dimension

BY DR.ABDULRAHMAN ALMUALM
MJDF MCQS WITH ANSWERS

1505The lamina dura seen on periapical radiograph as


A. Usual radiolucency between tooth root and surrounding bone as a thin
white line.
B. Cribriform plate of bone making the tooth socket
C. Dense crestal bone consistent with a healthy periodontal status
D. Pattern of radiopaque lines in supporting alveolar bone

1506Which of the following organisms are pathognomonic of acute necrotic


ulcerative gingivitis
A. Spirochaetes and fusobacterium SP
B. Spirochaetes and eikenella corrodes
C. Polymorphs and lymphocytes
D. Actinobacillus actinomycetes comitans oral capnocytophaga
E. Porphyromonas gingivalis and prevotella intermedia

1507In testing for mobility, which of the following statement is true


A. Heavy pressure must sometimes be used to test mobility
B. Only lateral mobility is significant in diagnosis and treatment of chronic
inflammatory periodontal disease
C. Hyper mobility indicates that the tooth supporting structure have been
weakened
D. During the periodontal examination each tooth should be tested individually
for hyper mobility
E. Reliance on radiograph is essential

1508The treatment of Localised Juvenile Periodontitis is frequently


supplemented with tetracycline because flora involved is predominant
A. Aerobic
B. Strictly anaerobic
C. Facultative or microaerophilic
D. Resistant to other antibiotic

1509The most accurate way to evaluate the effectiveness of root planning is


by
A. Inspect the root surface with an instrument for root smoothness
B. Use air for visual inspection
C. Activate a curette against root surface and listen for a high pitched sound
which indicates a smooth, hard surface.
D. Evaluate the soft tissue at the end of the appointment for a decrease
oedema and bleeding
E. Evaluate the soft tissues 10 to 14 days later

1510Probe pressure at the sulcus of pocket should not be more than enough
to
A. Feel the top of the crestal bone
B. Balance the pressure between fulcrum and grasp
C. Define the location of the apical and the calculus deposit
D. Feel the coronal end of the attached tissues

BY DR.ABDULRAHMAN ALMUALM
MJDF MCQS WITH ANSWERS

E. Limit the lateral pressure

1511Tetracycline hydrochloride conditioning of root surface in periodontal


surgery is to
A. Sterilise the root surface
B. May enhance binding of fibronectin and fibroblast
C. Aids in re-mineralising the root surface
D. Assist the biding of lamina dura
E. Prevents post operative infections

1512Of all the factors that increase the resistance of teeth to dental caries
THE MOST EFFECTIVE is
A. The general nutrition of a child during the period of tooth formation
B. The intake of fluoride during the period of enamel mineralization and
maturation
C. Periodic topical fluoride application by dental health care following tooth
eruption
D. Sufficient intake of calcium and Vitamin D during the period of enamel
mineralization and maturation

1513When the enamel of the tooth is exposed to preparation containing high


concentrations of fluoride; the major reaction is
A. Sodium fluoride
B. Calcium fluoride
C. Stannous fluoride
D. Fluorapatite

1514 76. When describing a removable partial denture, the minor connector refers to**

A. Rigid components anterior to the premolar teeth


B. Flexible components, in contrast to rigid major connectors
C. Smaller connectors which connect denture components to the major connector
D. The components of the denture base which provides reciprocation

BY DR.ABDULRAHMAN ALMUALM
MJDF MCQS WITH ANSWERS

1515. The means by which one part of a partial denture framework opposes the action
of the retainer in faction is**

A. Tripoding
B. Reciprocation
C. Stress breaking
D. Indirect retention
1516. In removable partial denture, the principle of an indirect retainer is that

A. Stabilise against lateral movement


B. Prevent settling of major connectors
C. Restrict tissue movement at the distal extension base of the partial denture
D. Minimise movement of the base away from the supporting tissue

1517. Distortion or change in shape of a cast partial denture clasp during its clinical
use probably indicates that the

A. Ductility was too low


B. Hardness was too great
C. Ultimate tensile strength was too low
D. Tension temperature was too high

E. Elastic limit was exceeded

1518. Which of the following is true regarding preparation of custom tray for
elastomeric impression

A. Adhesive is preferred over perforation


B. Perforation provides adequate retention
C. Adhesive is applied immediately before procedure
D. Perforations are not made in the area over the prepared tooth

BY DR.ABDULRAHMAN ALMUALM
MJDF MCQS WITH ANSWERS

1519. When a removable partial denture is terminally seated ; the retentive clasps tips
should

A. Apply retentive force into the body of the teeth


B. Exert no force
C. Be invisible
D. Resist torque through the long axis of the teeth

1520. Why do you construct a lower removable partial denture with lingual bar

A. It is used when the space between raised floor, mouth and gingival margin is
minimal
B. Plaque accumulation is less than lingual plate
C. Should be make thicker when short

1521. The Fovea Palatinae are

A. Foramina covering the lesser palatine nerves and vessels


B. Morphologically related to the formation of the premaxilla
C. Located on either sides of the midline close to the junction of the hard and soft
palate
D. Closely related to the rugae of the palate

1522. Which of following restoration material its strength is not effected by pins

A. Amalgam
B. Composite resin

1523.Which one of following statement about Overdenture is not correct

BY DR.ABDULRAHMAN ALMUALM
MJDF MCQS WITH ANSWERS

A. Greater occlusal loads can be applied by the patient


B. Retention and stability are generally better than with conventional complete denture
C. Alveolar bone resorption is reduced
D. The retained roots are covered by the denture thus protecting them from caries and
periodontal diseases

1524Which of the following is a major disadvantage to immediate complete denture


therapy

A. Trauma to extraction site


B. Increased the potential of infection
C. Impossibility for anterior try in
D. Excessive resorption of residual ridge

1525.. Brown skin pigmentation does not occur in

A. Hyperparathyroidism
B. Von Willebrand’s syndrome

1526.Which statement BEST prescribe plaque

A. It is a soft film composed mainly of food debris and can not be rinsed off teeth
B. It is a soft film composed mainly of food debris and can be rinsed off teeth
C. It is a soft film composed mainly of none calcified bacteria and can not be rinsed
off the teeth
D. It is a soft film composed mainly of dextran and can not be rinsed off the teeth
E. It is a soft film composed mainly of dextran and can be rinsed off teeth.

1527 The gingivae of child is diagnosed on the basis of all of these except of

BY DR.ABDULRAHMAN ALMUALM
MJDF MCQS WITH ANSWERS

A. Contour of gingival papilla


B. Sulcus depth
C. Contour of Nasmyth membrane
D. Tight filling of gingival collar

1528.Which one of the following statement is correct

A. The remnants of Ameloblast contribute to the primary enamel cuticle


B. the last secretion of the odontoblast is cementum X
C. The last secretion of the ameloblast is the acquired of enamel cuticle
D. The remnants of odontoblast form the primary enamel cuticle

1529.In regards to the glass of quartz particles of filling restorative resin; the microfill
resins tend to have

A. A higher coefficient of thermal expansion and a higher crashing strength


B. A higher coefficient of thermal expansion and a lower crashing strength

C. A lower coefficient of thermal expansion and a higher crashing strength


D. A lower coefficient of thermal expansion and a lower crashing strength

1530. Mercury is dangerous when it turns into vapour form because of

A. It is accumulative and cause liver poison


B. It is accumulative and cause kidney poison
C. It induces neoplasia in the liver
D. It is accumulative and cause brain poison
E. It induces neoplasia in the brain

1531. The elastic limit may be defined as the **

BY DR.ABDULRAHMAN ALMUALM
MJDF MCQS WITH ANSWERS

A. The maximum stress under tension that can be induced without failure
B. The maximum elongation under tension that can be measured before failure
C. The minimum stress required to induce permanent deformation of a structure
D. Minimum stress in structure
E. Maximum strain that can be measured.

1532. Rank the following impressions materials according to their flexibility

A. Alginate> Polysulphide> Silicone> Zinc Oxide Eugenol


B. Silicone> Alginate> Polysulphide> Zinc Oxide Eugenol
C. Alginate> Polysulphide> Zinc Oxide Eugenol>Silicone
D. Alginate> Silicone> Polysulfide> Zinc Oxide Eugenol
E. Alginate> Zinc Oxide Eugenol> Silicone> Polysulphide

1533.Denture resin are usually available as powder and liquid that are mixed to form
a plastic dough; the powder is referred to as**

A. Initiator
B. Polymer
C. Inhibitor
D. Monomer
E. Dimer

1534. Which one of the following is the major disadvantage of stone dies used for
crown fabrication

A. They lack accurate reproduction of surface details


B. Their overall dimensions are slightly smaller than the original impression
C. The strength of the stone

D. The hazard of aspiration of toxic materials during trimming of the dies.

BY DR.ABDULRAHMAN ALMUALM
MJDF MCQS WITH ANSWERS

1535.Glass Ionomer Cement sets because of**

A. Acid-Base reaction
B. Addition polymerisation reaction
C. Growth of glass crystals
D. Slip plane locking
E. Solvent evaporation

1536.The articular surface of the normal temporomandibular joint are lined with

A. A specially adapted, highly fibrous tissue


B. Hyaline cartilage
C. Chondroitin-6-phosphate
D. Highly vesiculated tissues

1537.When all other removable partial denture consideration remains unchanged;


clasps constructed of which material can be engage the deepest under cut

A. Chrome cobalt casts


B. Nickel chrome casts
C. Wrought stainless steel
D. Wrought gold

1538.Which one of the following types of pain is most likely to be associated with
cranio mandibular disorders

A. Exacerbated pain by hot or cold food


B. Keeps patient awake at night
C. Associated with muscle tenderness
D. Associated with trigger spots related to the trigeminal nerve

BY DR.ABDULRAHMAN ALMUALM
MJDF MCQS WITH ANSWERS

1539.The incisal guidance on the articulator is the**

A. Mechanical equivalent of horizontal and vertical overlap of upper and lower incisors
B. Mechanical equivalent at the compensating curve
C. Same as condylar guidance
D. Estimated by the equation: Incisal guidance = 1/8 of condylar guidance

1540.When immature permanent molars that have been treated with Ledermix pulp
capping, the most probable pathology is

A. Chronic inflammation of the pulp

B. Necrosis of the pulp

1541.Child with rampant caries taking medicine with high quantity of sugar; the best
way to help preventing caries is

A. Change sugar to sorbitol sweetener


B. Report the patient is having expectorant
C. Give him the syrup during sleep time
D. Give him inverted sugar

1542. How many ppm “ Part Per Million” of fluoride are present in water supply in case
of temperate climate**

A. 1 ppm
B. 2 ppm
C. 8 ppm

BY DR.ABDULRAHMAN ALMUALM
MJDF MCQS WITH ANSWERS

D. 1.2 ppm

1543.. The difference between deciduous and permanent teeth are

A. Deciduous teeth have a higher pulp horns and larger pulp chambers
B. Deciduous teeth have flatter contact areas
C. Deciduous teeth have thinner enamel surface
D. All of the above

1544. The most resistant filling materials to fill class IV cavities are

A. Resins with silicone dioxide (SiO2)


B. Resins with glass or quartz
C. Silico-phosphate
D. Silicates

1545. With dentin bonding agent, you apply

A. First acid etching to dentine and then bonding agent


B. Bonding agent directly to dentine
C. Chelating agent (EDTA) and bonding agent

1546.What is the best way to apply aspiration before injection

A. Short, sharp pressure backwards


B. Pressure for 2 to 3 seconds
C. Long pressure
D. Turning the needle 90° between two aspirations

1547. The method you will use to fill root canal of maxillary lateral incisor is

BY DR.ABDULRAHMAN ALMUALM
MJDF MCQS WITH ANSWERS

A. One major Gutta Percha cone

B. Laterally condensed
C. Laterally above condensed

1548.What controls the occlusion**

A. Teeth
B. Receptors in periodontal membrane
C. Proprioceptors
D. Neuromuscular receptors
E. TMJ
F. All of the above

1549. How would you extract 35

A. Rotation
B. Lingually
C. Labially

1550.Why the method of extracting lower 8’s by directing the extraction lingually is
used**

A. Because of the roots direction


B. Thinner bone
C. Lingual deviation

1551.What the maximum dose of 2% lignocaine without vasoconstrictors**

A. 5 ml

BY DR.ABDULRAHMAN ALMUALM
MJDF MCQS WITH ANSWERS

B. 10 ml
C. 50 ml
D. 100 ml

1552. Where do Maryland bridges lose retention often

A. Resin-metal
B. Resin enamel
C. Resin layer
Ans A
1553What is the function of gypsum-binder in the investment**

A. Setting and hydroscopic


B. Strength and rigidity
Ans B

1554. Where is the retentive position on tooth according to the survey line

A. Below the height of contour


B. Next to gingival margins
Ans A

1558. In regards to distal free end saddle; what is TRUE

A. Will require relining more often than a denture supported with teeth

1559.What are the most common errors when constructing partial denture
A. Improper survey
B. Bad positioning of the occlusal rests

BY DR.ABDULRAHMAN ALMUALM
MJDF MCQS WITH ANSWERS

C. Incorrect design
Ans B

1560.Which periodontal pockets are evident on periapical x rays

A. Buccal pockets
B. Lingual pockets
C. Mesial pockets
D. Distal pockets
E. Sinuous
Ans D (also C is correct (

1561.. What factor do you consider the most important when storing the occlusal part
of a tooth

A. Occlusal anatomy
B. Function
Ans A
1562. All dental plaque**

A. Produce acid
B. Produce caries
C. Produce chelation
D. Not necessarily produce acid
Ans A
1563. Treatment of gangrenous tooth

A. Pulp capping
B. Root canal therapy
C. Pulpotomy
Ans B

BY DR.ABDULRAHMAN ALMUALM
MJDF MCQS WITH ANSWERS

1564.Which material is not compatible with composite resin

A. Zinc Oxide and eugenol ZOE


B. Ca(OH)2
C. Carboxylate
D. Zinc phosphate cement

1565. Tooth under occlusal trauma shows

A. Bone resorption

B. Necrosis of the pulp


C. Hypercementosis
D. Triangulation
E. All of the above

1566.Which drug is specific for Trigeminal Neuralgia

A. Diazepam
B. Carbamazepine (Tegretol)
C. Ergotamine
D. Phenytoin

1567.Which Nerve is anesthetised in anterior border of ramus and 1 cm above


occlusal plane of lower posterior teeth

A. Lingual nerve
B. Long buccal nerve

BY DR.ABDULRAHMAN ALMUALM
MJDF MCQS WITH ANSWERS

1568.in an X ray; the mesio buccal root of upper first molars is elongated which is the
result of

A. Mesio angular horizontal


B. Too big vertical angulation
C. Too small vertical angulation
D. High angulation

1569 Which of the following is false in regards to Cleft-Palate

A. May be submucous
B. More common in males than females
C. Predispose to speech defects, orthodontics problem and hearing loss
D. Patients are more likely to have cardiovascular defect than the general population.

1570. Which of the following statement is correct for a periodontal disease**

A. The finger pressure is enough for mobility diagnosis


B. A communicable disease
C. X ray after intra alveolar surgery is sufficient for diagnosis healing
D. Systemic diseases have no effects on it
E. ZoE paste will accelerate healing

1571. The major cause of jacket crown breakage is

A. Inclusion of platinum foil


B. Use of weak cementum
C. Voids of porcelain

D. Porcelain is thinner than 1mm

BY DR.ABDULRAHMAN ALMUALM
MJDF MCQS WITH ANSWERS

1572.. Pontic replaces upper first molars in a bridge should be

A. Slightly compress soft tissues


B. Be clear of soft tissues
C. Just in contact with soft tissues

1573.Labially displaced anterior tooth is restored with a gold core porcelain jacket
crown so that it is in line with the arch; the crown will appears

A. Short
B. Long
C. Narrow
D. Wide

1574. Which is NOT characteristics of canal filing materials “obturation material”

A. Tacky adhesive to walls


B. Radio opaque
C. Not irritating
D. Quick in setting

1575 The best location of pin in class II inlay is

A. Where is the biggest thickness


B. Mesial and distal angle
C. Contact area

1576.Class V composite resin restorations can be polished

A. 24 hours after application


B. Immediately after application

BY DR.ABDULRAHMAN ALMUALM
MJDF MCQS WITH ANSWERS

C. 3 to 4 days
D. 3 to 4 weeks
E. Not at all

1577.Caries which is close to the pulp chamber; on x rays you find dent in dent; the
right treatment is

A. Zinc oxide eugenol cement and amalgam


B. Pulpectomy
C. Pulpotomy
D. Calcium hydroxide on pulp and amalgam

1578.Dental plaque produces

A. Chelation
B. Dental caries
C. Acids

1579.The main advantage of amalgam with high content of Cu is

A. Better marginal sealing


B. Less corrosion
C. Better tensile strength
D. Higher and immediate compressive strength

1580. The major disadvantage of self-threaded pin is

A. Friction locked
B. Too expensive
C. Not all sizes available
D. May cause tooth cracking

BY DR.ABDULRAHMAN ALMUALM
MJDF MCQS WITH ANSWERS

1581 In which class of cavities do composite restorations show most durability

A. I
B. II
C. IV
D. III
E. V

1582. How much space do you need to cap a weakened cusp with amalgam

A. 1mm
B. 1.5mm
C. 2mm
D. 2.5mm

1583. Upper premolar with MO cavity; what is important about the application of the
matrix band: “the question has shown too as
….What is complicated by”

A. The mesial concavity of the root surface


B. Small lingual pulp
C. High buccal pulp horn
D. High lingual pulp horn
E. Concavity of distal root surface

1584. Etching techniques are used always to

A. minimise the leakage of restorations


B. for aesthetic considerations

BY DR.ABDULRAHMAN ALMUALM
MJDF MCQS WITH ANSWERS

1585. Sjogren syndrome is characterised by

A. Dryness of the mouth


B. Dryness of the eyes
C. Rheumatoid arthritis
D. All of the above

1586.Long use of Tetracycline is characterised by**

A. Agranulocytosis
B. Candida Albicans

1587. The most common characteristic symptom of malignant tumours occurring in


lower jaw is

A. Pain
B. Bleeding
C. Paraesthesia

1588. Why Class IV gold can not be used in cavity as a filling material

A. Can not be polished “burnished”


B. The corrosive properties

1589.The type of gold that used for dental bridges is

A. Hard 18%
B. Type IV 75%

1590.In regards to Partial dentures, how do you establish reliable vertical dimension

BY DR.ABDULRAHMAN ALMUALM
MJDF MCQS WITH ANSWERS

A. Wax if the remaining teeth occlude

1591In regards to indirect compare to direct wax technique**

A. Low temperature solidifying point


B. Hard in room temperature
C. Higher flow in room temperature

1592.. If amalgam gets contaminated with moisture, the most uncommon result is**

A. Blister formation
B. Post operative pain
C. Secondary caries
D. Lower compressive strength

1593.The effects of tooth removal in healthy individuals can show as

A. Loss of contacts
B. Slight tilting
C. Pocket formation
D. TMJ problem
E. All of the above

1594. Which is not a malignant lesion

A. Leukoplakia
B. Erythema migrans /Geographic tongue/

1595.Anaesthesia 1 mm above last lower molars will anesthetize

BY DR.ABDULRAHMAN ALMUALM
MJDF MCQS WITH ANSWERS

A. Lingual Nerve
B. Long buccal nerve

1596.Posterior superior alveolar nerve supplies**

A. 8, 7 and 6 except the mesio buccal root of 6


B. 8, 7 and 6

1597. Patient complains of itching and vesicles on the upper labium (Vermillion region)
every year, your diagnosis would be

A. Herpes simplex
B. Recurrent ulceration aphthae
C. Impetigo

1598. What is the typical feature of Lichen planus**

A. Smooth rete pegs


B. Band of lymphocytes inflammation and hyper parakeratosis
C. Immunofluorescence of liquefied layer

1599.Denture stomatitis is treated with

A. Amphotencin
B. Tetracycline lozenges
C. Mycostatin

1600.Paget’s disease shows in the early stages in jaws

A. Cotton wool

BY DR.ABDULRAHMAN ALMUALM
MJDF MCQS WITH ANSWERS

B. Ground glass
C. Orange peel
D. Beaten copped

1601 The most serious complications which may occur from abscess of max canine
is

A. Cellulitis
B. Cavernous sinus thrombosis
C. Lacrimal duct stenosis
D. Damage to infra orbital nerves

1602.Granulomas, cysts and chronic periapical abscesses may mostly be


differentiated by

A. Radiographs
B. Electric pulp test
C. Biopsy
D. Thermal

1603.The most prominent feature of acute apical periodontitis is

A. Tenderness of tooth to pressure


B. Extra oral swelling
C. Intermittent pain

1604. Marsupialisation is a technique used in the treatment of

A. Pericoronitis
B. Cysts
C. Abscesses

BY DR.ABDULRAHMAN ALMUALM
MJDF MCQS WITH ANSWERS

1605. Diagnosis of oral candidiasis (candidosis) is BEST confirmed by

A. Microscopic examination of smears


B. Biopsy
C. Blood count
D. Serological exam

1606.Which antibiotic administered in childhood may result in tooth discolouration

A. Penicillin
B. Tetracycline
C. Streptomycin

1607. Ameloblastoma occurs MOST frequently

A. Near the angle of the mandible


B. In the maxilla
C. At the mandibular symphysis

1608. A patient with long standing rheumatoid arthritis and a history of steroid therapy,
until a week ago, he presents for multiple extractions. The dentist should consult the
patient’s physician because

A. Patient is more susceptible to infection


B. Patient may have a suppressed adrenal cortex
C. Patient will need haematological evaluation

1609. A patient whose hands fell warm and moist is MOST likely to be suffering from**

BY DR.ABDULRAHMAN ALMUALM
MJDF MCQS WITH ANSWERS

A. Anxiety
B. Congestive cardiac failure
C. Thyrotoxicosis

1610 An adult patient with a history of bacterial endocarditis requires prophylactic


administration of antibiotic prior to removal of teeth. indicate the pre-operative
regimen**

A. Amoxicillin 2 gram an hour before operation orally


B. Penicillin 250 mg orally six hours before operation
C. Tetracycline 250-500 mg orally 2 hours before treatment

1611. A 12 year old girl complains of sore mouth, she has painful cervical
lymphadenitis and a temperature of 39°c, oral examination shows numerous yellow
grey lesions. What is the MOST LIKELY diagnosis

A. Measles
B. Erythema multiform
C. Herpetic gingivostomatitis
D. Stevens-Johnson syndrome

1612.The causative micro organism for Herpetic gingivostomatitis is

A. Herpes simplex bacteria


B. Herpes simplex virus
C. Herpes zoster virus
D. Borrelia vincentii

1613. To reduce the side effects risk of local anaesthetic injections; you should follow
all of the following EXCEPT

A. Aspirate before injection

BY DR.ABDULRAHMAN ALMUALM
MJDF MCQS WITH ANSWERS

B. Use the smallest effective volume


C. Use the weakest efficient percentage strength
D. Inject rapidly

1614. The most potent viricidal properties: “another format of the same answer:
Indicate which of the following has viricidal properties”

A. Sodium hypochlorite
B. Chlorhexidine
C. Glutaraldehyde
D. Alcohol 70%
E. Quaternary ammonium

1615. Antibiotics should be used routinely to prevent infection arising from oral surgery
in patients suffering from all the following EXCEPT

A. Agranulocytosis
B. Sever uncontrolled diabetes
C. Aplastic anaemia
D. Mumps
E. Leukaemia

1616. At what rate is closed chest cardiac compression should be in an adult**

A. 12 times a minute
B. 24 times a minute
C. 50 times a minute
D. 80 times a minute

1617. Nitrous Oxide (N2O) is not used alone as a general anaesthetic agent because
of**

BY DR.ABDULRAHMAN ALMUALM
MJDF MCQS WITH ANSWERS

A. Difficulties in maintaining an adequate O2 concentration


B. Adverse affects on liver
C. Poor analgesics affects

1618. How can a periodontal pocket be recognised**

A. X-Ray
B. Periodontal probe / Calibrated probe/
C. Periodontal marker
D. Bitewing radiograph

E. Sharp explorer
F. Study cast

1619. The final material you use for endodontically treated deciduous molars is**

A. Amalgam
B. GIC
C. Composite resin
D. Wrought base metal crown

1620. Which type of cells does an abscess contain

A. Mast cells
B. Polymorphonuclear leukocytes
C. Eosinophils
D. Epithelial cells

1621. The presence of sulphur granules is diagnostic of**

BY DR.ABDULRAHMAN ALMUALM
MJDF MCQS WITH ANSWERS

A. Actinomycosis
B. Candidosis
C. Viral infection
D. Keratocyte

1622. Immediate aim of dry socket treatment is to**

A. Avoid Osteomyelitis
B. Control pain

1623. Which is the LEAST likely to cause Xerostomia

A. Sjogren’s syndrome
B. Emotional reaction
C. Antidepressants drugs
D. Submandibular sialolith

1624. Intact vesicles are MOST likely to be seen in**

A. Herpes simplex infection


B. Oral lichenoid reaction
C. Aphthous ulceration
D. Pemphigus vulgaris
E. Cicatricial pemphigoid

1625. Painful salivary gland are MOST likely to be indicate to**

A. Mucocele
B. Mumps

BY DR.ABDULRAHMAN ALMUALM
MJDF MCQS WITH ANSWERS

C. Sjogren’s syndrome

1626. A patient with an acetone odour would be suspected suffering from

A. Heart disease
B. Liver damage
C. Diabetes

1627. Chronic inflammatory periodontal disease originates in

A. The marginal gingiva


B. The crystal alveolar bone
C. Cervical cementum

1628. Which is the most important local factor in the aetiology of periodontal disease

A. Occlusal trauma
B. Calculus
C. Brushing habits
D. Coarse food

1629. Which of the following does state BEST the morphology of periodontal ligament
fibres

A. Elastic
B. Striated
C. Non striated
D. Levity
E. Wavy

1630. Which of the following is LEAST to cause toxicity from local anaesthetic injection

BY DR.ABDULRAHMAN ALMUALM
MJDF MCQS WITH ANSWERS

A. Injecting in supine position


B. Injecting in vascular area
C. Injecting without a vasoconstrictor
D. Intravenous injections

1631. If a child’s teeth do not form; this would MOSTLY affects the growth of**

A. Alveolar bone
B. Whole face
C. Mandible
D. Maxilla

1632. MOST common consequence arising from premature extraction of deciduous


molar is

A. Loss of arch length


B. Loss of speech sound
C. Loss of facial contour

1633. After the age of 6 years, the greatest increase in the size of the mandible occurs

A. At the symphysis
B. Between canines
C. Distal to the first molar

1634. Which is present in Angel’s Class II division 2 malocclusion

A. Open bite
B. Retrusion of maxillary central incisors
C. Reduced Overjet

BY DR.ABDULRAHMAN ALMUALM
MJDF MCQS WITH ANSWERS

D. Increased overbite

1635. When injecting without vasoconstrictor, the maximum safe dose of 2%


lignocaine solution for 70Kg adult is

A. 2.2ml
B. 22ml

1636. Several application has been suggested to increase the effectiveness of


prophylactic application of topical fluoride which include all EXCEPT

A. Increase Fluoride ions in solution “increase concentration”


B. Increase PH of fluoride
C. Increase exposure time to topical fluoride
D. Pre-treat enamel with 0.5% phosphoric acid
E. Use NH4F instead of NaF

1637. Which of the following ahs the highest sucrose content

A. Ice cream
B. Canned juice
C. Cough syrups
D. Breakfast cereal
E. Sweet potato

1638. The amount of fluoride required to reduce caries according to age and level of
fluoride in drinking water. Which of the following figures is incorrect**

A. 1 year old child requires no fluoride when the fluoride in drinking water is 0.3PPM
B. 3 years old child requires no fluoride when the fluoride in drinking water is 0.7PPM
C. 6 years old child requires 1mg of fluoride when drinking water containing 0.5mg

BY DR.ABDULRAHMAN ALMUALM
MJDF MCQS WITH ANSWERS

1639. The major etiological factor responsible for Class II division 2 malocclusion in
Angel’s classification is**

A. Thumb sucking
B. Growth discrepancy
C. Tongue thrust habit
D. Tooth to jaw size discrepancy
E. Skeletal cause (discrepancy)

1640. Ankylotic primary second molar in the mandible is not always a good space
maintainer because of

A. Mesial inclination of the 1st permanent molar


B. It does not keep up with the rest of occlusion

1641. Preschool child has an intruded upper incisor; what would your treatment be**

A. X-ray
B. Put it back in place and splint
C. Control bleeding and check after a month
D. Make the patient comfortable without disturbing the tooth.

1642. An upper deciduous molar has a caries exposure and on X ray the
corresponding 2nd permanent premolar is absent. What treatment would you do to
the deciduous tooth

A. Pulpotomy
B. Endodontic treatment
C. Pulp capping

BY DR.ABDULRAHMAN ALMUALM
MJDF MCQS WITH ANSWERS

1643. Where is the MOST probable place of bone resorption after a deciduous molar
has a pulpal gangrene

A. Interradicular septum
B. The periapical area

1644. How many pulp horns are presented in a typical mandibular deciduous second
molar

A. 2
B. 3

C. 4
D. 5

1645. All of the following are keratinised EXCEPT of

A. Crevicular epithelium
B. Palatal epithelium
C. Alveolar mucosa
D. Free gingiva
E. Attached gingiva

1646. The MOST cause of gingiva; irritation is

A. Calculus
B. Plaque
C. Caries
D. Restorative material

1647. How can you improve the adhesion of a fissure sealant

BY DR.ABDULRAHMAN ALMUALM
MJDF MCQS WITH ANSWERS

A. Acid etching technique

1648. The advantage of using dental floss over rubber point interdentally

A. Remove plaque and debris in interproximal surfaces


B. Polish
C. Massage of the interdental papillae
D. Aid and recognise subgingivally

1649. After prophylactic treatment, you decide to change the flora to a non-acidogenic
by changing the diet. How long does it take to achieve this change

A. Few weeks
B. Several months or longer

1650. Which one of the following is a non-calorie sweetener

A. Mannitol
B. Saccharin
C. Xylitol

1651. 6 year old child who had a history of primary herpes simplex has got a recurrent
infection. What is the likely cause

A. Herpes labialis

1652. A newly placed restoration interferes with occlusion. What will be the periodontal
response

A.Thickening of the periodontal membrane

BY DR.ABDULRAHMAN ALMUALM
MJDF MCQS WITH ANSWERS

1653. In class II restoration, all of the following considered to occur as probable causes
of periodontal problems except

A. Flat ridge
B. Faulty or not proper contour
C. Not properly polished restoration
D. Cervical wall is too deeply apical
E. Overextension of lining in cavity

1654. Angular type of bone resorption can be seen more often in**

A. Occlusal traumatism
B. Food particles retention
C. Periodontosis
D. All of the above
1655. What is the most important function of periodontal ligament

A. Keep teeth in the socket


B. Protect alveolar bone
C. Provide nutrition

1656. The periodontal ligament in a teeth without use appear to be

A. Narrow
B. Wide

1657. Which radiographic method would you use in assessing periodontal conditions
and lesions

A. Bitewing
B. Periapical

BY DR.ABDULRAHMAN ALMUALM
MJDF MCQS WITH ANSWERS

C. Occlusal
D. Panoramic

1658. What does CPITN stand for

A. Community Periodontal Index of Treatment needs

1659. Vertical incision of mucoperiosteal flap should be

A. Always extending to the alveolar mucoperiosteal


B. Bisect the middle of gingival papillae
C. Must be at the right angle of the tooth

1660. Apical migration of the epithelial attachment followed by atrophy of marginal


gingiva at the same level results in

A. False periodontal pocket


B. Periodontal pocket recession
C. Gingival cleft
D. True pocket

1661. Calculus attaches to teeth surface by**

A. Acquired pellicle
B. Interlocking to the crystals of the tooth
C. Penetrated into enamel and dentine
D. Mechanical interlocking
E. All of the above

1662. The width of normal periodontal ligament space is

BY DR.ABDULRAHMAN ALMUALM
MJDF MCQS WITH ANSWERS

A. 0.25 to 0.5mm
B. 1mm

1663. The incision angle in Gingivectomy is

A. 45° to the tooth in an apical direction

1664. The MOST common place for initiation of gingivitis is

A. Interdental papillae
B. The free gingival ridge
C. The attached gingiva
D. The marginal gingiva

1665. Which is the MOST local factor in the aetiology of periodontal disease

A. Occlusal trauma
B. Calculus
C. Brushing habits
D. Coarse food

1666. Incisive foramen when are superimposed over apex of root on radiograph may
be mistaken to be

A. Cyst
B. Cementoma
C. Odontoma

1667. Which of the following factors can affect the shape and size of the pulp canal

BY DR.ABDULRAHMAN ALMUALM
MJDF MCQS WITH ANSWERS

A. Chemical irritation and caries


B. Trauma and function
C. Attrition, wear and aging of the patient
D. All of the above

1668. Following a periodontal surgery; periodontal dressing will

A. Help in tissue adoption


B. Decrease the patient’s discomfort
C. Enhance the rate of healing
D. Control bleeding and maintain blood clot

1669. What is the MOST important role of saliva in preventing dental caries

A. Buffering action

1670. A patient comes with a lactobacillus of more than 100000. what is your advice

A. Reduce sugar in diet

1671. The MOST cariogenic sugar is

A. Sucrose

1672. How to detect the furcation involvement

A. Radiolucent area radiographically


B. Probe in mesial distal and mid facial areas of suspected tooth

1673. What is TRUE about topical fluoride**

BY DR.ABDULRAHMAN ALMUALM
MJDF MCQS WITH ANSWERS

A. It cooperates into plaque and resits acid demineralisation


B. Fluoride prophylaxis paste has been clinically proven to be more effective
preventing caries

1674. Which of the following is correct about Nitrous Oxide N2O

A. N2O has high analgesic property and low anastatic at its minimum anaesthetic
dose. “Low MAC; Max Anaesthetic Concentration”
B. Absolutely contraindicated in pregnancy
C. Has low blood diffusibility and result in hypoxia
D. It is good aesthetic and low MAC

1675. Which is CORRECT about the Lingual Nerve

A. Lingual nerve is anterior and medial to inferior alveolar nerve

1676. Which local anaesthetic agent is preferred for a confirmed hypersensitive patient

A. 3% prilocaine with felypressin


B. Mepivacaine 3% without vasoconstrictor **

1677. The MOST common side effects of local anaesthetic is a result of

A. Intravascular injection
B. Hypersensitivity

1678. Which is TRUE about disinfectant solution**

A. It destroys all pathogenic micro organism including high resistant


B. It reduces the number of micro organism to a non infective level

BY DR.ABDULRAHMAN ALMUALM
MJDF MCQS WITH ANSWERS

C. It kills all pathogens but not spores.

1679. What to do with instruments after surgically treating a patient with confirmed
diagnosis of hepatitis B**

A. Soak them in hypochlorite solution “Milton”


B. Sterilize, scrub and sterilize
C. Handle them with two pairs of household rubber gloves
D. Scrub them with iodine surgical solution

1680.What is the mode of action of autoclaving “Moist sterilisation”

A. Moist heat sterilization


B. Protein denaturation

1681.All of the following are requirements of an adequate mucosal periosteal flap


except**

A. Base is wider than the free margin


B. Mucous membrane is carefully separated from periosteum
C. Base containing blood supply

1682. The first thing to do after surgical removal of impacted 3rd molar in the mandible
is**

A. Cold application from the outside

1683. A primary molar with relatively un-resorbed roots encompassing the permanent
tooth bud. What extraction technique would you use to avoid the inadvertent removal
of a developing bicuspid

A. Section the tooth vertically and remove each root separately

BY DR.ABDULRAHMAN ALMUALM
MJDF MCQS WITH ANSWERS

1684. A young female patient presents with throbbing pain in the left lower posterior
jaw with trismus and associated lymphadenopathy. What would be your diagnosis

A. Tumour
B. Pericoronitis

1685. Patient presents to you with a history of local pain in the lower right posterior
region. Insisting that you extract his lower teeth. The teeth in question are vital without
any pathology. You diagnosis is

A. Odontalgia
B. Referred pain
C. Trigeminal neuralgia

1686. Which of the following are not supplied by the mandibular division of trigeminal**

A. Anterior part of digastric


B. Masseter muscle
C. Buccinator

1687. 30 years old male complains of painless swelling in the buccal mucosa. It has
been present for about six months. He admits “playing with it”. He is concerned
because this might represent cancer. The base is narrow; the most likely diagnosis is

A. Irritation fibroma

1688.A patient is complaining of an open sore on the buccal mucosa. The lesion is
painless, ulcerated, has indurated margins, 1.5 cm in diameter, covered by greyish-
white exudate, enlarged lymph nodes and tender, negative tuberculin test and positive
serology. The diagnosis is**

A. Chancre /Primary lesion of syphilis/

BY DR.ABDULRAHMAN ALMUALM
MJDF MCQS WITH ANSWERS

1689. An old male presents complaining of having numerous white lesions in the oral
cavity within past few days. Prior to this the family physician prescribed chlorite
tetracycline for an upper

respiratory infection, the patient is taking this antibiotic for the past two weeks; lesions
are relatively non-painful, slightly
elevated, adhere plaques on the lip mucosa, buccal mucosa and the tongue. MOST
LIKELY to be

A. Moniliasis /Which is candidiasis/

1690. Characteristic of Squamous Cell Carcinoma**

A. White skinned people


B. Alcoholic and smokers
C. It reacts far simply to radiotherapy

1691. How can differentiate between a benign epithelial tumour from a carcinomatous
one

A. Soft papillomatous mass, not indurated or not fixed /Move freely/ and pedunculated.

1692. What is the characteristic feature of gingivitis in AIDS patient**

A. Red band on the free gingiva associated with platelet.


B. Correlating with other pathogenesis lesions of AIDS and does not resolve to
periodontal conventional treatment.
C. Sever pain

1693.The characteristic feature of basal cell carcinoma is

BY DR.ABDULRAHMAN ALMUALM
MJDF MCQS WITH ANSWERS

A. Blood metastasis
B. Does not erode bone
C. Intensive involvement / inveterately characteristic/
D. Radio resistant

1694. What is the significance of erosive lichen planus**

A. High malignant potential


B. Some malignant potential

1695. Where does the bone resorption show in a necrosis pulp of deciduous molar

A. At the root apex


B. At the bifurcation
C. On the buccal side of the tooth
D. On the lingual side of the tooth

1696. To extract upper deciduous molar, the movement should be

A. Buccal first to move tooth


B. Palatal first to move tooth
C. Distal first to move tooth
D. Rotation movement
E. Fraction of the tooth

1697. 8 years old child, on examination you find 75 with carious exposure. On X-ray
you find 35 missing. Your treatment is

A. Extraction of 75 allowing 36 to move mesially


B. Pulpotomy on 75 and wait indefinitely
C. Extraction of 75 and place a fixed space retainer to be replaced with fixed bridge.

BY DR.ABDULRAHMAN ALMUALM
MJDF MCQS WITH ANSWERS

D. Extraction of 65 and 75

1698. Which of the following is true

A. Antibiotics are useful in the treatment of periodontitis


B. Trauma from occlusion causes thickening of the marginal gingivae
C. Periodontitis is the primary cause of teeth lost after the age of 35.
D. All periodontal pockets can be identified by x-ray
E. Periodontitis is the most common disease in the oral cavity

1699. Longest lasting resin restorations are

A. Class I
B. Class II
C. Class III
D. Class IV

1700. Pin Restoration with which material has the best retention

A. Amalgam
B. Gold inlay
C. Composite
D. Glass Ionomer

1701. Acute apical abscess-emergency treatment

A. Open and drain for two days


B. Antibiotic and analgesic
C. Clean and Ledermix

BY DR.ABDULRAHMAN ALMUALM
MJDF MCQS WITH ANSWERS

1702. TMJ dysfunction common symptom is

A. Clicking

B. Locking
C. Pain in the muscles of mastication

1703. Gagging reflex is caused by

A. Trigeminal nerve
B. Glossopharyngeal
C. Facial nerve
D. Recurrent laryngeal

1704. Which impression material should NOT be kept in water within on hour: “in
another paper was: 30 mins before pouring”

A. Polyether
B. Condensation silicone
C. Polyvinyl silicone

1705. High copper amalgam lasts longer than low copper amalgam because of**

A. Increased compressive strength


B. Increased corrosion resistance
C. High creep
D. Increased tensile strength
E. Decreased setting expansion

1706. Porosity in acrylic dentures is caused by

BY DR.ABDULRAHMAN ALMUALM
MJDF MCQS WITH ANSWERS

A. Contraction porosity in thickest point of the denture


B. Insufficient pressure during flasking causes it

1707. The shortest facial height is when

A. Teeth are overlapped


B. There is maximum cuspal interdigitation

1708. What is TRUE about vertical dimension

A. Does not change for the whole life


B. Decreases when head is tilted back
C. Increases when a lower denture is placed in mouth

1709. The sterilisation of Gutta Percha is achieved by

A. Heat
B. Chemical sterilisation
C. Flame
D. Boiling
E. Autoclave

1710. Why would you cast gold in hot mould**

A. To compensate for the expansion of investment.

1711. If temporary cementation is required, which cement will you use

A. ZOE

BY DR.ABDULRAHMAN ALMUALM
MJDF MCQS WITH ANSWERS

B. Zinc Polycarboxylate
C. GIC

1712. In the construction of an RPD, guiding planes are created by

A. Perpendicular to the occlusal plane


B. Parallel side towards the path of placement.

1713. When should pour polyether impression materials

A. Within 24 hours after taking impression


B. Within 30 minutes after taking impression
C. Should be stored dry and then poured
D. Should be stored in humid place

1714. Which of the following is a frequent cause of opaqueness in a porcelain jacket


crown

A. Porcelain layer is too thin over the opaque layer.


B. Porcelain layer is too thick

1715. Which cement is less soluble in the oral cavity

A. Polycarboxylate
B. Zinc phosphate
C. Silicate phosphate
D. GIC

1716. A patient with reasonable oral hygiene has a small proximal caries on the
premolar. The patient requests an aesthetic filling. Your preparation will be

BY DR.ABDULRAHMAN ALMUALM
MJDF MCQS WITH ANSWERS

A. Same as amalgam with cavo-surface bevels


B. Proximal caries removal with occlusal & gingival bevels

1717.A gingivally extended chrome cobalt cast clasp

A. Can extend 0.5 under the surveyor line


B. Can extend 0.25 under the surveyor line
C. Will resist deforming forces better compared to cast gold

1718. The first molars are extracted in both arches

A. The bone resorption will be the same for both arches


B. Resorption is more on the palatal side of maxillary molars
C. Resorption is more on lingual side of mandibular molars
D. The ridge height resorbs more in maxilla than mandible

1719. The use of nickel chromium in base plate should be judiciously considered
because

A. A significant number of females are allergic to nickel


B. A significant number of female are allergic to chromium
C. A significant number of males are allergic to nickel

1720. A seven year old boy fell of his bicycle 2 weeks ago and broke his maxillary
central incisor. The pulp horn is visible as a pin point. The tooth is vital. Your treatment
will be

A. Pulpectomy
B. Place calcium hydroxide and fill with composite resin
C. Calcium hydroxide pulpotomy

BY DR.ABDULRAHMAN ALMUALM
MJDF MCQS WITH ANSWERS

1721. During mouth preparation for RPD on tooth adjacent to edentulous area. There
is dentine exposure

A. Restoration is required
B. Proceed with rest seat preparation and fabrication if involved area is not more than
2mm

1722. After making an impression to reline an RPD the dentist notes that the indirect
retainers are not resting on the tooth. To avoid this what process should have
undertaken initially

A. Ask patient to bite firmly while impression is setting


B. Hold the metal base frame against the abutment tooth while setting
C. Fabricate new denture
D. Add impression material and close the gap

1723. Which is the only dental tissue that lose its formative cells as it matures

A. Enamel
B. Dentine
C. Pulp
D. Cementum

1724.The muscle responsible for maintaining the bolus of food while chewing is

A. Buccinator
B. Orbicularis oris

1725. Which muscle contracts during the tongue protrudes

A. Mylohyoid

BY DR.ABDULRAHMAN ALMUALM
MJDF MCQS WITH ANSWERS

B. Genioglossus
C. Digastric

1726. Which of the following is the most significant factor regarding filler in composite
for increased strength

A. Particle size 1-3 micron


B. Sub micron sized particles
C. High concentration of the filler particles

1727.A patient has a small incisal fracture of the maxillary incisor. Which is the best
material to resist fracture at the acid etched tooth composite interface

A. Micro-filled composite
B. Hybrid composite
C. GIC
D. Silicate

1728. The principal factor involved in oral Para-function is related to

A. Periods of stress
B. Occlusal pre-maturities during mandibular closure

1729. During manual palpation, the mucosa is thin in

I. Midline of the palate


II. Mylohyoid region
III. Over torus palatinus

A. I, II and III
B. None of the above

BY DR.ABDULRAHMAN ALMUALM
MJDF MCQS WITH ANSWERS

C. I and II
D. II and III

1730. The path of condyle during mandibular movements depends on

A. Articular eminence, meniscus/capsule of TMJ and muscle attachments

1731. While doing RCT you gave dressing with a paper point wetted with..??...solution.
The patient arrives the next day with severe

pain. There is no swelling but the tooth is tender to percussion. You will**

A. Replace with similar dressing and prescribe antibiotic


B. Replace with corticosteroid past
C. Retrieve paper point surgically
D. Remove the dressing and leave for several days before replacing it.
E. Provide incision and drainage

1732. The area of the posterior palatal seal includes which of the following

A. [left][right]
B. Hamular notch

1733. The best method of cleaning and toilet cavity

A. Alcohol
B. Citric acid
C. Water
D. Organic acid

1734. Herpetic infection is an iatrogenic infection spreads by the infected

BY DR.ABDULRAHMAN ALMUALM
MJDF MCQS WITH ANSWERS

A. Serum
B. Vesicle
C. Vesicle fluid and saliva

1735. Periapical abscess is differentiated from periodontal abscess by

A. Pulpal radiology
B. History and vitality test
C. X-ray and history

1736. You may suspect poor reaction to bleeding if there is a history of

A. Cirrhosis of liver
B. Hypertension

1737. Maxillary central incisor located palatally causes

A. Prolong stay of primary central incisor


B. Supernumerary teeth

1738. Toxicity of anaesthetic is assessed by

A. Dose which is given


B. Percentage of solution
C. Vasoconstrictions amount

1739. Children born with cleft palate, microdontia and glossoptosis have

A. Christian disease

BY DR.ABDULRAHMAN ALMUALM
MJDF MCQS WITH ANSWERS

B. Trenches-Collins Syndrome
C. Pierre-Robin Syndrome

1740. Which of the following penicillin are readily destructed by stomach acid

A. Methicillin
B. Cloxacillin
C. Phenoxy methyl
D. Penicillin G

1741. What is not correct about Long Buccal Nerve**

A. Passes through two heads of pterygoids muscles


B. Supplies mucosa over lower and upper molars
C. Supplies the buccinator muscle
D. Supplies skin over buccinator

1742. N2O excretes through

A. Urine
B. Lungs

1743. Radiopaque lesions are seen in

A. Multiple myeloma
B. Paget’s disease
C. Hyperparathyroidism
D. Chronic renal failure

1744. The causative organism in localised juvenile periodontitis is Actinomyces


actinomycete comitans which is**

BY DR.ABDULRAHMAN ALMUALM
MJDF MCQS WITH ANSWERS

A. Gram positive facultative aerobic


B. Gram positive facultative anaerobic non-motile rod
C. Gram negative facultative anaerobic non-motile

1745. Which of the following is NOT significant factor in determining virulence of


a.a.??** //I think a.a. stands for

Anaerobic Bacteria/

A. It effects chemotaxis
B. Produces leukous toxins
C. Destroys collagen
D. It is immuno-suppressive

1746. Density of film is decreased by increasing the

A. MA
B. Exposure time
C. Developing time
D. Rinsing time

1747. The best space maintainer is

A. Lingual holding arch


B. Pulpectomised primary tooth
C. Band and loop maintainer

1748. The laboratory findings in Paget’s disease show

A. Elevated calcium, elevated phosphate, and elevated alkaline phosphate.

BY DR.ABDULRAHMAN ALMUALM
MJDF MCQS WITH ANSWERS

B. Normal calcium, normal phosphate and elevated alkaline phosphate


C. Decreased calcium, increased phosphate and elevated alkaline phosphate
D. Increased calcium, normal phosphate and decreased alkaline phosphate
E. Normal calcium, increased phosphate and elevated alkaline phosphate

1749. While giving CPR which of the following is considered

A. It achieves 30% of cardiac output with 60 compressions per minute


B. It achieves normal blood oxygen levels with 12 respirations per minute
C. You have to check compression point by thumping before starting compression
D. Cardiac output has to be monitored regularly by checking radial pulse.

1750. A patient has developed a sever chest pain and difficulties in breathing while in
the dental chair. Your initial response is

A. Administer glycerine trinitrate and monitor patient in upright position


B. Patient has an acute episode of angina as demonstrated by curve in ECG
C. No treatment is required until confirmed as MI by ECG
D. Patient has myocardial infarction as confirmed by ECG

1751. On inspection of lateral boarder of the tongue at the base, which structure would
you expect to find

A. Filiform papillae
B. Fungiform papillae
C. Taste buds
D. Lymph nodes
E. Circumvallate papillae

1752. Delayed eruptions of at least part of the dentition is a recognised feature in**

BY DR.ABDULRAHMAN ALMUALM
MJDF MCQS WITH ANSWERS

A. Dentino-Genesis imperfecta
B. Anhidrotic ectodermal dysplasia
C. Rickets

1753. Which of the following is a radiographic feature of dentino- genesis imperfecta

A. Small pulp chambers and root canals, normal enamel


B. Enamel is missing but dentine formation is normal
C. Enamel and dentine show disturbances
D. Pulp is normal but dentine is abnormal

1754. A 10 year old boy presents with non-vital, non-mobile tooth. Treatment is
A. Pulpectomy with calcium hydroxide
B. Pulpectomy with Zinc oxide eugenol
C. Pulpotomy with formocresol
D. No treatment is required if tooth is asymptomatic

1755.A patient suffers a blow to his maxillary central incisor without resulting in
fracture. The pulp

A. Immediate necrosis
B. Becomes non-vital but only if treatment is delayed too long
C. Becomes non vital irrespective of treatment
D. No changes is seen later if fracture does not occur

1756. In the case f malignant melanoma occurring intra orally, which of the following
is true

A. Uncommon on the palate when occurs intra orally


B. Should not biopsied, as this will increase metasis
C. The 5 years survival rate is 20%

BY DR.ABDULRAHMAN ALMUALM
MJDF MCQS WITH ANSWERS

D. The incidence of oral melanoma is the same as those on the skin


E. Commonly occurs intra orally

1757. Patient on anti-coagulant therapy requires an extraction to be performed. Which


of the following is NOT true**

A. Post operative bleeding can be reduced somehow by using tranexemic acid


B. Prothrombin values of at least 2.5 is required to perform extraction
C. It takes at least 8 hours for heparin to take affects
D. Heparin should be administered sub-cutaneous

1758. Community water fluoridation MOST effectively achieves is**

A. 90-95% reduction of caries


B. 45-55% reduction of caries
C. Reduces pit and fissures caries more than smooth surfaces
D. Reduces smooth surfaces more than pit and fissures

1759. Patient presents with caries in many teeth. you will advise that

A. Fluoride toothpaste does not effectively prevent caries and topical fluoride is
required.

1760. What is the primary consideration in providing nutrition/dietary counseling to a


patient

A. Secure details of patient’s eating habits


B. Have the patient to fill in a diet survey
C. Eliminate sugar from diet

1761. Which of the following is true in regards to periapical cementoma

BY DR.ABDULRAHMAN ALMUALM
MJDF MCQS WITH ANSWERS

A. Teeth are vital.


B. Teeth are not vital

1762. Which of the following is not a side effects of lignocaine

A. Angioneurotic oedema
B. Nervousness

1763. A physician refers a nine year old boy to you to confirm diagnosis. The boy has
a fever of 40°C and coughing. When you focus your light into his eyes he turns away.
Intra-orally there are white spots surrounded by red margins. The disease and lesions
are

A. Measles and Koplik’s spots


B. AHGS vesicles

1764. In periodontal scalers and curettes; the blade is formed by which two surfaces

A. Two lateral surfaces


B. Lateral surface and face
C. Lateral surface, face and shank
D. Lateral surface, face, back and shank

1765. Which of the following is NOT TRUE in regards to lateral periodontal cyst**

A. It is more common in anterior region


B. It occurs more in maxilla than mandible
C. Probable origin is from dentigerous cyst which develops laterally
D. Encountered in the cuspid-premolar region of the mandible, derived from the
remnants of the dental lamina

BY DR.ABDULRAHMAN ALMUALM
MJDF MCQS WITH ANSWERS

1766. Middles aged woman gives a history of intermittent unilateral pain in the sub
mandibular region, most probable cause is

A. Calculus in the salivary duct resulting in sialolithiasis.


B. Ranula
C. Cyst
D. Mucocele

1767. What is TRUE in regards to branchial cyst

A. Situated on the anterior boarder of sternocleidomastoid muscle

1768. Damage/injury to which nerve causes dilation of pupils

A. Oculomotor
B. Ansa cervicalis
C. Abducens

1769. After an inferior alveolar nerve block; the patient develops paralysis of eyelid,
upper lip and lower lip on that side. This means that the L.A was deposited in

A. The parotid gland

1770. Aspirin reduces pain by which of the following mechanism

A. It is anti inflammatory by the release of histamine


B. It blocks the cyclo-oxygenase pathway.
B ans

1771. Patient with haemophilia presents which of the following findings**

BY DR.ABDULRAHMAN ALMUALM
MJDF MCQS WITH ANSWERS

A. Increased prothrombin time


B. Increased bleeding time
C. Increased clotting time
A

1772. The pulpal floor of the Class II cavity for a mandibular first premolar should be**

A. Parallel to occlusal plane


B. Perpendicular to long axis
C. Tilted lingually

1773. Marginal leakage at the proximal gingival cavosurface of a recently restored


class II can be caused by

I. Insufficient condensation
II. First proximal increment was too large
III. Neglecting to wedge the matrix
IV. Hand manipulation instead of mechanical
V. Debris contamination

A. I, II, III
B. II, III, IV
C. I, II, V
D. None of the above
E. All of the above

1774. What are the dangers of using air as a cooler during cavity cutting

A. Hypersensitivity
B. Odontoblast is drawn into the tubule

BY DR.ABDULRAHMAN ALMUALM
MJDF MCQS WITH ANSWERS

1775. In RCT the ideal root filling

A. Ends at the apex


B. Extends beyond apex to achieve a good seal
C. Ends at the dentino-cemental junction

1776. Where is the narrowest part of the pulp

A. At the radiographic apex


B. At the dentino-enamel junction
C. At the orifices

1777. Which of the following is MOST useful in differentiating between apical abscess
and periodontal

A. Percussion
B. Vitality tests
C. Cold tests
D. Heat tests

1778. What is the ideal length for a post in post-core in an endodontically treated tooth

A. 2/3 of the tooth length


B. ½ of the tooth length
C. 1.5 times that of the crown
D. Same as the anticipated crown

1779. Which is correct in regards to shade selection of crowns

A. It should be selected before starting preparation


B. Chroma is the lightness/darkness of colours

BY DR.ABDULRAHMAN ALMUALM
MJDF MCQS WITH ANSWERS

C. Value is the colour itself


D. Hue is the concentration of colours

1780. How many mg of fluoride ions are obtained from 2.2 mg tablet of NaF

A. 0.5mg
B. 1 mg
C. 1.5mg
D. 10mg

1781. Strain is defined as **

A. An external force
B. An internal force to oppose external load
C. Deformity opposed the applied load

1782. Size of pulp chamber within the tooth is influenced by

A. Age
B. Parafunctional
C. History of the tooth /abrasion, erosion, caries/
D. All of the above

1783. Self polymerising acrylic resins differs from heat cured resins because they
exhibit

A. Higher molecules weight


B. Higher content of residual monomers

1784. The advantage of firing porcelain in vacuum**

BY DR.ABDULRAHMAN ALMUALM
MJDF MCQS WITH ANSWERS

A. Reduces size of air-bubbles incorporated thus decreasing porosity


B. Removes water before firing, increasing the hardness of porcelain
C. Significantly lowers firing temperature

1785. The contraction (Gaseous) porosity in inlays is related to**

A. Overheating of the alloy


B. Molten gases
C. Diameter of the sprue
D. Overheating of investment

1786. Where would you expect to find the Mylohyoid on relation to periphery of
complete denture

A. Mandibular buccal in the midline


B. Mandibular lingual in the midline
C. Mandibular disto buccal area

1787. Class V lesion may originate

A. In lingual pits
B. In buccal fissures
C. Poor oral hygiene

1788. Retention for occlusal amalgam cavity in premolars is BEST provided by

A. Slightly undercutting of walls with inversed cone bur


B. Mesial and distal undercuts
C. Buccal and lingual undercuts

BY DR.ABDULRAHMAN ALMUALM
MJDF MCQS WITH ANSWERS

1789. What is true in regards to lateral mandibular incisor

A. 20% have 2 canals with one foramen


B. 20% have 2 canals with two foramina
C. 40% have two canals with 10% ending in two foramina
D. 40% have two canals with only one ending in two foramina

1790. Splinting the adjacent teeth in fixed bridge is primarily done to

A. Distribute the occlusal load


B. Achieve better retention

1791. Porcelain must not be contaminated by handling between which two stages

A. Pre-soldering and heat treatment


B. Heat treatment and opaque /bake/ stages
C. Opaque and bisque stages
D. Bisque and glazing stages
E. First opaque bake and second opaque bake

1792. What is the relationship of the retentive portion of the partial dentures retainers
to the survey line of abutment

A. Gingival /Below/
B. Occlusal /Above/
C. No relation

1793. Which of the following liquids is not suitable for prolonged immersion of cobalt
chrome partial dentures

A. Alkaline peroxidase

BY DR.ABDULRAHMAN ALMUALM
MJDF MCQS WITH ANSWERS

B. Hypochlorite solutions
C. Soap solutions
D. Water

1794. Dentures hyperplasia is generally attributed to

A. Poor oral hygiene


B. Denture movement

1795. In complete dentures, cheek biting is most likely a result of

A. Reduced Overjet of posterior


B. Increased vertical dimension
C. Teeth have large cusp inclines

1796. Resting face height in edentulous patients

A. Decreases when head is tilted back


B. Increases when lower denture is inserted
C. Does not change over time

1797. Ala-Tragal line is**

A. The line running from the tragus of the nose to ala of the ear
B. A guide used to orient the occlusal plane
C. Parallel to Frankfurt horizontal plane
D. A guide to the occluding face height in complete denture.

1798. Decision to employ cusped or without cusps teeth is influenced by**

BY DR.ABDULRAHMAN ALMUALM
MJDF MCQS WITH ANSWERS

A. Reverse Overjet
B. TMJ problems
C. Cranio mandibular skeletal relationship

1799. The first forming microbial elements of plaque are

A. Aerobic gram positive G+


B. Aerobic gram negative G-
C. Anaerobic gram negative G-
D. Spirochetes
E. Anaerobic gram positive G+

1800. Extracellular polysaccharides in plaque are formed by

A. Bacteria from sucrose


B. Precipitated from carbohydrate
C. Precipitated from glycoproteins

1801. What is important requisite for fillers in dental composite restorative resins in
load bearing area

A. Sub micro sized particles


B. High coefficient of thermal expansion
C. High in content (High filler)

1802. Sensitivity to hot and cold foods soon after cavity preparation and placement of
GIC and composite resin in an upper incisor tooth is due to

A. Mechanical trauma due to cavity preparation


B. Chemical
C. Heat from GIC settings

BY DR.ABDULRAHMAN ALMUALM
MJDF MCQS WITH ANSWERS

1803. After completing pulp extirpation, debridement and placing a dressing; apical
periodontitis is because

A. Over instrumentation extending into periapical area


B. Irritation from chemicals used
C. Entrapped bacteria
D. One or any combination of the above

1804. It is desirable to major connectors of upper partial dentures to**

A. At least 5 mm away from the gingival margin


B. Cover the anterior palate

1805. Which of the following statements is true

A. Last secretion of odontoblast forms cementum


B. Last secretion of odontoblast forms acquired enamel cuticle
C. Remnants of ameloblasts form primary enamel cuticle
D. Remnants of odontoblasts form primary enamel cuticle

1806. When you tries to seat a crown on tooth you find a discrepancy of 0.3mm at the
margin; you will

A. Reduce inner surface of crown


B. Remake a new crown
C. Smooth the enamel at the margin
D. Hand burnish crown margins

1807. In regards to Chlorhexidine mouth wash**

BY DR.ABDULRAHMAN ALMUALM
MJDF MCQS WITH ANSWERS

A. Is anionic
B. Used in 0.02% concentration
C. Used in 0.12 concentration
D. Penetrates the gingival crevice/pocket

1808. Glycerine trinitrate given to an angina patient acts by

A. Gives relief of pain by decreasing venous return


B. Decreases blood pressure and causes headache

1809. Which of the following is NOT complication of radiation to head and neck area**

A. Xerostomia
B. Mucositis
C. Increased caries
D. Heightened taste sensation
E. Increased risk of osteomyelitis

1810. A female patient is diagnosed with Addison’s disease which of the following
does not confirm this

A. Weakness, lassitude
B. Anorexia, nauseas, fatigue
C. Hypotension
D. Bony expansion
E. Amenorrhea

1811. Which of the following conditions is not associated with periodontal destruction
in primary teeth

BY DR.ABDULRAHMAN ALMUALM
MJDF MCQS WITH ANSWERS

A. Down’s syndrome
B. Steven Johnson’s syndrome
C. Hypophosphatasia
D. Papillon-Lefebvre syndrome
E. Cyclic neutropenia

1812. In patient with exposed root surfaces

A. Ask to use low abrasive dentifrices


B. It is because of dental hypersensitivity

1813. Which of the following is the best index to evaluate gingival health

A. Gingival index by Loe and Silness


B. Periodontal index
C. Periodontal disease index
D. OHI-S

1814. In surveying; calibration of examiners data is important because

A. It reduces the errors in gathered data.

1815. Patient is resistant to caries but has periodontal disease. In this case, sucrose
in diet is important because

A. Sucrose is greatly involved in plaque development


B. S. mutans produces Levans frictions which are used by periodontal pathogens
C. The streptococcus mutans cannot survive with a continual supply of sucrose
D. Existing plaque must continue to get sucrose in order to grow

1816. Cariogenicity of Streptococcus mutans is because of the production of

BY DR.ABDULRAHMAN ALMUALM
MJDF MCQS WITH ANSWERS

A. Glucans
B. Levans
C. Fructans
D. Sucrose

1817. A child consumes a toxic dose of fluoride. You will**

A. Induce vomiting
B. Gives a lot of fluids
C. Gives a lot of fluids and sodium bicarbonates
D. Ask patient not to eat for 45 minutes
E. Gives milk, calcium tablets or magnesium tablets

1818. Collimation is done to

A. Reduces the size of the beam, so it is easy to visualise the central X ray.
B. Avoids unnecessary exposure to radiation of surrounding tissues of the patient

1819. In X rays filtration is used to**

A. Remove low energy X rays


B. Reduce exposure time
C. Reduce size of the beam

1820. In calculus formatio, the epitaxic concept is one of the theories. Which of the
following is true

A. Mineralisation occurs when calcium and phosphate content is high


B. The presence of matrix would start initiate formation of nucleus

BY DR.ABDULRAHMAN ALMUALM
MJDF MCQS WITH ANSWERS

C. The amorphous materials would convert to calcium phosphate and hydroxy


phosphate

1821. Gemination is**

A. Division of single tooth, twining


B. Fusion of two or more crowns of teeth
C. Fusion of two or more roots

1822. In primary teeth, failure of Ca(OH)2 pulpotomy is MOST likely to produce**

A. External resorption
B. Internal resorption
C. Necrosis of the pulp
D. Ankylosis

1823. A raised dot on X ray film is present to

A. Orient exposure side


B. Differentiate between left and right side
C. Dip during developing

1824. What does the fixes solution in developing X-rays do

A. Removes unexposed silver halide crystals


B. Removes exposed silver halide
C. Fixes the developed film

1825. When the developing solution is correctly mixed and x ray film is being
developed for normal time; but the solution is too warm, the outcome film will be

BY DR.ABDULRAHMAN ALMUALM
MJDF MCQS WITH ANSWERS

A. Too light
B. Too dark
C. Fogged

1826. Kaposi’s sarcoma

A. Seen on buccal mucosa in HIV as purple lesion


B. Seen on palate of most HIV patient
C. Should be biopsy

1827. What is characteristic feature seen in pyloric stenosis**

A. Erosion of maxillary central incisors


B. Vomiting of undigested food
C. Loss of appetite
D. Weakness

1828. At birth, the oral cavity usually contains

A. S. mutans only
B. No micro organism
C. S. mutans and S. salivavis
D. Lactobacilli and S. mutans

1829. The papillae that are few in numbers, associated with MOST taste buds,
associated with Von Ebner’s glands are**

A. Fungiform
B. Circumvallate
C. Foliate
D. Filiform

BY DR.ABDULRAHMAN ALMUALM
MJDF MCQS WITH ANSWERS

1830. In class II preparation it is difficult to place the gingival seat when preparation is
extended too gingivally because the

A. Enamel rods are directed occlusally


B. Marked cervical constriction

1831. In maxillo fracture, if intra cranial pressure increases**

A. It is normal
B. Typically associated with tachycardia
C. Associated with blood pressure
D. Usually subsides spontaneously
E. Typically associated with constricted and un-reactive pupil

1832. Moist heat sterilization is achieved by

A. Denaturation of protein

1833. In regards to Benzodiazepines

A. Increases R.E.M. sleep


B. Has a hangover effects because of active metabolism
C. Includes carbamazepine
D. Can be used safely on children as it achieves reliable effects

1834. Which is NOT CORRECT in regards to lingual nerve

A. It is posterior and medial to the inferior alveolar nerve


B. It passes close to the mandibular 3rd molar

BY DR.ABDULRAHMAN ALMUALM
MJDF MCQS WITH ANSWERS

C. It may be anaesthetised by the mandibular nerve block


D. It provides supply to the lingual gingiva
E. Supplies anterior 2/3 of the tongue

1835. The maxillary and mandibular teeth get their blood supply from**

A. Separate branches of S. Palatina artery


B. Separate branches of maxillary artery
C. Branches of maxillary and mandibular arteries

1836. Haemophilia is characterised with**

A. Daughters affected from their carriers fathers


B. Presents on “y” chromosome
C. Hemarthrosis is common finding
D. Deficiency of factor VII
E. Neutrophil defect

1837. In minor oral surgery which is TRUE in regards to antibiotic

A. Amoxil is satisfactory against most oral infection


B. Metronidazole and Amoxil have the same penetrating power
C. It is evident that it will reduce post operative swelling
1838. In regards to third molars surgery

A. Maximum swelling is seen after 24-48 hours


B. Prophylactic antibiotic will reduce swelling
C. Antibiotic cover is compulsory

1839. A 65 year old patient needs extraction of 44; he has taken insulin in the morning.
What preoperative advice you should

BY DR.ABDULRAHMAN ALMUALM
MJDF MCQS WITH ANSWERS

give

A. Take more sugar


B. Maintain normal diet
C. Antibiotic 2 hours before
D. Medication increases preoperatively

1840. Patient with prosthetic heart valve taking 7.5 mg warfarin.


She has????. Patient needs extraction. What is your management

A. 3g Amoxil, suture after surgical removal


B. 3g Amoxil, suture when bleeding has stopped
C. Gentamycin/vancomycin cover, stop warfarin, give heparin and suture later
D. Ampicillin cover, stop warfarin, give heparin and suture later
E. Gentamycin/vancomycin cover, stop warfarin and suture later

1841. Loss of the gingival attachment is measured between

A. CEJ to base of pocket


B. Top of the gingiva to the base

1842. Absence of clearly defined crystal lamina dura is because

A. Pathognomonic of periodontal disease


B. Indicative of attachment loss
C. Associated with periodontal pocket
D. Commonly related to radiograph angulation

BY DR.ABDULRAHMAN ALMUALM
MJDF MCQS WITH ANSWERS

1843. A patient 37 year old; with paroxysmal pain on the left eye that he thinks is
related to his maxillary posterior teeth. The pain comes in recurrent bursts and
aggravated by stress and alcohol. Oral exam is negative. The probable diagnosis is

A. Migraine
B. Cluster headache
C. Trigeminal neuralgia
D. Temporal neuritis

1844. A mandibular permanent first molar had to be extracted, this will affect

A. Adjacent teeth
B. Teeth in the same quadrant
C. Both arches the same side
D. Full mouth

1845. The places for new erupted mandibular molars are created by

A. Resorption of anterior ramus and apposition posteriorly


B. Apposition of alveolar process
C. Apposition of inferior boarder of mandible

1846. A patient comes with a firm, painless swelling of lower lobe of parotid which has
grown progressively for the past year. He complains of paresthesia for the past 2
weeks. This is most likely to be

A. Pleomorphic adenoma
B. Carcinoma of the parotid
C. Lymphoma of parotid

1847. What is the histopathology of the pathogenesis of the plaque following 21 days
of plaque accumulate

BY DR.ABDULRAHMAN ALMUALM
MJDF MCQS WITH ANSWERS

A. Primarily infiltrate of plasma cells


B. Primarily infiltrate of lymphocytes
C. Infiltrate of plasma cells and early bone involvement
D. Infiltrate of neutrophils

1848. What is INCORRECT in HIV associated periodontitis**

A. Picture of ANUG superimposed with RPP


B. Spontaneous bleeding interproximal
C. Depression of T4/T8 lymphocytes
D. Deep Perio-pockets usually seen in advanced periodontitis

1849. What is true in treating a patient with secondary herpes simplex**

A. Acyclovir inhibits viral transcription when applied in the prodromal phase


B. Idoxuridine is better than acyclovir when applied topically
C. Antivirals are contra indicated in immuno-compromised patient

1850. The MOST common cause of gingival enlargement is

A. Hereditary
B. Drug induced
C. Plaque induced
D. Leukaemia

1851. A 13 year old has enlarged gingivae; gives a history of Dilantin sodium what is
you treatment

A. Oral prophylaxis and gingivoplasty


B. Oral prophylaxis, scaling, root planning

BY DR.ABDULRAHMAN ALMUALM
MJDF MCQS WITH ANSWERS

C. Stop medication

1852. A patient has improperly formed DEJ, reduction in size of pulp chamber,
chipping and attrition of enamel that would MOSTLY be

A. Fluorosis
B. Amelogenesis imperfecta
C. Dentinogenesis imperfecta

1853. Which is wrong in regards to (water jet spray) hydrotherapy

A. Does not harm gingivae


B. Removes plaque
C. Removes required pellicle

1854. Anhidrotic ectodermal dysplasia is characteristic by**

A. Hypodontia or anodontia

1855. During extraction of maxillary third molar, the tuberosity is fractured; however, it
remains in place attached to the mucoperiosteum. Which of the following procedures
should be employed**

A. Remove the tuberosity and suture


B. Leave the tuberosity and stabilize if required
C. Remove the tuberosity and fill the defect with Gelfoam then suture.
D. If fractured tuberosity is greater than 2 cm, leave in place and suture

1856. An incision biopsy of an ulcerated and intruded clinically suspicious lesion in 50


years old female reveals chronic inflammation; you would**

BY DR.ABDULRAHMAN ALMUALM
MJDF MCQS WITH ANSWERS

A. Inform the patient and her physician of your findings and instruct the patient to return
in six months
B. Surgically excise the entire lesion since you know it is not malignant
C. Dismiss the patient with instructions for warm saline rinses for re- examination
D. Repeat the biopsy

1857. What is the MOST common consequence of an allergic response to medication

A. Skin rash “dermatitis” with swelling of lips and eyes

1858. How many time do you breath in mouth to mouth resuscitation

A. 10-12 times a minute


B. 4-6 times a minute

1859. What cause a reduce of pulmonary ventilation

A. Laryngeal muscle paralysis


B. Air way obstruction

1860. What would you do if the systole is elevated**

A. Calm down the patient

1861. What would you do if the diastole is elevated

A. Investigate systemic cause

1862. Which are non-calcified areas in the child’s cranium

A. Fontanelles

BY DR.ABDULRAHMAN ALMUALM
MJDF MCQS WITH ANSWERS

1863. Koplik’s spots are associated with one of the following

A. Viral infection
B. Diabetes
C. Measles
D. Rubella
E. Candidosis

1864. What is Von Reckling hausen disease

A. Neurofibroma
B. Necrosis of bone produced by ionizing radiation

1865. How do treat the cause of airway obstruction

A. Extension of the neck


B. Flexion of the neck

1866. How do prepare a patient with rheumatic fever before extraction

A. 6000000 units of benzoyl penicillin


B. 2g Amoxicillin pre-operatively

1867. Which is LEAST likely to cause bleeding after surgical operation

A. Antibiotic therapy
B. Poor surgical techniques
C. Aspirin
D. Codeine

BY DR.ABDULRAHMAN ALMUALM
MJDF MCQS WITH ANSWERS

1868. Acute pyogenic bacteria infection may result in**

A. Leucopoenia
B. Neutropenia
C. Leukocytosis
D. Lymphocytosis
E. Eosinophilia

1869. Prophylactic administration of antibiotic is indicated in patient before oral


surgery with

A. Herpes simplex
B. Whooping cough
C. Bacterial endocarditis

1870. Oral mucosa and skin pigmentation occurs in patient with**

A. Diabetes mellitus
B. Addison’s disease
C. Multiple myeloma
D. Squamous cell carcinoma
E. Bright’s disease
F. Cushing’s disease

1871. Patient has fainted, the signs are, blanched face, weak pulse, moist skin,
shallow respiration; your first management is

A. 1 ml adrenaline subcutaneously
B. Mouth to mouth respiration
C. Nitro glycerine sub lingually

BY DR.ABDULRAHMAN ALMUALM
MJDF MCQS WITH ANSWERS

D. Recumbent position; supine

1872. Thrombo cytopenic purpura would complicate surgery by

A. Oedema
B. Haemorrhage
C. Acute infection

1873. Patient who has WBC count of just over 100000 is most likely suffering from**

A. Leucopoenia
B. Leukaemia
C. Polycythemia

1874. Which of the following is TRUE

A. Antibiotics are useful in the treatment of ANUG


B. Trauma of occlusal factors causes cleft or fibrous thickening of marginal gingivae
C. All Perio pockets can be detected by x rays
D. Periodontitis is the most common problem in teenage
E. Perio disease is a primary cause of loss of teeth after 35 years of age.

1875. In regards to the conditions where you have to prescribe antibiotic prior to dental
treatment

A. Rheumatic fever
B. Sub-acute bacterial endocarditis
C. By pass
D. Valve replacement
E. Uncontrolled diabetes

BY DR.ABDULRAHMAN ALMUALM
MJDF MCQS WITH ANSWERS

F. All of the above

1876. Herpangina is caused by

A. Coxsackie virus

1877. The main vitamin to synthesis prothrombin is

A. Vitamin K

1878. The immediate concern in the management of facial trauma should be

A. Securing a blood units to replace any loss


B. Fixation of fractures
C. Checking the breath and insure a free airways
D. Neurological consultation

1879. What is NOT A SIGN of neurological trauma

A. Excitement
B. Shock
C. Improper eye sight
D. Leaning
E. Sever headache
F. Vomiting
G. Euphonia
H. Fixed dilated pupils

1880. A young patient presented with rheumatic fever and suspected allergy to
penicillin. The antibiotic of choice is

BY DR.ABDULRAHMAN ALMUALM
MJDF MCQS WITH ANSWERS

A. Chloromycetin
B. Sulphonamide
C. Buffered penicillin
D. Erythromycin
E. Achromycin

1881. Patient under treatment with corticosteroids may develop

A. Adrenal suppression

1882. Disorder of steroid will result in

A. Adrenal suppression
B. Delayed healing
C. Osteoporosis
D. All of the above

1883. Esophagitis, herpes simplex, colitis during 5 weeks. You will find the same signs
of

A. Multiple myeloma
B. Erythema multiforme
C. AIDS

1884. What does not show in Cleidocranial dysplasia**

A. Defective formation of clavicles


B. Delayed closure of fontanelles
C. Retention of maxilla
D. Delayed eruption of permanent teeth
E. None of the above

BY DR.ABDULRAHMAN ALMUALM
MJDF MCQS WITH ANSWERS

1885. In regards to Plummer-Vincent syndrome or “Paterson and Kelly syndrome”

A. Iron deficiency is a feature


B. Atrophic oral and gastric mucosa
C. Dysphagia and angular cheilitis
D. Predisposing oral cancer
E. All of the above

1886. Steam under pressure sterilisation is the best method to kill microorganisms.
How does it work

A. Coagulation of plasma protein


B. Dehydration of DNA

1887. Patient with morphine coma; what is the medication of choice to reverse its act

A. Bradykinin
B. Epinephrine
C. Amphetamine
D. Naloxone

1888. Why are streptococci resistant to penicillin**

A. They produce penicillinase.

1889. When comparing the mesio distal length of second deciduous molar with the
length of 2nd premolar; we will find the deciduous tooth is

A. Longer
B. Shorter

BY DR.ABDULRAHMAN ALMUALM
MJDF MCQS WITH ANSWERS

C. Near the same size

1890. How do you diagnose trigeminal neuralgia MOST accurately

A. History

1891. How do you treat a child with severe Von Willebrand’s disease**

A. Like a normal child


B. Like a diabetic child
C. Like a haemophilic child

1892. The zygomatic process serves as

A. Origin of masseter muscle


B. Origin of temporalis
C. Protects parotid gland
D. Insertion of lateral pterygoid

1893. Treatment of patient with herpes simplex**

A. Symptomatic treatment and acyclovir


B. Idoxuridine

1894. Painless bluish lump filled with fluid on the lips; MOST likely is

A. Smoker’s keratosis
B. Squamous cell carcinoma
C. Mucocele
D. Fibroma

BY DR.ABDULRAHMAN ALMUALM
MJDF MCQS WITH ANSWERS

E. Fibro-epithelial polyp

1895. The diagnosis of pemphigus vulgaris is confirmed by**

A. Tzanck cells
B. Test dose of corticosteroid
C. Test of anti body
D. Histological immunofluorescence
E. Serological test for auto antibody

1896. Paget’s disease under microscope shows

A. Mosaic pattern

1897. Ameloblastoma on x-rays shows as


A. Soap bubbles

1898 Ankyloglossia is caused by

A. Edentulous ridge
B. Short lingual frenum
C. Short labial frenum

1898. What is NOT CHARACTERISTIC finding in carcinoma of the mouth

A. Elevation
B. Fixation
C. Invasion
D. Verrucoid appearance
E. Pain

BY DR.ABDULRAHMAN ALMUALM
MJDF MCQS WITH ANSWERS

1899. Blow to the mandible resulted in deviation to the left on opening; x-rays show
unilateral fracture, where would you expect the fracture**

A. Neck of the left condyle


B. Neck of the right condyle
C. Body of the left condyle
D. Body or the right condyle

1900. Marble bone disorder is**

A. Osteoporosis
B. Osteopetrosis

1901. In regards to dentinogenesis imperfecta on x-rays, What is TRUE

A. Short and blunted roots


B. The pulp canal is obliterated

C. Big pulp chamber, thin dentine and normal enamel


D. Type III, characteristic shell teeth
E. All of the above

1902. Exfoliative cytology will not help in the diagnosis of

A. Herpes simplex infection

1903. Treatment of Anaphylactic shock

A. Adrenalin 1mp IV

BY DR.ABDULRAHMAN ALMUALM
MJDF MCQS WITH ANSWERS

1904. The treatment of angioneurotic oedema

A. Anti histamine 10mg IV


B. Chlorphenamine maleate as Piriton by Allen
C. Hydrochloride 25 mg IM
D. Corticosteroid drugs or with adrenaline.

1905. Most congenitally missing teeth are

A. Mandibular 3rd molars


B. Mandibular 2nd premolars
C. Maxillary lateral incisor

1906. Which of the following is secondary to immune deficiency

A. Pseudo membrane deficiency


B. Herpes simplex
C. Squamous cell carcinoma
D. Elevated Epstein bar viruses incidence

1907. Odontogenic cyst develop from the following structures except

A. Reduced enamel epithelium of tooth crown


B. Dental lamina dura
C. Epithelium trapped after sutures
D. Hertwig’s root sheath

1908. The definition of Leeway space is

A. It is the difference in mandibular width between C, D, E and 3, 4, 5

BY DR.ABDULRAHMAN ALMUALM
MJDF MCQS WITH ANSWERS

1909. If the focal spot to film distance is increased from 20cm to 40cm, the intensity of
radiation is reduced by

A. ½
B. ¼

C. 1/3
D. 1/5

1910. Which vitamin is not produced and stored in organisms

A. Vitamin C

1911. The initial priority in treatment of horizontal fracture is

A. Preservation of pulp
B. Immobilisation
C. Root canal treatment
D. Calcium hydroxide treatment

1912. Healthy dental pulp responds to injury by

A. The formation of reparative dentine at the pulpal surface corresponding to area of


irritation

1913. In full dentures; porosity in the most thickest area is due to**

A. Gaseous porosity
B. Shrinkage porosity

BY DR.ABDULRAHMAN ALMUALM
MJDF MCQS WITH ANSWERS

1914. The most common cause of fracture at the isthmus of class II dental amalgam
restoration is

A. Delayed expansion
B. Inadequate depth at the isthmus area
C. Inadequate width at the isthmus area
D. Moisture contamination of the amalgam during placement

1915. The definition of incompetent lips is

A. Lips can not close in rest position

1916. Which drug may cause respiratory depression

A. Barbiturate

1917. What is Hutchinsonian triad

A. Combination of Hutchinson’s teeth, interstitial keratitis and nerve deafness in


children with congenital syphilis.

1918. For a 5 years old child who lives in a NON WATER FLUORIDATED are. What
is the recommended intake of fluoride

A. 0.25mg
B. 0.10mg
C. 0.50mg
D. 1.00mg

1919. Nitrous Oxide in contraindicated in

BY DR.ABDULRAHMAN ALMUALM
MJDF MCQS WITH ANSWERS

A. Heart disease
B. Asthma
C. Mental retardant
D. Sickle cell anaemia

1920. Green stain on tooth surface is due to**

A. Chromogenic bacteria

1921. Spread of infection 12, 22 is MOST LIKELY to be

A. Labial
B. Palatal

1922. Sinus tract is indication of

A. Chronic lesion

1923. The MOST common tumour of the parotid is

A. Pleomorphic oedema

1924. What does “DOUBLE BLIND” mean

A. A kind of clinical study in which neither the participants nor the person administering
treatment know which treatment any particular subject is receiving. Usually the
comparison is between an experimental drug and a placebo or standard comparison
treatment. This method is believed to achieve the most accuracy because neither the
doctor nor the patient can affect the observed results with their psychological bias.

1925. After 4 to 7 days, what type of cells you would find predominately in gingivitis

BY DR.ABDULRAHMAN ALMUALM
MJDF MCQS WITH ANSWERS

A. Leukocytes
B. Plasma cells

1926. What the age of patient who has all incisors, some premolars and some canine
erupted; note that no 2nd molars showing

A. 8 years

B. 11 years
C. 13 years
D. 14 years

1927. Patient with Class II div I malocclusion has ANB of

A. +2
B. -2
C. +8
D. -8

1928. In hairy tongue you will find

A. Elongated filiform papillae

1929. Which muscle has insertion in the pterygoid raphe**

A. Superior constrictor of the pharynx


B. Middle constrictor of the pharynx
C. Inferior constrictor of the pharynx

1930. Which micro-organisms in periapical lesion you would find microscopically

BY DR.ABDULRAHMAN ALMUALM
MJDF MCQS WITH ANSWERS

A. Aerobes
B. Aerobes to mainly anaerobes

1931. What is ESR? “erythrocyte sedimentation rate”

A. A test that measures the rate at which red blood cells settle through a column of
liquid. A non-specific index of inflammation

1932. The first thing to do when syncope occurs in apprehensive patient

A. Head should be lowered

1933. Which of the following is staphylococcal infection

A. Scarlet fever
B. Pericarditis
C. Pancreatitis
D. Carbuncle

1934. What is TRUE about Chrome-Cobalt partial denture

A. No immersion of dentures in hypochlorite

1935. Patient with eruption cyst; your treatment would be

A. Observation, mostly it bursts spontaneously

1936. The expected age of patient with rapid progressive periodontitis

A. Between 15 and 25 years of age

BY DR.ABDULRAHMAN ALMUALM
MJDF MCQS WITH ANSWERS

1937. Which of the following has proven to be the MOST important in community
preventive program

A. Dental awareness of the community


B. Institution of oral hygiene measures
C. Water fluoridation

1938. The water fluoridation is 0.5ppm; what is the recommended supplemental


fluoride concentrations for 3 year old child

A. 0.25mg
B. 0.50mg
C. 1.00mg
D. 0mg

1939. Rhomboid glossitis is

A. Candidal infection

1940. The image of x ray is too pale the MAIN cause is

A. Old expired film

1941. The beam that goes from cathode to anode is consisted of

A. Electrons

1942. In the mouth of new born baby; what sort of bacteria you expect to find

A. None

BY DR.ABDULRAHMAN ALMUALM
MJDF MCQS WITH ANSWERS

1943. The transmission of RNA into DNA called

A. Transcription

1944. How often a bitewing should be taken for children

A. Every visit routinely


B. Every year after parent’s permission

1945. To obtain the MOST accurate X rays of teeth; the tooth film distance should be
(Close/far) as anatomical restriction will permit. What is TRUE in this regard

A. The paralleling technique favours the bisecting technique.

1946. The pregnancy enlargement of gingivae is a result of

A. Hormonal disturbance

1947. Why do you give atropine in general**

A. To reduce the salivary secretion

1948. Pathogenic means

A. Pathological conditions of the disease

1949. Periodontitis is usually severe in patient with

A. Defective neutrophils

BY DR.ABDULRAHMAN ALMUALM
MJDF MCQS WITH ANSWERS

1950. Halothane anaesthetic by

A. Hepatotoxic reaction

1951. Sedation in children can be achieved by

A. Diazepam

1952. Which lymph node is involved in carcinoma of the lip: “or the first metastasis of
carcinoma of lips”

A. Submental node
B. Submandibular node

1953. Which of the following could cause the overall cellular damage to be greater

A. The specified dose delivered all at once


B. The same fatal dose given in divided smaller doses over a period of time

1954. Which of the following conditions would be considered for antibiotic prophylaxes

A. Malignancy recently removed


B. Congenital valve heart disease

C. Functional heart murmur

1955. All of the following should be considered for systemic antibiotic except

A. Extraction of tooth with acute dento alveolar abscess


B. Necrotic ulcerative gingivitis (NUG) unless it is acute.
C. Extraction of 38 or 48 with acute pericoronitis

BY DR.ABDULRAHMAN ALMUALM
MJDF MCQS WITH ANSWERS

D. Full mouth extraction for a patient with perio disease

1956. The tissue response to oral hygiene after periodontal treatment is BEST
assessed by

A. Decrease in the tendency to bleed on probing

1957. In regards to Metronidazole**

A. It is effective for the treatment of AUG/NUG

1958. The mode of act of drug may be defined as

A. How it produces its action

1959. Class III cavity is

A. Proximal cavity slightly gingival to the contact area

1960. Terminal Hinge Axis can be obtained by

A. Face bow
B. Kinematic face bow
C. Articulator

1961. Incisal colour differs from gingival colour in that the gingival part

A. Is thicker that the incisal part


B. Has dentine background

1962. Alveolar bone resorption is not seen in

BY DR.ABDULRAHMAN ALMUALM
MJDF MCQS WITH ANSWERS

A. Steven-Johnson syndrome (Erythema multiforme)

1963. The silver bromide crystals in x rays films after being expressed to radiation
forms

A. Latent image

1964. The best radiograph for maxillary sinus is

A. PA skull x ray
B. Occipitomental radiograph
C. Town’s view

1965. Fluoride in water community of 4ppm will result in

A. No mottling
B. Mottling in almost all permanent teeth except some molars
C. Mottling in permanent premolars only

1966.Hydrotherapy “Water Jet” is used to

A. Remove pellicle from tooth surface


B. Remove dental plaque
C. Causes no harm to gingiva

1967. Which of the following is not considered in the estimation of gingival index

A. Nasmyth’s membrane

BY DR.ABDULRAHMAN ALMUALM
MJDF MCQS WITH ANSWERS

1968. When examining intra orally between the side of the tongue and the lateral
border of the mandible, you expect to

A. Palpate the lymph nodes


B. Palpate the borders of the tongue

1969.Black hairy tongue is MOSTLY seen in

A. HIV patient

1970.What is TRUE about water fluoridation

A. Will have no effects after the eruption of permanent teeth

1971. When there is a fracture on condyle, the muscle responsible for elevation of
condyle is

A. Lateral pterygoid muscle


B. Medial pterygoid muscle
C. Masseter muscle

1972. Dentinogeneses imperfecta develops in

A. Initial stage
B. Proliferation stage
C. Histodifferentiation stage

D. Morphology stage

1973. Compared to dental plaster all die stones

BY DR.ABDULRAHMAN ALMUALM
MJDF MCQS WITH ANSWERS

A. Require less gauging water


B. Require more gauging water
C. Require the same quantity of gauging water
D. Are beta-hemihydrate
E. None of the above

1974. The MOST effective manner to produce a hard surface on a cast is by

A. Employ as much water as possible on mixing


B. Employ as little water as possible on mixing
C. Adding 2% of borax to the mix
D. Adding calcium tetraborate
E. None of the above

1975.When dry cast is immersed in water saturated with calcium sulphate**

A. There is contraction
B. There is negligible expansion
C. There is definite expansion
D. There is no change
E. None of the above

1976 Fusion temperature of impression compound should occur

A. Below mouth temperature


B. Above mouth temperature
C. As of the skin temperature
D. At the room temperature
E. None of the above

BY DR.ABDULRAHMAN ALMUALM
MJDF MCQS WITH ANSWERS

1977. The flow of the following percentage is allowable for impression compound (type
I) at the oral temp of 37º

A. 6%
B. 10%
C. 2%
D. 20%
E. None of the above

1978. The disadvantage of heating the impression compound in a water bath is

A. It may become brittle

B. It may become grainy


C. Lower moles with constituents are leached out
D. The plasticity of the compound may be altered
E. All of the above

1979. Generally there is ???? zinc oxide eugenol impression pastes between flow are

A. Working time
B. Accelerator
C. Setting time
D. Composition
E. None of the above

1980. Dental impression material are hydrocolloids of

A. The emulsoid type


B. The suspension type
C. The sol type

BY DR.ABDULRAHMAN ALMUALM
MJDF MCQS WITH ANSWERS

D. The get type


E. None of the above

1981. Elastomers are**

A. Hydrophilic
B. Hydrophobic
C. Water-loving impression material
D. Potassium alginates
E. None of the above

1982.The polysulfide rubber impression material are

A. Not sensitive to temperature when curing


B. Quite sensitive to temperature when curing
C. Less sensitive to temperature than silicone rubber
D. The same sensitivity to temperature as silicone rubber
E. None of the above

1983. The elastic properties of rubber impression material

A. Improves with time


B. Deteriorates with time
C. Deteriorates when exposed to temperature
D. Improves when exposed to temperature
E. None of the above

1984. The effect of temperature rise above 100ºC on heat cured denture base acrylic
resins is

A. Produces porosity on the external portion of the resin.

BY DR.ABDULRAHMAN ALMUALM
MJDF MCQS WITH ANSWERS

B. Produces porosity on the internal portion of the resin.


C. Produces porosity on the surface of the resin.
D. Prevents porosity on the interior of the resin

1985.The principle cause of failure of amalgam restoration is

A. Improperly prepared amalgam


B. Improper cavity preparation
C. Perio involvement
D. Particles of amalgam
E. None of the above

1986 Reduced occlusal area means

A. More fracture potential for amalgam


B. Less fracture potential for amalgam
C. Pulpal involvement
D. Perio involvement
E. None of the above

1987 .The less mercury remaining in condensed amalgam

A. The stronger the restoration which contains fewer matrixes alloys and fewer voids
B. The weaker the restoration is
C. The more matrixes alloys
D. The more voids
E. None of the above

1988. High copper amalgams are superior if

BY DR.ABDULRAHMAN ALMUALM
MJDF MCQS WITH ANSWERS

A. Copper is available for a secondary reaction


B. Copper is not available for a secondary reaction
C. Copper is burnished
D. Copper is fractured
E. None of the above

1989. Concerning condensation of restorative gold

A. It may vary widely vary widely and has no influence on the final restoration
B. The degassing procedure is not important
C. It is the Achilles heel of direct gold restoration
D. Clinical tech are more important than the physical properties of restorative gold
E. All of the above

1990. In regards to the enamel surface

A. It is a perfect substance for bonding


B. It does not conform to the bonding requirements
C. It is the most inorganic, rough part
D. It is free from contamination and roughness
E. None of the above

1991. Acid conditioning of enamel and eroded dentine

A. Provides an none traumatic, conservative clinical approach to the bonding of


restorative material
B. Is traumatic approach to bonding materials
C. Is not safe and simple method of bonding
D. Bonding fails to produce a highly significant retention and good marginal integrity
and clinical durability
E. None of the above

BY DR.ABDULRAHMAN ALMUALM
MJDF MCQS WITH ANSWERS

1992. The effectiveness of the acid etch is dependent on which of the following factors

A. Material must be used to clean the surface of the tooth prior to etching
B. The effectiveness of the itchant
C. The chemical and physical nature of the tooth
D. The area and surface of the enamel to be itched

1993. Creep in amalgam is the greatest in

A. Low copper lathe cut alloy

1994.The surface of enamel rods prisms in permanent teeth is

A. Perpendicular to the outer surface of the tooth


B. Parallel to the outer surface of the tooth
C. Parallel to enamel contour
D. Parallel to enamel-dentine contour

1995. All of the following are properties of fluoride except

A. Crosses the placental barrier


B. It deposits rapidly in bone
C. It is excreted rapidly by kidneys
D. It is bacteriostatic
E. It produces extrinsic tooth stain

1996. A patient indicates that he takes methyldopa (Aldomet) he is being probably


treated for

A. Hypertension

BY DR.ABDULRAHMAN ALMUALM
MJDF MCQS WITH ANSWERS

B. Angina pectoris
C. Myocardial infection

1997.A patient with history of angina suffers an attack while in the dental chair. Prompt
relief can be anticipated in MOST instances from

A. Oral administration of short acting barbiturates


B. Intra muscular administration of morphine sulphate
C. Subcutaneous administration of epinephrine
D. Sublingual administration of glyceryl bi-nitrate
E. Putting the patient in upright position

1998. Proximal caries on x rays appear

A. Smaller than clinically seen


B. Larger than clinically seen
C. The same

1999. The lamina dura is absent in which condition

A. Von Recklinghausen
B. Paget’s
C. Periapical granuloma

2000.You notice radiolucent area close to the apex of central incisor, on a second x
ray the radiolucent area moves; it is likely to be

A. Cyst
B. Abscess
C. Granuloma

BY DR.ABDULRAHMAN ALMUALM
MJDF MCQS WITH ANSWERS

D. Incisive foramen

2001Treatment of Osteoradionecrosis is

A. Antibiotic coverage
B. Conservative treatment including antibiotic coverage and resection of jaw segment.
C. Conservative treatment with sequestrectomy

2002 Widening of perio membrane can be seen in

A. Osteosarcoma
B. Scleroderma

2003. Lamina dura is actually

A. Cortical bone

B. Spongy bone
C. Immature bone
D. Cribriform plate perforated by nutrient carnally

2004. Bitewing x rays are taken to assist in the detection of caries**

A. Occlusally
B. Lingually
C. Buccally
D. Gingivally
E. Interproximally

2005. A periapical x ray of 11 and 12 region shows the vimen, floor of the nasal fossa
and the median palatine suture. The other feature that can be seen is

BY DR.ABDULRAHMAN ALMUALM
MJDF MCQS WITH ANSWERS

A. Maxillary sinus
B. Incisive foramen
C. Zygomatic process
D. Wall of maxillary sinus

2006. At the end of four years, the x rays reveal calcification of

A. All deciduous and first permanent molars


B. All permanent except of 3rd molars
C. All deciduous
D. All permanent

2007. The diagnosis of ortho cases is by

A. Measurement of cranium size


B. Recording profile
C. The relation of dentition and the jaw to the cranium
D. Determination of overbite size
E. Determination of jaw size

2008. Full mouth x ray survey at birth reveals

A. Ten teeth are present


B. Twenty teeth are present
C. Twenty four teeth are present
D. Twelve teeth are present

2009. When adhesive is used with polysulphide impression material

BY DR.ABDULRAHMAN ALMUALM
MJDF MCQS WITH ANSWERS

A. Should be thin and dry

2010. When a probe penetrate between tooth and amalgam

A. not always an indication of caries

2011 Reversible hydrocolloids impression material in comparison to alginate are

A. Better for undercuts areas

2012. You can increase the retention of Maryland bridge by

A. Incorporate mesh work in wax pattern


B. Perforation techniques in the metal cast

2013. Maryland bridges are made of

A. Nickel chrome

2014. Initiation of curing process in self cure acrylic resin is achieved by**

A. Benzyl peroxide

2015. The objective of pulp capping is to

A. Preserve vitality of coronal pulp


B. Preserve vitality of entire pulp
C. Preserve vitality of radicular pulp
D. Regenerate a degenerated and necrotic pulp
E. None of the above

BY DR.ABDULRAHMAN ALMUALM
MJDF MCQS WITH ANSWERS

2016. The objective of pulpotomy is to

A. Preserve vitality of coronal pulp


B. Preserve vitality of entire pulp
C. Preserve vitality of radicular pulp
D. Regenerate a degenerated and necrotic pulp
E. None of the above

2017. What contra indicate pulp capping

A. Accidental exposure on vital young molars


B. When inflammation of radicular pulp is already present
C. When roots are greatly curved and tortuous
D. When anterior tooth is vital and immature with wide open apices
E. None of the above

2018. Tow successive negative cultures are

A. Absolutely necessary for successful endodontic treatment


B. Not always necessary for successful endodontic treatment
C. Not questioned today as a dogmatic requirement in endodontics
D. Unquestioningly it adhered for successful endodontic treatment
E. None of the above

2019. What indicates for a periapical surgery

A. Where performing an endodontic treatment on existing root canal filling may lead
to fracture of the root
B. When root canal treatment is faulty
C. When there is danger of involving other structures

BY DR.ABDULRAHMAN ALMUALM
MJDF MCQS WITH ANSWERS

D. When the bony defect is so extensive that the edges of the incisors will collapse
E. None of the above

2020. In regards to external resorption**

A. Continues after successful endo treatment


B. Stops in most cases following successful endodontic treatment
C. Continues only in mandibular incisors after successful endo treatment
D. Stops in maxillary lateral incisors after successful endodontic treatment
E. None of the above

2021. The concomitant perio-periapical lesion as the cause of endodontic failure

A. Cannot be discovered prior to endo treatment


B. May be discovered prior to endo treatment
C. Is most commonly found in maxillary teeth
D. Is most commonly found in mandibular teeth
E. None of the above

2022. X rays are used in endodontic treatment to

A. Aid in the diagnosis of periapical hard tissue lesion


B. Determine the number, location, shape, size and direction of roots and root canals
C. Confirm the length of root canals
D. Evaluate the adequacy of the complete root canal filling
E. All of the above

2023. To achieve optimum cavity preparation which of the following factors of internal
anatomy must be considered

A. Outline form

BY DR.ABDULRAHMAN ALMUALM
MJDF MCQS WITH ANSWERS

B. The age and shape of pulp chamber; in addition to the direction of individual root
canals.
C. Internal external relationship
D. Intra-coronal preparation
E. None of the above

2024. Irrigation in root canal treatment, should be undertaken at frequent intervals


during instrumentation to**

A. Removes cementum falling from the canal


B. Remove noxious material since it may be forced to the apical foramen resulting in
periapical infection
C. Destroy all micro organism in the canal
D. Stop instruments from going beyond the apical foramen
E. None of the above

2025. The length of the tooth is established by

A. Good undistorted pre-operative x ray


B. Adequate coronal access to all canals
C. Adjustable endo millimetre ruler
D. Definite repeatable plane of reference to anatomical landmark on tooth
E. All of the above

2026. Which of the following Gold casting alloys are available

A. Medium alloy “Type II”


B. Hard alloy “Type III”
C. Extra Hard alloy “Type IV”
D. All of the above

BY DR.ABDULRAHMAN ALMUALM
MJDF MCQS WITH ANSWERS

2027. Which of the following varieties should be made in the proximal occlusal cavity
preparation in deciduous teeth compared to permanent ones

A. The occlusal isthmus should be proportionally wider


B. The occlusal lingual walls need not to be extended to self cleansing areas
C. It is not necessary to include fissures in the occlusal outline
D. The lingual angle should be sharper
E. The axio pulpal line angle should not be bevelled

2028 10 years old boy looses permanent mandibular molar; what is affected

A. Teeth adjacent to extracted teeth


B. Teeth on both arches on same side
C. The remaining teeth in the mouth
D. Teeth directly opposite to the extracted tooth

E. Teeth on the same quadrant

2029. MOST common consequence arising from premature extraction of deciduous


molars is

A. Loss of arch length


B. Loss of speech sound
C. Loss of facial contour
D. Loss of vertical height
E. Loss of free way space

2030. During teeth eruption, the reduced enamel epithelium merges with the oral
epithelium and consequently….????

BY DR.ABDULRAHMAN ALMUALM
MJDF MCQS WITH ANSWERS

A. Down growth of oral epithelium which replaces the reduced enamel epithelium
B. Proliferation of inner enamel epithelium
C. Proliferation of outer enamel epithelium
D. Down growth of oral epithelium which undermines the reduced enamel epithelium
E. Gradual transformation of the reduced enamel epithelium

2031. Essential for the diagnosis and treatment plan of orthodontics is

A. Classifications
B. X rays
C. Plaster models

2032. As a general practitioner; you decide at an initial appointment that you can not
handle a child due to lack of co- operation. Which of the following approaches would
seem to be your alternative

A. Refer child to pedodontist


B. Send child home until he/she has to co-operate
C. Schedule child for General Anaesthetic session

2033. The most common reason to refer a child to a pedodontist is problem with

A. Rampant caries
B. Behaviour management
C. Endodontic treatments in primary teeth
D. Space maintainers

2034. A very quick and wide separation of teeth causes

A. Gingival inflammation

B. Vasodilation

BY DR.ABDULRAHMAN ALMUALM
MJDF MCQS WITH ANSWERS

C. Wider spaces
D. Necrosis of bone

2035. A patient who has lost several teeth in an otherwise healthy mouth, can get

A. TMJ dysfunction
B. Changes in the vertical dimension
C. Change in the interocclusal dimension

2036. The final deposition of ameloblast occurs**

A. Primary enamel cuticle


B. Secondary enamel cuticle
C. Acquired enamel cuticle
D. Cementum

2037. Oral epithelium fused with reduced epithelium to form

A. Junctional enamel epithelium

2038. The increase of mandible about year 5 and 6 is mainly at

A. Depth
B. Width
C. Length

2039. 8 years child has a badly broken deciduous molar what is the best material to
restore it

A. Amalgam
B. Gold

BY DR.ABDULRAHMAN ALMUALM
MJDF MCQS WITH ANSWERS

C. Composite
D. GIC

2040. When tooth is twisted along its long axis; it is called

A. Mesio version
B. Disto version
C. Lingo version
D. Torsion

2041. A full x rays is recommended in children by age of**

A. 2 years- first visit


B. 2 years for uncooperative kids
C. 3-5 years

2042. The percentage of malocclusion after early loss of deciduous teeth is

A. 60%

2043. A tooth is expected to erupt when root development is

A. ¾ of its full development

2044. Crowding of anterior permanent teeth is directly affected by

A. Premature loss of deciduous molars

2045. Hawley appliances are used**

BY DR.ABDULRAHMAN ALMUALM
MJDF MCQS WITH ANSWERS

A. To close midline diastema


B. Maintain the normal relationship of the adjacent teeth until the canine erupts

2046. Helical spring is used to

A. Ectopically erupting permanent molars

2047. The function of varnish

A. To reduce initial marginal leakage “Short-term leakage”


B. To prevent long term leakage

2048. Turner’s tooth is**

A. Due to infection of primary tooth

2049. The outcome of rapid wax burn out is

A. Cracking of the investment


B. Back pressure porosity

2050. The MAIN purpose of burnishing is

A. To help eliminating excess mercury


B. To condense margins
C. Polishing of filling

2051. What happen to etched composite after settings

A. Expand

BY DR.ABDULRAHMAN ALMUALM
MJDF MCQS WITH ANSWERS

B. Contract
C. Contract and expand
D. Expand and contract

2052. Which of the following muscles elevates the lower lip

A. Orbicularis oris

2053. The MOST common cause for midline fracture is

A. Impact
B. Fatigue

2054. The function of face bow is

A. Orient maxilla to TMJ

2055. Hypoplasia as seen in x rays**

A. Thick enamel surface


B. Thin enamel surface
C. Sometimes large pulp chamber
D. Can not be detected on X rays

2056. Function of matrix band

A. Substitute for the missing wall so adequate condensation forces can be applied
B. Permit re-establishment of proper contact lines
C. Restrict extrusion of amalgam and prevent formation of an “overhang”

BY DR.ABDULRAHMAN ALMUALM
MJDF MCQS WITH ANSWERS

D. Provide adequate physiological contour for the proximal surface


E. Provide an acceptable surface texture to the proximal surface
F. All of the above

2057. Which composite is used in load bearing areas**

A. Hybrid composite

2058. The palatal canal of maxillary molars is found Under

A. Disto lingual cusp


B. Mesio lingual cusp

2059. Obturator in cleft palate plate is maintained by

A. Cohesion
B. Atmospheric pressure
C. Retention in the defect
D. Patient support it with the tongue

2060. In a fixed moveable bridge where should the moveable connectors “non rigid”
be placed

A. Distal to anterior retainers


B. Mesial to posterior retainers

2061. What do expect after successful pulpectomy in the periapical area

A. Apical foramen is closed by cementum calcified tissues

2062. Frenum is consisted of what kind of tissues

BY DR.ABDULRAHMAN ALMUALM
MJDF MCQS WITH ANSWERS

A. A fold of mucous membrane

2063. What is the minimal labial reduction for porcelain metal crowns

A. 1mm
B. 1.5mm
C. 0.5mm

2064. What is the function of flux

A. To protect alloy from oxidation, and distribute metallic oxides as they are formed

2065. What is TRUE

A. Boiling point of acrylic > boiling point of water


B. Boiling point of acrylic is similar to that of water
C. Boiling point of acrylic < boiling point of water

2066. Porcelain bonded to metal is strongest

A. In compression
B. In tension

2067. What is the MOST adverse reaction to lignocaine

A. Drug interaction with patient’s medicines


B. Injecting into vein
C. Hypersensitivity
D. Toxicity

BY DR.ABDULRAHMAN ALMUALM
MJDF MCQS WITH ANSWERS

2068. A 29 year old lady presents with mandibular second molar associated with
radiolucency of 1 cm diameter and paraesthesia of mental nerve. There is no other
symptoms

A. Extraction and curettage


B. Root canal treatment and antibiotics
C. Blood test, extraction and biopsy

D. Extract and pack with white head’s varnish

2069. Periodontitis is a disease that has

A. A slow progression
B. Rapid progression
C. Cyclic or burst progression “active and inactive phases”
D. Intermittent progress

2070. In regards to apically displaced flap; which is TRUE

A. Does not preserve attached gingivae


B. Does not lengthen crown of tooth
C. Is a pocket elimination procedure
D. A & C

2071. Which is NOT TRUE about occlusal trauma

A. Cemental tears
B. Bone loss
C. Mobility
D. True pocket formation

BY DR.ABDULRAHMAN ALMUALM
MJDF MCQS WITH ANSWERS

E. Bleeding in periodontal ligament

2072. Which is the MOST significant clinical feature of periodontal disease**

A. Bleeding
B. True pocket formation and apical migration of attached gingiva

2073. Hypodontia can be seen in

A. Cleidocranial dysplasia “dysostosis”


B. Down’s syndrome
C. Papillon le fever syndrome
D. Rickets

2074. Hyperdontia can be seen in

A. Down’s syndrome
B. Cleidocranial dysplasia “dysostosis”

2075. Which of the following does not carry a risk of infection from hepatitis B patient

A. HBs Ag antigens
B. HBs Ag
C. HBe Ag

2076. Which is the MOST conservative treatment for periodontal disease

A. Oral hygiene, sub-gingival debridement, regular review and maintenance


B. Surgery, sub-gingival debridement, regular review and maintenance

BY DR.ABDULRAHMAN ALMUALM
MJDF MCQS WITH ANSWERS

C. Oral hygiene, sub-gingival debridement

2077. Filter is used in x ray machine to

A. Reduce exposure time


B. Removes low energy x rays

2078. A patient 8 years old has 3 of first premolars erupted with swelling on the ridge
of the un-erupted premolar. X ray shows a fully developed crown and ¾ roots
development with no other pathology. What is your management

A. Remove the dentigerous cyst


B. Soft tissues recision to allow eruption
C. Soft tissues recision accompanied with orthodontic appliance to help with eruption

2079. Loss of the first deciduous molar in 10 years old child required

A. Band and loop to maintain space


B. Evaluate the case radiographically and then decide whether space maintainer is
needed or not
C. No treatment

2080. Palatal root displaced into the antrum while extracting; what is your decision to
retrieve it

A. Through the alveolar


B. Surgical opening of canine fossa
C. Nasal antrostomy

2081. Which one of the following is expansile lesion of jaw bone**

A. Odontogenic keratocyte

BY DR.ABDULRAHMAN ALMUALM
MJDF MCQS WITH ANSWERS

B. Central haemangioma
C. Radicular cyst
D. Osteomyelitis

2082. The MOST frequent retained deciduous teeth in permanent dentition are

A. Upper lateral incisors


B. Upper central incisors
C. Lower central incisors
D. Second lower molars
E. Second upper molars

2083. The MOST frequently synthesized substance by Streptococcus mutans is

A. Liven
B. Fructose
C. Glucan
D. Glycogen

2084. Benzodiazepine and diazepam in 5-10mg oral dose used for oral sedation in
dentistry DOES NOT give

A. A good analgesic effect if given 1 hour prior to dental sessions


B. Would be reversed by flumazepil because it is a Benzodiazepam
C. Post operative headache
D. There is a profound amnesic action and no side affects

2085. Formcresol fixation is used in deciduous dentition in

A. Necrotic pulp
B. Carious exposure

BY DR.ABDULRAHMAN ALMUALM
MJDF MCQS WITH ANSWERS

C. Mechanical exposure
D. Periapical disease

2086. After you have successfully treated an Angle’s Class II division I malocclusion.
The ideal Class I incisor relationship has been produced and 14, 24 were extracted.
The arches are now well aligned. What molar occlusion will there be at the end of
treatment when all spaces are closed

A. Full unit Class II


B. ½ unit class II
C. Class I
D. ½ unit Class III
E. Full unit Class III

2087. The tensile nodes are located at

A. The mandible angle


B. The jugular-digastric interaction
C. Mylohyoideus intersection
D. Internal carotid level

2088. Herpangina is the MOST reliable diagnosis is by**

A. Immunofluorescence
B. Microscopy
C. Serology

2089. Metallic Plato backing the intra oral films are for**

A. Reduces the flexibility of films


B. Reduces patient exposure to x rays

BY DR.ABDULRAHMAN ALMUALM
MJDF MCQS WITH ANSWERS

C. Increases the bending capacity of films

2090. The fixing time for dental x ray should be

A. 5 minutes at 20ºC
B. At least 10 minutes
C. Until it clears up
D. 2 minutes at 40ºC

2091. The developing time for dental x ray should be

A. 5 minutes at 20ºC
B. At least 10 minutes
C. Until it clears up
D. 2 minutes at 40ºC

2092. What is the range of the visible light cure beam

A. 100-120 nm
B. 200-300 nm
C. 400-430 nm
D. 470 nm or 450-500 nm

2093. When is LEAST required gingival groove

A. When restoring with GIC for abrasion


B. When restoring with GIC for root caries
C. When restoring with GIC base and composite lamination
D. When restoring with amalgam

BY DR.ABDULRAHMAN ALMUALM
MJDF MCQS WITH ANSWERS

2094. Corrosion and discolouration of amalgam restorations is usually caused by

A. Sulphur oxides
B. Oxygen
C. Chlorides
D. Over trituration

2095. When you find ditching in an amalgam filing you would

A. Replace the defective filing


B. Repair defect with unfilled resins

2096. What is the reason that pulp calcified after trauma

A. The intensity of the blow was too low to cause pulp death

2097.Which is TRUE in regards to the preparation of occlusal rests

A. Use an inverted cone bur


B. Use a flat fissure bur
C. Parallel to occlusal plane
D. At right angle to the long axis of tooth
E. None of the above

2098. Patient presents with fever of 39ºC, pain, swelling of upper lip and nose.
Radiograph shows an enlargement of periodontal ligament space of 11 which has a
large restoration without a base. What would your treatment be

A. Recision and antibiotic


B. Antibiotic, analgesic followed by root canal treatment after remission of acute
phase.

BY DR.ABDULRAHMAN ALMUALM
MJDF MCQS WITH ANSWERS

C. Complete debridement of root canal, analgesic and antibiotic


D. Remove restoration, apply a sedative dressing with corticosteroids

2099. Cervical finish line of full veneer crown preparation should be placed

A. Just supragingival whenever is possible


B. According to the depth of gingival crevice
C. Subgingival to reduce ability of recurrent caries
D. At the junction of tooth and amalgam core

2100. Why do we itch enamel for composite restorations

A. To increase surface area


B. To decrease surface area
C. Does not really change the surface area
D. Increase the chemical bonding capability
E. Decrease the chemical bonding capability

2101. All of the following are requirements of mucoperiosteal flap except of

A. Base is wider than free margin


B. Mucous membrane carefully separated from periosteum

C. Base has an adequate blood supply


D. Flap wider than bony defect that will be present at conclusion of operation
E. Mucoperiosteum is carefully separated from bone

2102 Which of the following will NOT be used in determination of vertical dimension

A. Aesthetic

BY DR.ABDULRAHMAN ALMUALM
MJDF MCQS WITH ANSWERS

B. Phonetics
C. Gothic arch tracing
D. Swallowing

2103. Zinc oxide impression material

A. May cause irritation to mucosa


B. Is a thermoplastic material

2104. The adhering of tissues on the surgical electrode usually means

A. Current intensity is too high


B. Current intensity is too low
C. Dispersion plate not applied to patient
D. None of the above

2105. How do remove the smear layer in root canal treatment

A. Use of 0.5% hypochlorite sodium


B. Hedstrom file
C. EDTA

2106. Why do you over pack amalgam

A. To ensure excess mercury reaches the surface

2107. A lateral incisor labial to the arch needs to be restored in normal alignment with
PFM retraction. How will the tooth appear

A. Too wide
B. Too short

BY DR.ABDULRAHMAN ALMUALM
MJDF MCQS WITH ANSWERS

C. To narrow
D. To long

2108. Which of the following is more prone to crack

A. Buccal of lower molars


B. Lingual of lower molars

C. Lingual of upper molars


D. Buccal of upper molars

2109. Flexibility of the retentive clasp arm depends on

A. Length
B. Cross section
C. Material
D. Degree of taper
E. All of the above

2110. In vital pulp therapy; what is the optimum depth for a pin hole in a tooth

A. 4-5mm
B. Approximately 2mm
C. Less than 2mm
D. 1-1.5mm

2111. Proximal cavosurface walls in Class II preparation for the reception of an


amalgam. Should be finished at which angle to external surface

A. Acute angle
B. Right angle

BY DR.ABDULRAHMAN ALMUALM
MJDF MCQS WITH ANSWERS

C. Obtuse angle
D. 45 angle

2112. Why are three tripod marked on a cast being surveyed

A. To orient cast to articulator


B. To orient cast to surveyor
C. To provide guide planes

2113. An irregular shaped void on surface of a gold cast would indicate that

A. A fragment of investment had been carried into the mould


B. Air carried into mould
C. Burning out of wax was inadequate
D. The powder/water ratio for the investment was too high

2114. The MAJOR disadvantage of Gutta Percha is

A. Soluble in chloroform
B. Too weak for narrow canals

2115. What effect do fissure sealants have on caries progression

A. Reduces new caries and hamper the progress of freshly established caries
B. Reduces new caries and hamper the progress of existing caries

2116. In regards to connectors on dentures; which of the following is correct

A. Major connector should be rigid as possible


B. Minor connector should engage undercuts

BY DR.ABDULRAHMAN ALMUALM
MJDF MCQS WITH ANSWERS

2117. What control tooth or teeth should be used when testing a suspected pulpally
involved tooth

A. Adjacent tooth and contralateral teeth


B. Contralateral and opposing teeth
C. Opposing and adjacent teeth
D. Test only suspected tooth/teeth
E. All of the above

2118. What is CORRECT in regards to periodontal surface area in mandibular teeth

A. First molar> first premolar> second premolar


B. Canine> first premolar> second premolar
C. Canine> lateral incisor> central incisor

2119. Cast crown fits on die but not on tooth; discrepancy is about 0.3mm what would
you do

A. Relieve cast from the inside


B. Take a new impression and make new crown
C. Burnish margins
D. Use thick mix of cement

2120 In regards to marginal leakage in amalgam

A. The wider the gap the better the chance of secondary caries
B. Seal the margin with fissure sealant would prevent further breakdown
C. Secondary caries may develop

2121 Pit and fissure caries start at

BY DR.ABDULRAHMAN ALMUALM
MJDF MCQS WITH ANSWERS

A. bottom of the fissure


B. walls of the fissure

2122. What interferes with maxillary denture in posterior vestibular fold

A. Coronoid process
B. Condyle
C. Masseter muscle

2123. In regards to shade; Chroma is

A. Brightness
B. Saturation of hue
C. Value

2124. Acrylic self-cure special trays; how long should have been made prior taking
impression

A. 12 hrs
B. Immediately after fabricating it
C. After been left in water for an hour
D. Wait for an hour before pouring

2125. The MAIN CAUSE of gingivitis in partial dentures patients is

A. Placement of dentures
B. Plaque accumulation

2126. Movable component of the non-rigid connector in a fixed bridge is placed. Which
of the following is TRUE

BY DR.ABDULRAHMAN ALMUALM
MJDF MCQS WITH ANSWERS

A. Should be placed on the longer retainer


B. Mesial drift causes unseating of the distally placed connector

2127. When lateral incisor is lost; patient has Class II Division II type with deep bite.
Which of the following is contra indicated

A. Fixed bridge with canine and central incisor as abutment


B. Non-rigid connector with central incisor as abutment

2128. Which is the neutral zone

A. The zone where displacing forces are neutral


B. The zone where buccal and lingual forces are balanced

2129. What is the Bilaminar Zone

A. Formed of, or having, two laminae, or thin plates. Which is the distal attachment of
superior hard lateral plate

2130. Which of the following DOES NOT cause depression of the mandible

A. Contraction of lateral pterygoid


B. Contraction of temporalis
C. Contraction of the suprahyoid muscles
D. Contraction of the infrahyoid muscles
E. Relaxation of all muscles so that the only forces on the mandible are the forces
against the gravity

2131. Which of the following is the MOST appropriate related to hardness

BY DR.ABDULRAHMAN ALMUALM
MJDF MCQS WITH ANSWERS

A. Tungsten carbide>Porcelain>Human enamel>acrylic


B. Porcelain>Enamel>Tungsten carbide>amalgam>acrylic
C. Porcelain>Enamel>Tungsten Carbide>Amalgam>Acrylic

2132. How much would you reduce a cusp to be replaced with amalgam onlay
A. 2 mm to achieve a good retention form
B. 2mm to achieve a good resistance form
C. 1mm

2133. How long it would take to notice significant reduction in radiolucency after
finishing a root filing for a tooth with a periapical lesion

A. 6 months
B. 1 month
C. 3 months

2134. The major cause of mentalis muscle hyperactivity is

A. Class II Division I
B. Tongue thrust

3135. When treating a tooth with a non-vital pulp with a fistula presented; fistula should
be treated by

A. Surgical incision
B. Antibiotic coverage
C. The usual root canal procedures for non-vital teeth and no special procedures for
fistula

2136. To increase the setting time of phosphate cements you would

BY DR.ABDULRAHMAN ALMUALM
MJDF MCQS WITH ANSWERS

A. Use a cold glass slab

2137. 27 years old female; shows sudden oedematous rash and collapses after an
injection of barbiturates. Your management is

A. I.M. o.5ml of 1:1000 adrenaline with oxygen administration

2138. Which of the following procedures will not achieve sterilization

A. Hot air at 160ºC “320ºF” for 90mins


B. Boiling water at 100ºC “210ºF” for 2 hours
C. Autoclave at 121ºC “250ºF” under 15psi for 20 mins
D. Dry heat at 177ºC “350ºF” for 60mins
E. All of the above will achieve sterilisation

2139. 50 years old man presented after a full mouth extraction complaining that he
“bled all night”. Which of the following pre existing conditions could be responsible for
the post operative bleeding

A. Blood pressure reading of 180/110


B. Gastric ulcer
C. Elevated prothrombin time
D. A & D are correct
E. None of the above

2140. Long bone growth by

A. Mitosis in osteoblast
B. Mitosis of osteoblast
C. Appositional growth in cartilage epiphysis
D. Interstitial growth in cartilage epiphysis

BY DR.ABDULRAHMAN ALMUALM
MJDF MCQS WITH ANSWERS

2141. What is TRUE in regards to oral lesions of reticular lichen planus

A. Never accompanied with skin lesions


B. Always accompanied with skin lesions
C. Lesions may present anywhere
D. Lesions may present on legs
E. Lesions may present on arms

2142. Bone graft method that has shown the greatest osteogenetic potential is

A. Lymphocytic bone graft


B. Freeze-dried bone graft
C. Heltozygo?? Marrow graft
D. Cortical bone graft

E. Cancellous bone graft

2143. A patient states that for ALMOST a year now, she has had a rubbery, firm,
painless nodule within the substance of parotid gland. This MOST likely is

A. Mucocele
B. Lymph node
C. Benign mixed tumour
D. Squamous cell carcinoma
E. Sialolith with encapsulations

2144. The best method to radiate a specific area of the head is

A. Use lead collimator

BY DR.ABDULRAHMAN ALMUALM
MJDF MCQS WITH ANSWERS

2145. The best reading on radiograph to diagnose ankylosis in deciduous molar is

A. Density of lamina dura

2146. Which is NOT CHARCTERISTIC of dentinogenesis imperfecta

A. Dentinal tubules are more than usual

2147. Child with less than normal number of teeth, mandibular lateral incisor is larger
than usual; on x rays it shows with two roots and two roots canals; your diagnosis is**

A. Dilaceration
B. Gemination
C. Fusion
D. Concrescence
E. Taurodontism

2148. The MOST stable area to evaluate the craniofacial growth is

A. Nasal floor
B. Cranial vault
C. Occlusal plane
D. Naso maxillary complex
E. Anterior cranial base

2149. 7 years child with Class I malocclusion, slight version of maxillary Class I;
adequate arch length. What is your management

A. Oral screen
B. Head cap therapy

BY DR.ABDULRAHMAN ALMUALM
MJDF MCQS WITH ANSWERS

C. Inclined plane on mandibular anterior teeth


D. Hawley plate
E. Expansion screw plate

2150. Tongue thrust with tongue to lip swallow is seen in

A. Incompetent lips

2151. Clinical indications of pathogenic chronic periodontitis

A. Mobility
B. Dull pain on closing
C. Presence of true pocket
D. Apical migration of gingival epithelium
E. Presence of subgingival calculus
F. C, D & E

2152. To prevent exposure of a ?????? on a permanent root; the dentist BEST


approach for elevating flap is to use

A. Double flap
B. Stripping procedure
C. Full thickness flap
D. Apically positioned flap
E. Split thickness flap

2153. What is TRUE in regards to primary occlusal trauma

A. Mobility caused by excessive forces on normal periodontal ligament

2154. In advance periodontitis with marked mobility; teeth should be splinted

BY DR.ABDULRAHMAN ALMUALM
MJDF MCQS WITH ANSWERS

A. To improve comfort and function

2155. Examination reveals area of gingival recession, exposed wide area of dental
roots. Which is the procedure of choice to obtain coverage of the root surface

A. Free gingival autograft


B. Sub-epithelial tissue graft
C. Apically positioned graft
D. Free gingival graft
E. Modified wide flap

2156. What does contra indicate distal wedge in molars’ area

A. Distal fluting
B. Long attached gingiva

C. Sharply ascending ramus that limits space distal to molars


D. Supra bony pockets distal to molars

2157. Which of the following is not a property of Fluoride ion

A. Crosses placental barrier


B. Deposits in bone
C. Excretes rapidly by kidney
D. Bacteria static
E. Produces extrinsic tooth stain

2158. Two conditions of enamel facilitate post eruptive uptake of fluoride element

A. Hyper mineralisation and surface dentine

BY DR.ABDULRAHMAN ALMUALM
MJDF MCQS WITH ANSWERS

B. Surface demineralisation and hypo mineralisation


C. Dental fluorosis and enamel opacities

2159. In regards to topically applied fluoride

A. Effective in incorporated into dental plaque


B. Inhibits acid demineralisation of enamel

2160 Flexibility of the retentive clasp arm Does not relate to

A. Length
B. Cross section
C. Material
D. Degree of taper
E. Under cut area
Ans e

2161. Protrusive movement in wax

A. Can not be perforated

2162. LEAST use of blood count

A. Infectious mononuclears

2163. The FIRST advantage of using 100000 rpm and over rotors is

A. Less vibration on patient

2164. How long it would take to see the dentinal bridge after direct pulp capping by
using Calcium hydroxide**

BY DR.ABDULRAHMAN ALMUALM
MJDF MCQS WITH ANSWERS

A. 6-8 weeks
B. 4 weeks

C. 6-8 months
D. 4 months

2165. The OPTIMUM crown to root ratio for abutment tooth is**

A. 2:3
B. 1:1

2166. What does contraindicate bridge works

A. Long edentulous span which will lead to damage of abutments

2167. Elasticity of impression material will lead to ideally

A. Prevents distortion when impression is removed out of the mouth

2168. Patient has been coming to your clinic for several times complaining about
soreness under the denture; what would you do

A. Check occlusion of lower buccal cusps

2169. What is the difference between arcon and non arcon articulator

A. In arcon the condylar element is in the lower compartment

2170. Purplish lesions on the buccal mucosa that have been there since birth; the
diagnosis is

BY DR.ABDULRAHMAN ALMUALM
MJDF MCQS WITH ANSWERS

A. Haemangioma

2171. Elevators are not used in

A. Dividing third lower molars roots

2172. What is contraindicated to the use of calcium hydroxide for pulp capping

A. Accidental exposure of pulp


B. Carious exposure of pulp in otherwise asymptomatic tooth
C. Carious exposure of pulp in tooth that has been painful for weeks

2173. How would you treat hyperaemia “hyperaemic tooth”**

A. Zinc Oxide and eugenol cement


B. Calcium hydroxide
C. Corticosteroid paste

2174. Patient comes to you complaining of pain in a tooth, the tooth is filled with
composite long time ago; what would you do

A. X ray, remove filling and restore with temporarily filling

2175. Use of inhalation general anaesthesia

A. Halothane should not be less than 5%


B. Oxygen must not be less than 30%

2176. Bilateral symmetrical swelling of the mandible of a child is likely to be caused


by

BY DR.ABDULRAHMAN ALMUALM
MJDF MCQS WITH ANSWERS

A. Acromegaly
B. Paget’s disease
C. Giant cell lesion
D. Primordial cysts
E. Dental cysts

2177. For fissure and sealant treatment to be a part of the primarily retentive care

A. Place sealant on teeth which are at high risk of caries


B. Place sealant on newly erupted teeth

2178. Periodontal pocket is measured between

A. CEJ to base of pocket


B. Top of the gingiva to the base

2179. When you apply a pressure of 0.25N to measure pocket depth

A. 4 mm indicates periodontitis

2180. Which of the following elements is not found in normal periodontal membrane

A. Fibroblast
B. Epithelial cells
C. Erythrocytes
D. Vest cells of malaise
E. Inflammatory plasma cells and lymphocytes

2181. Which of the following situations make periodontal disease more sever

BY DR.ABDULRAHMAN ALMUALM
MJDF MCQS WITH ANSWERS

A. Enough proximal surface


B. Too wide bucco lingual embrasure

C. Missing proximal contacts

2182. The auxiliary occlusal rest on tooth for partial denture should be placed

A. Away from edentulous space


B. Adjacent to edentulous space
C. Near fulcrum line
D. Away from fulcrum line

2183 A vital tooth has a crown cemented to a pin retained amalgam cored; where does
failure occur

A. Between crown and cement


B. Between core and cement
C. In the crown and the root
D. In the core and the margin preparation

2184. Which is NOT a result of toxic dosage of local anaesthetic

A. Angioneurotic oedema
B. Hypotension
C. Respiratory depression
D. Hypertension
Ans D
2185. Swallowing will aid in the diagnosis of

A. Branchial cyst

BY DR.ABDULRAHMAN ALMUALM
MJDF MCQS WITH ANSWERS

B. Thyroglossal duct cyst


C. Ranula
D. Retention cyst
E. Globulomaxillary cyst
Ans B

2186. What is not true about tobacco smoking

A. Redox potential favours growth of anaerobic bacteria


B. It is caries immuno-suppressive
C. It is adrenergic
D. Affects neutrophils and chemotactic factors
Ans

2187. On X ray you found the Gutta Percha cone extending 1mm beyond the apex
without any symptoms; what would you do

A. Remove restoration material until you are able to withdraw the Gutta Percha cone
B. Apiectomy
C. Leave as is until any complications occur

2188. On X ray you found the cement of previous root canal treatment is extending
1mm beyond the apex without any symptoms; what would you do

A. Remove restoration material and retreat


B. Apiectomy
C. Leave as is until any complications occur

2189. What is the main purpose of using corticosteroids in pulpal obturation material

A. For their antibiotic action

BY DR.ABDULRAHMAN ALMUALM
MJDF MCQS WITH ANSWERS

B. For their antiinflammatory action


C. To relief pulp pressure

2190. What nerve supplies upper first molars

A. Posterior and mid superior alveolar nerve:

2191. The roughest surface on cut tooth structure

A. Cross cut fissures at ultra speed

2192. The main of damaged gingival tissues after placing rubber dam is

A. The distance between holes is big


B. The distance between holes is small
C. The punctured holes are too big in size
D. Not using lubricant when placing rubber dams

2193. The advantage of using the lingual plate on lingual bar is

A. It acts as indirect retention

2194. Retention in precision attachment is achieved by

A. Frictional resistance

2195. How much under cut area a clasp arm should engage**

A. As much under the undercut as possible


B. Anywhere beyond the survey line

BY DR.ABDULRAHMAN ALMUALM
MJDF MCQS WITH ANSWERS

C. A predetermined amount of undercut

2196. What is characteristic of fibrotic gingivitis

A. Is phenytoin induced gingivitis and only seen on intra lateral papilla


B. Can only be treated surgically

2197. Bone is characterised by**

A. Haversian canal around bony canals


B. Irregularly arrayed tabullae??

2198. Why we do not use porcelain in long span bridge works

A. Because of the high casting shrinkage of porcelain

2199. You have patient with Class II division 2; which of the following is contraindicated

A. Cantilever bridge
B. Maryland bridge

2200. How will cover buccal bicuspid for lower premolar when making a metallic
porcelain crown

A. Cover the occlusal and buccal cuspid by porcelain


B. Cover just buccal cuspid by porcelain
Ans b

2201. What is the main cause of bilateral cheilosis

BY DR.ABDULRAHMAN ALMUALM
MJDF MCQS WITH ANSWERS

A. Short vertical dimension


B. Vitamin B deficiency
Ans B/ if it was on denture wearer it will be A

2202. What sort of alloys do you use for bridges

A. Ductile
B. Hard
C. High sensitivity
Ans A

2203. What sort of material do you use for the fabrication of Maryland bridges

A. Single phase materials


B. Multi phase materials
C. Extra hard
D. The same as bonding martial
Ans C

2204. When the neck of the condyle is fractured; what muscles determine the
movement of the superior segment

A. Lateral pterygoid
B. Medial pterygoid
C. Temporalis
D. Mylohyoid
Ans A

2205. Patient with prosthetic heart valves, with INR value of 3.0; requires surgery,
what is the your management

BY DR.ABDULRAHMAN ALMUALM
MJDF MCQS WITH ANSWERS

A. Give Amoxicillin or Vancomycin and suture carefully


B. Stop warfarin, start heparin, carefully suture and give Amoxicillin or Vancomycin
C. Stop warfarin, carefully suture and give Amoxicillin or Vancomycin
Ans A

2206. Chronic oral antral fistula for some time after the extraction of maxillary first
molar. What is your management

A. Surgical closure
B. Anti-biotic and nasal decongestant
C. Wash the antrum
Ans B

2207. Pigmented naevus can undergo malignant

A. Always
B. Never
C. 10 to 15%
Ans C
2208. The MOST common sites for squamous carcinoma in the oral cavity are**

A. Palate and gingivae


B. Tongue and floor of the mouth
C. Tongue and palate
Ana B
MOST common sites are, lateral tongue, floor of the mouth, lower vermilion
and/or lip border and the alveolar ridge respectively.

2209. A patient has painful lesions on her buccal mucosa. Biopsy report shows
acantholysis and supra basilare; your diagnosis is

BY DR.ABDULRAHMAN ALMUALM
MJDF MCQS WITH ANSWERS

A. Pemphigus vulgaris
B. Bulla lichen planus
C. Erythema multiform
D. Systemic lupus erythematosus
Ans A

2210. Oral mucosal pigmentation; what is TRUE

A. Commonly seen in ethnic groups


B. Commonly an amalgam tattoo
C. Commonly oral melanoma
D. Commonly melanotic naevus
Ans A

2211. What I TRUE in regards to osteogenesis imperfecta

A. Manifests with blue sclera


B. May be associated with deafness
C. Sex linked disorder of bones that develop in cartilage (ans is B)

2212. Increase which of the following will decrease density of radiograph

A. Milliampere
B. Time
C. KvP Kilovoltage
D. Object-film distance
E. Focal spot-object distance
Ans E

2213. Which of the following will increase sharpness

BY DR.ABDULRAHMAN ALMUALM
MJDF MCQS WITH ANSWERS

A. Larger focal spot


B. Smaller focal spot
C. Increase object-film distance
Ans B

2214. The MOST common staphylococcal infections is

A. A localised purulent infection of the skin


B. Diffuse purulent infection of the skin
C. Staphylococcal osteomyelitis
D. Impetigo
Ans A
2215. What is TRUE in regards to Basal Cell Carcinoma

A. Metastases is common
B. Erodes bone
C. More common in oriental races
D. Cannot occur in oral mucosa according to definition (ans is D

2216. In severe periodontitis; probe

A. Get stopped by calculus


B. Goes beyond connective tissues of junctional epithelium
C. Touches coronal end of junctional epithelium
D. Touches the middle of junctional epithelium
E. Touches sulculuar epithelium
Ans C

2217. Characteristic of mucogingival involvement

A. A pocket of more than 4 mm depth

BY DR.ABDULRAHMAN ALMUALM
MJDF MCQS WITH ANSWERS

B. Only 1mm of attached gingiva remains


C. Pocket extends to the mucogingival junction
Ans C

2218. The role of Guided Tissue Regeneration G.T.R. is

A. Prevents apical migration of junctional epithelium


B. Allow the growth of connective tissue in contact with surface
C. Prevent apical migration of junctional epithelium
Ans a
2219. The critical plaque PH is

A. 6
B. 5.5
C. 4
D. 4.5

2220. When it is acceptable for patient to hold radiographic film packet in the patient’s
mouth

A. Patient is very young and can not understand direction


B. Patient is physically handicapped and unable to hold the film
C. Film should never be held by the dentist
D. There is a lack of time and radiograph is essential

2221. Common cause of poor diagnosis in avulsion Replantation

A. External resorptive defects

2222. Posterior vital molar with core the best material to restore it is

BY DR.ABDULRAHMAN ALMUALM
MJDF MCQS WITH ANSWERS

A. Amalgam

2223. The function of incisor pin of an articulator

A. Horizontal and vertical overlap

2224. Which of the following is important consideration when deciding whether to


design an upper partial denture without anterior flang

A. The amount of labial alveolar bone resorption

2225. Zinc Oxide and eugenol impression paste


A. Can not be used in areas with undercuts

2226. When restoring with composite resins, why do we do the cavo surface beveling

A. Aesthetic

2227. In regards to denture stomatitis

A. Due to over growth of some normal commensal of oral cavity

2228. The MOST unfavourable root fracture

A. Cervical third

2229. The GREATEST reliable finding to confirm a necrotic pulp is

A. Area of radiolucency surrounding the apex of tooth

BY DR.ABDULRAHMAN ALMUALM
MJDF MCQS WITH ANSWERS

2230. When preparing Class II cavity, you notice a hard dark brown spot on the
adjacent tooth just below the contact point; MOST LIKELY it is

A. Demineralised enamel

2231. When opening the mouth; in TMJ area

A. Initial rotation followed by translation of condyle

2232. In cavity preparation 1mm below DEJ what is seen

A. More dentinal tubules, some intertubular and peritubular


B. Some dentinal tubules, more intertubular and less peritubular
C. More peritubular, some intertubular and dentinal tubular
D. Equal amount of dentinal tubules, intertubular and peritubular

2233. Pulp with multiple microabscesses will cause eventually

A. Necrosis

2234. Endodontic therapy completed on tooth with periapical radiolucency. Marked


reduction in size of radiolucency is expected in approximately

A. One year

2235. Well constructed complete denture

A. Needs little maintenance


B. Less than a week for adjustment and total success
C. Adverse effects and decrease taste sensations

BY DR.ABDULRAHMAN ALMUALM
MJDF MCQS WITH ANSWERS

2236. To prevent cervical resorption defects following bleaching


A. Remove Gutta Percha at least 2mm below CEJ or above the crest of alveolar bone
and isolate

2237. Which muscle acts on the disto lingual contour of lower denture

A. Mentalis
B. Masseter
C. Mylohyoid
D. Buccinator

2238. The MOST common curvature of palatal root of maxillary first molar is

A. Distal
B. Mesial
C. Buccal
D. Palatal

2239. The reason that endodontically treated teeth are weak is

A. Loss of blood supply


B. Loss of coronal tissues

2240. 58 years old male has had a 60 yo WM course of radiation given for carcinoma
of tongue. Patient complains of pain associated with poor dentition. The dental
management would be

A. Immediate extraction of any poor teeth under local anaesthetic with antibiotic
coverage
B. Segmental dental clearance and closure to eliminate problems
C. No dental treatment may be due to neuronic of neoplasms

BY DR.ABDULRAHMAN ALMUALM
MJDF MCQS WITH ANSWERS

D. Clearance of poor dentition followed by hyperbaric oxygen treatment plus a primary


closure of wounds under antibiotic coverage
E. No extraction as radionecrosis is an important sequelae

2241. On examination of composite restoration you find a dark attain

A. Replace the composite


B. Repair with unfilled resin
C. Apply topical fluoride at the margin

2242. Occasional sensitivity in a shallow class I amalgam restoration after two days
would be managed by

A. Replace old filing immediately


B. Oxide Zinc and eugenol
C. Using thicker mix of cements
D. Tell patient the discomfort will disappear after 4 t o6 weeks
E. Ledermix

2243. Pulp capping in mature tooth may be followed by**

A. Pulpalgia
B. Internal resorption
C. Hypercalcification within root canals
D. All of the above

2244. The MOST common occurrence after direct pulp capping is

A. Signs of reversible pulpitis

BY DR.ABDULRAHMAN ALMUALM
MJDF MCQS WITH ANSWERS

2245. When should not contaminate metallic framework during fabrication of porcelain
fused to metal crown

A. Between bisque stage and glazing stage


B. Between preheat and opaque stages
C. Between opaque and bisque stages
D. Between one opaque and two opaque stages

2246. “Pop off” of a porcelain veneer from under the lying gold crown is due to**

A. Too thick application of pure gold surface conditioner


B. Contamination at the porcelain metal interface
C. Under firing the opaque layer
D. All of the above

2247. Attrition in elderly, why do teeth maintain contact

A. Building bone around the fundus of alveolar bone and deposition of cementum
B. Increased interocclusal distance
C. Formation of dentine

2248. The MOST likely factor contributes to tooth eruption is**

A. The growing root


B. Bone growth
C. Vascular pressure
D. The developing periodontal ligament

2249. Initial condylar guidance of 25 degree was wrong is changed to 45 degree. What
changes will you make to achieve balanced occlusion

BY DR.ABDULRAHMAN ALMUALM
MJDF MCQS WITH ANSWERS

A. Decrease incisal guidance


B. Reduce cusps height
C. Increase compensate curve

2250. Good oral hygiene and fluoridation is LEAST useful in preventing caries of

A. Pit and fissure


B. Smooth surface
C. Inaccessible area

2251. Patient complains of sensitivity; on examination you found a composite restoring


a good cavity preparation without any secondary caries; what is your next step

A. Extirpate the pulp that is obviously inflamed


B. Place ZOE dressing to sedate the pulp
C. Ask patient to come back in six months
D. Repeat restoration

2252. What is the shape of occlusal rest

A. Spoon shape with rounded margin

2253. Regeneration periodontal surgery

A. Regeneration of cementum
B. Long junctional epithelium

2254. What is NOT TRUE about gingivitis

A. Mobility

BY DR.ABDULRAHMAN ALMUALM
MJDF MCQS WITH ANSWERS

2255. Why is the frequency of carbohydrates intake more important quantity

A. Low number of streptococcus mutans


B. Hetero formation is better at low sugar concentration
C. Homo formation is better at high sugar intake
D. Restricted diffusion of acid through plaque

2256. Gingivitis is not caused by

A. Diabetes
B. Viral infection

2257. The elimination half life of Diazepam is in the range of

A. 2-5 hours
B. 5-12 hours
C. 12-30 hours
D. 30-48 hours
E. 48-96 hours

2258. Myxoedema occurs due to**

A. Hypersecretion of the thyroid


B. Hypersecretion of the adrenal
C. Hyposecretion of thyroid-hypothyroidism

D. Hyposecretion of the adrenal

2259. Ulcers, necrosis and plasma cells at the basal membrane with atrophic thin
areas, reduced rete pegs will be diagnosed as

BY DR.ABDULRAHMAN ALMUALM
MJDF MCQS WITH ANSWERS

A. Desquamative gingivitis

2260. Which of the following is seen in benign mucosal membrane pemphigoid**

A. Tzanck cells
B. Intraepithelial vesicles
C. Histopathology like aphthous ulcer
D. Scarring of the conjunctiva

2261. In syphilis

A. Primary lesion is not contagious


B. Oral lesions are not seen in less than 1%
C. Spirochetes disseminate in 24 hours

2262. Which of the following is TRUE about syphilis

A. The spirochetes disseminate rapidly throughout the body within 24hour after
contact
B. Both the primary chancre and the secondary mucous patch stages of the disease
are highly infectious
C. Only the lesions of the primary and secondary stages are contagious
D. All of the above

2263. Which of the following is not true about warfarin

A. INR of 3 is enough to start any extraction


B. Affects extrinsic system and increases prothrombin time
C. Heparin can be given subcutaneously and acts rapidly
D. It takes at least 12 hours for Vitamin K to reverse the effects of coumarin

BY DR.ABDULRAHMAN ALMUALM
MJDF MCQS WITH ANSWERS

2264. Staphylococcus aureus can cause which of the following infection

A. Thyroiditis
B. Pancreatitis
C. Osteomyelitis
D. Scarlatina
E. Pneumonia

2265 A 10 year old child presents with crowding of the


dentition and desires correction. What your next step would be

A. Perform mixed dentition analysis


B. Extract the deciduous teeth
C. Ask the patient to come after the deciduous teeth fall off and complete permanent
dentition erupts
D. Apply a fixed appliances
E. Review in yearly intervals

2266 In regards to paracetamol

A. Liver damage in mild overdose

2267. In regards to periapical lesions, what is TRUE

A. Are predominantly anaerobic


B. Must be treated by antibiotics
C. Must always treated by surgery
D. Change fro aerobic into anaerobic

2268. Patient with weak pulse, moist skin and dyspnoea; what is the first thing to do

BY DR.ABDULRAHMAN ALMUALM
MJDF MCQS WITH ANSWERS

A. Maintain airway and place in supine position


B. Give insulin injection
C. Administer oxygen
D. Inject adrenaline

2269. Single retroclined upper incisor in 9 years old, space is sufficient. What is your
management

A. Anterior inclined plane on mandibular teeth


B. Bite plane
C. Expansion screw
D. Hawley appliance

2270. The angle of blade for closed curettage is**

A. Less than 35 degree


B. Less than 45 degree
C. Less than 90 degree
D. Less than 100 degree

2271. In which of the following conditions vesicles/bullae are never seen prior to
ulceration

A. HSV 1
B. Aphthous ulcer

C. Pemphigus

2272. Patient complains of finger-like growth on the lateral aspect of the tongue. The
lesion is painless and of normal colour. The MOST PROBABLE diagnosis is

BY DR.ABDULRAHMAN ALMUALM
MJDF MCQS WITH ANSWERS

A. Folate papillae
B. Filiform papillae
C. Neurofibroma
D. Papilloma

2273. Thiamine is useful in

A. Collagen synthesis
B. Clotting factor production
C. Epithelial integrity
D. Cellular energy production

2274. Topical fluorides are MOST beneficial when

A. Directly applied on decalcified enamel


B. Applied after eruption

2275. Child presented to you with sore throat, fever and joint swelling; the MOST
probable diagnosis is

A. Rheumatic fever
B. Rheumatic arthritis
C. Osteoarthritis

2276. End product of amino acids metabolism is

A. Urea
B. Uric acid
C. Allantoin

BY DR.ABDULRAHMAN ALMUALM
MJDF MCQS WITH ANSWERS

2277. Patient shows a lesion on the tongue adjacent to sharp tooth. You “rounded off”
the sharp area and recall patient after one month to see the lesion turning smaller in
size. What your next step would be

A. Keep observing
B. Perform an excision biopsy
C. Prescribe Kenalog and Orabase

2278. On HIV patient which of the following IS NOT RECOGNISED

A. Squamous cell carcinoma


B. HIV gingivitis

C. Osteosarcoma
D. External lymphoma
E. Kaposi sarcoma

2279. The MOST primary treatment of ANUG in HIV patient is

A. Prescribe antibiotics
B. Debridement and antimicrobial rinses
C. Gingivoplasty
D. Flap surgery

2280. A patient on dicoumarol treatment needs extraction. Which of the following is


MOST valuable in evaluating surgical risks

A. Clotting time
B. Bleeding time
C. Prothrombin time
D. Sedimentation rate
E. Complete blood cell count

BY DR.ABDULRAHMAN ALMUALM
MJDF MCQS WITH ANSWERS

2281. Infection with new bone formation is

A. Garr’s osteomyelitis
B. Condensing osteitis
C. Torus

2282. You want to place a post on an endodontically treated tooth which has a good
silver point “Ag point”; there is no evidence of failure of the previous root filing. What
would you do

A. Remove and replace the Ag point with Gutta Percha before the post preparation.

2283. The placement of metal stops at a location remote to direct retainers to increase
retention is termed

A. Indirect retainers

2284. The hamular notch is important in full dentures construction because it aids in
the setting position of the artificial teeth

A. First statement is true, but the reason given is false

2285. When setting up teeth for complete dentures having bilateral balanced
occlusion, separation of posterior teeth during protrusion is done by

A. Increasing the anterior posterior occlusal curve

B. Decreasing the angle orientation of the occlusal plane

2286. When patient bites in protrusion you notice that posterior teeth do not meet,
what would you do to solve this

BY DR.ABDULRAHMAN ALMUALM
MJDF MCQS WITH ANSWERS

A. Increase the compensatory curve


B. Decrease the angle of the occlusal plane

2287. The MOST common reason for full denture failure

A. Inadequate interocclusal clearance

2288. A complaint of burning tongue in an elderly female would be a result of

A. A systemic allergy
B. Allergy because of denture
C. Psychogenic

2289. In posterior crossbite situation which are the supporting cusps

A. Upper buccal and lower lingual cusps

2290. The bilaminar zone in reference to TMJ refers to

A. The upper and lower joint spaces


B. The distal attachments of the lateral pterygoid to the condyle

2291. What is the MOST COMMON configuration of the mesial buccal canal of upper
first molar**

A. Two canals and one foramina

2292. What does “SYNERESIS” in prosthodontics mean

A. Loss of water and contraction

BY DR.ABDULRAHMAN ALMUALM
MJDF MCQS WITH ANSWERS

2293. Why would you invest the wax pattern as soon as possible in an indirect inlay
fabrication

A. Minimise distortion
B. Avoid contraction
C. Avoid expansion

2294. Upon palpation which of the following areas would be found to have overlying
mucosa

I. Midline of the palate


II. Mylohyoid ridge
III. Mental foramen IV.Incisive foramen
V. Tori

A. I and II
B. I, II, III
C. I, II, V
D. None of the above
E. All of the above.
Ans C

2295. Why do people with cleft palate/lip have speech difficulties

A. Difficulties in keeping the intraoral pressure.

2296 Patient needs a minimal preparation metal crown for his LL6 the best cement
is
a) GIC
b) zinc phosphates
c) RMGIC
d) zinc polycarboxylate cement

BY DR.ABDULRAHMAN ALMUALM
MJDF MCQS WITH ANSWERS

e) resin cement
Ans e

2297.What is the best type of restoration for restoring buccal root caries in the lower
mandibular molar in elderly patient:
a) Glass ionomer
b) Amalgam
c) Composite
d) Porcelain veneer crown
e) Zinc phosphate cement
ans A

2298.To record the occlusal plane of upper denture we use:


a) Face bow
b) Fox’s bite plane
c) Articulator
d) Freeway space
e) Adjust to the lower rim
Ans B

2299.The best way to determine hinge axis during prosthetic construction


a) Face bow
b) Fox bite
c) Articulator
d) Wax rim
e) Photos and study models
ans C

2300.The liquid which is present in the adhesive dentin systems to wet the dentin is:
a) Water
b) Acetone
c) Polymers liquid
d) Ketones
Ans B
2301.During the try-in stage of complete Denture you need to do slight grinding of
back teeth.
Which teeth cusps should be reduced
a) Reduce teeth randomly
b) Upper buccal and lower lingual
c) Upper palatal and lower buccal
d) Upper palatal and lower lingual
e) Upper buccal and lower buccal
ans B

2302.the obturation technique which gives the best hermetic seal for gutta-percha in
endodontic treatment
a) warm lateral compaction
b) warm vertical compaction
c) thermoplasticised gutta-percha technique
d) cold lateral compaction

BY DR.ABDULRAHMAN ALMUALM
MJDF MCQS WITH ANSWERS

e) chlorpercha technique
Ans C
2303.The use of latex gloves does has the following effect when a polyvinyl siloxane
impression is taken
a) Retard the set of the impression material
b) Enhance the set of the impression material
c) Result in porosities in the impression material
d) Has no effect on impression
e) Latex gloves stick to the polyvinyl siloxane impression material
Ans A

2304.After crown replacement in how many months should you review it


radiogaphically?
a) 1 month
b) 3 months
c) 6 months
d) 1 year
e) Only if become symptomatic
f) Radiolucent band across the tooth
Ans D

2305.Female patient with missing upper lateral incisors, gaps between centrals and
canines
and pointed canine the best treatment option would be:
a) Move the canines by orthodontic means to close gaps
b) Provide fixed-fixed bridge
c) Provide single crowns over the canines to close the gaps to a limit
d) Close the gaps with tooth colored restorations
e) Restorative and orthodontic intervention
f) flumazenil
Ans e

2306.The metal framework of metallic dentures is formed of


a) Nickel chromium
b) Cobalt chromium
c) Stainless steel
d) gold
e) iron
Ans B

2307.The metal framework of Ceramo-metallic crowns is formed of


a) Nickel chromium
b) Cobalt chromium
c) Stainless steel
d) gold
e) iron

BY DR.ABDULRAHMAN ALMUALM
MJDF MCQS WITH ANSWERS

Ans A

2308.First line treatment for patient with NCTL in palatal surfaces of upper front
teeth:
a) Direct Composite palatal veneers
b) Full coverage crowns
c) Gold veneers
d) Indirect composite palatal veneers
e) Indirect porcelain veneers
Ans A

2309.Regarding Using balanced force technique in endodontic treatment


a) Decrease incidence of perforations and fractures in root
b) Ensure better apical seal
c) Ensure moving the irrigation solution coronally
d) Ensure better shaping especially in curved canals
e) Ensure widening of canals coronally to make filling the canal easy
ans D

2310.A 35 year old female patient requires replacement of 6 upper anterior


crowns with a confirmative occlusal scheme. What would be the most
appropriate way to articulate study and working casts?
a) Average value articulator with incisal guidance table.
b) Fully adjustable articulator
c) Plane line articulator.
d) Semi adjustable articulator with facebow transfer and custom incisal
guidance table.
e) Semi adjustable articulator with facebow transfer.
Ans D

2311.Placement of maxillary anterior teeth in complete denture too far superiorly and
anteriorly might result in difficulty in pronouncing :
a) F and V sounds
b) S sound
c) T and S sounds
d) D and T sounds
e) Th and V sounds
Ans A

2312.You are going to do full clearance and immediate denture fabrication, the teeth
that
should be removed last are:
a) Canines and 1st molars
b) Canines and second molars

BY DR.ABDULRAHMAN ALMUALM
MJDF MCQS WITH ANSWERS

c) Start from posterior to anterior


d) Start from anterior to posterior
e) Let 1st premolars last to avoid fractures
ans C

2313.Preparation of margins for full gold crown should be


a) 1 mm
b) 2 mm
c) Rest on sound tooth structure
d) Shoulder
e) subgingival
Ans C
2314.You performed endodontic treatment and crown for a tooth, there’s a 0.5 cm
radiolucent area after 3 years; what is the best action :
a) Apicectomy
b) Apicectomy with MTA retrograde filling
c) Apicectomy with amalgam retrograde filling
d) Remove the crown and provide orthograde filling
e) Give him strong antibiotic and monitor
Ans B
2315.female patient with localized fluorosis and brown grooves affecting her upper
incisor
teeth, what is the treatment of choice :
a) full crowns
b) composite veneers
c) vital bleaching
d) direct composite
e) direct composite application
2316.what is the type of instrument will you use to do scaling under implant retained
bridge
a) metal instrument
b) plastic instrument
c) ultrasonic scaler
d) titanium instrument
e) probe
Ans B
2317.during cavity preparation for amalgam filling, when is the cavity ready to
receive the
restoration?
a) Brown ,dry, firm to the probe
b) brown, wet, soft to the probe
c) dark brown ,wet, soft to the probe
d) yellow brown, dry, firm to the probe
e) yellow brown, wet, soft to the probe
Ans D

2318.The most important factor in post retention


a) Post length
b) Cement material
c) Post material

BY DR.ABDULRAHMAN ALMUALM
MJDF MCQS WITH ANSWERS

d) Post width
e) Post design
ans A

2319.The most difficult post to be removed in case of failure


a) Long parallel threaded
b) Long taper
c) Short tapered
d) Short parallel
e) Custom made post
Ans A

2320.The most important advantage of NITI files in endodontic treatment:


a) Flexibility
b) Enhanced hand control
c) Enhanced features in rotary systems
d) Shape memory
e) Better biocompatibility
Ans A

2321.A patient lost his two lower central incisors in a fight; what do you recommend
a) Fixed fixed Resin bonded bridge
b) Two separate cantilever bridge
c) Fixed fixed conventional
d) Spring cantilever
e) 6 units bridge from canine to canine |ans is C?

2322.A 90-year old gentleman presented to clinic who is edentulous and has
dentures
upper and lower full for 15-years. Denture bit uncomfortable, tooth structures little bit
worn out, freeway space 2-4mm, polished surfaces satisfactory and occlusal wear
minimal. How would you proceed?
a) Copy denture
b) Hard reline
c) Soft reline
d) Construction of new denture
e) Construction of new denture with thermo-press material
Ans A

2323.When preparing the Apical Zone, the use of the files sequentially from apex to
backwards. what is the best distance to achieve good apical seal.
a) 0.5
b) 1

BY DR.ABDULRAHMAN ALMUALM
MJDF MCQS WITH ANSWERS

c) 1.5
d) 2
e) Flush apex
Ans B
2324.Copy denture is indicated for a patient with:
a) Oral Cancer
b) TMJ problems
c) Artificial limbs
d) Hemophilic patient
e) Heavy smoking habit
Ans B
2325.The best way to achieve retention in patient with mandibular free end saddle is:
a) Occlusal rest mesially
b) RPI system, gingival approaching
c) Occlusal rest distally
d) Wide lingual coverage
Ans B

2326.The best way to achieve retention in patient with maxillary free end saddle is:
a) Occlusal rest mesially
b) Occlusal rest distally
c) gingival approaching clasp
d) Attachments
e) RPI
E?

2327.The rest seat preparation should be:


a) 0.5
b) 1
c) 1,5
d) 2
e) 2.5

2328.A 18 years old girl lost her upper canine in accident, she has good oral hygiene
and minimal restored
dentition; to restore the tooth we use:
a) Implant
b) Resin based fixed fixed bridge
c) Resin based cantilever bridge
d) Fixed fixed conventional bridge
e) Conventional cantilever bridge

2329.the best material to be used as medication for non-vital pulpotomy


a) ferric sulphate
b) formecresol
c) Ca/OH catalyst tube
d) MTA

BY DR.ABDULRAHMAN ALMUALM
MJDF MCQS WITH ANSWERS

e) Beechwood creosote

2330.The component of amalgam which decrease creep is:


a) Tin
b) Silver
c) Copper
d) Zinc
e) Gold

2331. 50 years old patient who started smoking since he was 20, he smoked 20
cigarettes for
15 years before he reduced the number to only 10 cigarettes per day. His pack/year
value
is:
a) 10
b) 20
c) 22.5
d) 15
e) 25

Ans C

2332. A reciprocal clasp arm on a removable partial denture will provide


1. resistance to horizontal force.
2. indirect retention.
3. stabilization.
4.direct retention.
A. (1) (2) (3)
B. (1) and (3)
C. (2) and (4)
D. (4) only A
. E. All of the above.
Ans is A

2333. The maxillary cast partial denture major connector design with the
greatest potential to cause speech problems is
A. a thick narrow major connector.
B. an anterior and a posterior bar
C. a thin broad palatal strap.
D. narrow horseshoe shaped.
Ans A

2334. Which of the following structures lies inferior to the mylohyoid muscle at
the level of the mandibular second molar?
A. Lingual artery.

BY DR.ABDULRAHMAN ALMUALM
MJDF MCQS WITH ANSWERS

B. Lingual vein.
C. Lingual nerve.
D. Submandibular duct.
Ans B

2335A healthy, 23 year old male patient experiences a warm sensation,


diaphoresis, nausea, light headedness and then loses consciousness
approximately 30 seconds following the injection of 1.8ml of 2% lidocaine with
1:100,000 epinephrine for an inferior alveolar nerve block. What is the most
likely diagnosis for his situation?
A. Allergic reaction to local anesthetic.
B. Overdose of local anesthetic.
C. Syncope.
D. Intravascular injection of local anesthetic.
Ans C

2336. In an acute periapical abscess, which of the following teeth is most


likely to spread infection to the submandibular space?
A. Mandibular second bicuspid.
B. Maxillary third molar.
C. Mandibular first molar.
D. Mandibular third molar
. E. Mandibular lateral incisor.
Ans D

2337. The most appropriate management for a patient in an early mixed


dentition stage with mild anterior crowding is to
A. strip all proximal contacts.
B. extract the deciduous canines.
C. perform a space analysis.
D. regain space in the arch.
Ans C^

2338. Primate spacing in the primary dentition is observed between


1. maxillary canines and first molars.
2. maxillary canines and lateral incisors.
3. mandibular canines and first molars.
4. mandibular canines and lateral incisors.
. A. (1) and (3)
B. (1) and (4)
C. (2) and (3)
D. (2) and (4)
E. (4) only

2339. Roots of the permanent maxillary central incisors are completed by what
age?
A. 8 years.

BY DR.ABDULRAHMAN ALMUALM
MJDF MCQS WITH ANSWERS

B. 10 years.
C. 12 years.
D. Later than 12 years.
Ans B

2340. Hypothyroidism affects the dental developmental pattern by


A. interfering with jaw growth.
B. delaying the eruption timetable.
. C. causing sclerotic bone to form over the occlusal surface of erupting teeth.
D. accelerating the eruption timetable.
Ans B^

2341. Mandibular condylar region grows by


A. sutural and interstitial proliferation.
B. interstitial and appositional proliferation.
C. appositional and sutural proliferation.
D. interstitial proliferation only.
E. appositional proliferation only
Ans B^

2342. Which of the following is/are associated with the presence of


microorganisms in the bloodstream?
A. Anachoresis.
B. Cavernous sinus thrombosis.
C. Bacteremia.
D. All of the above.|| Ans is D^
2343. Severe throbbing tooth pain which increases when the patient lies down
is a symptom of
A. a pulp polyp (chronic hyperplastic pulpitis).
B. late stage of acute pulpitis (acute suppurative pulpitis).
C. chronic pulpitis (chronic ulcerative pulpitis).
D. chronic apical abscess.
E. pulp hyperemia.
Ans is B

2344.
What clinical evidence would support a diagnosis of acute dento-alveolar
abscess? 1. 2. 3. 4.
1. A negative reaction to the electric vitality tester.
2.A positive reaction of short duration to cold.
3.A positive reaction to percussion.
4.Presence of a draining fistula.
A. (1) (2) (3)
B. (1) and (3)
C. (2) and (4)
D. (4) only
E. All of the above.
Ans B

2345. In the restoration of a tooth, cavity varnish reduces

BY DR.ABDULRAHMAN ALMUALM
MJDF MCQS WITH ANSWERS

A. ion migration from amalgam to tooth.


B. transfer of thermal changes.
C. amalgam corrosion.
D. galvanic stimulation of the pulp.
Ans A

2346. In an 80-year old patient you would expect


A. a reduced size of the pulp chamber.
B. increased incidence of pulp stones.
C. increased tendency to pulpal fibrosis.
D. All of the above.
Ans D

2347. In teeth with complete pulp necrosis, the periapical area is involved if
1. there is pain to thermal stimuli.
2. there is pain on percussion.
3. the tooth throbs when the patient is lying down.
4. the radiograph shows an apical radiolucency.
A. (1) (2) (3)
B. (1) and (3)
C. (2) and (4)
D. (4) only
E. All of the above.
Ans C

2348. Radiographs of the mandibular incisor teeth of a 45 year old healthy black
female patient reveal periapical radiolucencies. The teeth are vital and
asymptomatic. You would
A. perform a biopsy of the radiolucent lesion.
B. perform endodontic therapy on the four incisors.
C. place a drain in the affected area.
D. observe periodically
Ans D

2349. What are the purposes of using occlusal splints?


1. To change the pattern and degree of tactile afferent neural impulses.
2. To immobilize teeth.
3. To produce a permanent change in the occlusion.
4. To prevent teeth from disturbing occlusal sensory input.
A. (1) and (2)
B. (3) only
C. (1) (2) (4)
. D. All of the above.
Ans C

2350. Histologically, a pulp polyp consists of


1. a mass of collagenous fibres.
2. Russell bodies.
3. proliferating capillaries.

BY DR.ABDULRAHMAN ALMUALM
MJDF MCQS WITH ANSWERS

4. fibroblasts.
5. polymorphonuclear leucocytes.
A. (1) (2) (3) (4)
B. (1) (3) (4)
C. (1) (3) (4) (5)
D. (2) and (5)
E. All of the above.
Ans C

2351. Which of the following muscles comprise the retromolar pad?


1. Lateral (external) pterygoid.
2 Buccinator.
3 Palatoglossus.
4 Superior constrictor.
A. (1) (2) (3)
B. (1) and (3)
C. (2) and (4)
D. (4) only
. E. All of the above.

Ans C

2352. Which of the following is NOT a sign or symptom of the myofascial pain
dysfunction syndrome?
A. Pain.
B. Muscle tenderness.
C. Limitation of jaw motion.
D. "Clicking" or "popping" noise in the joints.
E. Radiographic changes of the join
Ans E

2353. Which of the following is/are associated with xerostomia?


1. Atropine administration.
2. Acute anxiety state.
3. Mikulicz's disease.
4. Sjögren's syndrome.
A. (1) (2) (3)
B. (1) and (3)
C. (2) and (4)
D. (4) only
E. All of the above
Ans E

2354. Tissue from a multilocular radiolucent area of the posterior mandible


shows microscopically follicular areas lined with cylindrical cells resembling the
enamel organ. The most likely diagnosis is a/an
A. neurofibroma.
B. ameloblastoma.
C. central fibroma.
D. periodontal cyst.

BY DR.ABDULRAHMAN ALMUALM
MJDF MCQS WITH ANSWERS

E. dentigerous cyst.
Ans B

2355. Unconsciousness in syncope results from


A. electrolyte imbalance.
B. neurogenic shock.
C. cerebral hyperemia.
D. cerebral hypoxia.
Ans D.

2356. Tetracycline will cause crown discolouration when prescribed at the age
of
1. 6 months in utero.
2. 2 years.
3. 7 years.
4. 14 years.
A. (1) (2) (3)
B. (1) and (3)
C. (2) and (4)
D. (4) only
E. All of the above.
Ans A

2357. Prolonged use of nitrous oxide has been shown to produce


1. abortions in females.
2. cancer in occupationally exposed females.
3. liver disease in occupationally exposed males.
4. birth defects in exposed females.
5. suppression of white blood cells.
A. (1) (2) (3)
B. (1) (4) (5)
C. (2) (3) (4)
D. (1) (2) (5)
E. None of the above.
Ans B

2358. Which of the following statements is/are true regarding diazepam?


1. Its long duration of action is partly due to active metabolites.
2. It does not produce antianxiety effects after intramuscular administration.
3.Intravenous administration is more reliable than oral.
4. Its sedative effect can be reversed by naloxone.
A. (1) (2) (3)
B. (1) and (3)
C. (2) and (4)
D. (4) only
E. All of the above.

BY DR.ABDULRAHMAN ALMUALM
MJDF MCQS WITH ANSWERS

Ans B

2359. Which of the following can be mistaken on a radiograph for a chronic


alveolar abscess?
A. Mental foramen.
B. Cementoma in its early stages.
C. Posterior palatine foramen.
D. All of the above.
Ans D

2360. Displacement of mandibular fractures is dependent upon


1. direction of fracture line.
2. proprioceptor nerve action.
3. muscle pull.
4. tooth in line of fracture.
5. direction of blow.
A. (1) and (3)
B. (1) (3) (5)
C. (1) (3) (4)
D. (2) (3) (5)
E. All of the above.

Ans B

2361 In a standard dental cartridge (carpule) containing 1.8ml 2% lidocaine with


epinephrine 1/100,000, the amount of vasoconstrictor is
A. 18.0 mg.
B. 0.018 mg.
C. 1.8 mg.
D. 0.18 mg.
E. 180.0 mg.
Ans B

2362. An infected root is accidentally displaced into the maxillary sinus.


Examination of the socket reveals perforation of the sinus lining with an
oro-antral communication. Therapy should consist of
1. antrostomy for retrieval of root.
2. closure of oro-antral communication and antibiotic coverage.
3. antibiotic coverage and observation.
4. acrylic template to cover socket opening and saline rinses.
A. (1) only
B. (3) only
C. (4) only
D. (1) and (2)
E. (1) and (3)
Ans D.

2363. Bilateral dislocated fractures of the mandibular condyles result in

BY DR.ABDULRAHMAN ALMUALM
MJDF MCQS WITH ANSWERS

1. anterior open bite.


2. anesthesia of the mental nerves.
3. inability to protrude the mandible.
4. inability to bring the molars into contact.
A. (1) (2) (3)
B. (1) and (3)
C. (2) and (4)
D. (4) only
E. All of the above.
Ans B

2364. Difficulty in mouth opening, dysphagia, tongue stiffness and generalized


induration of the skin are characteristic of
A. lupus erythematosus.
B. scleroderma.
C. erythema multiforme.
D. lichen planus.
E. malignant disease.
Ans B

2365. Which drug should be administered as the initial management for a


patient with chest pain consistent with a myocardial infarction?
A. Epinephrine.
B. Atropine.
C. Diphenhydramine.
D. Lidocaine.
E. Acetylsalicylic acid.
Ans E

2366 .The study using data from entire population to compare


between lung cancer deaths in relation to per capita cigarette
consumption is:
Case report
Cohort study
Correlation studies
Cross-sectional studies

2367.A random selection of 1200 adults agreed to participate in a


study of the possible effects of drug X. They are followed
prospectively for a period of five years to see if there is an

BY DR.ABDULRAHMAN ALMUALM
MJDF MCQS WITH ANSWERS

association between the incidence of cataract and the use of


drug X. This type of study is a:
Case-control study.
Randomized controlled clinical trial.
Cross-sectional study.
Cohort study.

2368.You are interested in finding out what the risk indicators


are for a rare form of oral cancer and decide to undertake a
study to examine this. What type of study would be the most
appropriate for addressing this issue?
Cohort
Prevalence study
Clinical trial
Case-control study

2369.A study was conducted with the following objective, to


determine the individual and combined effects of potential
risk factors in relation to the temporomandibular disorder,
Pain Dysfunction Syndrome (PDS). The first group comprised
new referrals between May 1997 and August 1999 to the
temporomandibular disorder clinic of the University Dental
Hospital, diagnosed with PDS. The second group consisted of
patients without PDS randomly selected from twenty-four
dental practices. Using a postal questionnaire information
was collected on socio-demographic, local mechanical,
psychological factors, co-morbidities and illness
behaviour.What type of study is this?
Case series
Cross-sectional study
Case-control study
Case report
Randomised clinical trial

2370.Bitewings in Proximal caries detection has


a. Moderate sensitivity & low specitivity
b. Moderate sensitivity & moderate specitivity
c. Moderate sensitivity & high specitivity
d. High sensitivity & moderate specitivity
e. High sensitivity & high specitivity

BY DR.ABDULRAHMAN ALMUALM
MJDF MCQS WITH ANSWERS

2371.Bitewings in Occlusal caries detection has


a. Moderate sensitivity & low specitivity
b. Moderate sensitivity & moderate specitivity
c. Moderate sensitivity & high specitivity
d. High sensitivity & moderate specitivity
e. High sensitivity & high specitivity

2372.Mobility in peri-implantitis has


a. Moderate sensitivity (50%) & low specitivity (25%)
b. Moderate sensitivity & moderate specitivity
c. Moderate sensitivity & high specitivity
d. High sensitivity & moderate specitivity
e. High sensitivity & high specitivity

2373.In studying the impact of dental caries on communities,


Care index is used sometimes? What is the most appropriate
interpretation of care index?
Shows the prevelance of decay in community
Shows the extent to which decay is prevented
Shows the extent to which decay is treated by extraction
It shows the extent to which decay is treated restoratively
Shows the incidence of new decay

2374.In dental epidemiology, indices are used to measure the


oral health of a population. The DMF index is commonly used
to measure the prevalence and severity of dental caries. In
dental epidemiology the main limitation of DMFT index is
It does not allow statistical analysis
It gives equal weight to decayed missing and filled teeth
It is difficult to calibrate examiners
It is reversible
It is not gold standard

2375.What is the % lies between the 3 standard deviation of the


mean? 99.7%

What is the % lies between the 2 standard deviation of the


mean? 95%

What is the % lies between the 1 standard deviation of the


mean? 68%

BY DR.ABDULRAHMAN ALMUALM
MJDF MCQS WITH ANSWERS

What is the % lies between the less than 1 standard deviation of


the mean? 68.27%

2376.In a randomised double-blind single-centre clinical trial, the


effect of pre-operative administration of Ibuprofen 600mg or
Diclofenac 100mg tablets was compared for post-operative
pain relief in 119 patients having day surgery under general
anaesthesia for surgical removal of impacted third molars.
Pain was assessed using visual analogue scales (VAS) pre-
operatively, 15minutes, 30 minutes, 1 and 3 hours post-
operatively.Which answer is correct? The trial was double-
blind because
Each patient received both treatments
The patients could choose the treatment
The patients did not know they were in a trial
Both patients and the nurse assessing did not know treatment
allocation
The nurse assessing the patients knew which treatment they have
received

2377.A dental company has claimed that ozone is better than


conventional methods for treating decay in peoples'
mouths.Which of the following would provide the best
evidence to support these claims?
Systematic review of Randomised Controlled Trials
Randomised controlled trial
Cohort study
Case/control study

2378.A GDP in Manchester randomly selects 5000 patients,


aged between 45 and 55 years, who are free from
periodontitis. He determines that 300 of them are smokers.
He follows all 5000 patients for 10 years, by which time 150
had developed periodontitis. Of the patients with
periodontitis, 60 had previously been identified as smokers

BY DR.ABDULRAHMAN ALMUALM
MJDF MCQS WITH ANSWERS

and 90 as non-smokers.What type of study is this? Cohort


study

Floss and future plaque relation = Cohort study

2379.Impact of smoking on the discoloration of teeth using


patients arriving for their first appointment following
registration at a practice = Cohort study

A crossover study, also referred to as crossover trial, is a longitudinal


study in which subjects (same individual) receive a sequence of different
treatments (or exposures).

1. Wisdom teeth removal in adults during their 20s-30s = Cross over


2. Longitudinal study in 20s n 30s = Cross over
3. The effectiveness of dental floss and dental tape in reducing the
development of plaque = Cross over

2380.When drug is evaluated for uselessness in controlled


condition, it is termed as trial signifying?
Efficacy
Effectiveness
Efficiency
Effect modification
accuracy

2381. Parasympathetic nerve supply to the salivary glands is by

a)Vagus and glossopharyngeal nerves

b)facial and glossopharyngeal

c)facial and lingual


ans B

2382. Which of the following combinations of milliamperage and kilovoltage will give
Xradiation with the maximum penetration?
A. 10kVp - 65ma.
B. 85kVp - 5ma.

BY DR.ABDULRAHMAN ALMUALM
MJDF MCQS WITH ANSWERS

C. 90kVp - 10ma. .
D. 65kVp - 15ma. .
E. 75kVp - 40ma.
Ans is C

2383. Submandibular salivary gland gives the major contribution at rest?

True

False

Ans True

2384-parotid salivary gland gives the major contribution at function


True
False
Ans True

2385. - Loss of taste sensation is attributed to which nerve? CHONDRA TYMPANI


TRUE

2386. The occlusal parameter that is most useful to differentiate between an overbite
of dental or skeletal origin is the
A. mandibular curve of Spee.
B. mandibular curve of Wilson.
C. molar sagittal relationship.
D. mandibular anterior lack of space.
E. maxillary curve of Wilson.(
ans is A)

2387. Pleomorphic tumour is most affects theminor salivary glands?


A- False, it affects the parotid gland in 75% of cases.

.2388 What is the time for the lower permanent canine to erupt?
A- 9-11 years.
2389. You want to make root canal treatment in the first lower permanent molar, what
nerves you need to anaesthetise?
A- Inferior Dental.

.2390 A dry and crumbly mix of amalgam can be the result of


A. under trituration
. B. over trituration.
C. high copper content.
D. lack of zinc content.
Ans A

.2391 Which antibiotic is CONTRAINDICATED for a patient with a history of a Type 1


anaphylactic reaction to penicillin?

BY DR.ABDULRAHMAN ALMUALM
MJDF MCQS WITH ANSWERS

A. Azithromycin
. B. Cephalexin.
C. Clindamycin.
D. Erythromycin.
Ans B

2392 Which of the following penicillins is most effective against Gram-negative organisms?
A. Nafcillin.
B. Ampicillin
C. . C. Methicillin.
D. D. Penicillin V.
E. E. Phenethicillin.
Ans B

2393 The most common cause of persistent post-operative sensitivity following the
placement of posterior composite resin restorations is
A. hyperocclusion.
B. microleakage
. C. acidic primers.
D. residual caries.
Ans B

2394 Space closure is LEAST likely to occur following the premature loss of the primary
A. maxillary second molar.
B. mandibular second molar.
C. maxillary first molar.
D. mandibular canine.
E. maxillary central incisor.
Ans e

2395 A 14 year old boy presents with bilateral pearly-white thickening of the buccal mucosa
which has been present since birth. His brother has similar lesions. The most likely diagnosis
is A. leukoplakia.
B. lichen planus.
C. mucous patches.
D. white sponge nevus.
Ans D

2396 The white appearance of the oral mucosa following extended local application of
acetylsalicylic acid is the result of
A. hyperparakeratosis.
B. acanthosis.
C. coagulation necrosis
. D. edema.

BY DR.ABDULRAHMAN ALMUALM
MJDF MCQS WITH ANSWERS

Ans C

2397 Thinned cortical bone, decreased cancellous trabeculation, enlargement of the


medullary cavity and decreased bone density are radiographic features of
A. osteomalacia
B. . B. osteopetrosis.
C. C. primary occlusal trauma.
D. D. osteoradionecrosis.
E. E. osteoporosis.
Ans E

.2398 Sharpey’s fibres run in 5 directions: crestally, apically, horizontally, obliquely and
interradicularly? A- True.

2399The lamina dura appears as a dark thin line on X- Ray? A-False, it appears as a white
line arround the teeth indicating low rate of turn over in the bone.

2400.The cementum could be cellular or acellular? A-True.

2401-The first formed cementum is the acellular one? A-True, and the cellular forms later.

2402-Cementum is a static tissue like enamel?


A- False, it responds to the functional requirements of teeth.

. 2403 Amoxicillin and ampicillin are best avoided in which of these:


a- Alcholisim,
b- - Asthma
c- - Gout,
d- Addison disease.
Ans C
2404Tetracycline is best avoided in children under 7 and in myasthenia gravis.
True

.2405 Ampicillin and amoxycillin are contraindicated in infectious mononucleosis


True

2406Metronidazole is contraindicated in pregnancy and alcoholism.


True

2407Oral penicillin is the main cause of anaphylactic shock?


A- False. Parenteral penicillin not the oral is the main cause.

2406Anaemia, ischemia, or poisoning CO in blood can cuses hypoxia and cell death?
A- True.

BY DR.ABDULRAHMAN ALMUALM
MJDF MCQS WITH ANSWERS

2407Megamitochonria are seen in Alcoholic liver disease and nutritional defieceincies?


True

2408In the mandibulr first permanent molar, the buccal cusps are smaller and less pointed
that the lingual cusps?
True

2409What is the Dryopithecus pattern in the mandibular first permanent molar?


A- It is a primitive pattern where the mesiobuccal and the distobuccal cusps join at the
floor of the central fossa.

2410which muscle elevates the mandible during closure?


a. masseter
b. temporalis
c. digastric
d. lateral pterygoid

Ans A

2412what is given sublingually in the treatment of ischemic heart attack


A. Nirtoglycerine
B. IsoSorbide mononitrate
C. Both
D. None
Ans is C

2413. histological differences between pemiphigiod and pemphigus?

Pemiphigius is intraepthilial ,intercellular IgG and C3 ,heals without scaring ,skin lesion
involved

Pemiphigoid is supraepithilial, linear IgG and C3 at the basement membrane zone,heals


with scaring ,skin lesion is uncommon

2414. Patient very anxious long term treatment?


A : fluxetin
B: diazepam
C: amitryptillin

Ans C

2415. Pt with lower incisor radiolucency.. in x ray it is Acceptable percentage it will be?
A: not more than 50%
B: not more than 20%
C: not more than 10%

BY DR.ABDULRAHMAN ALMUALM
MJDF MCQS WITH ANSWERS

2416 Examining a patient in dental office is equivalent to community dental health:

A. Analysis of data
B. Program planning
C. Program operation
D. Surveying
E. Evaluation

Ans is D

2417. 59. Patient come to you with the request of replacing her old partial denture with a
new one. Which denture design is best for taste and phonetics while eating?
a. Relieving the anterior area
b. Relieving the posterior area
c. Relieving the center area

Ans C.

2418. 72. The Department of Health publication “A First Class Service” can be defined as a
framework through which NHS organisations are accountable for continuously improving
the quality of their services and safeguarding high standards of care by creating an
environment in which excellence in clinical care will flourish.”. What does the statement
indicates?
a. Clinical Quality Assurance
b. Clinical Audit
c. Clinical governance
d. Clinical Operational Service

Ans c

BY DR.ABDULRAHMAN ALMUALM

You might also like